Anda di halaman 1dari 160

CONSTITUTIONAL LAW

Professor: Atty. Rene B. Gorospe


Source: Gorospe, Rene B. (2006). CONSTITUTIONAL LAW Notes and Readings on the Bill of Rights, Citizenship and Suffrage (Vol. 1 and 2).
Quezon City: Rex Printing Company, Inc.

Chapter 1
The Fundamental Powers and the Bill of Rights Police Power

The  Bill  of  Rights  is  a  “charter  of  liberties  for  the  individual  and   It   has   been   defined   as   the   ‘state   authority   to   enact   legislation  
a   limitation   upon   the   power   of   the   state.”   The   purpose   is   to   that may interfere with personal liberty or property in order to
protect the people against arbitrary and discriminatory use of promote  the  general  welfare’
political power.
It includes: (1) an imposition or restraint upon liberty or
Calalang v. Williams property, (2) in order to foster the common good
70 Phil. 726 (1940) Being what it is, police power cannot stand still. It also has to
adjust to the demands and realities of changing times
Commonwealth Act No. 548 prohibits animal-drawn vehicles from
passing along certain Manila streets during certain hours
It may be delegated to the (1) President and (2) administrative
Maximo Calalang assails its constitutionality on the ground that it boards as well as (3) the law-making body of municipal
is an unlawful interference with legitimate business or trade and corporations or local government units. Once delegated, the
abridge the right to personal liberty and freedom of locomotion agents can exercise only such legislative powers as are
conferred on them by the national lawmaking body
Held: No. It was passed in the exercise of the paramount police
power of the state Requisites -- lawful ends through lawful means
Persons and property may be subjected to all kinds of restraints
United States v. Toribio
and burdens, in order to secure the general comfort, health, and
15 Phil. 85 (1910)
prosperity of the state
Act No. 1147 regulates the registration, branding and slaughter of
Liberty should not be made to prevail over authority because then
large cattle. The provisions of the said law requires, before large
society will fall into anarchy. Neither should authority be made to
cattle may be slaughtered or killed for food at the municipal
prevail over liberty because then the individual will fall into
slaughterhouse, a permit be obtained from the Municipal
slavery
Treasurer

The Fundamental Powers Appellant was convicted of slaughtering an animal without the
requisite permit
They are inborn in the very fact of statehood and sovereignty.
They are necessary and indispensable as there can be no Appellant contends the constitutionality on the ground that it
effective government without them violates   the   provision   the   “no   law   shall     be   enacted   which   shall  
deprive any person of life, liberty, or property without due
process  of  law”
They are all exercised primarily by the national legislature
Held:  No.   The   act   primarily   seeks  to   protect   the   “large   cattle”   of  
Police Power Eminent Domain Taxation the Philippine Islands against theft and to make easy the recovery
Maintenance of a Just Form of and return of such cattle to their proper owners, when lost,
healthy economic compensation for protection and strayed, or stolen
standard of society the property benefits from the
taken government All property is acquired and held under the tacit condition that it
shall not be so used as to injure the equal rights of others or
Regulates both Affects only greatly impair the public rights and interests of the community
liberty and property rights
property Rights of property are subject to such reasonable limitations in
May be exercised May be May be exercised their enjoyment as shall prevent them from being injurious xxx
only by the delegated to only by the
government some other government Confronted by such conditions, there can be no doubt of the right
entities in the of the Legislature to adopt reasonable measures for the
preservation of work animals, even to the extent of prohibiting
private sector
and penalizing what would, under ordinary conditions, be a
Property taken is Property taken is meant for public use perfectly legitimate and proper exercise of rights of ownership
destroyed or purpose and control of the private property of the citizen
Compensation not Receipt of market Immediate and
immediate; value of his apparent in the Police power v. Due process
sometimes leaving property that is form of
the reward to be taken protection and Due   process   is   the   “embodiment   of   the   sporting   idea   of   fair  
reaped through his benefits derived play.”   The   minimum   requirements   of   due   process   are   notice  
recognition that he from the use of and hearing which, generally speaking, may not be dispersed
has done taxes paid with because they are intended as a safeguard against official
something for the arbitrariness
public good
Previous judicial hearing, however, may be omitted without “The   drift   is   towards   social   welfare   legislation   geared   towards  
violation of due process in view of the nature of the property state policies to provide adequate social services, the
involved or the urgency of the need to protect the general promotion of the general welfare, social justice as well as
welfare from a clear and present danger human  dignity  and  respect  for  human  rights”

The protection of the general welfare is the particular function Police power is the power to prescribe regulations to promote
of the police power which both restrains and is restrained by health, morals, peace, education, good order or safety and
due process general welfare of the people

The justification is found in the venerable Latin maxims, Salus Police Power v. General Welfare Clause
populi est suprema lex and Sic utere tuo ut alienum non laedas,
which call for the subordination of individual interests to the Police power may be delegated to and exercised by local
benefit of the greater number government units through the so-called General Welfare Clause

Ynot v. IAC Villacorta v. Bernardo


148 SCRA 569 (1987) 143 SCRA 480 (1986)

EO 626-A prohibited the interprovincial transportation of carabao The Municipal Board of Dagupan City adopted Ordinance No. 22
(the  poor  man’s  tractor)  and  carabeef  and  subjected  carabao  and   seeking to regulate the subdivision plans over parcels of land
carabeef transported in violation of its provisions to confiscation located therein, which requires, among others, an approval from
and forfeiture, to be distributed to charitable institutions xxx the City Engineer and payment of a service fee and a certification
Whereas, the present conditions demand that the carabaos and from the City Engineer
buffaloes be conserved for the benefit of small farmers who rely
on them for energy needs An action was brought against its constitutionality

Six carabaos transported by Ynot in a pump boat from Masbate to Held: Yes. Ordinance No. 22 is null and void being in conflict with
Iloilo were confiscated by a police station commander of Barotac Section 44 of Act 496
Nuevo, Iloilo
So many excesses are attempted in the name of the police power
Held: Yes. We find that the challenged measure is an invalid xxx
exercise of the police power because the method employed to
conserve the carabaos is not reasonably necessary to the purpose Police Power v. Vices
of the law and, worse, is unduly oppressive. Due process is
violated because the owner of the property confiscated is denied
The power to tax (the power to destroy) cannot be allowed to
the right to be heard in his defense and is immediately
condemned and punished defeat an instrumentality or creation of the very entity which
has the inherent power to wield it
Police power is inherent in the state but not in municipal
corporations. A valid delegation of police power may arise from What is settled is that the matter of regulating, taxing or
express delegation, or be inferred from the mere fact of the otherwise dealing with gambling is a State concern and hence,
creation of the municipal corporation it is the sole prerogative of the State to retain it or delegate it
to local governments
Binay v. Domingo
201 SCRA 508 (1991) Basco v. PAGCOR
197 SCRA 52 (1991)
The Municipality of Makati approved Resolution No. 60 ratifying
the ongoing Burial Assistance Program, extending financial The PAGCOR was created by virtue of PD 1067-A (and PD 1869)
assistance coming from the municipal treasury to bereaved and  was  granted  a   franchise   “to   establish,   operate   and   maintain  
families with gross family income of less than P2,000.00 gambling casinos on land or water within the territorial
jurisdiction  of  the  Philippines”
The COA disapproved Resolution 60. It held that the resolution
cannot be sustained as a legitimate exercise of the police power Petitioners alleged that such law is   “null   and   void”   for   being  
due to a lack of perceptible connection or relation between the “contrary   to   morals,   public   policy   and   public   order”;   it   further  
objective sought to be attained and the alleged public safety, contends that its exemption from paying any tax is violative of the
general welfare, etc. of the inhabitants of Makati, and, that the principle of local autonomy (waiver of right of City of Manila to
disbursement of funds was not for a public purpose since it was impose tax)
for the benefit of only a few individuals and not the whole or
majority of the inhabitants of the Municipality Held: No. Gambling in all its forms, unless allowed by law, is
generally prohibited. But the prohibition of gambling does not
Held: Yes. The care for the poor is generally recognized as a public mean that the Government cannot regulate it in the exercise of its
duty. The support for the poor has long been an accepted exercise police power
of police power in the promotion of the common good. PAGCOR has a dual role, to operate and to regulate gambling
Resolution No. 60 is a paragon of the continuing program of our casinos. The latter role is governmental, which places it in the
government towards social justice category of an agency or instrumentality of the Government, thus,
exempt form local taxes
COA is not attuned to the changing of the times. Public purpose is
not unconstitutional merely because it incidentally benefits a Before: Tax Credit (Taxation & Eminent Domain exercised)
limited number of persons After: Tax Deduction (Taxation & Police Power exercised)

Eminent Domain

2|P LATON
or self-preservation
It is an inherent power of the State that enables it to forcibly Entails payment of The property is precisely destroyed
acquire private lands intended for public use upon payment of just compensation as a way of promoting the greater
just compensation to the owner welfare of the populace who might
be endangered or otherwise placed
In times of national emergency, the government temporarily in  harm’s  way
takes over a public utility imbued with public interest pursuant Primarily exercised May be exercised by private
to Article XII, Section 17 of the Constitution, it exercises police by the government individuals
power and not its power of eminent domain. Accordingly the
private entity-owner cannot claim any just compensation for Taxation
the use of the said business and its properties
The power of the State to impose a charge or burden upon
The police power being the most active power of the person, property, or property rights, for the use and support of
government and the due process being the broadest limitation the government
on governmental power, the conflict between this power of
government and the due process of the Constitution is Taxation is a destructive power which interferes with the
oftentimes inevitable personal and property rights of the people and takes from them
a portion of their property for the support of the government.
City Government of QC v. Ericta Tax statutes must be construed strictly against the government
122 SCRA 759 (1983) and liberally in favor of the taxpayer
The Quezon City Council passed Ordinance No. 6118, S-64
regulating the establishment, maintenance and operation of Limitations on the Power to tax
private memorial type cemetery or burial ground. Section 9 of the
said ordinance required that at least 6% of the total area of every The rule of taxation shall be uniform and equitable. The
memorial park cemetery must be set aside for charity burial, Congress shall evolve a progressive system of tax
otherwise, such cemetery will be prohibited from selling memorial
park lots Uniformity means that persons or things of the same class shall
be taxed at the same rate. It requires that all subjects or objects
Himlayang Pilipino contends that Section 9 is not a valid exercise
of taxation, similarly situated, are to be treated alike or put on
of police power
equal footing both in privileges and liabilities
Held: Yes. The power to regulate does not include the power to Uniformity, however, is not equality, the latter term signifying
prohibit. A fortiori, the power to regulate does not include the that the taxes shall be strictly proportional to the relative value
power to confiscate of the taxable property

Section 9 is not a mere police regulation but an outright It is also an inherent limitation on the power to tax that the
confiscation. It deprives a person of his private property without proceeds be for public purpose. They could not be used for
due process of law, nay, even without compensation
purely private purposes xxx the real purpose of taxation is the
promotion of the common good
Expropriation requires payment of just compensation
Taxation is said to be equitable when its burden falls on those
This is a power that may be exercised by entities other than the
better able to pay. Taxation is progressive when its rate goes up
government itself or its subdivisions and instrumentalities. The
depending on the resources of the person affected
private corporations serving the public, such as public utilities,
may validly be delegated the power.
It is the strongest of all powers of government
xxx those engaged in the supply of electricity, water,
The taxing power has the authority to make a reasonable and
telecommunications services and some transportation firms
natural classification for purposes of taxation but the
whose services might require the acquisition of private
government’s  act  must  not  be prompted by a spirit of hostility,
property for the efficacious service to the public, may also be
or at the very least discrimination that finds no support in
vested with the power of expropriation
reason
Before a municipal corporation may exercise its power of
Taxes are the lifeblood of the government and so should be
eminent domain, it must be sanctioned and must not violate
collected without unnecessary hindrance. However, such
any law
collection should be made in accordance with law xxx
Private lands, for purposes of socialized housing, rank last in the Reyes v. Almanzor
order of priority for acquisition, and expropriation proceedings 196 SCRA 322 (1991)
are to be resorted to only after the other modes of acquisition
have been exhausted Petitioners are owners of parcels of land in Manila which are
leased and occupied as dwelling sites by tenants. RA 6359 was
Eminent Domain v. Destruction by Necessity enacted prohibiting from increasing in monthly rentals of dwelling
units and also disallowing the ejectment of lessees upon the
expiration of the usual legal period (amended by PD 20).
EMINENT DOMAIN DESTRUCTION BY NECESSITY
Thereafter, City Assessor of Manila increased tax rates.
Connotes taking Speaks for itself--condemnation of a
for use property as a means of self-defense Petitioners  averred  that  the  reassessments  made  were  “excessive,  

3|P LATON
unwarranted, inequitable,  confiscatory  and  unconstitutional” rights accruing to the owner in favor of the farmer-beneficiary, an
exercise of the power of eminent domain
Held: Yes. Public respondents would have this Court completely
ignore the effects of the restrictions of the said law on the market The power being exercised is eminent domain if the property
value of properties within its coverage involved is wholesome and intended for public use. Property
condemned under the police power is noxious or intended for a
Tax exemptions noxious purpose which should be destroyed in the interest of
public safety, morals, etc. The confiscation of such property is
xxx as broad as the power to tax not compensable, unlike the taking of property under the
power of expropriation, which requires the payment of just
Like any other power, it is one that may not be exercised compensation to the owner
arbitrarily or whimsically
The Bill of Rights
The  Constitution  declares  outright  that:  “Charitable  institutions,  
churches and parsonages or covenant appurtenant thereto, As a Check on Governmental Powers Only
mosques, non-profit cemeteries, and all lands, buildings, and
improvements, actually, directly, and exclusively used for It governs the relationship between the individual and the State
religious, charitable, or educational purposes shall be exempt and its agent. The Bill of Rights only tempers governmental
from  taxation” power and protects the individual against any aggression and
unwarranted interference by any department of the
No law granting any tax exemption shall be passed without the government and its agencies
concurrence of a majority of all the Members of the Congress
Partnership among Fundamental Powers In the absence of governmental interference, the liberties
guaranteed by the Constitution cannot be invoked against the
Ermita-Malate Hotel and Motel Operators Association, Inc.
State. It concerns not the relation between individuals,
v. City Mayor of Manila
20 SCRA 849 (1967)
between a private individual and other individuals. What the
Bill of Rights does is to declare some forbidden zones in the
The Municipal Board of the City of Manila enacted Ordinance No. private sphere inaccessible to any power holder
4670 regulating the operation of hotels and motels
People v. Marti
Petitioners sought to invalidate the ordinance 193 SCRA 57 (1991)

Held: No. The presumption is all in favor of validity xxx The local The appellant sought to have (4) gift -wrapped packages sent to a
legislative body, by enacting the ordinance, has in effect given friend in Zurich, Switzerland through services of a forwarding
notice that the regulations are essential to the well being of the company,   the   “Manila   Packing   and   Export   Forwarders.”   Upon  
people inspection of the box, dried leaves of marijuana were found in the
packages
Negatively   put,   police   power   is   “that   inherent   and   plenary  
power in the State which enables it to prohibit all that is hurtful He questions the admissibility of the marijuana, contending that it
was a product of an illegal search and seizure
to the comfort, safety, and welfare of society
Held: No. The evidence sought to be excluded was primarily
There is no controlling and precise definition of due process. It discovered and obtained by a private person, acting in his private
furnishes though a standard to which governmental action capacity and without the intervention and participation of State
should conform in order that deprivation of life, liberty or authorities
property, in each appropriate case, be valid
As a Yardstick of Validity and the Standards of Review
Taxation  may  be  made  to  implement  the  state’s  police  power
As a counterweight to the great powers of the government, the
Bill of Rights would pose a constant standard of measurement
to determine the validity of any governmental act which may
The liberty of the citizen may be restrained in the interest of limit rights and liberties, or intrude into privacies of persons, or
the public health, or of the public order and safety, or otherwise impair their freedoms
otherwise within the proper scope of the police power
Determining whether there is sufficient justification for the
Association of Small Landowners of the Philippines government’s   action   depends   very   much   on   the   level   of  
v. Secretary of Agrarian Reform scrutiny or the standards of review used
175 SCRA 343 (1989)
Standards of review--the mere rationality deferential review
PD No. 27 was enacted to provide for the compulsory acquisition
of private lands for distribution among tenant-farmers (CARP)
standard, the middle-level review or intermediate review or
heightened scrutiny standard, and the strict scrutiny standard
The constitutionality of such law is challenged raising, among
others, issues of due process and just compensation Strict scrutiny--used today to test the validity of laws dealing
with the regulation of speech, gender, or race and facial
Held: No. The taking contemplated is not a mere limitation of the challenges are allowed for this purpose; for determining the
use of the land. What is required is the surrender of the title to quality and the amount of governmental interest brought to
and the physical possession of the said excess and all beneficial
justify the regulation of fundamental freedoms; focus is on the

4|P LATON
presence of compelling, rather than substantial governmental limitations except treaty obligations that the revolutionary
interest and on the absence of less restrictive means for government, as the de jure government in the Philippines,
achieving that interest assumed under international law

We hold that the Bill of Rights under the 1973 Constitution was
Deferential review--laws are upheld if they rationally further a not operative during the interregnum. However, we rule that the
legitimate governmental interest, without the courts seriously protection accorded to individuals under the Covenant and the
inquiring into the substantiality of such interest and examining Declaration remained in effect during the interregnum
the alternative means by which the objectives could be
achieved; the substantiality of the governmental interests is xxx During the interregnum, a person could not invoke any
seriously looked into and the availability of less restrictive exclusionary right under a Bill of Rights because there was neither
alternatives are considered a constitution nor a Bill of Rights during the interregnum

To hold that the Bill of Rights xxx remained operative during the
Rational Basis Test has been described as adopting a interregnum would render void all sequestration orders issued by
“deferential”   attitude   towards   legislative   classifications;   it   the PCGG before the adoption of the Freedom Constitution
remains a primary standard for evaluating the constitutionality
of a statute Nevertheless, even during the interregnum the Filipino people
continued to enjoy, under the Covenant and the Declaration,
Strict scrutiny is applied when the challenged statute either (1) almost the same rights found in the Bill of Rights xxx
classifies on the basis of inherent suspect characteristic or (2)
Clearly the raiding team exceeded its authority when it seized
infringes fundamental constitutional rights, i.e. the right to
[such] items
procreation, the right to marry, free speech, etc
Political, Civil, Economic and Other Rights
Intermediate scrutiny or heightened scrutiny standard is
applied  when  the  challenged  statute’s  classification  is  based  on  
The Bill of Rights is basically about political and civil rights as
either (1) gender or (2) legitimacy; Intensified mean Test, in
contradistinguished from economic rights which are dealt with
which, the court should accept the legislative end, but should
in the constitutional provisions on the national economy and
closely scrutinize its relationship to the classification made
patrimony, as well as in the provisions on social justice and
human rights
What is important to bear in mind is this: To the extent that a
particular liberty interest is considered more important and
Human rights can be understood to include those that relate to
more valuable to society, to that extent must the courts utilize
an   individual’s   social, economic, cultural, political and civil
a more demanding and exacting standard with which to
relations; the universally accepted traits and attributes of an
measure that governmental intrusion protected spheres
individual, along with what is generally considered to be his
inherent and inalienable rights, encompassing almost all
The Bill of Rights and the 1986 Interregnum
aspects of life
Can the rights and freedoms guaranteed by the Bill of Rights
What the law guarantees as human right in one country should
exist when there is no constitution?
be also guaranteed by law in all other countries
Republic v. Sandiganbayan
407 SCRA 10 (2003) “Right   which   inheres   in   persons   from   the   fact   of   their  
humanity”
Major Gen. Josephus Q. Ramas, who was the Commanding
General of the Philippine Army, was investigated by the PCGG “Civil  rights”  refers  to  those  rights  that  belong  to  every  citizen  
through its AFP Anti-Graft Board for alleged ill-gotten and of the state or country including the rights of property,
unexplained wealth, after which the PCGG filed a complaint for marriage, equal protection of the law, freedom to contract, etc;
forfeiture under RA 1379 those rights appertaining to a person by virtue of his citizenship
in a state or community or the right of his being a member of
Elizabeth  Dimaano,   alleged  to   be   Ramas’   mistress,   was   also   later  
impleaded. The raiding team seized the items detailed in the society
seizure receipt together with other items not included in the
search warrant xxx “Political   rights”   are   said   to   refer   to   the   right   to   participate,  
directly or indirectly, in the establishment or administration of
The Sandiganbayan declared the properties confiscated xxx as government, the right to suffrage, the right to hold public
illegally seized and therefore inadmissible xxx office, etc
[The Republic] asserts that the revolutionary government
“Natural  rights”  are  those  rights  that  appertain  to  man  in  right  
effectively withheld the operation of the 1973 Constitution which
guaranteed  private  respondents’  exclusionary  right of his existence, i.e. the rights to freedom of thought, to
freedom of religious belief, etc
WON the revolutionary government was bound by the Bill of
Rights of the 1973 Constitution during the interregnum xxx; WON American Bill of Rights -- The First Ten Amendments
the protection accorded to the individuals under the xxx
(“Covenant”)   and   the   xxx (“Declaration”)   remained   in   effect   The Philippine Bill of Rights has been basically patterned after
during the interregnum the American Bill of Rights which is contained in the first ten
amendments to the United States Constitution
Held: No. The resulting government was indisputably a
revolutionary government bound by no constitution or legal

5|P LATON
Accordingly, in view of the Philippine reliance and reference National Blood Service Act. Section 7 of RA 7719 provides phase-
every now and then to American case law on certain out of Commercial Blood Banks. Petitioners assail the
constitutional issues relative to the Bill of Rights, parallel constitutionality of the said provision on the ground, among
others, that such represents undue delegation if not outright
citations to the pertinent provisions of the US Bill of Rights have
abdication of the police power of the state.
to be made occasionally.
ISSUE: WON RA 7719 is a valid exercise of police power
International Bill of Rights
HELD: Petitions dismissed. The court upholds the validity of RA
While the Bill of Rights as found in the Constitution is the 7719.
primary basis for the determination if there is any violation of
the rights of persons, it does not necessarily mean that such is RATIO: The promotion of public health is a fundamental obligation
of the State. The health of the people is a primordial
the sole source of rights that may be recognized
governmental concern. RA 7719 was enacted in the exercise of
the  State’s  police  power  in  order  to  promote and preserve public
The Court also considers the pertinent international health and safety.
conventions and declarations in trying to determine if there is a Police power of the state is validly exercised if (a) the interest of
violation  of  a  person’s  rights the public generally, as distinguished from those of a particular
class, requires the interference of the State; and (b) the means
The most prominent of these is the Universal Declaration of employed are reasonably necessary to the attainment of the
Human Rights objective sought to be accomplished and not unduly oppressive
upon individuals
The individual may still avail of the guarantees provided by the Police power is the State authority to enact legislation that may
international instruments and covenants--from the so-called interfere with personal liberty or property in order to promote
International Bill of Human Rights to some other declarations the general welfare.
and conventions--to buttress whatever claims to freedom and
liberty that he or she may have Thus, persons may be subject to certain kinds of restraints and
burdens in order to secure the general welfare of the State and to
The Bill of Rights, Vigilance and Government as Teacher its fundamental aim of government, the rights of the individual
may be subordinated
A close and literal construction deprives them of half their
A2. PHCAP v. Duque III1
efficacy, and leads to gradual depreciation of the right, as if it
535 SCRA 265 (2007)
consisted more in sound than in substance. It is the duty of the
courts to be watchful for the constitutional rights of the citizen, Health is a legitimate subject matter for regulation by the DOH
and against any stealthy encroachments thereon (and certain other administrative agencies) in exercise of police
powers delegated to it. The superiority of breastfeeding and
The lead should come from the Government itself if ever it correct information as to infant feeding and nutrition, as in this
wants the citizens to follow and abide by its commands and case,  is  infused  with  public  interest  and  welfare.  The  DOH’s  power  
demands under the Milk Code to control information regarding breastmilk
vis-à-vis breastmilk substitutes is not absolute as the power to
control does not encompass the power to absolutely prohibit the
Decency, security, and liberty alike demand that government advertising, marketing, and promotion of breastmilk substitutes.
officials shall be subjected to the same rules of conduct that are Implementing rules and regulations imposing labeling
commands to the citizens. In a government of laws, existence requirements and limitations, as well as a prohibition against
of the government will be imperiled if it fails to observe the law certain health and nutrition claims are inconsistent with the Milk
scrupulously. Our government is the potent, the omnipresent Code.
teacher
Nonetheless, the DOH, in imposing an absolute prohibition on
advertising, promotion, and marketing, the same went beyond its
If the government becomes a lawbreaker, it breeds contempt
authority since the same was not within the provisions of the Milk
for law; it invites every man to become a law unto himself; it Code itself.
invites anarchy
A3. Carlos Superdrug Corp. v. DSWD
The rights and guarantees may exist. But it takes some 526 SCRA 130 (2007)
vigilance, some action on the part of the people in order that
those guarantees may come to life and become part of the FACTS: Petitioners are domestic corporations and proprietors
human spirit operating drugstores in the Philippines. Petitioners assail the
constitutionality of Section 4(a) of RA 9257, otherwise known as
Additional Cases the   “Expanded   Senior   Citizens   Act   of   2003.”   Section   4(a)   of   RA  
9257 grants twenty percent (20%) discount as privileges for the
(A) The Bill of Rights and the Fundamental Powers
Senior Citizens. Petitioner contends that said law is
unconstitutional because it constitutes deprivation of private
A1. Beltran v. Secretary of Health property.
476 SCRA 168 (2005)
ISSUE: WON RA 9257 is unconstitutional
FACTS: Petitioners comprise the majority of the Board of Directors
of the Philippine Association of Blood Banks, a duly registered HELD: Petition is dismissed.
non-stock and non-profit association composed of free standing
blood banks. Public respondent Secretary of Health is being sued RATIO: The law is a legitimate exercise of police power which,
in his capacity as the public official directly involved and charged
with the enforcement and implementation of RA 7719 or the
1 UST Golden Notes 2010, Political Law
6|P LATON
similar to the power of eminent domain, has general welfare for
its object. HELD: Petition denied. EO 179 is null and void.

Accordingly,  it  has  been  described  as  “the  most  essential,  insistent   RATIO: MMDA has no police power, let alone legislative power. In
and the least limitable of powers, extending as it does to all the light of the administrative nature of its powers and functions, the
great  public  needs.”  It  is  the  power  vested  in  the  legislature  by  the   MMDA is devoid of authority to implement the Project as
constitution to make, ordain, and establish all manner of envisioned by the EO; hence it could not have been validly
wholesome and reasonable laws, statutes, and ordinances, either designated by the President to undertake the Project. It follows
with penalties or without, not repugnant to the constitution, as that the MMDA cannot validly order the elimination of the
they shall judge to be for the good and welfare of the respondents’  terminals
commonwealth,  and  of  the  subjects  of  the  same.”
Police power rests primarily with the legislature, such power may
For this reason, when the conditions so demand as determined by be delegated, as it is in fact increasingly being delegated. By virtue
the legislature, property rights must bow to the primacy of police of a valid delegation, the power may be exercised by the
power because property rights, though sheltered by due process, President and administrative boards as well as by the lawmaking
must yield to general welfare. bodies of municipal corporations or local government under an
express delegation by the LGC of 1991
A4. BANAT v. COMELEC2
586 SCRA 210 (2009) Measures calculated to promote the safety and convenience of
the people using the thoroughfares by the regulation of vehicular
The Court therefore strikes down the two percent threshold only traffic present a proper subject for the exercise of police power
in relation to the distribution of the additional seats as found in
the second clause of Section 11 (b) of RA 7941. The two percent On  Constitutional   Law,   “The   true   role   of   Constitutional   Law   is   to  
threshold presents an unwarranted obstacle to the full effect an equilibrium between authority and liberty so that rights
implementation of Section 5 (2), Article VI of the Constitution and are exercised within the framework of the law and the laws are
prevents  the  attainment  of  the  “broadest  possible  representation   enacted  with  due  deference  to  rights.”
of party, sectoral or group interests in the House of
Representatives” A7. Yamane v. BA Lepanto Condominium Corporation
474 SCRA 258 (2005)
A5. Mirasol v. DPWH
490 SCRA 318 (2006) FACTS: Petitioner City Treasurer of Makati holds respondent, in a
Notice of Assessment, liable to pay the correct business taxes,
FACTS: Petitioners sought the declaration of nullity of certain fees and charges totaling to P1.6M in which the respondents
administrative issuances of the DPWH for being inconsistent with protested contending that condominium does not fall under the
RA  2000,  entitled  “Limited  Access  Highway  Act.”  Among  others,  is   definition of a business, thus, they are not liable for such taxes
AO1 which requires motorcycles shall have an engine
displacement of at least 400cc. ISSUE: WON the City Treasurer of Makati may collect business
taxes on condominium corporations
ISSUE: WON said administrative issuances are unconstitutional
HELD: Petition denied. Accordingly, and with significant degree of
HELD: Petition partly granted. It is the DOTC, not the DPWH, comfort, we hold that condominium corporations are generally
which has authority to regulate, restrict, or prohibit access to exempt from local business taxation under the LGC, irrespective
limited access facilities. of any local ordinance that seeks to declare otherwise.

We find that AO1 does not impose unreasonable restrictions. It RATIO: The power of the local government units to impose taxes
merely outlines several precautionary measures, to which toll way within its territorial jurisdiction derives from the Constitution
users must adhere. These rules were designed to ensure public itself,   which   recognizes   the   power   of   these   units   “to   create   its  
safety and the uninhibited flow of traffic within limited access own sources of revenue and to levy taxes, fees, and charges
facilities. subject to such guidelines and limitations as the Congress may
RATIO: The use of public highways by motor vehicles is subject to provide,  consistent  with  the  basic  policy  of  local  autonomy.”
regulation as an exercise of the police power of the state. The
police power is far-reaching   in   scope   and   is   the   “most   essential,   A8. PPI v. Fertphil Corporation
insistent  and  illimitable”  of  all  government  powers.  The  tendency 548 SCRA 485 (2008)
is to extend rather than to restrict the use of police power. The
sole standard in measuring its exercise is reasonableness. FACTS: Petitioner and private respondent are private corporations
incorporated under Philippine laws. They are both engaged in the
A6. MMDA v. Viron Transportation Co., Inc. importation and distribution of fertilizers, pesticides and
530 SCRA 341 (2007) agricultural chemicals. President Marcos issued LOI 1465 which
provided, among others, for the imposition of a capital recovery
FACTS: PGMA issued EO 179, which provided for the component on the domestic sale of all grades of fertilizers in the
establishment of a Mass Transport System for Greater Manila. Philippines. Pursuant to the LOI, private respondent paid P10 for
Pursuant to this EO, the Metro manila Council of the MMDA cited every bag of fertilizer it sold in the domestic market to the
the need to remove the bus terminals located along major Fertilizer and Pesticide Authority (FPA). After the 1986 Edsa
thoroughfares of Metro Manila. Respondents, provincial bus Revolution, FPA voluntarily stopped the imposition of the P10
operators who had bus terminals that were threatened to be levy. Private respondent then demanded from petitioner a refund
removed, alleges that EO should be declared unconstitutional and of the amounts it paid under LOI 1465
illegal for transgressing the possessory rights of owners and
operators of public land transportation units over their respective ISSUE: WON the issuance of LOI 1465 is a valid exercise of police
terminals3 power of the State
ISSUE: WON EO 179 is a valid exercise of police power
HELD: Petition denied. The RTC and the CA did not err in ruling
against the constitutionality of the LOI
2 UST Golden Notes 2010, Political Law
3 Stef Macapagal

7|P LATON
RATIO: Police power and the power of taxation are inherent ordinary citizens against arbitrary government action, but not
powers of the State. These powers are distinct and have different from acts committed by private individuals or entities.
tests for validity. Police power is the power of the State to enact
legislation that may interfere with personal liberty or property in The right to due process guards against unwarranted
order to promote the general welfare, while the power of taxation encroachment by the state into the fundamental rights of its
is the power to levy taxes to be used for public purpose. The main citizens and cannot invoked in private controversies involving
purpose of police power is the regulation of a behavior or private parties.
conduct,   while   taxation   is   revenue   generation.   The   “lawful  
subjects”   and   “lawful   means”   tests   are   used   to   determine   the   The discipline of members by a political party does not involve the
validity of a law enacted under the police power. The power of right to life, liberty or property within the meaning of the due
taxation, on the other hand, is circumscribed by inherent and process clause.
constitutional limitations.
Chapter 2
An inherent limitation on the power of taxation is public purpose. Due Process
Taxes are exacted only for a public purpose. They cannot be used
for purely private purposes or for the exclusive benefit of private
persons. “No  person  shall  be  deprived  of  life,  liberty  or  property  without  
4
due  process  of  law.”
The power to tax exists for the general welfare; hence, implicit in
its power is the limitation that it should be used only for a public Due Process of Law
purpose.
Person
A9. Yrasuegui v. PAL
569 SCRA 467 (2008) Life, Liberty and Property
FACTS: Petitioner was a former international flight steward of
PAL, herein respondent. Petitioner was dismissed because of his
Life
failure to adhere to the weight standards of the airline company.
Petitioner claims that he was illegally dismissed. Liberty

ISSUE: WON petitioner was discriminated against when he was Roe v. Wade
dismissed. 410 US 113, 35 L Ed 2d 147, 93 S Ct 705 (1973)

HELD: Petition denied. Caunca v. Salazar


82 Phil. 851, 1 SCUD 177 (1 January 1949)
RATIO: To make his claim more believable, petitioner invokes the
equal protection clause guaranty of the Constitution. However, in
Lupangco v. CA
the absence of governmental interference, the liberties
160 SCRA 848 (1988)
guaranteed by the Constitution cannot be invoked. Put
differently, the Bill of Rights is not meant to be invoked against
acts of private individuals. Indeed, the US Supreme Court, in Property
interpreting the 14th Amendment, which is the source of our equal
protection guarantee, is consistent in saying that the equal Ayog v. Cusi, Jr
protection erects no shield against private conduct, however 118 SCRA 492 (1982)
discriminatory or wrongful. Private actions, no matter how
egregious, cannot violate the equal protection guarantee. Public Office

A10. Atienza, Jr. v. COMELEC Layno, Sr. v. Sandiganbayan


612 SCRA 761 (2010) 136 SCRA 536 (1985)

FACTS:   Drilon,   as   president   of   the   LP,   announced   his   party’s  


Licenses
withdrawal of support for the administration of PGMA. Petitioner,
LP chairman, and a number of party members denounced  Drilon’s  
move. In a party conference, petitioner moved to declare all Corona v. United Harbor Pilots Association of the Philippines
positions  in  the  LP’s  ruling  body  vacant  and  elected  new  officers,   283 SCRA 31 (1997)
with petitioner as LP president. Drilon filed a petition before the
COMELEC and the latter nullified the elections. Eventually, Roxas Right-Privilege Dichotomy
was installed as the new LP president. Petitioners were deemed
resigned for holding the illegal election of LP officers and were RIGHT PRIVILEGE
dropped from the roster of LP members. The former come under the They are just by way of grant
ISSUE:  WON  respondents  violated  petitioners’  constitutional  right
protection of the Due Process by the State
to  due  process  by  the  latter’s  expulsion  from  the  party. Clause

HELD: Petition denied. The requirements of administrative due The American Supreme Court now has rejected the concept
process do not apply to the internal affairs of political parties. that constitutional rights turn upon whether a governmental
benefit   is   characterized   as   a   “right   “   or   as   a   “privilege.”   .   .   .  
RATIO: The constitutional limitations on the exercise of the state’s   Whether any procedural protections are due depends on the
powers are found in Article III of the Constitution or the Bill of
extent   to   which   an   individual   will   be   “condemned to suffer
Rights. The Bill of Rights, which guarantees against the taking of
life, property, or liberty without due process under Section 1 is
grievous  loss”
generally   a   limitation   on   the   state’s   powers   in   relation to the
rights of its citizens. The right to due process is meant to protect 4 CONSTITUTION, Art. III, § 1
8|P LATON
Hierarchy of Rights It started off originally as simply a guarantee of procedural
fairness
The  Due  Process  Clause  protects  “life,  liberty  and  property”
‘Due   process   law’   was   originally   used   a   shorthand   expression  
PBMEO v. Philippine Blooming Mills Co., Inc. for  governmental  proceedings  according  to  the  ‘law  of  the  land’  
51 SCRA 189 (1973) as it existed at the time of those proceedings
PBMEO allegedly informed the respondent Company of the proposed
The guaranties of due process, though having their roots in
demonstration. A day before the planned demonstration, the Company
informed the workers that even as their right to demonstrate is
Magna  Carta’s  ‘per legem terrae’  and  considered  as  procedural  
recognized, the normal operations of the Company should not be safeguards  ‘against  executive  usurpation  and  tyranny,’  have  in  
unduly prejudiced this  country  ‘become  bulwarks  also  against  arbitrary  legislation’

Company filed with the COR a complaint for violation of the CBA, PROCEDURAL SUBSTANTIVE
particularly  the  “No  Strike-No  Lockout”  clause The method or manner by Requires that the law itself,
which the law is enforced not merely the procedure by
(2) The Bill of Rights is designed to preserve the ideals if liberty, equality
which the law would be
and   security   “against   assaults   of   opportunism,   the   expediency   of   the  
passing hour, the erosion of small encroachments, and the scorn and enforced, is fair, reasonable,
derision of those who have no patience with general principles and just
Basically addressed to those Primarily directed at the
Held: Yes. The respondent is the one guilty of unfair labor practices. who adjudicate lawmakers
Because the refusal on the part of the respondent firm to permit all its Revolves around the right to Implicates fundamental
employees and workers to join the mass demonstration against alleged be heard notions of fairness and justice
police abuses and the subsequent separation of the eight (8)
petitioners from the service constituted an unconstitutional restraint
Focuses on rules that are Concerns itself with the law,
on their freedom of expression, freedom of assembly and freedom of established in order to ensure its essence, and its
petition for redress of grievances, the respondent firm committed, meaningful adjudications concomitant efficacy
[among   others],   unfair   labor   practice   for   an   employer   “to   interfere   appurtenant thereto
with, retrain or coerce employees on the exercise of their rights
guaranteed  in  Section  Three” In other words, substantive due process looks to whether there
is  a  sufficient  justification  for  the  government’s  action
Property and property rights can be lost thru prescription; but
human rights are imprescriptible Procedural Due Process

In the hierarchy of civil liberties, the rights of free expression Procedural due process is basically associated with the right to
and of assembly occupy a preferred position as they are be notified and heard. This accordingly presupposes that one
essential to the preservation and vitality of our civil and has been adequately and meaningfully informed of a case or
political institutions;  and  such  priority  “gives  these  liberties  the   matter in which his rights are involved and that jurisdiction has
sanctity  and  the  sanction  not  permitting  dubious  intrusions.” been validly acquired over him

As heretofore stated, the primacy of human rights over ‘Jurisdiction’   is   the   right   to   hear   and   determine,   not   to  
property rights has been sustained determine without hearing

xxx habeas corpus is the remedy to obtain the release of an That due process is the equivalent of law of the land which
individual, who is convicted by final judgment through a forced means  ‘the  general  law;  a  law  which  hears  before  it  condemns,  
confession, which violated his constitutional right against self- which proceeds upon inquiry and renders judgment only after
incrimination; or who is denied the right to present evidence in trial xxx that every citizen shall hold his life, liberty, property,
his defense as a deprivation of his liberty without due process and immunities under the protection of the general rules which
of law, even after the accused has already served sentence for govern  society’
twenty-two years
Classic procedural due process issues are concerned with what
The liberties of any person are the liberties of all of us kind of notice and what form of hearing the government must
provide when it takes a particular action
“When  freedom  of  the  mind  is  imperilled  by  law,  it  is  freedom  
that commands a momentum of respect; when property is The fundamental requirement of due process is the opportunity
imperilled   it   is   the   lawmakers’   judgment   that   commands   to  be  heard  ‘at  a  meaningful  time  and  in  a  meaningful  manner’
respect. This dual standard may not precisely reverse the
presumption of constitutionality in civil liberties cases, but It is a well-established doctrine that rules of procedure may be
obviously it does set up a hierarchy of values within the due modified at any time to become effective at once, so long as
process  clause.” the change does not affect vested rights

Related to the hierarchy of rights are the so-called   “standards   Additional Cases
of  review”  discussed  earlier  in  Chapter  1 (B) Due Process [14]

The Two Faces or Components of Due Process -- Substantive and B1. Republic v. Cagandahan
Procedural 565 SCRA 72 (2008)

9|P LATON
People v. Cayat
68 Phil. 12 (1939)

The accused, Cayat, a native of Baguio, Benguet, Mountain Province,


and a member of the non-Christian tribes, was found guilty of violating
Act No. 1639 for having acquired and possessed one bottle of A-1-1 gin,
an intoxicating liquor, which is not a native wine

Cayat challenges the constitutionality of Act 1639 on the grounds that it


is discriminatory and denies equal protection of the laws xxx

HELD: No. Act 1639 satisfies all the requirements:


1.  The  ‘non-Christian  tribes’  refers,  not  to  religious  belief,  but  to natives
of the Philippine Islands of a low grade of civilization
2. It is designed to insure peace and order in and among the non-
Christian tribes xxx free use of highly intoxicating liquors by the non-
Christian tribes have often resulted in lawlessness and crimes, thereby
hampering the efforts of the government to raise their standard of life
and civilization
3. It is intended to apply at all times as long as those conditions exist
4. The Act applies equally to all members of the class

Ormoc Sugar Co., Inc. v. Treasurer of Ormoc City


22 SCRA 603 (1968)

The Municipal Board of Ormoc City passed Ordinance No. 4 imposing


“on  any  and  all  products  of  centrifugal  sugar  milled  at the Ormoc Sugar
Company Inc., in Ormoc City a municipal tax equivalent to 1% per
export sale to the United States of America and other foreign
countries”
Chapter 3
Equal Protection Petitioner alleged that the said ordinance is unconstitutional for being
violative of the equal protection clause xxx insofar as Ormoc Sugar Co.,
“No  person  shall  be  deprived  of  life,  liberty,  or  property  without   Inc. is singled out
due process of law, nor shall any person be denied the equal
5 HELD: Yes. A perusal of the requisites instantly shows that the
protection  of  the  laws.”
questioned ordinance does not meet them, for it taxes only centrifugal
sugar produced and exported by the Ormoc Sugar Company, Inc. and
The guarantee is against class legislation, or such legislation none other
which denies rights to one which are accorded to others, or The taxing ordinance should not be singular and exclusive as to exclude
inflicts upon one individual a more severe penalty than is any subsequently established sugar central, of the same class as
imposed upon another in like case offending plaintiff, from the coverage of the tax. As it is now, even if later a
similar company is set up, it cannot be subject to the tax because the
Equality of operation of statutes does not mean indiscriminate ordinance expressly points only to Ormoc Sugar Company, Inc. as the
entity to be levied upon
operation on persons merely as such, but on persons according
to the circumstances surrounding them. It guarantees equality,
JM Tuason & Co., Inc. v. Land Tenure Administration
not identity of rights 31 SCRA 413 (1970)

The equal protection of the law clause is against undue favor RA 2616 authorized the expropriation of the Tatalon Estate in Quezon
and individual or class privilege, as well as hostile discrimination City jointly owned by herein petitioner, Gregorio Araneta and Company,
or the oppression of inequality Inc. and Florencio Deudor

It does not demand equality among residents; it merely Petitioner sought to declare said RA unconstitutional as it is violative of
the equal protection clause since it applies only to the Tatalon Estate
requires that all persons shall be treated alike, under like
and not to any other lands in Quezon City or elsewhere
circumstances and conditions both as privileges conferred and
liabilities enforced HELD: No. The legislature is not required by the Constitution to adhere
to   the   policy   of   ‘all   or   none.’   Thus,   to   reiterate,   the   invocation  by   the  
Requisites for Valid Classification petitioner of equal protection clause is not attended with success

1. The classification is based on substantial distinctions which Gender


make real differences
2. It is germane to the purpose of the law The equality of the sexes is something that the Constitution
3. It applies not only to present conditions but also to future itself promotes. And this means basically having to treat
conditions which are substantially identical to those of the women on equal footing with men even as it still maintains a
present special solicitude for them
4. It applies equally to everyone or every member belonging to
the same class Bradwell v. Illinois
83 US (16 Wall) 130, 21 L Ed 442 (1873)

5 CONSTITUTION, Art. III, § 1 Mrs. Myra Bradwell, residing in the State of Illinois, applied with the
10 | P LATON
Supreme Court of that State for a license to practice law. Mrs. Stanley v. Illinois
Bradwell’s  application  for  a  license  was  refused,  and  it  was  stated  as  a   405 US 645, 31L Ed 2d 551, 92 S Ct 1208 (1972)
sufficient reason that under the decision of the Supreme Court of
Illinois, the applicant--“as  a  married  woman  would  be  bound  neither  by   Under the [challenged] scheme, the children of unmarried fathers,
her express contracts nor by those implied contracts which is the policy upon the death of the mother, are declared dependents without any
of  the  law  to  create  between  attorney  and  client.” hearing on parental fitness and without proof of neglect, though such
hearing and proof are required before the State assumes custody of
HELD: No. The right to control and regulate the granting of license to children of married or divorced parents and unmarried mothers
practice law in the courts of a State is one of the powers which are not
transferred for its protection to the Federal government, and its HELD:  Yes.  Stanley’s  claim  in  the  state  courts  and  here  us  that  failure  to  
exercise is in no manner governed or controlled by citizenship of the afford him a hearing on his parental qualifications while extending it to
United States in the party seeking such license other parents denied him equal protection of the laws. We have
concluded that all Illinois parents are constitutionally entitled to a
Michael M. v. Superior Court hearing on their fitness before their children are removed from their
450 US 464, 67 L Ed 2d 437, 101 S Ct 1200 (1981) custody. It follows that denying such a hearing to Stanley and those like
him while granting it to other Illinois parents is inescapably contrary to
Petitioner, then a 17-year-old male, was charged with violating the Equal Protection Clause
Clifornia’s   “statutory   rape”   law,   which   defines   unlawful sexual
intercourse  as  “an  act  of  sexual  intercourse  accomplished  with  a  female   Marriage and Legitimacy
not the wife of the perpetrator, where the female is under the age of
18  years.” GSIS v. Montesclaros
434 SCRA 441 (2004)
Petitioner   contends   that   the   law   “discriminates   on   the   basis   of   sex,  
because only females may be victims and only males may violate the Nicolas Montesclaros, 72-year old widower, married Milagros Orbiso,
section” then   43   years   old.   GSIS   approved   Nicolas’   application   for   retirement  
granting a lump sum payment of annuity for the first five years and a
HELD: No. The justification for the statute offered by the State and monthly annuity thereafter. Nicolas died. Milagros filed with the GSIS a
accepted by the Supreme Court of California, is that the legislature claim of survivorship pension. GSIS denied the claim because under
sought to prevent illegitimate teenage pregnancies Section 18 of PD 1146, the surviving spouse has no right to survivorship
pension if the surviving spouse contracted the marriage with the
Geosaert v. Cleary pensioner within three years before the pensioner qualified for the
335 US 464, 93 L Ed 163, 69 S Ct 198 (1948) pension

A Michigan law forbade any female to act as a bartender unless she be HELD: Yes. We hold that the proviso is unconstitutional because it
“the   wife   or   daughter   of   the   male   owner”   of   a   licensed   liquor   violates the due process clause. The proviso is also discriminatory and
establishment denies equal protection of the law. The proviso in question does not
satisfy these requirements. The object of the prohibition is vague.
HELD: While Michigan may deny to all women opportunities for There is no reasonable connection between the means employed and
bartending, Michigan cannot play favourites among women without the  purpose  intended  (“deathbed  marriages”)
rhyme or reasons.
Labine v. Vincent
Philippine Association of Service Exporters, Inc. v. Drilon 401 US 532, 28 L Ed 2d 288, 91 S Ct 1971 (1971)
163 SCRA 386 (1988)
Ezra Vincent died intestate, survive only by collateral relations and an
Petitioner,   PASEI,   a   firm   “engaged   principally   in   the   recruitment   of   illegitimate minor daughter, whose guardian sued to have her declared
Filipino workers, male  and  female,  for  overseas  placement,”  challenges   Vincent’s  sole  heir
the constitutional validity of Department Order No. 1 of the DOLE
“Guidelines   Governing   the   Temporary   Suspension   of   Deployment   of   Appellant   contends  that   Louisiana’s   intestate   succession   laws  that   bar  
Filipino   Domestic   and   Household   Workers”   for   “discrimination   against   an illegitimate child from sharing equally with legitimate children in the
males  or  females”  and  for  “not  applying  to  all  Filipino  workers  but  only   father’s   estate   constitutes   an   invidious   discrimination   violative   of   the  
to  domestic  helpers  and  females  with  similar  skills” Due Process and Equal Protection Clauses of the Constitution

HELD: No. As a matter of judicial notice, the Court is well aware of the HELD: No. To further strengthen and preserve family ties, Louisiana
unhappy plight that has befallen our female labor force abroad, regulates the disposition of property upon the death of a family man
especially domestic servants, amid exploitative working conditions
marked by, in not a few cases, physical and personal abuse Age

Tuan Anh Nguyen v. Immigration and Naturalization Service A classification based on age could very well be justified
522 US 53 150 L Ed 2d, 121 S Ct 2053 (2001)
depending on particular subject matter which may require a
Tuan Anh Nguyen was born out of wedlock in Vietnem to a Vietnamese
sense of maturity and responsibility, or vigor, health or strength
mother and an American Father.
Dumlao v. COMELEC
Petitioner claims that § 1409--which imposes a set of requirements on 95 SCRA 392 (1980)
the children of citizen fathers born abroad and out of wedlock to a non-
citizen mother that are not imposed under like circumstances when the Section4   of   BP   52   disqualifies   “any   retired   elective   provincial,   city   or  
citizen parent is the mother--violates equal protection municipal official who has received payment of the retirement benefits
to which he is entitled under the law and who shall have been 65 years
HELD:  No.  The  challenged  classification  serves  ‘important  governmental   of age at the commencement of the term of office to which he seeks to
objectives   and   that   the   discriminatory   means   employed’   are   be  elected”  to  run  for  the  same  elective  local  office  from  which  he  has  
‘substantially  related  to  the  achievement  of  those  objectives’ retired

11 | P LATON
Petitioner contends that it is discriminatory and contrary to the equal disputed law was enacted to remedy a real actual threat and danger to
protection and due process guarantees of the Constitution national economy posed by alien domination and control of the retail
business and free citizens and country from such dominance and
HELD: No. The purpose of the law is to allow the emergence of younger control
blood in local governments. It is within the competence of the
legislature to prescribe qualifications for one who desires to become a Office and Employment
candidate for office provided they are reasonable, as in this case
One’s  office,  occupation  or  employment  may also provide basis
Method or Mode of Dying for some differences in treatment, such as whether it is in the
public or private sector
Vacco v. Quill
521 US 793, 138 L Ed 2d 834, 117 S Ct 2293 (1997)
It has also been held that elective officials may be treated
In New York, as in most States, it is a crime to aid another to commit or differently from appointive officials with respect to the effect of
attempt suicide, but patients may refuse even lifesaving medical the filing of their certificates of candidacy, i.e., while the latter
treatment. Respondents are physicians who practice in New York may be deemed resigned, the former may still hold on to their
positions
Respondents’   claim   that   the   distinction   between   refusing   lifesaving  
medical  treatment  and  assisted  suicide  is  “arbitrary”  and  “irrational” Nuñez v. Sandiganbayan
111 SCRA 433 (1982)
HELD: No. The overwhelming majority of the state legislatures have
drawn a clear line between assisting suicide and withdrawing or Petitioner, accused before the Sandiganbayan of Estafa through
permitting the refusal of unwanted lifesaving medical treatment by Falsification of Public and Commercial Documents, assails the validity of
prohibiting the former and permitting the latter. PD 1486, as amended by PD 1606, which created the Sandiganbayan

Academic Performance The Snadiganbayan proceedings   violates   petitioner’s   right   to   equal  


protection, because--appeal as a matter of right became minimized into
Academic ratings could very well determine how a student gets a mere matter of discretion;--appeal likewise was shrunk and limited
classified or accepted only to questions of law, excluding a review of the facts and trial
evidence; and there is only one chance to appeal conviction, by
certiorari to the Supreme Court, instead of the traditional two chances;
Tablarin v. Gutierrez while other estafa indictees are entitled to appeal as a matter of right
152 SCRA 730 (1987) covering both law and facts and to two appellate courts
The petitioners sought admission into colleges or schools of medicine HELD: No. The general guarantees of the Bill of Rights, included among
for the school year 1987-1988. However, petitioners either did not take which are the due process of law and equal protection clauses must
or did not successfully take the National Medical Admission Test “give  way  to  a  specific  provision”
(NMAT) required by the Board of Medical Education which is
administered by private respondent Center for Educational
Measurement (CEM) International School Alliance of Educators v. Quisumbing
333 SCRA 13 (2000)
Petitioners alleged that it is violative of the Equal Protection Clause by
reason of the yearly changes in the cut-off scores for successful The School hires both foreign and local teachers as members of its
applicants faculty, classifying the same into two: (1) foreign-hires and (2) local-
hires. Foreign-hires are then granted certain benefits not accorded
HELD: No. We conclude that prescribing the NMAT and requiring local-hires.
certain minimum scores therein as a condition for admission to medical
schools in the Philippines do not constitute an unconstitutional Petitioner contested the difference in salary rates between foreign and
imposition local-hires

HELD: No. There is no reasonable distinction between the services


Nationality and Alienage rendered by foreign-hires and local-hires. The practice of the School of
according higher salaries to foreign-hires contravenes public policy and,
The Constitution itself has made classifications based on certainly, does not deserve the sympathy of this Court
citizenship, such a in the political field
Crimes and Punishments
Ichong v. Hernandez
101 Phil. 1155 (1957) Different offenses could not possibly be meted the same
penalty but crimes of the same nature should be treated no
RA  No.  1180  “An  Act  to  Regulate  the  Retail  Business”  prohibits  persons,  
not citizens of the Philippines, and associations, partnerships, or
differently
corporations the capital of which are not wholly owned by citizens of
the Philippines, from engaging directly or indirectly in the retail trade Skinner v. Oklahoma
unless such aliens have actually been engaged in said business on 15 316 US 535, 86 L Ed 1655, 62 S Ct 1110 (1942)
May 1954
Oklahoma’s   Habitual   Criminal   Sterilization   Act   provides   for   the  
Petitioner attacks the constitutionality of the Act contending that it sterilization,   by   vasectomy   or   salpingectomy,   of   “habitual   criminals.”  
denies to alien residents the equal protection of the laws and deprives Petitioner, convicted of the crimes of stealing chickens, and robbery
them of their liberty and property without due process of law with firearms twice, challenged the constitutionality of the Act but the
Oklahoma Supreme Court sustained the Act
HELD: No. The difference in status between citizens and aliens
constitutes a basis for reasonable classification in the exercise of police HELD: The equal protection clause does not prevent the legislature
power. Resuming what we have set forth above we hold that the from  recognizing  “degrees  of  evil”

12 | P LATON
People v. Ching Kuan In the case at bar, the challenged proviso operates on the basis of the
74 Phil. 23 (1942) salary grade or officer-employee status. It is akin to a distinction based
on economic class and status, with the higher grades as recipients of a
Ching Kuan was accused of violating section 86 of the Revised benefit specifically withheld from the lower grades
Ordinances of the City of Manila in that he constructed a 297-square-
meter building of strong materials in the district of Tondo without the Inverse Equal Protection
proper permit from the city engineer
“All   persons   or   things   differently   situated   should   be   treated  
Petitioner alleged the unconstitutionality of Article 66 of the revised
differently”
Penal Code which permits the court to take into consideration the
wealth and means of the culprit in the imposition of fines
Philippine Judges Association v. Prado
HELD: No. It may seem paradoxical, but the truth is that the codal 227 SCRA 703 (1993)
provision in question, in authorizing the imposition of unequal fines,
aims precisely at equality before the law The petitioners, members of the lower courts who feel that their official
functions as judges will be prejudiced by the withdrawal of the franking
privilege of the Judiciary [while retaining the same for the President of
People v. Jalosjos
the Philippines, et. al.], assail the constitutionality of Section 35 of RA
324 SCRA 689 (2000)
7354 as implemented by the Philippine Postal Commission [as
discriminatory]
the accused-appellant, Romeo F. Jalosjos is a member of Congress,
confined at the national penitentiary while his conviction for statutory
HELD: Yes. The respondents are in effect saying that the franking
rape on two counts and acts of lasciviousness on six counts is pending
privilege should be extended to only those who do not need it very
appeal
much, if at all, but not to those who need it badly.
Petitioner filed a motion asking that he be allowed to fully discharge
the duties of a Congressman, including attendance at legislative Equalizing Illegality
sessions and committee meetings despite his having been convicted in
the first instance of a non-bailable offense. This he primarily anchors on What is illegal is plainly against the law and the fact that others
the  “mandate  of  sovereign  will” were able to get away with it is no justification to provide equal
chances for others under the Equal Protection Clause
HELD: No. Allowing the accused-appellant to attend congressional
meeting for five (5) days or more in a week will virtually make him a
Alunan III v. Mirasol
free man with all the privilege appurtenant to his position
276 SCRA 501 (1997)

People v. Vera The DILG through then Secretary Rafael M. Alunan III, issued a letter-
65 Phil. 56 (1937) resolution exempting the City of Manila from holding election for the
SK on the ground that the election previously held on 26 May 1990 was
Mariano Cu Unjieng applied for probation. Respondent Judge denied to be considered he first under the newly enacted LGC
the application for probation Private respondents claimed equal protection clause violation

HELD: Yes. In the case at bar, the resultant inequality may be said to HELD: No. Even assuming that only barangays in Manila were not
flow from the unwarranted delegation of legislative power, although permitted to hold SK elections on December 4, 1992 while the rest of
perhaps this is not necessarily the result in every case the 5,000 barangays were allowed even if KB elections had already
been held there before, this fact does not give the youth voters in the
One province may appropriate the necessary fund to defray the salary 897 Manila barangays ground for complaint because what the other
of a probation officer, while another province may refuse or fail to do barangays did was contrary to law. There is no discrimination here
so. In such a case, the Probation Act would be in operation in the
former province but not in the latter. this means that a person
Delegated Discrimination
otherwise coming within the purview of the law would be liable to
enjoy the benefits of probation in one province while another person
similarly situated in another province would be denied those same A violation of the guarantee of equal protection may be seen
benefits on the face of the law itself, or perceived and felt in the manner
in which what pretends to be a just and fair regulation is
This basic individual right sheltered by the Constitution is a actually utilized as a tool to camouflage a discriminatory act
restraint on all three grand departments of our government
and in the subordinate instrumentalities and subdivisions Affirmative Action--A Case of Compensatory Discrimination
thereof, and on many constitutional powers, like the police
power, taxation and eminent domain Affirmative action is a sort of reverse discrimination in the
sense that minorities have been discriminated against in the
Central Bank Employees Association, Inc v. BSP past, are given preference in certain areas, like employment
446 SCRA 299 (2004) and layoff, as well as admission to educational institutions

The   thrust   of   the   petitioner’s   challenge   is   that   the   proviso (on Classifications necessarily mean making distinctions. And
exemption from the coverage of the Salary Standardization Law) makes
grouping people or things always raises the issue as to whether
an unconstitutional cut between two classes of employees in the BSP
there is too much or there is too little in it
HELD: Yes. It is clear in the legislative deliberations that the exemption
of officers (SG 20 and above) from the SSL was intended to address the There should be no hesitation in using the equal protection
BSP’s  lack  of  competitiveness  in  terms  of  attracting  competent  officers clause as a major cutting edge to eliminate every conceivable
and executives. It was not intended to discriminate against the rank- irrational discrimination in our society
and-file.
13 | P LATON
Additional Cases HELD: No. The equal protection clause is not violated, because there is
(C) Equal Protection a substantial basis for a different treatment of a member of a foreign
military armed forces allowed to enter our territory and all other
accused
C1. Serrano v. Gallant Maritime Services, Inc.
582 SCRA 254 (2009)
The rule in international law is that a foreign armed forces allowed to
th enter   one’s   territory   is   immune   from   local   jurisdiction,   except   to   the  
Antonio Serrano, a Filipino seafarer, claims that the last clause in the 5
extent agreed upon
paragraph of Section 10, RA 8042 deprives them of equal protection
and denies them due process, for it treats OFWs differently from local
Filipino workers Chapter 4
Searches and Seizures
Petitioner was hired by Gallant Maritime Services, Inc. and Marlow
Navigation Co., Ltd. under a POEA approved Contract of Employment. “The  right  of  the  people  to  be  secure  in  their  persons,  houses,  
He as constrained to accept a downgraded employment contract for papers, and effects against unreasonable searches and seizures
the position of Second Officer with a monthly salary of $1,000.00, upon of whatever nature and for any purpose shall be inviolable, and
the assurance and representation of respondents that he would be
no search warrant or warrant of arrest shall issue except upon
made Chief Officer by the end of April 1993. Respondents did not
deliver. Hence, petitioner refused to stay on as Second Officer and was probable cause to be determined personally by the judge after
repatriated to the Philippines on May 26, 1998. examination under oath or affirmation of the complainant and
the witnesses he may produce, and particularly describing the
6
Petitioner filed with the Labor Arbiter a Complaint against respondents place  to  be  searched  and  the  persons  or  things  to  be  seized.”
for constructive dismissal and for payment of his money claims in the
total amount of $26,442.73. The dismissal was declared illegal; and This constitutional guarantee is a liberty safeguard that
awarding him monetary benefits amounting to $8,770.00 based on the prohibits and directs. While laying down the general rule
salary period of three months only (in accordance with the 5th
against unreasonable searches and seizures, it also provides the
paragraph of Section 10, RA 8042)
guidelines for acceptable conduct on the part of government
HELD: Yes. Section 18, Article II and Section 3, Article XIII accord all authorities in regard to any intrusion into or invasion of the
members of the labor sector, without distinction as to place of people’s   right   “to   be   secure   in   their   persons, houses, papers,
deployment, full protection of their rights and welfare xxx All monetary and  effects”
benefits should be equally enjoyed by workers of similar category,
while all monetary obligations should be borne by them in equal It has also been held that the warrant is not meant to authorize
degree; none should be denied the protection of the laws which is fishing expeditions
enjoyed by, or spared the burden imposed on, others in like
circumstances
The finding of evidence cannot be the immediate reason for
The subject clause has a discriminatory intent against, and an invidious issuing a search warrant. To use a search warrant for this
impact on, OFWs purpose would be  ‘unreasonable’  use  of  the  remedy  by  search  
warrant, which is prohibited by law
C2. Santos v. People
563 SCRA 341 (2008) It  assures  one’s  entitlement  to  privacy  and  right  to  be  left  alone  
and do whatever he wishes within bounds without having to be
Criminal prosecution was recommended against MS. JUDY ANNE subjected to the prying eyes of Big Brother
LUMAGUI SANTOS for substantial underdeclaration of income, which
constitutes as prima facie evidence of false or fraudulent return
The  embodiment  of  a  “spiritual  concept:  the  belief  that  to  value  
The information is void ab initio, being violative of due process and the the privacy of home and person and to afford its constitutional
equal protection of the laws protection against the long reach of government in no less than
to value human dignity, and that his privacy must not be
HELD: No. Petitioner cannot aver that she has been denied equal disturbed except in case of overriding social need, and then
protection of the laws. Recognizing the existence of real differences only  under  stringent  procedural  safeguards”
among men, the equal protection does not demand absolute equality.
It merely requires that all persons shall be treated alike, under like
The privacy of the home has always been regarded by civilized
circumstances and conditions, both as to the privileges conferred and
liabilities enforced
nations as one of the most sacred personal rights to which men
are entitled
The  only  basis  for  petitioner’s  claim  of  denial  of  equal  protection  of  the   “A  man’s  home  is  his  castle,”  has  become  a  maxim  among  the  
laws was the dismissal of the charges against Velasquez while those civilized people of the earth
against her were not
The right extends to all persons, including aliens. So are
C3. Nicolas v. Romulo corporations also included within its protection, though to a
578 SCRA 438 (2009) lesser extent
Lance Corporal (L/CPL) Daniel Smith is a member of the United States
Requirements for Issuance of Warrants
Armed Forces. He was charged with the crime of rape committed
against a Filipina, petitioner herein, sometime on November 1, 2005
1. There should be a search warrant or warrant of arrest
Petitioners argue that to allow the transfer of custody of an accused to 2. Probable cause supported the issuance of such warrant
a foreign power is to provide for a different rule of procedure for that
accused, which also violates the equal protection clause of the
Constitution
6 CONSTITUTION, Art. III, § 2
14 | P LATON
3. Such probable cause had been determined personally by a opposition candidate. This should have put the judge on guard as to the
judge motivations of the witnesses and alerted him of possible
4. The judge personally examined the complaint and his misrepresentations from them. One may wonder why it did not occur
to the respondent judge to ask how the witnesses would be so certain
witnesses
even to the caliber of the guns, or how far he was from the window or
5. The place to be searched and the persons or to be seized whether it was on the first or second floor, or why his presence was not
have been particularly described noticed at all, or if the acts related were really done openly, in the full
view of the witnesses, considering that these acts were against the law.
The Constitution requires that [the judge] carefully determine These would have been judicious questions that were omitted and
the basis for the issuance of such warrant in order that the instead, the declarations of the witnesses were readily accepted and
protection declared be not set at naught the search warrant sought was issued forthwith. The above discussed
defects have rendered the search warrant invalid. Nonetheless, the
Solicitor general argues that whatever defect there was, was waived
The test of sufficiency of an affidavit or testimony to justify the
when the petitioner voluntarily submitted to the search and manifested
issuance of a warrant is whether the affiant or the witness could his conformity in writing. We do not agree because what we see here is
be held for perjury and made liable for damages in case his pressure exerted by the military authorities, who coerced the petitioner
statements are false to sign the supposed waiver was a guaranty against a possible challenge
later to the validity of the search they were conducting. Confronted
Nature of Search Warrants Proceedings with the armed presence of the military and the presumptive authority
of the writ, the petitioner had no choice but to submit. The
A search warrant proceeding is, in no sense, a criminal action or respondents argued that the possession of colt magnum pistols and 18
live bullets are illegal per se, being malum prohibitum, hence it could be
the commencement of a prosecution. The proceeding is not
taken even without a warrant. The SC held that it does not follow that
one against any person, but is solely for the discovery and to because an offense is malum prohibitum, the subject thereof is
get possession of personal property. It is a special and peculiar necessarily illegal per se. Motive is immaterial in mala prohibita but the
remedy, drastic in nature, and made necessary because of subjects of this kind of offense may not be summarily seized because
public necessity they are prohibited. A search warrant is still necessary. WHEREFORE,
the search warrant issued by the respondent judge is declared null and
Determination of Probable Cause void.

This responsibility of determining such cause has been vested Bache and Co (Phil.) v.Ruiz
solely in judges, a duty which cannot be delegated to anybody 37 SCRA 823 (1971)
else
FACTS: The Commissioner of Internal Revenue wrote the respondent
judge requesting the issuance of the search warrant against petitioners
If the determination is done by one directly interested in law for violating Sec. 46 (a) of the National Internal Revenue Code. The
enforcement the process would very likely be tainted and following day, National Revenue examiner De Leon and his witness,
clouded by his need to produce result in line with his job, and, Logronio, went to the court together with documents i.e. application
in the process, obscure some other considerations which would for search warrant, affidavits, and other documents already filled out
otherwise be seen and appreciated by one not similarly but unsigned. At that time, Judge Ruiz was hearing a certain case and so
circumstanced he simply instructed the deputy clerk of court to take their depositions.
After the session adjourned, the judge asked for the stenographer to
read to him the stenographic notes and without propounding any
Roan v. Gonzales
questions, asked Logronio to take the oath and warned him that is his
145 SCRA 687 (1986)
deposition was found to be false, he would be charged with perjury.
The search warrant was then signed and BIR agents served the same on
FACTS: Petitioner challenged the admission of one Colt Magnum
the   petitioners’   office.   Petitioner’s   lawyers   protested   on   the   ground  
revolver and 18 live bullets which were found after a search in his
that no formal complaint or transcript of testimony was attached to the
house pursuant to a search warrant issued by the respondent judge,
warrant. Nevertheless, the agents proceeded with the search which
which warrant he also assails. None of the articles listed in the warrant
yielded 6 boxes of documents. Hence, this petition for certiorari,
was discovered during the search. However, the officers conducting the
prohibition and mandamus to declare the search warrant void.
search found in the premises the revolver and the bullets which they
confiscated. They are now the bases of the charge against the
HELD: The petition should be granted because the respondent judge
petitioner.
failed to personally examine the complainant and his witness. As
provided in the Constitution and the Rules of Court, the examination of
HELD: To be valid, the search warrant must be supported by probable
the complainant and witness he may produce shall be conducted by the
cause to be determined by the judge or some other authorized office
judge himself and not by others. The participation of the judge in the
after examining the complainant and the witnesses he may produce.
proceedings which led the to the issuance of the search warrant was
No less important, there must be a specific description of the place to
thus  limited   to   listening   to   the   stenographer’s  reading   her   notes,   to   a  
be searched and the things to be searched to prevent arbitrary and
few words of warning against the commission of perjury and to
indiscriminate use of the warrant. Probable cause, in the case of Burgos
administering the oath to the complainant and his witness. This cannot
V.   Chief   of   Staff   refers   to   “such   facts   and   circumstances   which would
be considered a personal examination. The reading of the stenographic
lead a reasonably discreet and prudent man to believe that an offense
notes to respondent judge did not constitute sufficient compliance with
has been committed and the objects sought in connection with the
the constitutional mandate and the rule for by that manner respondent
offense are in the place sought to be searched. As held in a long line of
judge did not have the opportunity to observe the demeanor of the
decision, the probable cause must refer to only one specific offense.
complainant and his witness, and to propound initial and follow up
The petitioner has claimed that no deposition was taken by the
questions which the judicial mind on account of its training, was in the
respondent judge , however, this is not entirely true because
best position to conceive. These were important in arriving at a sound
depositions were taken of the complainants 2 witnesses in addition to
inference on the all important question of whether or not there was
the affidavit executed by them. However, there is still the question of
probable cause. Petition is granted, search warrant is declared null and
the sufficiency of their deposition. A study of one of the depositions of
void.
the witnesses, who claimed that their intelligence informers show that
they were suspicious of the petitioner because he was a follower of the

15 | P LATON
Particularity of Description and General Warrants namely 1) that no warrant shall issue but upon probable cause, to be
determined by the judge in the manner set forth in said provision and
Notwithstanding the authority granted by a warrant, resulting 2) that the warrant shall particularly describe the things to be seized.
None of these requirements has been complied with in the contested
search or seizure may still be violative of the guarantee if such
warrants. Relying on the Moncado doctrine, the respondents
instrument is in the nature of a general warrant, or one which petitioners maintain that even if the search and seizures were
practically authorizes a blanket search or seizure, a roving unconstitutional, the documents, papers and things thus seized are
authority of officers serving it to look everywhere and get admissible in evidence against petitioners. We are unanimously in the
anything that may be associated with what is being sought, opinion that this position be abandoned. Common law jurisdictions
“authorizing  searches  in  any  place  for  anything have already given up in this approach and eventually adopt the
exclusionary rule realizing that this is the only practical means of
The general warrants, whereby any officer or messenger may enforcing the constitutional injunction against unreasonable searches
and seizures.
be commenced to search suspected places without evidence of
a fact committed, or to seize any person or persons not named,
Central Bank v. Morfe
or whose offense is not particularly described and supported by
20 SCRA 507 (1967)
evidence, are grievous and oppressive, and ought not to be
granted FACTS: Respondent is a non stock corporation, the main purpose of
which is to encourage and implement savings and thrift among its
To obviate and prevent an avenue for abuse and arbitrary members and to extend financial assistance in the form of loans. CB
exercise of power, the Constitution specifically requires then published an announcement that such associations and others
particularity of description of things or people to be searched similar in nature have never been authorized to engage in the banking
and seized business. Moreover, CB directed the investigation and gathering of
evidences on the activities of such associations which are operating
contrary to law. Thereafter, a member of the intelligence division of CB
The warrant may be said to particularly describe the things to filed a verified application for a search warrant alleging that after close
be seized when the description therein is as specific as the observation and personal investigation, the organization is illegally
circumstances will ordinarily allow, or when the description engaged in banking activities. A search warrant was issued. Petitioner
expresses conclusion of fact--not of law--by which the warrant filed an action to annul the said search warrant on the grounds that it is
officer may be guided in making the search and seizure, or a  roving  commission,  general  in  its  terms  and  the  use  of  the  word    “and  
when the things described are limited to those which bear others”   in   the   search   warrant   permits   the   unreasonable search and
direct relation to the offense for which the warrant if being seizure of documents which have no relation whatsoever to any specific
criminal act.
issued
HELD: It is to noted that the action taken by CB is predicated on the
Stonehill v Diokno theory that the organization is illegally engaged in banking. The
20 SCRA 383 (1967) respondent judge found the searches and seizure in question to be
unreasonable because the depositions given in support of the search
FACTS; Upon application of the respondents-prosecutors, several warrant states that the deponent personally knows threat the premises
judges issued on several dates, a total of 42 search warrants against the of the organization were being used unlawfully for banking purposes.
petitioners and/or corporations of which they are officers, to search the He deduced that the deponent know specific baking transaction of the
persons above named and/or premises of their offices, warehouses petitioner with specific persons and then concluded that said deponent
and/or residences, and to seize and take possession of certain personal could have if he really knew of actual violation of the law applied for a
properties i.e. books of accounts, ledgers, financial records, vouchers , search want to search and seize only books. To authorize and seize all
correspondences and other documents and/or papers showing all the records without referred to specific alleged victims of the
business transactions as the subjects of the offense. Petitioners alleged purported illegal banking transactions would be to harass the
that said warrants are null and void because, inter alia, they do not petitioners.
describe with particularity the documents, books and things to be
seized. Hence, this petition for certiorari, prohibition and mandamus. The aforementioned order would seem to assume that an illegal
banking transaction of the kind contemplated in the contested action of
HELD: The documents, papers and things under the alleged authority of the  offices  of  the  bank,  must  always  connote  the  existence  of  a  “victim”  
the warrants in question may be split into 2 major groups, namely a) If this term is used to denote a party whose interests have been
those found and seized in the offices of the aforementioned actually injured, the assumption is not necessarily justified. The law
corporations and b) those found seized in the residences of petitioners requiring compliance with certain requirements before anybody can
herein. As regards the first group, we hold that the petitioners have no engage in banking obviously seeks to protect the public against actual,
cause of action to assail the legality of the contest warrants and of the as well as potential, injury.
seizures made for the simple reason that said corporations have their
respective personalities, separate and distinct from the personality of It is true that such funds referred to are their savings and the
petitioners, It is well settled that the legality of the seizure can be depositors thereof are members, but even a cursory examination of
contested only by the party whose rights have been impaired and that said documents will readily show that anybody can be a depositors and
the objection to the unlawful search is purely personal and cannot be thus be a participating member. Hence, it is open to the public.
availed of by third parties. With respect to the documents, papers and Moreover, the power to so dispose of the funds is placed exclusive
things seized in the residences of the petitioners, the respondent authority of the founding members and participating members are
prosecutors are restrained from using them in evidence against the expressly denied the right to vote or be voted for. This situation is
petitioners. In connection with the documents, papers and things , 2 fraught, precisely with the very dangers or evils which RA 337 seeks to
important questions need to be settled 1) whether the search warrants forestall, by exacting compliance with the requirements of said act,
in questions and the search and seizures made under the authority before the transaction in question could be undertaken. WHEREFORE,
thereof are valid or not and 2) if the answer to the preceding questions the order is hereby annulled.
is in the negative, whether said documents, papers and things may be
used in evidence against petitioners herein. Petitioners maintain that
the search warrants are in the nature of general warrants an that A search warrant may be partially void, such as when it
accordingly the seizures effected are null and void. We agree. 2 points provided for the search and seizure of items which have not
must be stressed in connection with this constitutional mandate been testified to during the application for such warrant

16 | P LATON
159 SCRA 599 (1988)
The constitutional requirement of reasonable particularity of
the things to be seized is primarily meant to enable the law FACTS: A shooting incident occurred in Lanao del Sur which left at least
five persons dead and two others wounded. In the course of events, a
enforcers serving the warrant to: (1) readily identify the
warrant of arrest was issued against 64 persons: 14 (the petitioners in
properties to e seized and this prevent them from seizing the this case) who were identified by three witnesses, and 50 John Does.
wrong items; and (2) leave said peace officers with no
discretion regarding the articles to be seized and thus prevent ISSUE:   WON   the   warrant   of   arrest   is   valid;   i.e.   “Can   a   court   issue   a  
unreasonable searches and seizures warrant  of  arrest  against  an  unknown  accused?”  

The rule is that a description of a place to be searched is HELD: NO, it is not valid as far as the 50 John Does are concerned (as for
sufficient if the officer with the warrant can, with reasonable the 14 petitioners in this case, it’s   an   entirely   different   story   because  
their names and identities are already known; the warrant of arrest is
effort, ascertain and identify the place intended and distinguish
valid as regards to them). The warrant in question is of the nature of a
it from other places in the community general warrant, one of a class of writs long prescribed as
unconstitutional   and  once   anathematized  as   “totally   subversive   of   the  
“John  Doe”  Warrants liberty   of   he   subject.”   Clearly   violative   of   the   constitutional   injunction  
that warrants of arrest should particularly describe the person or
The warrant should, as a rule, identify the person whose place persons to be seized, the warrant as against unidentified subjects will
is to be searched, or the individual to be arrested. It may be considered as null and void.
happen, however, that the name could not be readily
ascertained, in which event some description sufficient to Service Warrants - Time, Place and Manner
identify the person may be resorted. If the description contains
enough details so that the officer serving the warrant would The  time  must  not  be  one  which  is  intrusive  or  violative  of  one’s  
have no difficulty identifying the person, then the warrant privacy, like at the middle of the night
would be considered valid
As for place, the warrant limits the area that may be searched
People v. Veloso and the warrant could not be utilized as authority to search
48 Phil. 169 (1925) everywhere until the item or articles sought for are found

FACTS: Manila police, armed with a search warrant, raided the The manner of service should not also be oppressive or
Parliamentary Club, an establishment managed by Jose Ma. Veloso, a otherwise abusive
member of the House of Representatives, acting on reliable
information that said club was actually a gambling house. They
apprehended nearly 50 people, including Veloso. Veloso asked what It could be served out at any time within its ten-day lifetime,
they wanted and was shown the search warrant; he read it and told the and if its object or purpose cannot be accomplished in one day,
police that he was Representative Veloso and not John Doe (since the the same may be continued the following day or days until
name of Veloso did not appear in the search warrant but instead the completed
pseudonym John Doe was used), and that the police had no right to
search the establishment. Being armed with a warrant does not give leeway to a law enforcement
officer in doing whatever he wants without regard to the time, place,
The police noticed   that   Veloso’s  pockets  were   bulging   as   if   containing   and manner of execution of the warrant.
gambling utensils and as such they required him to show the evidence
they are searching for. After five minutes of conversation (with Veloso 1. The time must not also be one which is intrusive or violative
constantly refusing to be searched all the while), the officers’  patience   of   one’s  privacy (e.g.   in  the   middle   of   the   night).   It’s  also   a  
reached its limit and one of them took hold of Veloso who promptly given that what is considered as reasonable time in one
retaliated, injuring the officer quite severely. It took two officers to locality may not be deemed the same way in another.
restrain Veloso, and with that, it was found that gambling utensils really 2. As for place, the warrant limits the area that may be
were in his pockets. searched and the warrant could not be utilized as authority
to search everywhere until the items or articles sought for
All of the persons apprehended – Veloso included – were arrested, are found.
promptly searched and taken to the patrol wagons. Veloso again 3. Lastly, the manner of execution should not be oppressive or
refused to obey and shouted offensive epithets against the police otherwise abusive.
department, resisting so tenaciously that it took three policemen were
needed to place him in the patrol wagon. Because of his actions, Veloso Q: May the implementation of search warrants be done on different
was charged and convicted of the crime of resistance to agents of the days?
authority (governed by Art. 252 of the Penal Code back then). On A: Yes, it could be served at any time within its 10-day lifetime. If its
appeal, Veloso held on to the proposition that his resistance was purpose cannot be accomplished in one day, it may be continued in the
justifiable on account of the illegality of the John Doe search warrant. next following day/s until completed.

ISSUE: WON the search warrant and the arrest of Veloso were valid. Q: May the search warrant be used or a different purpose on each day?
A: No. A warrant can no longer be utilized as authority to make another
HELD: YES. Both the affidavit for the search warrant and the search search once the items specified in it have already been seized.
warrant itself described the address of the building where the
Parliamentary Club is. That sufficed as a sufficient designation of the
People v. Court of Appeals
premises to be searched. As the search warrant stated that John Doe
347 SCRA 453 (2000)
had gambling utensils in his possession in the building occupied by him
(which, of course, is the building where the Parliamentary Club is) and
FACTS: Private respondent Ortiz was arrested carrying a pistol and
as that John Doe was Jose Ma. Veloso, the manager of the club, the
shabu. Soon after his arrest a search warrant was obtained from the
police could identify John Doe as Jose Mar. Veloso without difficulty.
MTC   of   Parañaque   commanding   the   PNP   officers   “to   make   an  
immediate search at any reasonable hour of day or night of the
Pangandaman v. Casar house/s, closed receptacles and premises above-described and
17 | P LATON
forthwith   seize   and   take   possession”   of   the   property   subject   of   the   to abuse of the search process, and grant to officers executing a search
offense described (that is, unlicensed firearms – a Baby armalite M-16, warrant that discretion which the Constitution has precisely removed
a shotgun, a pistol cal. 45 and the like). Armed with the warrant, the from them. The particularization of the description of the place to be
police – accompanied by a representative of the MTC judge and a searched may properly be done only by the Judge, and only in the
barangay security officer – went  to  Ortiz’s  residence  at  about  7:30  PM   warrant itself; it cannot be left to the discretion of the police officers
to search the premises, which resulted in the seizure of several conducting the search.
unlicensed firearms and ammunition. Ortiz was subsequently charged
with illegal possession of firearms; he moved to quash the search Wilson v. Layne
warrant but the trial court denied it. The Court of Appeals (hereinafter 526 US 603, 143 L Ed 2d 818, 119 S Ct 1692 (1999)
CA) reversed, holding as inadmissible in evidence the seized firearms
and ammunition. There is a warrant of arrest  for  the  petitioner’s  son  in  their  home.
Respondents,   deputy   federal   marshals   and   local   sheriff’s   deputies,  
ISSUE: WON the CA a quo erred in holding that the firearms and invited the Washington Post (reporter and photographer) as part of
ammunition  seized  from  private  respondent’s  house  are  inadmissible  as   Marshal’s  Service  “ride-along”  policy.
evidence for being the fruits of an illegal search.
Warrant was silent about the media ride-along.
HELD: YES. In this case, there is no illegal search. The general rule is that
search warrants must be served in the daytime. However, when the 6:45 am petitioners were still in bed. Husband wearing pair of briefs ran
application asserts that the property is on the person or place ordered to the living room to investigate discovering 5 men wearing street
to be searched, a search at any reasonable hour of the day and night clothes with guns in their living room. The respondents, believing him
can be ordered. As to whether the time during which the search was to be the person sought by the warrant, immediately subdued him on
executed was unreasonable or not, the Supreme Court (hereinafter SC) the floor.
stated that the exact time of the execution of the warrant should be
left at the discretion of the law enforcement officers. Judicial notice Wife follows wearing only nightgown saw her husband being restrained
may be taken not just from the realities of law enforcement but also by the armed officers.
the prevailing conditions in the place to be searched. As such, the SC
deemed that 7:30 PM was a reasonable time, taking notice that said Reporters observed and photographed the incident BUT WERE NOT
time in a suburban subdivision in Metro Manila in an hour at which the INVOLVED IN THE EXECUTION OF THE WARRANT. The newspaper
residents are still up and about. NEVER published the photographs.

People v. Court of Appeals After  a  protective  sweep  of  the  house  revealed  that  the  petitioner’s  son  
291 SCRA 400 (1998) is not in the house, the officers left.

FACTS: Quezon City police have procured a search warrant against Azfar ..Petitioners sued the officers in their personal capacities for money
Hussain who had allegedly in his possession firearms and explosives at damages.
Abigail Variety Store (hereinafter AVS) in Bulacan. However, the next ..District court denied on basis of qualified immunity.
day, the search warrant against Hussain was served not at AVS but at ..Court of Appeals declined to decide whether it is a violation of the 4th
the adjacent apartment (hereinafter Apartment 1), resulting in the amendment but concluded that because no court had held at the time
arrest of four Pakistani nationals and the seizure of their personal of the search that media presence during a police entry into a residence
belongings, papers and effects, including cash (apparently quite a lot, constituted such   a   violation,   the   right   alleged   was   NOT   “CLEARLY  
too) which was never mentioned in the warrant. ESTABLISHED”   and   thus   the   respondents   are   entitle   to   qualified  
immunity
Included allegedly among those taken were a piece of dynamite stick, CHIEF JUSTICE REHNQUIST delivered the opinion of the court.
two pieces of C-4-type explosives and one fragmentation grenade, ………..  such  a  media  ride-along does VIOLATE the 4th amendment. BUT
along with some firearms and ingredients for explosives. On because the state of the law was not clearly established at that time
arraignment, the four Pakistani nationals pleaded not guilty and the search in this case took place, the officers are entitled to the
submitted a motion to quash the search warrant and declare the defense of qualified immunity.
evidence obtained by the police officers as inadmissible; the trial court
granted the motion. The People brought forth a petition for certiorari Petitioners sued the federal officials under Bivens (Hanlon v. Bivens 525
but the CA dismissed it, basing on the proposition that the place US 981, [1988]) and the state officials under §1983. Both allows a
actually searched was different and distinct from the place described in plaintiff to seek money damages from government officials who have
the search warrant. violated the 4th amendment. But government officials are shielded from
liability for civil damages insofar as their conduct does not violate
ISSUE: WON a search warrant was validly issued as regards the CLEARLY ESTABLISHED statutory or constitutional right of which a
apartment in which private respondents were then actually residing (or, reasonable person would have known. (Harlow v. Fitzgerald 457 US
put differently, WON the apartment had been specifically described in 800, [1982])
the warrant).
The court evaluating a claim for immunity Must first determine
HELD: NO. (The SC noted that the discrepancy concerning the premises whether the alleged right was clearly established at the time of
to  be  searched  appeared  to  have  resulted  from  the  police  officers’  own   alleged violation. (Conn v. Gabbert 525 US 286 [1999])
faulty description of said premises in their application for the search
warrant, which was exactly what the Judge of the Quezon City Regional In 1604 an English court made the now-famous   observation   that   “the  
Trial Court reproduced – at AVS. The Judge also made the scope of the house of everyone is to him as his castle and fortress, as well for his
search more particular and restrictive by his admonition in the warrant defense against injury and violence, as for  his  repose”
that  the  search  be  “limited  only  to  the  premises  herein  described”.)  The  
place to be searched, as set out in the warrant, cannot be amplified or “The   law   of   England   has   so   particular   and   tender   a   regard   to   the  
modified  by   the   officers’   own  personal   knowledge   of   the   premises,   or   immunity   of   a   man’s   house,   that   it   stiles   in   its   castle,   and   will   never  
the evidence they adduced in support of their application for the suffer  it  to  be  violated  with  impunity………….from  this  reason  no  doors  
warrant. Such a change is proscribed by the Constitution which requires may be broken to execute any civil process; though, in criminal cases,
inter alia the search warrant to particularly describe the place to be the  public  safety  supersedes  the  private.”  William  Blackstone
searched as well as the persons or things to be seized. It would concede
to police officers the power of choosing the place to be searched, even The 4th amendment:   “   the   right   of   the   people   to   be   secured   of   their  
if it not be that delineated in the warrant. It would open wide the door persons, houses, papers, and effects, against unreasonable search and

18 | P LATON
seizures shall not be violated, and no warrants shall issue, but upon This exception requires that there must be a lawful arrest which
probable cause, supported by Oath or affirmation, and particularly precedes the search, the search must be contemporaneous with
describing the place to be searched, and the persons or things to be the arrest, and the area searched is within the immediate
seized.
control of the person arrested
Respondents concede that the reporters did not assist them in their
task. But they are there to assert that the officers should be able to Nolasco v. Paño
exercise reasonable discretion, for good public relations, and also could 139 SCRA 152 (1985)
serve as to minimize abuse of the officers.
Own purpose of the Reporters, and the fact that it is not the officers Aguilar-Roque was one of the accused in the criminal case for rebellion
who kept the photographs, though in some cases, presences of 3rd before Special Military Commission No.1, and also one of the accused in
parties are justifiable, this is not. a criminal case for subversion before the Military Commission no. 25.

The court held that it is a violation of the 4th amendment for police to ….August  6,  1984..  She  and  Nolasco  was  arrested  at  the  intersection  of  
bring members of the media or other 3rd parties into a home during the Mayon street and Piy Margal Street, Quezon City at 11:30 am. At noon
execution of a warrant when the presence of the third parties in the on the same day, elements of the Constabulary Security Group
house was not in aid of the execution of the warrant. searched the premises at 239-B mayon Street, Quezon City.

It is not unreasonable for the respondents at that time to believe that ….The   search   resulted   in   the   seizure   of428   documents   and   written  
bringing media along is lawful. materials, and additionally a portable typewriter, and 2 wooden boxes,
making 431 items in all, and the arrest of Tolentino, the person in
It is common practice to them. And there is no persuasive authority yet charge of the premises.
on the case.
…  Aguilar-Roque raised the issue of inadmissibility of evidence obtained
-judgment of CA is affirmed pursuant to the search warrant.
……………                                                                                    ………………
… Judge   Paňo   admitted   to   the   amended   Return   and   ruled   that   the  
Outright entry also is not justified seized documents be subject to disposition of the tribunal trying the
case against the respondent.
knock and announce rule (US)
Rule 126, section 7 of the rules of criminal procedure (Philippines) …Petitioners   filed   the   instant   petition   for   certiorari,   prohibition   and  
mandamus to annul and set aside the
Sec7. Right to break door or window to effect search – the officer, if 1. search  warrant  issued  by  the  respondent  Judge  Paňo
refused admittance to the place of directed search after giving notice of 2. his Order admitting the Amended Return and granting
his purpose and authority, may break open any outer or inner door or the Motion to Retain Seized Items
window of a house or any part of a house or anything therein to 3. Order of the judge in the subversive documents case
execute the warrant or liberate himself or any person lawfully aiding denying  petitioner’s  Motion  to    Suppress.
him when unlawfully detained therein.
…..balancing  of  person’s  right  and  public  interest.
Further, being armed with a warrant does not also justify Petitioners principally argue that Search warrant is void because it is a
outright entry or barging into the place to be searched. In the general warrant since it did not sufficiently describe with particularity
U.S., there is the so-called  “knock  and  announce”  rule  whereby   the things subject of the search and seizure, and that probable cause
an officer executing a warrant must knock and introduce had not been properly established for lack of searching questions
himself and announce his purpose and only in exceptional propounded  to  the  applicant’s  witness.  The  SC  held  the  warrants  void,  
circumstances may he dispense with the same, as when his but did not order the return of the seized items. Temporary restraining
safety is at stake, or there is danger of the evidence being order enjoining the respondents from introducing evidence obtained
pursuant to the search warrant in the subversive documents case is
destroyed
made permanent, personalties seized may be retained by the CSG for
possible introduction as evidence in Criminal case no. SMC 1-1. Pending
Warrantless Searches and Seizures before Special Military Commission No. 1, without prejudice to
petitioner Mila Aguilar-Roque objecting to their relevance and asking
The recognized exceptions to the warrant requirement include: said Commission to return to her any and all irrelevant documents and
1. Search incident to a lawful arrest articles.
2. Evidence in plain view
3. Search of moving vehicles Teehankee, J., concurring and dissenting:
4. Customs search
Search warrant = void
5. Stop and frisk Warrantless search = still void because she was arrested on board a
6. Exigent and emergency circumstances public vehicle. Warrantless search cannot be made in a place other
7. Consented search than the place of arrest.

Search Incident to Lawful Arrest Resolution on Motion for Reconsideration


147 SCRA 509 (1987)
This is primarily justified by the need to ensure the safety of the
arresting officers against any possible harm arising from the use The solicitor general, on behalf of  public  respondents,  “in  deference  to  
the dissenting opinion of then SC Justice (now Chief Justice) Claudio
by the arrested individual of any weapon that he might have
Teehankee,”   now   offer   no   further   objection   to   a   declaration   that   the  
concealed in his person or which is within immediate reach as subject search is illegal and to the return of the seized items to the
well as the need to preserve evidence that might otherwise be petitioners. Respondents state, however, that they cannot agree to
destroyed having the arrest of petitioners declared illegal

Search warrant =void

19 | P LATON
FACTS: Detective Belland received information from a confidential
Temporary restraining order enjoining the respondents from informer that unknown persons were smoking opium in a hotel. The
introducing evidence obtained pursuant to the search warrant in the detective called for federal narcotic agents and went to the hotel with
subversive documents case is made permanent. such agents. The agents, who are experienced in narcotic work
recognized the smell of opium. The odor led them to Room 1. As
Personalities seized by virtue of the illegal search warrant are hereby Belland went to the door and introduced himself, there was a slight
ordered returned to petitioners. delay in the door before the defendant open the door. The defendant,
a woman, stepped back and admitted the officers. Belland then told her
People v. Leangsiri about the opium smell, the defendant denied that there is such a smell.
252 SCRA 213 (1996) Belland told her to consider herself under arrest because they were
going to search the room. The officers found opium and smoking
Leangsiri was arrested at the arrival area of NAIA bringing 8225.31 apparatus, the latter being warm, apparently from recent use. The
grams of heroin hidden under the false bottom of a black suitcase. defendant objected to the admission of the evidence but the District
Informed of the authorities that he is to deliver the contraband to 3 Court refused to suppress the same. She was convicted and the Court
people at the Las Palmas Hotel in Manila. of Appeals affirmed.

An entrapment was devised by NARCOM (narcotics command) and ISSUE: Whether or not there was a violation of a valid search and
agents of the bureau of customs then ensued. seizure (Specifically yung sa violation ng fourth amendment rights.)

The 3 were arrested. Decision: Search and arrest not valid.

They learned that Amidu stays at room 413 of the same hotel. While Rationale: No reason for not obtaining a search warrant except
Omogbolahan and Bhola were billeted at Royal Palm Hotel. inconvenience to the officers and some slight delay necessary to
prepare papers and present the evidence to magistrate. There are no
Accompanied by hotel owner and security officer, they searched convincing reasons to bypass the constitutional requirement. No
Amidu’s  room. evidence or contraband was threatened with removal or destruction,
except perhaps the fumes which suppose in time will disappear. The
Got there a piece of paper with the name Suchinda Leangsiri written on evidence of their existence before the search was adequate and the
it tucked within the pages of his telephone and address book. And testimony of the officers to that effect would not perish from the delay
other possessions were confiscated. of getting a warrant.

NARCOM went to Royal Palm Hotel. Coordinated with security officers Since it was a search without warrant, it could be valid only if for a
of the hotel who stood as witnesses when they entered and searched crime committed in the presence of the arresting officer for a felony of
the room. They yielded 2 black suitcases, each with false bottoms and which he had reasonable cause to believe defendant guilty. –The
both smaller than that confiscated from Leangsiri. The appellants were arresting officer did not have probable cause to arrest petitioner until
convicted of conspiring to transport heroin. he had entered her room and found her to be the sole occupant.

Whether or not the piece of paper found in the room of Amidu is An officer gaining access to private living quarters under color of his
admissible as evidence. office and of the law which he personifies must then have valid basis in
law for the intrusion. Valid basis is lacking.
Revised Rules of Court provides that a person lawfully arrested may be
searched for dangerous weapons or anything which may be used as Plain View Doctrine
proof of the commission of an offense, without a search warrant.
Under this exception, contraband in plain view of officers who
Strict application of the exception that such warrantless search
have a right to be in that place where they are and see the
obviously cannot be made at a place other than the place of arrest.
contraband have the right to seize it without having to secure a
Immediate Control testwas enunciated in the American case of Chimel warrant
vs. State of California. (Arrested at his house.. searched the entire
house and things found were allowed to be admitted as evidence. But Requisites:
the SC reversed the conviction and held that the search went far 1. Prior justification for the entry or intrusion
beyond his person and the area from which he might have obtained 2. Inadvertent discovery of the evidence
either a weapon or something that could have been used as evidence 3. Immediate apparent illegality of the item as evidence of a
against him.)
crime, contraband or otherwise subject to seizure
The exception obtains when the plain view doctrine applies.
4. Plain view justifies mere seizure of evidence without further
search
In the case a car, appellants were arrested in room 504 of the Las
Palmas Hotel. The piece of paper bearing the name of leangsiri People v. Musa
obtained in room 413 of the same hotel found through warrantless 217 SCRA 597 (1993)
search is illegal and the piece of paper is held to be inadmissible as
evidence against the appellants. The inadmissibility of the paper as In a buy-bust operation, a poseur buyer went to the house of Musa
evidence will not however exculpate the appellants. Its exclusion does (Present in the house of Musa: a boy, two women, one of which is his
not  destroy  the  prosecutor’s  case  against  the  appellants.  The  remaining   wife) and asked for marijuana. After receiving the 20 Php marked
evidence still established their guilt beyond reasonable doubt. money,  Musa  gave  the  marijuana  to  the  “buyer”,  the  “buyer”  went   to  
the Narcotics Command and showed them marijuana. NARCOM team
Johnson v. US then   rushed   to   the   buyer’s   house.   The   marked   money   could   not   be  
333 U.S. 10, 92 L Ed, 436, 68 S Ct 367 (1948) found. Musa said that he has given the money to his wife na wala na
dun nung panahon nay un. Apparently, two team members went to a
Petitioner challenges her conviction on four counts charging violation of kitchen and noticed  a  “cellophane  colored  white  and  stripe  hanging  at  
federal narcotic laws on the ground that there were violations of her the   corner   of   the   kitchen.”   The   two   asked   Musa   but   Musa   did   not  
Fourth Amendment rights. answer. They opened it and found dried marijuana leaves. Musa was
convicted of selling marijuana.

20 | P LATON
12 small plastic bags containing and paper clip bags containing shabu
ISSUE: Whether or not the evidence was admissible. and two bricks of dried leaves which appeared to be marijuana. The
accused was charged with the unlawful possession of shabu and
Decision:  No  it’s  not.  (Under  the  Plain  View  Doctrine) marijuana. Accused was convicted. On appeal, he questions the validity
of the search warrant and the seizure of the bricks of marijuana.
Rationale: In a buy bust operation, the authorities may seize the
marked money found on the person of the pusher immediately after ISSUE: whether or not the seizure of the latter drug (marijuana) was
the arrest even without arrest and search warrants. Apparently, the justified on the ground that the drug was seized within the plain view of
NARCOM agents searched the person of the appellant after arresting the searching party.
him in his house but found nothing.  That’s  the  time  they  searched  the  
whole house and found the plastic bag hanging in the corner which Decision: Not justified. The decision of the trial court was reversed and
contains the marijuana. set aside. Accused is acquitted.
Rationale: For the plain view doctrine to apply, there must be: (a) prior
The warrantless  search  and  seizure,  as  an  incident  to  a  suspect’s  lawful   justification, (b) inadvertent discovery of the evidence, (c) immediate
arrest, may extend beyond the person of the one arrested to include apparent illegality of the evidence before the police.
the premises or surroundings under his immediate control. Objects in
the  “plain  view”  of  an  officer  who  has  the  right  to  be  in  the position to The question is whether these requisites have been complied with.
have that view are subject to seizure and may be presented as (Hindi, lalo na yung a at b)
evidence.
The only justification for an intrusion by the police is the conduct of a
The plain view doctrine cannot justify the seizure of the object where search   pursuant   to   accused   appellant’s   lawful   arrest   for   possession   of  
the  incriminating  nature  of  the  object  is  not  apparent  from  the  “plain   shabu. The police failed to allege in this case the time when the
view”   of   the   object.   THE   PLASTIC BAGS WERE NOT WITHIN THEIR marijuana was found; whether or prior to, or contemporaneous with,
“PLAIN  VIEW”  WHEN   THEY   ARRESTED   THE   APPELANT  TO   JUSTIFY   ITS   the shabu subject of the warrant, or whether it was recovered on
SEIZURE. THE NARCOM AGENTS HAD TO MOVE FROM ONE PORTION accused-appellant’s  person  or  in  an  area  within  his  immediate  control.  
OF THE HOUSE TO ANOTHER BEFORE THEY SIGHTED THE PLASTIC Its recovery, therefore, presumably during the search conducted after
BAG. THE AGENTS HAD NO CLUE AS TO THE PLASTIC BAGS’  CONTENTS,   the shabu, had been recovered from the cabinet. Moreover, the
THEY HAVE TO OPEN IT FIRST TO KNOW WHETHER ITS CONTENTS IS A marijuana was in the form of two bricks wrapped in newsprint. Not
CONTRABAND  OR  NOT.  WHAT’S  WITHIN  THEIR  PLAIN  VIEW  WAS  THE   being in a transparent container, the contents wrapped in newsprint
PLASTIC BAG, NOT THE MARIJUANA. could not have been readily discernible as marijuana. The recovery of
marijuana is INVALID. It is inadmissible in evidence against accused and-
People v. Doria appellant.
301 SCRA 668 (1999)
Search of Moving Vehicles
Buy-bust operation.
If a warrant were first required before a car may be searched, it
View of the Prosection: PO3 Manlangit acted as the buyer, gave 1600
may only be an exercise in futility as by the time the warrant is
pesos to Doria who thereafter told the former to wait for him as he
went to get the marijuana from his associate. After an hour, Doria gave
issued the vehicle to be searched may have been driven to
PO3 an object wrapped in plastic and gave it to PO3 who forthwith some far away and unknown places
arrested Doria. Marked money could not be found so they went to the
associate’s  house.  PO3  was  standing  by  the  door  and  saw  a  box  which   One has a lesser expectation of privacy in a motor vehicle
has something wrapped in plastic which appear to be what just like because its function is transportation, and it seldom serves as
what Doria gave him. As  PO3’s  suspicion  aroused,  he  entered  Neneth’s   one’s   residence   or   as   the   repository of personal effects. A car
house and took hold of the box. Other officers recovered the marked has little capacity for escaping public scrutiny. It travels public
money bills from Neneth. Defense denies these allegations.
thoroughfares where both its occupants and its contents are in
ISSUE: Whether or not what happened is a reasonable seizure.(Plain
plain view
view doctrine again)
Related to this kind of searches is the issue of checkpoints
Decision: no. where the rule is that only visual searches or inspection may be
had unless there is justifiable reason for conducting a more
Rationale: It is clear that an object is in plain view if the object itself is extended search
plainly exposed to sight.
Standing  by  the  door  of  appellant  Gaddao’s  house,  PO3  Manlangit  had  
The Court differentiated the checkpoint--which was primarily
a view of the interior of the said house. Two and a half meters away
was the dining table and underneath it was a carton box. The box was
intended   to   determine   if   the   vehicle’s   occupants   were  
partially open and revealed something wrapped in plastic. In a cross committing a crime--from  an  “information-seeking”  checkpoint  
examination, Manlangit admitted that he merely presumed the where police stop vehicles and ask the occupants for help in
contents to be marijuana because it had the same plastic wrapping as providing information about a crime that has been committed
the   “buy-bust”   marijuana.   A   close   scrutiny   of   the   records   reveals   that   by others. This latter form was considered valid
the plastic wrapper was not colorless and transparent as to clearly
manifest its contents to a viewer. The marijuana was not in plain view People vs. Barros
and its seizure without requisite search warrant was in violation of the 231 SCRA 557 (1994)
law and the constitution.
FACTS: 2 police officers while riding a bus saw a man(Barros) carrying a
People v. Salanguit carton and placed it under his seat. When the officers reached their
356 SCRA 683 destination, they informed another policeman to inspect the carton
carried by Barros. When the said policeman inspected the carton, he
Police   officers   went   to   Salanguit’s   house   with   a   search   warrant   for   found that it contained marijuana. When asked whether he owned the
shabu. When they knocked on the door, no one opened the door. They carton of marijuana, he denied the same. But later on admitted the
heard people panicking so they forced the door open and entered the ownership  of  such  after  the  bus  driver  affirmed  Barros’  ownership.  He  
house. After showing the warrant, they searched the house, they found was charged for the violation of the Dangerous Drug Act of 1972. The

21 | P LATON
trial court convicted him. On appeal he claims that, among others, his which has been declared no to be illegal per se, for as long as it is
Constitutional right; against unreasonable search and seizures was warranted by the exigencies of public order and conducted in a way
violated by the police authorities. least intrusive to the motorists.

ISSUE: whether or not the act of the policemen constitutes Routine  inspections  are  not  regarded  as  violative  of  an  individual’s  right  
unreasonable search and seizures? against unreasonable search. Limitations of routine inspections:
a. Where the police officer merely draws aside the curtain of a
HELD: there was an unreasonable search and seizures. vacant vehicle which is parked on the public fair grounds.
b. Simply looks into a vehicle
General rule, a search and seizure must be carried out through a c. Flashes  a  light  therein  without  opening  the  car’s  door
judicial warrant. d. Where the occupants are not subjected to a physical or body
search
Exceptions (1): in case of moving vehicles, warrant less search can be e. Where the inspection of vehicles is limited to a visual search
conducted because it is not practicable to secure a judicial warrant or inspection
before searching the same since such can be quickly moved out of the f. Routine check is conducted in a fixed area
locality or jurisdiction in which the warrant may be sought.
None of the following circumstances are present in the case at bar. The
Limitations for the exception: police did not merely conducted a visual inspection of the vehicle, they
a. it is limited only to routine checks, that is, the vehicle had   to   reach   inside   the   vehicle…it   is   not   considered   a   simple   routine  
are neither really searched not their occupants subject check.
to physical or body searches, the examination of the
vehicles being limited to visual inspection. The fact that the vehicle looked suspicious simply because it is not
common for such to be covered   with   “kakawati”   leaves   does   not  
Exception (2): if there is probable cause. constitute probable cause as would justify the conduct of a search
without warrant.
In the case at bar, there was no probable cause for the policemen to
suspect that the carton he carried contained marijuana. Neither did Rule: accused is acquitted.
the carton emanate a distinct smell as that of marijuana. Nor did Barros
act suspiciously while boarding the bus. It did not indicate nor suggest Valmonte v. De Villa
the presence of any such probable cause. 178 SCRA 221 (1989)

Waiver  of  unlawful  search  and  seizure…  to  constitute  a  waiver,  it  must   FACTS: certain checkpoints in certain parts of Valenzuela and other
appear first that the right exists; second, that the person involved had cities were set-up by the NCRDC which some of its residents complain
knowledge, actual or constructive, of the existence if such right; last, because they claim they are violation of their right against search and
the person had an actual intention to relinquish the right. The fact that seizures. Furthermore, they claim they are worried of being harassed
the accused failed to object to the entry to his house does not amount and of their safety being placed in arbitrary, capricious and whimsical
to permission to make a search therein. disposition of the military. The checkpoints by the military have been
issued to maintain, among others, peace and order.
Ruling: Barros is acquitted.
ISSUE: whether the military checkpoints are constitutional and not an
Caballes v. Court of Appeals infringement upon the right from warrantless search and seizures?
373 SCRA 221 (2002)
HELD: there was no issue to begin with because the petitioners are not
FACTS: 2 policemen, while performing a routine patrol, spotted a the real party in interest. It was ruled that petitioners who do not
passenger  jeep  covered  with  “kakawati”  leaves  driven  by  the  appellant.     allege that any of their rights were violated are not qualified to bring
The policemen stopped the jeep, and when asked what was loaded on action as real party in interest. In the case at hand, no proof to show
such, the appellant did not answer appearing pale and nervous. They that, in the course of the routine checks, the military indeed committed
checked the cargo and discovered that it contained specific   violations   of   petitioner’s   right   against   unlawful   search and
aluminum/galvanized conductor wires exclusively owned by seizure or other rights.
NPC(national power corporation) He was convicted of theft. However
on appeal, he raised the validity of the warrantless search and the Between the inherent right of the state to protect its existence and
admissibility of the evidence thus obtained. promote   public   welfare   and  an   individual’s  right   against   a   warrantless  
search, which is however reasonably conducted, the former should
ISSUE: whether or not police officers who were on routine patrol, prevail.
merely   on   “suspicion”   that   “it   might   contain   smuggled   goods,”  
constitutes probable cause that will justify warrantless search and Rule: petition dismissed.
seizure?
Anag, Jr. v. COMELEC
HELD: there is no probable cause and therefore illegal. 237 SCRA 424 (1994)

The mere mobility of vehicles, does not give the police officers FACTS: COMELEC  issued  resolution  2323  otherwise  known  as  the  “Gun  
unlimited discretion to conduct indiscriminate searches without Ban.”  Pursuant  to  such  resolution,  the  petitioner  instructed  his  driver  to  
warrant if made within the interior of the territory and the absence of immediately send his gun that was given to him by the House of
probable cause. Representatives. Following such order, the driver immediately went to
Manila. At about 20 meters from the Batasang Pambansa, police
Probable cause signifies a reasonable ground of suspicion supported by authorities conducted a checkpoint. About 14 men search the car
circumstances sufficiently strong, in themselves, to warrant a cautious driven by the driver, thus the gun that was to be delivered was seized
man’s  belief  that  the  person  accused  is  guilty  of  the  offense  with  which   and he was charge for the violation of the gun ban.
he is charged. The required probable cause will justify a warrantless
search and seizure is not determined by a fixed formula but is resolved ISSUE: whether or not the search and seizure of the gun was in violation
according to the facts of each case. of  an  individual’s  right  against  warrantless  search?

“stop-and-search”   without   warrant   at   military   or   police   checkpoint   HELD: It was a violation of such right, since there were no justifying

22 | P LATON
circumstances specifically pointing the culpability of the petitioner and Almeida – Sanchez v. United States
his driver, therefore, the search was invalid. The action of the 413 US 266, 37 L Ed 2d 596, 93 S Ct 2535 (1973)
policemen unreasonably intruded to the privacy and the security of his
property. Consequently, the firearms obtained in the search cannot be FACTS: Mexican and holder of a valid work permit, was stopped by the
admitted as evidence for any purposes of proceeding. Border Patrol on a state highway 78 in California and hi car was
thoroughly searched. The point where the petitioner was stopped, the
The submissive actions of the driver with regards to the search, seems road meanders north as well as east. But nowhere does the road reach
like a waiver of the need for a warrant to search. However, considering the Mexican border, and at all points it lies north of US 80 a major east
the circumstance wherein 14 armed men who were there to search the – west highway entirely within the United States that connects the
vehicle, without his master, herein petitioner, thus he is in no way Southwest with the west coast. It is obvious that the Border Patrol had
capable of resisting such actions. no search warrant, and that there was no probable cause of any kind
for the search. Marihuana was uncovered from the search. The only
asserted justification is 287a3 of the Immigration and Nationality Act,
Aniag, Jr. Vs Comission on Elections 66 Stat. 233 8 U.S.C. 1357a3, that provides for warrantless searches of
237 SCRA 424 (1994) automobiles and other conveyances within reasonable distance from
any external boundary of United States.
FACTS: In the preparation for the synchronized election for the national
and local elections in 1992. COMELEC issued resolution No. 2323 also ISSUE: W/N the Border Patrol search is constitutional
known   as   “Gun   Ban”   containing   rules   and   regulations   on   bearing,  
carrying and transporting of firearms or other deadly weapons. HELD: No claim is made nor could one be that the search of the
COMELEC also issued resolution No. 2327 providing disqualification of petitioner’s   car   was   constitutional   under   any   previous   decision   of   this  
candidates engaged in gunrunning, using and transporting of firearms, court involving the search of an automobile. The search in this case was
organizing special strike forces, and establishing spot checkpoints. conducted in the unfettered discretion of the members of the Border
Patrol without warrant, probable cause or consent. Search on a
A congressman of 1st district of Bulacan was compelled to return 2 California road that lies at all points at least 20 miles north of the
firearms issued to him by the House of Representatives. Petitioner Mexican border was a wholly of different sort. In the absence of a
instructs his driver Arellano to pick the firearms and return it to the probable cause search violated the right to be free from unreasonable
congress. In the same day PNP set up a check point outside Batasan, search and seizures.
where Arellano was stop and searched finding the two firearms
properly packed. Arellano was detained and the firearms confiscated. United States v, Brigoni
Later Arellano was released and the petitioner was invited to give light 422 US 873 (1975)
to the situation. Petitioner explained that Arellano did not commit any
crime and that the driver actually complying with the law apprehended Case involving the not the Border Patrol but to question the occupants
by returning the firearms to the congress. about their citizenship and immigration status. The officer may
question the driver and passengers about their citizenship and
ISSUE: W/N such search and seizure is a violation to the constitutional immigration status.
right to due process.
United States v. Martinez – Fuente
HELD: Extensive search without warrant could only be resorted to if 428 US 543 (1976)
the officers conducting the search had reasonable or probable cause. In
the case at hand there was no evidence to show that the policeman Brief questioning is consistent with the fourth amendments, thus not
were impelled to do such, for there is no confidential report leading need ne authorized by warrant.
them to reasonably believe that certain motorist is engage in
gunrunning. Therefore the search could not be valid. Customs Searches

People v. Usana Items which are imported and which are to be subjected to
323 SCRA 754 (2000)
payment of customs duties are not considered as properly
FACTS: During the COMELEC gun ban period, policemen set up a check
within the territory of the taxing authority if the appropriate
point at the corner of Senator Gil Puyat Ave. Those manning said taxes have not yet been paid
checkpoint, check merely those they found to be suspicious an
imposing merely a running stop on others. Past midnight they stopped “Search   and   seizure   without   search   warrant   of   vessels   and   air  
a KIA Pride driven car by Escano with two passengers with it namely crafts for violations of the customs laws have been the
Usana and Lopez. One of the Policemen saw a long firearm on the lap traditional exception to the constitutional requirement of a
of Usana who was seated at the passenger seat. The three were search warrant, because the vessel can be quickly moved out of
brought to the police station together with the car. The car was
the locality or jurisdiction in which the search warrant must be
searched and found within is hashish.
sought before such warrant could be secured; hence it is not
ISSUE: W/N such search and seizures is a violation of the constitutional practicable to require a search warrant before search or seizure
rights of the accused. can be constitutionally effected

HELD: Court ruled that not all checkpoints are illegal. Those which are Papa v. Mago
warranted by exigencies of public order and are conducted in a way 22 SCRA 857 (1968)
least intrusive to motorists are allowed. As long as the inspection is
limited to a visual searched it cannot be regarded as violative of FACTS: Counter intelligence unit of the Manila Police Department
individual’s   right   against   unreasonable   searched.   There   are   six   misdeclared that a certain shipment of personal effects would be
recognize exceptions to warrant requirement 1) searched incidental to release from the customs zone pier in manila. When the trucks left the
an arrest 2) searched moving vehicles 3) evidence in plain view 4) vicinity other counter – intelligence group seize the said trucks.
custom searches 5) consented warrantless searched and 6) stop – frisk
– situations. Therefore checkpoint conducted was in pursuance of the ISSUE: W/N such act by the counter intelligence of Manila Police
gun ban enforced by the COMELEC. prejudiced the right to be free from unreasonable search and seizure.

HELD: 2d section of the Act of March 3, 1815 it was made lawful for

23 | P LATON
customs offices not only to board and search vessels within their own search is warranted for his and others' safety.
and adjoining examine any vehicle beast or person which is suspected In delivering the opinion of the Court, Chief Justice Warren stated:
which are introduce contrary to the law. In the instant case the "Where a reasonably prudent officer is warranted in the circumstances
petitioner could not question the search for their only complain they of a given case in believing that his safety or that of others is
were just intercepted without any search warrant. But even if there is a endangered, he may make a reasonable search for weapons of the
search there is still authority to the effect that no search warrant would person believed by him to be armed and dangerous regardless of
be needed under the circumstances obtaining in the instant case. Thus whether he has probable cause to arrest that individual for crime or the
freedom from unreasonable searches and seizures is construed as to absolute certainty that the individual is armed.
recognize the difference between search in the dwelling and search of a "(a) Though the police must, whenever practicable, secure a warrant to
ship, motorboat, wagon or automobile where it is not practicable to make a search and seizure, that procedure cannot be followed where
secure a search warrant. For the reason a vehicle could quickly move swift action based upon on-the-spot observations of the officer on the
out. beat is required."

Stop and Frisk Exception - The Terry Patdown The majority of the Court concluded the "stop and frisk" technique was
constitutional as long as the action could be rationally justified by
circumstances.
In certain situations, a police officer on the beat may observe
certain unusual and suspicious activity which his training and
Adams v. Williams
experience would indicate that something wrong is afoot. He 407 US 143, 32 L Ed 2d 612, 92 S Ct 1921 (1972)
need not wait for an explicit criminal conduct be manifested
before he may take action. In such an instance, he may FACTS: While on duty, a police officer was approached by a reliable
approach the suspicious person and undertake a limited check-- informant who told the officer that a person sitting in a nearby car, i.e.,
a stop and frisk--patting down the outside clothing of the the defendant, was carrying drugs and a gun. The officer went to
person for possibly concealed weapons. This is justified by the defendant’s  car  and  grabbed  a  gun  from  exactly  the  same  place  where  
need  to  act  expeditiously  in  a  case  where  a  minute’s  delay  may   the informant said the gun would be. Then the officer searched the car
and found additional weapons and drugs.
prove dangerous if not disastrous
ISSUE: Can the officer rely on information obtained from a reliable
A Terry stop   must   be   justified   at   its   inception   and   ‘reasonably   informant for reasonable suspicion for a search?
related  in  scope  to  the  circumstances  which  justified’ the initial
stop HELD: Yes. The Court held that the officer had reasonable suspicion to
The reasonableness of seizures that are less intrusive than a conduct the search. Here, the Court noted that the officer had
traditional   arrest   depends   ‘on   a   balance   between   the   public   reasonable suspicion because the officer knew the informant to be
interest   and   the   individual’s   right   to   personal   security   from   reliable. Thus, the officer had reason to believe that the suspect was
armed and dangerous and he could constitutionally frisk the suspect for
arbitrary  interference  by  law  officers’
weapons.
Terry v. Ohio
392 U. S. 1, 20 L Ed 2d 889 S Ct 1868 (1968) Malacat v. Court of appeals
283 SCRA 159 (1997)
FACTS: A Cleveland police officer (Martin McFadden) observed two
unidentified men on a street corner in his beat. As the officer watched, FACTS: On 27 August 1990, at about 6:30 p.m., allegedly in response to
the men took turns walking a short distance down the sidewalk and bomb threats reported seven days earlier, Rodolfo Yu of the Western
peering in a particular store window. After each circuit, the men would Police District, Metropolitan Police Force of the Integrated National
appear to confer about something. This occurred a total of twenty-four Police, Police Station No. 3, Quiapo, Manila, was on foot patrol with
times (according to the Supreme Court opinion; McFadden's earlier three other police officers (all of them in uniform) along Quezon
testimony indicates the men walked to the store and peered through Boulevard, Quiapo, Manila, near the Mercury Drug store at Plaza
the window approximately 3-5 times each). Miranda. They chanced upon two groups of Muslim-looking men, with
each group, comprised of three to four men, posted at opposite sides
McFadden watched as the men were briefly joined by a third person of the corner of Quezon Boulevard near the Mercury Drug Store. These
who left quickly. The suspicious activity caused McFadden to suspect men  were  acting  suspiciously  with  “their  eyes  moving  very  fast.”  Yu  and  
the men were planning to rob the store. his companions positioned themselves at strategic points and observed
both groups for about 30 minutes. The police officers then approached
After the third individual left, the men began walking away in the one group of men, who then fled in different directions. As the
direction of the store. McFadden followed and observed from a policemen gave chase, Yu caught up with and apprehended Sammy
distance until they met up with the third party. McFadden then Malacat y Mandar. Upon searching Malacat, Yu found a fragmentation
approached the group, identified himself as police, and asked the men's grenade   tucked   inside   the   latter’s   “front   waist   line.”   Yu’s   companion,  
names. police officer Rogelio Malibiran, apprehended Abdul Casan from whom
a .38 caliber revolver was recovered. Malacat was charged with
The men mumbled something unintelligible, at which point McFadden violating Section 3 of Presidential Decree 1866. The trial court ruled
turned one of the suspects and patted down the outside of his clothing. that  the  warrantless  search  and  seizure  of  Malacat  was  akin  to  a  “stop  
In the process, the officer found a gun in the pocket of the suspect's and   frisk,” where   a   “warrant   and   seizure   can   be   effected   without  
jacket. He then directed all three men to enter a store with their hands necessarily  being  preceded  by  an  arrest”  and  “whose  object  is  either  to  
raised, and proceeded to pat down the other two suspects, recovering maintain the status quo momentarily while the police officer seeks to
a revolver from one. The three were taken to the local police station, obtain   more   information”;   and   that   the   seizure   of   the   grenade from
where two were charged with carrying a concealed weapon. Malacat was incidental to a lawful arrest. The trial court thus found
Malacat guilty of the crime of illegal possession of explosives under
ISSUE: Whether it is always unreasonable for a policeman to seize a Section 3 of PD 1866.
person and subject him to a limited search for weapons unless there is
probable cause for an arrest. ISSUE: Whether the search made on Malacat is valid, pursuant to the
exception of “stop  and
HELD: The Court held that the stop, or seizure, and frisk, or search, was frisk.”
valid when a "reasonably prudent officer" has cause to believe a limited
HELD: The general rule as regards arrests, searches and seizures is that

24 | P LATON
a warrant is needed in team. No search warrant was secured by the raiding team. Accused was
order to validly effect the same. The Constitutional prohibition against found guilty of illegal possession of firearms.
unreasonable arrests, searches and seizures refers to those effected
without a validly issued warrant, subject to certain exceptions. As ISSUE: Whether or not there was a valid search and seizure in this case.
regards valid warrantless arrests, these are found in Section 5, Rule 113
of theRules of Court. A warrantless arrest under the circumstances Ruling: YES. It is admitted that the military operatives who raided the
contemplated under Section 5(a) has been denominated   as   one   “in   Eurocar Sales Office were not armed with a search warrant at that time.
flagrante  delicto,”  while  that  under  Section  5(b)  has  been  described  as   The raid was actually precipitated by intelligence reports that said
a   “hot   pursuit”   arrest.   Turning   to   valid   warrantless   searches,   they   are   office was being used as headquarters by the RAM. Prior to the raid,
limited to the following: (1) customs searches; (2) search of moving there was a surveillance conducted on the premises wherein the
vehicles; (3) seizure of evidence in plain view; (4) consent searches; (5) surveillance team was fired at by a group of men coming from the
a   search   incidental   to   a   lawful   arrest;   and   (6)   a   “stop   and   frisk.”   The   Eurocar building. When the military operatives raided the place, the
concepts   of   a   “stop-and-frisk”   and   of   a   search   incidental   to   a   lawful   occupants thereof refused to open the door despite requests for them
arrest must not be confused. These two types of warrantless searches to do so, thereby compelling the former to break into the office. The
differ in terms of the requisite quantum of proof before they may be Eurocar Sales Office is obviously not a gun store and it is definitely not
validly effected and in their allowable scope. In a search incidental to a an armory or arsenal which are the usual depositories for explosives
lawful arrest, as the precedent arrest determines the validity of the and ammunition. It is primarily and solely engaged in the sale of
incidental search. Here, there could have been no valid in flagrante automobiles. The presence of an unusual quantity of high-powered
delicto or hot pursuit arrest preceding the search in light of the lack of firearms and explosives could not be justifiably or even colorably
personal knowledge on the part of Yu, the arresting officer, or an overt explained. In addition, there was general chaos and disorder at that
physical act, on the part of Malacat, indicating that a crime had just time because of simultaneous and intense firing within the vicinity of
been committed, was being committed or was going to be committed. the office and in the nearby Camp Aguinaldo which was under attack by
Plainly, the search conducted on Malacat could not have been one rebel forces. The courts in the surrounding areas were obviously closed
incidental to a lawful arrest. On the other hand, while probable cause is and, for that matter, the building and houses therein were deserted.
not   required   to   conduct   a   “stop   and   frisk,”   it   nevertheless holds that Under the foregoing circumstances, it is our considered opinion that
mere   suspicion   or   a   hunch   will   not   validate   a   “stop   and   frisk.”   Here,   the instant case falls under one of the exceptions to the prohibition
there   are   at   least   three   (3)   reasons   why   the   “stop-and-frisk”   was   against a warrantless search. In the first place, the military operatives,
invalid:  First,  there  is  grave  doubts  as  to  Yu’s  claim  that  Malacat  was  a   taking into account the facts obtaining in this case, had reasonable
member of the group which attempted to bomb Plaza Miranda 2 days ground to believe that a crime was being committed. There was
earlier. This claim is neither supported by any police report or record consequently more than sufficient probable cause to warrant their
nor corroborated by any other police officer who allegedly chased that action. Furthermore, under the situation then prevailing, the raiding
group.   Second,   there   was   nothing   in   Malacat’s   behavior   or   conduct   team had no opportunity to apply for and secure a search warrant from
which could have reasonably elicited even mere suspicion other than the courts. Under such urgency and exigency of the moment, a search
that  his  eyes  were  “moving  very  fast”  — an observation which leaves us warrant could lawfully be dispensed with.
incredulous since Yu and his teammates were nowhere near Malacat
and it was already 6:30 p.m., thus presumably dusk. Malacat and his Consented Searches
companions were merely standing at the corner and were not creating
any commotion or trouble. Third, there was at all no ground, probable
Where the person to be searched acquiesces in the search of
or otherwise, to believe that Malacat was armed with a deadly weapon.
None was visible to Yu, for as he admitted, the alleged grenade was his person or property, then obviously no warrant need be
“discovered”   “inside   the   front   waistline”   of   Malacat,   and   from   all   procured. He in effect waives his right to otherwise have a
indications as to the distance between Yu and Malacat, any telltale warrant justify the invasion of his liberty and privacy
bulge, assuming that Malacat was indeed hiding a grenade, could not
have been visible to Yu. What is unequivocal then are blatant violations It   should   not   be   presumed   from   a   person’s   silence   that   he  
of   Malacat’s   rights   solemnly   guaranteed   in   Sections   2   and   12(1)   of   waived the illegality of a search
Article III of the Constitution.
Such a passive conformity given under coercive or intimidating
Exigent and Emergency Circumstances circumstances is considered no consent at all within the
purview of the constitutional guarantee
The Court justified the warrantless search by reason of the
“urgency  and  exigency  of  the  moment” Consent must be made voluntarily, knowingly and intelligently
People v. De Gracia
Requisites:
233 SCRA 716 (1994)
1. It must appear first that the right exists
FACTS: The incidents involved in this case took place at the height of 2. The person involved had knowledge, actual or constructive,
the  coup  d’état  staged  in  December,  1989.  Accused-appellant Rolando of the existence of such right
de Gracia was charged in two separate informations for illegal 3. Said person had an actual intention to relinquish the right
possession of ammunition and explosives in furtherance of rebellion,
and for attempted homicide. Appellant was convicted for illegal Consent to a search is not to be lightly inferred but must be
possession of firearms in furtherance of rebellion, but was acquitted of shown by clear and convincing evidence
attempted homicide. Surveillance was undertaken by the military along
EDSA because of intelligence reports & nbsp; &n bsp; about a coup.
Members of the team were engaged by rebels in gunfire killing one
Relevant to this determination are the following characteristics
member of the team. A searching team raided the Eurocar Sales Office. of the person giving consent and the environment In which
They were able to find and confiscate six cartons of M-16 ammunition, consent is given:
five bundles of C-4 dynamites, M-shells of different calibers, and 1. The age of the defendant
"molotov" bombs inside one of the rooms belonging to a certain Col. 2. Whether he was in public or secluded location
Matillano. De Gracia was seen inside the office of Col. Matillano, 3. Whether he objected to the search of passively looked on
holding a C-4 and suspiciously peeping through a door. The team 4. The education and intelligence of the defendant
arrested appellant. They were then made to sign an inventory, written
5. The presence of coercive police procedures
in Tagalog, of the explosives and ammunition confiscated by the raiding

25 | P LATON
6.  The  defendant’s  belief  that  no  incriminating  evidence  will  be   online tickets(Sec.9 RA 6235) that they are subject to search and, if any
found prohibited materials or substances are found, such would be subject to
7. The nature of the police questioning seizure. These announcements place passengers on notice that
ordinary constitutional protections against warrantless searches and
8. The environment in which the questioning took place
seizures do NOT apply to routine airport passengers.
9. The possibly vulnerable subjective state of the person
consenting
The Terry search   or   the   “stop   and   frisk”   situation   refers   to   a  
case where a police officer approaches a person who is acting
It is the State which has the burden of proving, by clear and
suspiciously, for the purpose of investigating possibly criminal
positive testimony, that the necessary consent was obtained
behavior in line with the general interest of effective crime
and that it was freely and voluntarily given
prevention and detection. To assure himself that the person
with whim he is dealing is not armed with a weapon that could
United States v. Drayton
536 U.S. 194, 153 L Ed 2d 242, 122 S Ct 2105 (2002) unexpectedly and fatally be used against him, he could validly
conduct a carefully limited search of the outer clothing of such
At a scheduled stop, police officers board the bus as part of a routine person to discover weapons which might be used to assault him
drug and weapons interdiction effort. Lang approached the
respondents – Drayton and Brown – who were seated together, he Miscellaneous Searches and Seizures
declared that the police were looking for drugs and weapons and asked
if   the   respondents  had  any   bags.  Lang   searched  the   bag   with  Brown’s   Extends to non-tangible things and other forms of searches and
consent, the bag revealed no contraband. The officer noticed that both
seizures involving alcohol and drug testing, for instance, or
respondents were wearing heavy jackets and baggy pants despite the
warm weather. He asked Brown whether he minded if Lang checked his transportation and communication, as well as in electronically
person. Brown agreed, and a pat-down revealed hard objects similar to enhanced intrusions
drug packages in both thigh areas. Brown was arrested. Same thing
happened to Drayton. A further search revealed that respondents had Alih v. Castro
taped cocaine between their shorts. 151 SCRA 279 (1987)

Respondents moved to suppress the cocaine on the ground that the In 1984, two hundred (200) Philippine marines and elements of the
pat-down searches was invalid. Lang did not inform the respondents home defense forces conducted   “zona”7. The initial reaction of the
(passengers in that bus) of their right to refuse to cooperate. people in the compound was to resist invasion with a burst of gunfire –
no one was hurt; intended to warn the intruders. Unfortunately, the
Although Officer Lang did   not   inform   respondents’   right   to   refuse   the   situation aggravated. The soldiers returned fire and a bloody shoout-
search, he did request permission to search, and the totality of the out followed, resulting to a number of casualties. The besieged
circumstances indicates that their consent was voluntary, so the compound surrendered the next morning, sixteen (16) male occupants
searches were reasonable. The Court has rejected in specific terms the were arrested, finger-printed, paraffin-tested, and photographed over
suggestion that police officers must always inform citizens of their right their objection. The military also inventoried and confiscated nine M16
to refuse when seeking permission to conduct a warrantless consent rifles, an M14 rifle, and nine rifle grenades, and several round of
search. While knowledge of the right to refuse consent is one factor to ammunitions found in the premises.
be taken into account, the government need not establish such
knowledge as sine qua non of an effective consent. The precarious state of Zamboanga City at the time in question
certainly did not excuse the non-observance of the constitutional
Airport Searches guaranty against unreasonable searches and seizures. The record does
not disclose that the petitioners were wanted criminals or fugitives
from justice. At the time of the zona they were merely suspected of the
Correlated to the lessened expectation of privacy which a mayor’s  slaying  and  had  not  in  fact  even  been  investigated for it. Search
passenger must necessarily have to recognize and accept, part of   petitioners’   premises   is   declared   illegal   and   all   articles   seized   as   a  
of the price for traveling in a mode of transportation that has result thereof are inadmissible evidence against the petitioners in any
special concerns for safety and security proceedings. However, said articles shall remain in custodia legis
pending the determination of legality of such.
People v. Johnson
348 SCRA 526 (2000) Guazon v. De Villa
181 SCRA 623 (1990)
Leila Johnson was about to fly back to USA. At NAIA departure area,
Olivia Ramirez, the lady in charge of frisking deporting passengers Petitioners, claiming to be bona fide residents of Metro Manila and
frisked Johnson.   Ramirez   felt   something   hard   on   Johnson’s   abdominal   taxpayers and leaders in their respective communities, seek to prohibit
area. Upon inquiry, Johnson explained she needed to wear girdle as she the military and police officers represented by public respondents from
had undergone an operation. Not satisfied with the explanation and conducting   “Areal   Target   Zonings”   or   “Saturation   Drives”   in   Metro  
with  the   consent   of   her   superior,   took   Johnson  to   the   lady’s  room for Manila. Saturation Drives were conducted in 1987in several critical
inspection. Ramirez asked her to bring out the thing under her girdle. areas pinpointed by police and military as places where the subversives
Johnson brought out three plastic bags, which when examined turned were hiding. The petitioners claim that the saturation drives follow a
out to be methamphetamine hydrochloride (shabu). She was taken to common pattern of human rights abuses.
the to the Security office where her passport and ticket were taken.
The petition is REMANDED to the RTCs of Manila, Malabon, and Pasay.
She questions the legality of the warrantless search conducted at her The remedy is not an original action for prohibition brought through a
person. taxpayers’  suit.  (1)No  proper  parties. Where one victim complains and
not one violator is properly charged, the problem is not initially for SC.
Persons may lose the protection of the search and seizure clause by It is basically one for the executive departments and for trial courts. (2)
exposure of their persons or property to the public in a manner There is no proof. A method of pinpointing human rights abuses and
reflecting a lack of subjective expectation of privacy, which expectation identifying violators is necessary.
society is prepared to recognize as reasonable. Such recognition is
implicit in airport security procedures. Travellers are often notified 7Zona- military operation – raid of a compound in search of loose firearms,
through airport public address systems, signs, and notices in their ammunition and other explosives.
26 | P LATON
In the meantime and in the prima facie showing that some abuses were In our view, requiring an employer to obtain warrant whenever
probably committed and could be committed during future police the  employer  wishes  to  enter  an  employee’s  office,  desk,  or  file  
actions, banging on walls, kicking in of doors, violation of residences,
cabinets for a work-related purpose would seriously disrupt the
etc. are temporarily restrained.
routine conduct of business and would be unduly burdensome
Oliver v. United States
Camara v. Municipal Court Of The City And Country Of San Francisco
466 US 170 (1984)
387 US 523 (1967)
Whether the open fields doctrine permits police officers to enter and
Appellant was charged with violating san Francisco housing code for
search marijuana fields without warrant where the fields are secluded
refusing after 3 efforts by inspectors to secure his consent, to allow
and  contain  “no  trespassing”  signs.
warrantless inspection of the ground floor quarters which he leased
and  whose   residential   used  allegedly   violated   the   apartment’s  permit.    
Reports that marijuana was being raised on the farm of oliver, two
He argued that the sec503 of housing code is contrary to the 4th and
narcotics agents went to the farm to investigate. They entered the farm
14th amendments in that it authorizes municipal officials to enter
gate that has no trespassing sign and found a field of marijuana 1 mile
private dwelling without a search warrant and without probable cause
away from his house.
to believe that a violation of the code exist.
SC: as sated in Hester vs US, the government’s   intrusion   upon   open  
Administrative searches are significant intrusions upon the interest
fields   is   not   one   of   those   “unreasonable”   searches   proscribed   in   the  
protected by the 4th amendment that such searches when authorized
fourth amendment. The amendment does not protect the merely
and conducted without a warrant; lack the traditional safeguards which
subjective expectation of privacy but only those expectations that
the 4th amendment guarantees to the individual.
society is prepare to recognize as reasonable. That an individual may
not legitimately demand privacy for activities conducted out of doors in
The warrant procedure is designed to guarantee that decision to search
fields, except in the area immediately surrounding the home. An
private property is justified by a reasonable governmental interest. The
individual has no legitimate expectation that open fields will be free
ultimate standard is the reasonableness.
from warrantless intrusion by the government.
Area   inspection   is   “reasonable   search   of   private   property   within   the  
Oliver : circumstances may indicate reasonable expectations of privacy
meaning of the 4th amendment. It is obvious  that  “probable  cause”   to  
were violated, it should be decided case to case basis.
issue warrant to inspect must exist if reasonable legislative or
Sc: no. it will make it difficult for the policeman to discern the scope of
administrative standards are satisfied.
his authority; it also creates a danger that constitutional rights will be
arbitrarily and inequitably enforced.
However, there was no emergency demanding immediate access; they
in fact made 3 trips to obtain consent. Yet no warrant was obtained and
“no  trespassing”  sign  and    fences  do  not  effectively  bar  the  public  from  
thus the appellant was unable t verify either the need for or
viewing open fields and do not demonstrate that the expectation of
appropriate limits of inspection... Appellant has constitutional right to
privacy was legitimate. The test is whether the intrusion of the
insist them to obtain a warrant to search.
government infringes upon the personal and societal values protected
by the fourth amendment.sc finds no basis for concluding that a police
inspection of open fields accomplishes such infringement.
Alcohol, Drugs and Related Tests
People v. Valdez
341 SCRA 25 (2000) Requiring a person to submit urine or blood, or to undergo
breathalyzer testing for the purpose of determining whether he
A tip was given about a plantation of marijuana allegedly owned by is under the influence of alcohol or drugs are considered a
valdez. The police were instructed to uproot the plants and arrest species of search that is governed by the constitutional
cultivator. They found him in a nipa hut then looked around and saw proscription against unreasonable searches and seizures. This
marihuana plant He admitted ownership but later on alleged that he
means, generally, that such a search may only be had pursuant
only admitted ownership out of fear.
General rule: search and seizure nust be carried on a judicial warrant.
to a probable cause, or an individualized suspicion
Otherwise, it is unreasonable.
They first located the marijuana before appellant was arrested without Canine  “Sniff  Test”
a warrant. There was no valid warrantless arrest which preceeded the
search   of   appellant’s   premises.   The seizure of evidence in plain view While this may also implicate the privacy interest of the owner,
applies only where the police officer is not searching for evidence this is deemed as not approaching constitutional dimensions so
against the accused, but inadvertently comes across an incriminating as to rule out the practice
object. The marijuana plants were not in plain view because further
search was needed. There was illegal search and seizure thus the plant
cannot b admitted as evidence against him. The confession of
“A   sniff   by   a   dog   that   simply   walks   around   a   car   is   much   less  
ownership without a counsel is also violative of the bill of rights. intrusive  than  a  typical  search”

Administrative and Other Searches The  “Special  Needs”  Exception

Searches and seizures by government employers or supervisors Under this exception, the Court upheld an extended
of the private property of their employees are subject to the warrantless search   of   a   student’s   purse   after   a   report   of  
restraints of the Fourth Amendment smoking   in   the   school   lavatory   and   following   the   student’s  
denial of said report
Public  employees’  expectations  of  privacy  in  their  offices,  desks,  
and file cabinets, like similar expectations of employees in Our precedents establish that the proffered special need for
private sector, may be reduced by virtue of actual office drug testing must be substantial--important enough to override
practices and procedures, or by legitimate regulation the individual’s  acknowledged  privacy  interest,  sufficiently  vital  

27 | P LATON
to   suppress   the   Fourth   Amendment’s   normal   requirement   of  
individualized suspicion Umil v. Ramos
187 SCRA 311 (1990)
Warrantless Arrest
Dural, a member of the NPA liquidation squad responsible for killing 2
soldiers on Jan. 31, 1988, was arrested on Feb. 1, while being treated
These arrests without warrant are justified by the fact that the for a gunshot in a hospital. Dural was arrested for being a member of
person to be arrested is caught in the act (in flagrante delicto), the NPA, an outlawed subversive organization. Subversion as a
is apprehended pursuant to a hot pursuit, or is an escaped continuing offense, the arrest without warrant is justified.
prisoner
The case filed involves subversion and illegal possession of firearm and
In Flagrante Delicto ammunition. On Aug. 17, 1988, the lower court found them guilty of
the charge.
Under this exception, the arrest is justified by the very fact that
the crime is committed or is about to be committed in the very Resolution on the Motion for Reconsideration
202 SCRA 292 (1991)
presence of the person making the arrest
The   court’s   decision   on   July   9,   1990   rules   that   the   arrest   of   Dural  
There is no more need for a warrant as the culprit is caught red- without warrant is justified because under Section 5 (a) Rule 113, Dural
handed was committing an offense, when arrested, because he was a member
of the NPA.
“Reliable   information”   alone   is   not   sufficient   to   justify   a   Sec 5 (b), Rule 113 of the Rules of Court requires 2 conditions for a valid
warrantless arrest under Section 5(a), Rule 113. The rule arrest without warrant:
requires, in addition, that the accused perform such overt act 1) That the person arrested had just committed an offense
2) That the arresting officer or the private person has personal
that would indicate that he has committed, is actually
knowledge of facts (based on probable cause)
committing, or is attempting to commit an offense
Accordingly, the motion for reconsideration of the decision dated July 9
Elements: is denied.
1. The person to be arrested must execute an overt act
indicating he has just committed, is actually committing, or is People v. Aminnudin
attempting to commit a crime 163 SCRA 40 (1988)
2. Such overt act is done in the presence or within the view of
the arresting officer Aminnudin was arrested after disembarking from M/V Wilcon. Police
officers received a tip that he was carrying marijuana, and conducted a
warrantless search and arrest. The only justification was the tip
People v. Burgos
received 2 days before the arrest.
144 SCRA 1 (1986)
The accused-appellant was not caught inflagrante nor was a crime to be
committed or had just been committed to justify the warrantless arrest
Cesar masalmok personally and voluntarily surrendered to the
under Rule 113 of the Rules of Court.
authorities and gave intelligence information that he was forcibly
recruited by Burgos using a firearm. They found the accused plowing his
Without the evidence of marijuana allegedly seized from Aminnudin,
field, he denied possessing firearm. His wife pointed where the gun was
the case of prosecution shall fall. The evidence cannot be admitted for
then the accused showed subversive documents that were allegedly
the simple fact that the marijuana was seized illegally.
issued to him by a team leader of NPA. He was convicted of the crime
of illegal possession of firearm in furtherance of subversion.
State of Rebellion
In the constitutional provision against wanton and unreasonable
invasion of privacy and liberty of a citizen, his person, property papers In quelling or suppressing the rebellion, the authorities may
and   effects,   What   is   sought   to   be   guarded   is   a   man’s   prerogative   to   only resort to warrantless arrests of persons suspected of
choose who is allowed entry to his residence. rebellion, as provided under Section 5, Rule 113 of the Rules of
Court, if the circumstances so warrant
Under rule112, the officer arresting a person who has just committed,
is committing or is about to commit an offense must have a personal
knowledge of that fact. There is no personal knowledge in this case, it Sanlakas v. Executive Secretary
came from masalmok and the location of gun was given by the wife. At 421 SCRA 656 (2004)
the time of his arrest, he was not in actual possession of the firearm
and the documents. Neither was he committing any act subversive. On July 27, 2003, some 300 junior officers and enlisted men of the AFP,
armed with ammunitions and explosives stormed into the Oakwood
The right of a person to be secure against unreasonable seizure of his Apartments in Makati City. Bewailing the corruption in the AFP, the
body and any deprivation of liberty is most basic and fundamental. The soldiers demanded for the resignation of high officials, including the
statute granting exceptions should be strictly construed. President. The President then issued Proclamation No. 427 and General
Order   No.   4   declaring   a   “state   of   rebellion”   and   calling   out   the   AF   to  
In arrest without a warrant, it is not enough that there is a reasonable suppress the rebellion. On Aug. 1, 2003, Proclamation No. 435 lifted the
ground to believe that the person to be arrested has committed a declaration of the state of rebellion. Several petitions were filed
crime. A crime must in fact or actually have been committed first. The challenging   the   validity   of   the   Proclamation,   the   President’s   authority  
fact of commission must be undisputed. The test of reasonable ground and the consequence of such declaration, specially in relation to the
applies only to the identity of perpetrator. arrest of those implicated in the rebellion.

No compelling reason for haste and not secure warrant of arrest. Arrest The mere declaration of the state of rebellion cannot diminish or
of the accused while he was plowing is illegal. The arrest was unlawful, violate constitutionally protected rights. Simple declaration of the state
the search and seizure is likewise not legal since these are mere of rebellion does not suspend the operation of the constitution or
incidents of a valid arrest. automatically suspend the privilege of writ of habeas corpus.

28 | P LATON
with the Prosecutor, the latter should have immediately scheduled a
Hot Pursuit preliminary investigation to determine whether there was a probable
cause for charging petitioner. Since petitioner had not been arrested,
with or without warrant, he was entitled to be released immediately
When a crime has just been committed, the law enforcers
and concerned only to his appearance in the preliminary investigation.
ordinarily would have to try to get the culprit as soon as
possible before he eludes them
Posadas v. Ombudsman
341 SCRA 388 (2000)
They may be in possession of enough information or knowledge
by which to identify their quarry and make the arrest before FACTS: Dennis Venturina, a member of Sigma Rho at UP was killed in a
their job would become more difficult through the passage of rumble between his fraternity & another on Dec. 8, 1994. Roger
time Posadas, petitioner, and the chancellor of UP Diliman, asked the NBI for
the identification & apprehension of the suspects. Respondent Dizon,
Go v. CA the Chief of the Special operations group of the NBI and his men went
206 SCRA 138 (1992) to UP and upon the basis of the identification by 2 eyewitnesses,
attempted to arrest Taparan and Narag who were members of the
FACTS: Rolito Go, petitioner, was driving in the opposite direction along other fraternity, as suspects but they just came to the UP police station
a one-way street in San Juan Metro Manila, when he nearly bumped for a peace talk between their fraternities. Posadas objected because
into  Eldon  Maguan’s  car.  Petitioner  got  out  of  his  car,  shot  Maguan,  and   the NBI agents do not have warrants of arrest and he & his lawyer
left. A security guard nearby was able to take down the plate number of promised to take the 2 to the NBI office the next day. However, the
Go’s   car   then   the   police   came   at   the   crime   scene.   After obtaining next day, they were not surrendered & have apparently escaped. Dizon
information from (1) the verification of LTO that the car was registered filed to the Office of the Prosecutor a complaint charging Petitioner for
under  the  petitioner’s  wife’s  name,  (2)  the  impression  of  the  credit  card   violating PD 1829 which states that obstruction of the apprehension &
used by him in the bakeshop where he went before the incident prosecution of criminal offenders is unlawful. Prosecutor
happened, and (3) the positive identification of the guard therein, the recommended for dismissal but was disapproved and was ordered by
police launched a manhunt. 6 days after the incident, Go, with 2 the Ombudsman to proceed with the prosecution in Sandiganbayan.
lawyers, presented himself to the San Juan police station for
verification. He was then detained and the police filed a complaint for Issues: (1) WON the attempted warrantless arrest of the student
frustrated homicide in the Office of the Provincial Prosecutor of Rizal. suspects by the NBI could be validly made; (2) WON there was probable
The Prosecutor filed before the RTC, an information for murder instead cause for prosecuting petitioner for violation of PD 1829
of frustrated homicide, since Maguan died after a few days.
Nevertheless, petitioner was allowed to bail. The RTC judge (1) recalled HELD:
the bail, and gave petitioner 48 hrs from receipt of the Order to (1) Negative.
surrender, (2) recalled and cancelled the Order which granted the leave Art.3 Sec.2 of the Constitution:
of the Prosecutor to conduct preliminary investigation, and (3) treated No arrest may be made except in the case of a warrant issued by a
as   petition   for   bail   the   petitioner’s   motion   for immediate release and judge after examining the complainant and the witnesses he may
preliminary investigation and set it for hearing. produce after finding probable cause to believe that the person to be
arrested has committed the crime. The case does not fall within the
Petitioner then filed for a petition for certiorari, prohibition and exceptions provided in Rule 113 Sec. 5 of the Rules of Criminal
mandamus before the SC, contending that the information was null and Procedure since neither the arresting officers witnessed the crime
void because no preliminary investigation has been previously being committed nor the students are fugitives from justice or
conducted. The SC remanded the petition to the CA wherein petitioner prisoners who had escaped from confinement.
was found not guilty since he refused to enter to a plea. The CA
dismissed  the  petitions  and  held  that  Go’s  warrantless  arrest  was  valid   The respondents invoked the ruling in People vs. Tonog,Jr wherein the
because   the   (1)   offense   was  ‘’freshly   committed’’,   (2)   his   identity   was   accused therein voluntarily went with the police after being invited and
established through investigation, (3) when he showed up, there was the arresting officer found bloodstains on the pants of the accused
an existing manhunt and (4) there were witnesses. which made the former conclude that the latter is the suspect and the
arrest was also made on the day when the crime was committed. Thus,
ISSUE: WON a lawful warrantless arrest had been effected by the San Tonog case is not applicable in the case at bar since the NBI agents
Juan police in respect of petitioner tried to arrest Taparan & Narag 4 day after the commission of the
crime, they had no personal knowledge about the suspects to indicate
HELD: The reliance of the petitioner and the Solicitor General in Umil v. that they are guilty and they had obtained the information merely from
Ramos, is misplaced since in the said case, it was held that warrantless eyewitnesses, which is insufficient to justify a warrantless arrest.
arrests made from 1- 14 days after the actual commission of the crime Their attempt to arrest Taparan and Narag without a warrant was
is legal in as much as such crime is a continuing crime. In the case at illegal for their failure to comply with constitutional and procedural
bar, the crime committed was that of murder and cannot be considered requirements.
as a continuing crime since it was commenced and completed at once.
(2) As petitioners are also being prosecuted under PD 1829, it is a rule
The warrantless arrest in this case does not fall under Sec. 5 of Rule 113 that a criminal prosecution cannot be enjoined but it has been held that
of the 1985 Rules on Criminal Procedure. Since the police arrested respect for the citizen's right to be free from arbitrary arrest and
petitioner 6 days after the shooting incident, it is apparent that the punishment and unwarranted and vexation prosecution is more impt.
‘’arresting   officers’’   were   not   present   during   such   incident   and   than criminal procedure. As held in Venus vs. Desierto that the Court
therefore   cannot   be   also   regarded   as   one   which   ‘’…had   just   been   does not interfere with the discretion of the Ombudsman in the
committed’’.   determination of the existence of a reasonable ground to believe that a
crime has been committed. The exceptions as stated in Brocka vs.
Likewise,   the   said   officers   do   not   have   ‘’personal   knowledge’’   of   the   Enrile are as follows: (a)to afford protection to the constitutional rights
facts indicating that petitioner was the gunman. The information of the accused, (b) when necessary for the orderly administration of
derived   from   eyewitnesses   did   not   constitute   ‘’personal   knowledge’’.   justice or to avoid oppression or multiplicity of actions, (c) when there
Thus, there was no lawful warrantless arrest. is a prejudicial question which is sub judice, (d)where the acts of the
officer are without or in excess of authority, (e) where the prosecution
Petitioner was not arrested at all. He walked in, with 2 lawyers, and is under an invalid law, ordinance or regulation, (f)when double
placed himself at the police disposal without stating the he is jeopardy is clearly apparent, (g) where the court has no jurisdiction
surrendering. When the police filed complaint for frustrated homicide over the offense, (h) where it is a case of persecution rather than

29 | P LATON
prosecution, (i) where the charges are manifestly false and motivated can be ordered only in the event that the prosecutor files the
by the lust for vengeance, (j) where there is clearly no prima facie case case and the Judge of the Regional Trial Court finds probable
against the accused and a motion to quash on that ground has been cause  for  the  issuance  of  the  warrant  of  arrest”
denied, and (k) preliminary injunction has been issued by the SC to
prevent the threatened unlawful arrest of the petitioners.
Lim, Sr. v. Felix
194 SCRA 292 (1991)
In the case at bar, PD 1829 was not violated since petitioner had a right
to prevent the arrest bec. it was illegal.
FACTS: Vicente Lim, petitioner, was one of those who were charged
with multiple murder with frustrated murder in connection with the
It was found out however by the Office of the Ombudsman that the
ambush of Masbate Congressman Espinosa and his bodyguards (only
intervention by the petitioners allowed the escape of Taparan and
one survived) in the domestic airport of the said province Preliminary
Narag. However, the student suspect, a certain Joel Carlo Denosta, was
investigation was conducted and the RTC judge was able to find a
not one of those who were attempted to be arrested by the NBI.
probable cause for the issuance of the warrant of arrest. The Fiscal
ruled that the crime of the suspects must be murder for each of the 4
The NBI agents are at fault bec. they were unable to arrest Taparan and
victims killed and physical injuries for the survivor. The Fiscal however
Narag. If they believed the information given to them, they should have
filed 4 separate Informations for murder against the 12 accused, with
applied first for a warrant before attempting to arrest.
no bail. As the petition for change of venue by Lim was granted by the
SC, the cases were transferred to Judge Felix of Makati. Petitioner filed
Sanchez v. Demetriou a motion and manifestation for the transmittal of initial records of
227 SCRA 627 (1993) preliminary investigation for the best enlightenment of the court in its
determination of the existence of a probable cause based on the
FACTS: Petitioner Sanchez, mayor of Calauan, Laguna, and other 6 Constitutional   mandate   that   “no   warrant   shall   issue   unless   the   issuing  
people were accused of rape with homicide. Charges were filed against magistrate   have   been   personally   convinced   of   such   probable   cause’’  
them in connection with the rape-slay of Mary Eileen Sarmenta and the but it was opposed by the prosecution & denied by the respondent
killing of Allan Gomez. Preliminary investigation was conducted as court. It later issued warrants of arrest against the petitioner and the
petitioner was represented by his counsel. The PNP sent petitioner an others who were accused.
invitation requesting him to appear for an investigation. When he was
taken to the camp, he was positively identified by 2 witnesses. He was ISSUE: WON a judge may issue a warrant of arrest without bail by
later placed on "arrest status" and taken to DOJ in Manila. An inquest simply relying on the prosecution's certification & recommendation
was conducted upon his arrival and a warrant of arrest was issued after that a probable cause exists
the hearing. He remained confined in Camp Crame while information
charges were filed with the others who were accused. A warrant arrest HELD: A  judge  may  rely  on  the   fiscal’s  certification  of   the   existence  of  
was then issued and the SC ordered the transfer of such case to Pasig probable cause and issue a warrant of arrest. However, such
City, M. Manila. Petitioner filed motion to quash the information since certification does not bind the judge to come out with the warrant of
his warrantless arrest was illegal and the court has no jurisdiction over arrest (Placer vs. Villanueva 126 SCRA 463 [1983]).
him. However, Judge Demetriou denied the petition.
The judge must have a personal determination of the existence of a
Issues:  (1)  WON  petitioner’s  warrantless  arrest  was  illegal;  (2)  WON  the   probable cause for a warrant of arrest to be issued, but it does not
court has no jurisdiction over him necessarily mean that he must personally examine the complaint
(Soliven vs. Makasiar 167 SCRA 393 [1988].
HELD: (1) The warrantless arrest was illegal. The arresting officers were
not present during the commission of the alleged crime and they have The determination of probable cause is a function of the judge.
no personal knowledge that petitioner is responsible because their Preliminary investigation is done by the prosecutor and does not bind
basis was the statement by the witnesses. However, (2) the RTC has the judge. Also, there must be distinction between (1) the preliminary
jurisdiction over him because it issued a warrant arrest against him and inquiry which determines the probable cause for the issuance of the
the others. It was delayed, but legal though. warrant of arrest and (2) the preliminary investigation which ascertains
if the offender should be held for trial or be released (People v.
Probable Cause for Issuance of Warrants of Arrest and Role of Honorable Enrique Inting GR No. 88919, July 25 1990).
Judges
RTC judges no longer have authority to conduct preliminary
What is required is that the judge must have sufficient investigations (Castillo v. Villaluz 171 SCRA 39 [1989]).
supporting documents upon which to make his independent
The judge may rely on the COMELEC's resolution to file for the
judgment, or at the very least, upon which to verify the findings information in the same way that he may rely on the Prosecutor's
of the prosecutor as to the existence of probable cause certification (People v Delgado GR Nos. 93419-32, Sept. 18, 1990.

If the judge disagrees, or finds the evidence insufficient, *The constitutional mandate has not been satisfied and the judge
contrary to the conclusions of the prosecutor, the judge should committed a grave abuse of discretion for relying solely on the
not dismiss the case but instead require the fiscal to present Prosecutor's certification where all the records of investigation are in
additional evidence to show probable cause Masbate. He has not personally determined the probable cause but it
was the Provincial Prosecutor who had done such.
The Court explained that the issuance of a warrant is not a The extent of reliance depends on the circumstances of each case and
mere ministerial function. It calls for the exercise of judicial subject to the sound discretion of the judge. But when he issues a
discretion on the part of the issuing magistrate warrant of arrest without evidence before him, he abuses such
discretion.
“While  before,  it  was  mandatory  for  the  investigating  Judge  to  
issue a warrant for the arrest of the accused if he found Webb v. De Leon
probable   cause,   the   rule   now   is   that   the   investigating   Judge’s   247 SCRA 652 (1995)
power to order the arrest of the accused is limited to instances
in  which  there  is  a  necessity  for  placing  him  in  custody  ‘in  order   FACTS: The National Bureau of Investigation (NBI) filed with the
Department of Justice a letter-complaint charging petitioners Hubert
not  to  frustrate  the  ends  of  justice.’  The  arrest  of  the  accused  

30 | P LATON
Webb, Michael Gatchalian, Antonio Lejano and six (6) other persons, HELD:   Respondent’s   issuance   of   Warrant   of   Arrest   was   in   violation   of  
with the crime of Rape with Homicide. Meanwhile, petitioner Webb the constitutional requirement of personal determination as to the
claimed during the preliminary investigation that he did not commit the existence of probable cause.
crime as he went to the United States on March 1, 1991 and returned
to the Philippines on October 27, 1992. Thereafter, the DOJ Panel REASON: “No  warrant  of  arrest  shall  issue  except  upon  probable  cause  
issued a 26-page   Resolution   “finding   probable   cause   to   hold   to be determined personally by the judge after examination under oath
respondents  for  trial”  and  recommending  that  an  Information  for  rape   or affirmation of the complainant and the witnesses he may produce,
with homicide be filed against petitioners and their co-respondents. It and  particularly  describing  xxx  the  persons  xxx  to  be  seized.”  (Section 2,
then filed the corresponding Information against petitioners and their Article III 1987 Constitution)
co-accused with the Regional Trial Court. Respondent judge issued
warrants of arrest. Petitioner Webb voluntarily surrendered to police The   phrase   “personal   determination”   emphasizes   the   EXCLUSIVE  
authorities. Petitioners Gatchalian and Lejano likewise gave themselves and PERSONAL RESPONSIBILTY of the issuing judge to satisfy himself as
up to the authorities after filing their petitions before the Supreme to the existence of probable cause.
Court. The Warrant of Arrest issues not on the strength of the certification
standing alone but because of the records that sustain it. In the case at
ISSUE:   “Whether   or   not   the   warrants   of   arrest   issued   by   respondent   bench,   there   was   not   even   a   prosecutor’s   certification   to   rely   upon  
Judge Raul de Leon and later, respondent Judge Amelita Tolentino met since no information had even been filed yet in court.
the constitutional requirement of probable  cause”. Respondent cannot exculpate himself from administrative liability by
contending that the mistake was entirely attributable to the Criminal
HELD: Docket   clerk   who   failed   to   faithfully   comply   with   her   ‘duty”   of   going  
The Constitution, the Rules of Court, and our case law over the records of the criminal case and ensuring first that an
repudiates the submission of petitioners that respondent information had already been filed in court before preparing the
judges   should   have   conducted   “searching   examination   of   warrant of arrest.
witnesses”  before  issuing  warrants  of  arrest  against  them.
The Court   also   rejects   the   petitioner’s   contention   that   a   * Options available to the judge upon personal determination of
judge must first issue an order of arrest before issuing a probable cause:
warrant of arrest. There is no law or rule requiring the (1) Personally evaluate the report and the supporting documents
issuance of an Order of Arrest prior to a warrant of arrest. submitted by the prosecutor regarding the existence of probable cause,
DOJ   Panel’s   26-page report, testimonies of witnesses and and on the basis thereof, issue a warrant of arrest.
counter- affidavits of petitioners satisfied both respondent (2) If on the basis thereof he finds no probable cause, disregard the
judges that there is probable cause in issuing said warrants prosecutor’s  report  and  require  the  submission  of  supporting  affidavits  
of arrest. of witnesses to aid him in determining its existence.
Before issuing warrants of arrest, judges merely determine
personally the PROBABILITY, NOT THE CERTAINTY OF GUILT Administrative Warrants
of the accused. They just personally review the initial
determination of the prosecutor finding a probable cause to
The Constitution is explicit that it is only a judge who can issue
see if it is supported by substantial evidence.
warrants
In search cases:
(1) Items sought are in fact seizable by virtue of their being The   1973   Charter   allowed   such   other   “responsible   officer   as  
connected with criminal activity. may  be  authorized  by  law”  to  determine  probable  cause
(2) The items will be found in the place to be searched.
Qua Chee Gan v. Deportation Board
In arrest cases: 9 SCRA 27 (1963)
(1) There must be probable cause that a crime has been
committed. FACTS: Petitioners were charged before the Deportation Board of
(2) The person to be arrested committed it. having purchased US Dollars in the total amount of $130,000.00
without the necessary license from the Central Bank of the Philippines,
*”Upon   filing   of   an   information   the   Regional Trial Court may issue a and of clandestinely remitting the same to Hong Kong. A warrant of
warrant  for  the  arrest  of  the  accused.” arrest of said aliens was issued by the presiding member of the
Deportation Board. Petitioners filed a motion to dismiss the charges
Talingdan v. Eduarte against them in the Deportation Board on the grounds of lack of
366 SCRA 559 (2001) jurisdiction and that the charges do not constitute legal basis for
deportation. The lower court held that the Board has the power to
FACTS: Petitioner, a private practitioner, charged respondent Judge issue warrants of arrest and fix the amount of the bond for the
Eduarte, with improvidently issuing a warrant of arrest in a criminal temporary release of the alien.
case for libel without the requisite preliminary investigation being first
conducted by the Office of the Public Prosecutor. He alleged that ISSUE:
sometime in April 2000, elements of PNP stormed into his residence to (1) “Whether   or   not   the   President   has   the   power   to   deport  
arrest him and his client on the strength of a Warrant of Arrest issued aliens and consequently, the validity of delegation to the
by respondent Judge. Complainant then filed a Very urgent Motion to Deportation  Board  of  the  ancillary  power  to  investigate”
Quash and/or Set Aside Warrant  of  Arrest  and  Direct  Prosecutor’s  Office  
to Conduct Preliminary Examination since they had not been previously (2) “Whether   or   not   the   President’s   power   to   conduct  
notified of the charge against them and no preliminary investigation investigation carries with it the power to order the arrest of
was   ever   conducted   by   the   public   prosecutor’s   office   yet.   The   the  alien  complained  of”
respondent granted the motion and recalled the warrant of arrest,
admitting that he issued the same under the mistaken belief that a * Pertinent Laws:
preliminary investigation had already been conducted and an CA No. 613 (Immigration Act of 1940) – Commissioner of Immigration
information filed in court. Thus, when he saw the Warrant of Arrest, he was empowered to effect the arrest and expulsion of an alien, after
signed the same honestly thinking that the Criminal Docket Clerk had previous determination by the Board of Commissioners of the existence
faithfully complied first with her duty of going over the records of the of grounds therefore.
case. Section 69 of Act No. 2711 (Revised Administrative Code) – Lays down
the procedure to be observed should there be deportation

31 | P LATON
proceedings.
* Order of Deportation-The return to his country of an alien who has
HELD: broken the conditions upon which he could continue to reside within
(1) our borders.
The charges against the herein petitioners constitute in effect an act
of profiteering, hoarding or blackmarketing of US dollars –an economic Exclusionary Rule - The Fruit of the Poisonous Tree Doctrine
sabotage –which is a ground for deportation.
There   seems   to   be   no   doubt   that   the   President’s   power   of  
In the past it was held that sanctions against erring law
investigation may be delegated. This is clear from a reading of Section
69   of   the   Revised   Administrative   Code   which   provides   for   a   “prior   enforcers would be enough vindication of a violated right while
investigation, conducted by said Executive (the President) or his allowing the results of such an unreasonable search and seizure
authorized agent.” to be admissible in evidence
(2)
Section 69 of the Revised Administrative Code, upon whose A fruit of an illegal or unconstitutional act could not and should
authority   the   President’s   power   to   deport   is   predicated,   does   not   not be given any form of legitimacy by its admission in evidence
provide for the exercise of the power to arrest.
An implied grant of power, considering that no express authority
Along with the discarding of the old rule came the demise of
was granted by the law on the matter under discussion, that would
serve as a curtailment or limitation upon the fundamental right of a the so-called Silver Platter Doctrine which allowed federal
person, such as his security to life and liberty, must be viewed with judicial use of evidence seized in violation of the Constitution
caution. Then, a delegation of that implied power must be REJECTED as by state agents
inimical to the liberties of the people.
It is said that the exclusionary rule has three purposes:
*The Executive Order insofar as it empowers the Deportation Board to
issue warrant of arrest upon the filing of formal charges against an alien First, the rule is calculated to prevent, not repair. Its purpose is
or aliens and to fix bond and prescribe the conditions for the temporary
to deter--to compel respect for constitutional guaranty in the
release of said aliens, is declared ILLEGAL. The order of arrest issued by
the respondent Deportation Board is declared NULL AND VOID. only effective available way--by removing the incentive to
disregard it.
Harvey v. Defensor-Santiago
162 SCRA 840 (1988) Second, the   “imperative   of   judicial   integrity,”   i.e.,   that   the  
courts  do  not  become  “accomplices  in  the  willful  disobedience  
FACTS: Petitioners were apprehended from their respective residences of a Constitution they   are   sworn   to   uphold…by   permitting  
on February 27 1988 by agents of the Commission on Immigration and unhindered  governmental  use  of  the  fruits  of  such  invasions…A  
Deportation (CID) by virtue of Mission Orders issued by Commissioner ruling  admitting  evidence  in  a  criminal  trial…has  the  necessary  
Miriam Defensor-Santiago. Petitioners were among the twenty-two
effect of legitimizing the conduct which produced the evidence,
(22) suspected alien pedophiles who were rounded up after three
months of close surveillance by CID agents. Seized during the
while an application of the exclusionary rule withholds the
apprehension were photo negatives, pictures, posters and other constitutional  imprimatur”
literature advertising the child prostitutes. After being denied bail,
petitioners availed a petition for a Writ of Habeas Corpus. Third, that   “of   assuring   the   people--all potential victim of
unlawful government conduct--that the government would not
HELD: The petition is dismissed and the Writ of Habeas Corpus is profit from its lawless behavior, thus minimizing the risk of
denied. seriously  undermining  popular  trust  in  government”
REASON:
*Probable Cause –such facts and circumstances antecedent to the
No man is to be convicted on unconstitutional evidence
issuance of the warrant that in themselves are sufficient to induce a
cautious man to rely on them and act in pursuance thereof. Silverthorne Lumber Co., Inc. v. United States
In this case, the arrest of petitioners was based on probable cause 251 US 385, 64 L Ed 319, 40 S Ct 182 (1920)
determined after close surveillance for three (3) months during which
period their activities were monitored. The existence of probable cause Facts: Indictment was filed against Frederick W. Silverthorne and his
justified the seizure of the photo negatives, photographs and posters father, who were arrested at their home. Representatives of
without warrant. Those articles were seized as an incident to a lawful Department of Justice and the United States marshal without authority
arrest and therefore, admissible in evidence. go to the office of the company, made a clean sweep of all the books,
The requirement of probable cause, to be determined by a Judge papers and documents found and directed all employees to the office
does not extend to deportation proceedings. of the district attorney of US. Photographs and copies of material
What is essential is that there should be a specific charge against the papers were made, and a new indictment was framed based upon the
alien intended to be arrested and deported, that a fair hearing be knowledge thus obtained. The District Court ordered the return of the
conducted with the assistance of counsel, if desired, and that the originals, but impounded the photographs and copies. Subpoenas to
charge should be substantiated by competent evidence. produce the originals then were served, and on the refusal of the
In deportation proceedings, the right to bail is not a matter of right plaintiffs in error to produce them, the Court made an order that the
but a matter of discretion on the part of the Commissioner of subpoena should be compiled with. Contempt was filed against the
Immigration and Deportation. corporation and its owner
The power to deport aliens is an act of State, an act done by and
under the authority of the sovereign power. Issue: W/N there is an infringement of constitutional rights of the
Writ of habeas corpus will not be granted when the confinement is parties under the Fourth Amendment which constitutes indictment?
or has become legal, although such confinement was illegal from the
beginning. Ratio: Taken from the dissenting opinion of CJ Holmes: The protection
of the Constitution covers physical possession, but not any advantages
*Deportation proceedings are administrative in character. An order of that the government can gain over the object of its pursuit by doing a
deportation is never construed as a punishment. It is preventive, not a forbidden act.
penal process.

32 | P LATON
If knowledge of them is gained from an independent source they may encompassing protection extends against intrusions directly done both
be proved like any others, but the knowledge gained by the government and indirectly by private entities.
Government’s  own  wrong  cannot  be  used  by  it  in  the  way  proposed
GROH v. Ramirez
Additional Sanctions for Violations of the Guarantee 540 U.S. 552, 257 L Ed 2d 1068, 124 S Ct 1284 (2004)

In addition to the exclusionary rule as a means of deterrence, Facts: The petitioner, a Bureau of Alcohol, Tobacco and Firearms agent,
prepared and signed an application for a warrant to search the
the erring officers may also be subjected to criminal and civil
respondent’s   ranch   for   specified   weapons,   explosives   and   records  
liabilities for violating the constitutional proscription against which is based on the information of a concerned citizen. Application
unreasonable searches and seizures was supported by the   petitioner’s   affidavit   that   such   items   are   there  
together with a warrant form he has completed. The Magistrate (judge)
MHP Garments, Inc. v. Court of Appeals signed the warrant form even it did not identity any of the items that
236 SCRA 227 (1994) the   petitioner   intended   to   seize.   The   description   of   the   “person or
property”  described  respondents’  two  story  blue  house  rather  than  the  
Facts: MHP Garments, Inc was awarded to be the exclusive franchiser alleged stockpile of firearms. The petitioner led federal and local law
to sell and distribute official Boy Scouts uniforms, supplies, badges and enforcement officers to the ranch the next day but found no illegal
insignias by the Boy Scouts of the Philippines. MHP has been given the weapons or explosives, then left the copy of the warrant but not the
authority  to  “undertake  or  cause  to   be  taken  the  prosecution  in  court   application. Respondents sued petitioner in violation of the Fourth
of  all  illegal  sources  of  scout  uniforms  and  other  scouting  supplies.”  In   Amendment.
October 1983, petitioner received information that the private
respondents (Agnes Villa Cruz, Mirasol Lugatiman and Gertrudes Issue: (1) W/N the search violated the Fourth Amendment; (2) W/N is
Gonzales) are selling unauthorized Boy Scout items and paraphernalia. entitled to qualified immunity, given the Magistrate Judge, relying on
The petitioner, who was tasked to conduct surveillance and report, an affidavit that particularly described the items in question; found
together with two Philippine Constabulary officers (PC), went to the probable cause to conduct the search.
respondents store and   without   warrant,   seized   boy   and   girl   scouts’  
pants, dresses and suits which are on display. The respondents files a
criminal complaint for unfair competition, but was dismissed by the Ruling: Affirmed (decision of the Court of Appeals). Warrant is invalid
Provincial Fiscal of Rizal, then later order returned the seized items to
the respondent. But seized items were not immediately returned, thus Ratio:
private   respondents   personally   went   to   the   petitioner’s   place   of   *Warrant was plainly invalid.
business to recover the goods. Not all goods were returned and the *Fourth Amendment states: no Warrants shall issue, but upon the
items that were returned were of that inferior quality. probable cause, supported by Oath or affirmation, and particularly
describing the place to be search and the persons or things to be seized
Issue: W/N there is reasonable search and seizure even without *The warrant complied with 1st three of the requirements:
warrant -based on probable cause
-supported by sworn affidavit
Ruling: Search and seizure is illegal -particularly described the place of search
*Warrant failed in particularity, because it did not provide description
Ratio: The constitutional protection of our people against unreasonable of the type of evidence sought.
search and seizure is not merely a pleasing platitude. It vouchsafes our *Fourth Amendment by its terms requires particularity in the warrant,
right to privacy and dignity against undesirable intrusions committed by not in supporting documents.
any public officer or private individual. An infringement of this right *The   stated   description   of   items   to   be   seized   in   the   warrant   “single  
justifies an award for damages. dwelling   residence…blue   in   color”   did   not   describe   the   items   to   be  
*Section 2, Article III of the 1987 Constitution protects not only those seized at all.
who appear to be innocent but those who appear guilty, but are *The mere fact that the Magistrate issued a warrant does not
nevertheless to be presumed innocent until the contrary is proved. necessarily establish that he agreed that the scope of the search should
*There is a progression of time between the receipt of information and be  as  broad  as  the  affiant’s  request.  Even  though  petitioner  acted  with  
the raid of the stores of private respondents. It shows sufficient time restraint   in   conducting   the   search,   “inescapable   fact   is   that   this  
for the petitioners and the PC raiding party to apply for a judicial restraint was imposed by the agents themselves, not by a judicial
warrant. officer”  (Katz  v.  United  States,  389  U.S.  347  [1967]).
*Purpose of the particularity requirement is not limited to the
Citing   case   of   “Lim   vs.   Ponce   de   Leon”,   recovery   of   damages   for   prevention of general searches.
violation of constitutional rights and liberties from public officer or *Petitioner did not have in his possession a warrant particularly
private individual as provided under Art. 32 of the Civil Code, in relation describing the things he intended to seize; proceeding with the search
to Article 2219 (6) of the same code. Recovery for Moral damages was  clearly  “unreasonable”  under  the  Fourth  Amendment.  
*No reasonable officer could believe that a warrant that plainly did not
*Art. 32 of the Civil Code make the persons who are directly, as well as comply with that requirement was valid.
indirectly responsible for the transgression joint tortfeasors. *“If   the   law   was   clearly   established,   the   immunity   defense   ordinarily
*Neither can it be said that only those shown to have participated should fail, since a reasonably competent public official should know
“directly”  should  be  held  liable.  Art.   32  of  the  Civil  Code  encompasses   the   law   governing   his   conduct”   (Harlow   v.   Fritzgerald,   457   U.S>   800,  
within the ambit of its provisions those directly, as well as indirectly, 818-819 [1982])
responsible for its violations. *Petitioner himself prepared the warrant and cannot reasonably relied
*Petitioners miserably failed to report the unlawful peddling of on  the  Magistrate’s  assurance that the warrant contained an adequate
scouting goods to the Boy Scouts of the Phil. for the proper application description of the things to be seize and was therefore valid. (Cf.
of a warrant. Sheppard, 468 U.S., at 989-990)
*Moral damages are not awarded to penalize the defendant but to
compensate the plaintiff for the injuries he may have suffered. “The   uniformly   applied   rule   is   that   a   search   conducted   pursuant   to   a  
warrant that fails to conform to the particularity requirement of the
The wantonness of the wrongful seizure justifies the award of Fourth  Amendment  is  unconstitutional”  (Cf.  Sheppard,  468  U.S.,  at  988,  
exemplary damages. It will also serve as a stern reminder to all and n. 5)
sundry that the constitutional protection against unreasonable search
and seizure is a virile reality and not a mere burst of rhetoric. The all Extra-territorial Reach of the Guarantee
33 | P LATON
of those territories.
The Court held that the Fourth Amendment does not have
extraterritorial effect sp as to cover searches made in another Scientific and Technological Advancements and the Search and
country involving non-American citizen. The social impact is Seizure Clause
only between the government and those governed, including
aliens who have gone into the territory of the United States and As technology advances, the level of reasonably expected
developed substantial connections with that country. Thus, if privacy decreases. The measure of protection granted by the
the person affected is a citizen, it might be an entirely different reasonable expectation diminishes as relevant technology
matter becomes more widely accepted

United States v. Verdugo-Urquidez In this area, again the courts would have to see how the
494 U.S. 259, 108 Led 2d 222, 110 S Ct 1056 (1990) constitutional guarantee of privacy could be adjusted to meet
modern needs and demands, for as always there would be the
Facts: Respondent is a citizen and resident of Mexico. He was believed never-ending push and pull between the need of government
by the United States Drug Enforcement Agency (DEA) to be one of the
to maintain its role as protector against equally modernizing
leaders of a large and violent organization in Mexico that smuggles
narcotics into the United States. He was apprehended by the Mexican criminal elements and the constant demand to safeguard
Police and transported him to United States Border Patrol station in enduring liberty interests
Calexico, California, then arrested by the United States Marshals and
moved him to a correctional center in San Diego, California, pending his Klyllo v. US
trial. DEA agents, working with Mexican officials, with Director General 533 US 27, 150 L Ed 2d 94, 121 S Ct 2038 (2001)
of the Mexican Federal Judicial Police (MFJP), authorizing the searches,
searched his Mexican residences in Mexicali and San Felipe and seized Suspicious that marijuana was being grown in petitioner Kyllo`s home in
certain documents. The search of his residence uncovered a tally sheet, a triplex, agents used a thermal imaging device to scan the triplex to
which the Government believes reflects the quantities of marijuana determine if the amount of heat emanating from it was consistent with
smuggled by the respondent into the United States. District Court the high – intensity lamps typically used for indoor marijuana growth.
granted  respondent’s  motion  to  suppress  the  evidence,  concluding  that  
the Fourth Amendment applied to the searches and DEA agents had Kyllo was indicated on a federal drug charge of manufacturing
failed to justify searching of the premises without a warrant. Court of marijuana, he unsuccessfully moved to suppress the evidence seized
Appeals for the Ninth Circuit Court, divided panel, held that American from his home and then entered a conditional guilty plea.
citizens tried abroad by United States military officials were entitled to
Fifth and Sixth Amendment protections – The court concluded that the This case presents the question whether the use of a thermal – imaging
Constitution imposes substantive constraints on the Federal device aimed at a private home from a public street detect relative
Government, even it operates abroad. (citing Reid v. Covert, 354 U.S. 1, amounts of heat   within   the   home   constitutes   a   “search”   within   the  
[1957]). Majority assumed that illegal aliens in the United States have meaning of the fourth amendment.
Fourth Amendment rights. (relying on INS v. Lopez-Mendoza, 468 U.S.
1032 [1984]). Majority recognized that American search warrant would The fourth amendment is to be construed in the light of what was
be no legal validity in Mexico, but it is deemed sufficient that a warrant deemed an unreasonable search and seizure when it was adopted, and
would  have  “substantial  constitutional  value  in  this  country”,  because  it   in a manner which will conserve public interest as well as interests and
would  reflect  a  magistrate’s  determination  that  there  existed probable rights of individual citizens.
cause to search and would define the scope of the search.
The government uses a devise that is not in general public use, to
Issue: W/N Fourth Amendment applies to the search and seizure by the explore details of the home that would previously have been
United States agents of property that is owned by a non-resident alien unknowable without physical intrusion, the surveillance is a “search”  
and located in a foreign country. and is presumptively unreasonable without warrant

Ruling: Reversed (Decision of the Court of Appeals) The judgment of the court of appeals is reversed: the case is remanded
for further proceedings consistent with this opinion.
Ratio:
*The Fourth Amendment operates in a different manner that the Fifth The Big Brother Spectre and the Right to Privacy
Amendment, because the Fifth Amendment guaranteed the privilege
against self-incrimination, which is a fundamental trial right of criminal
The limits of tolerable governmental intrusions and the extent
defendants, which the constitutional violation will occur only at trial.
*As   suggested   by   Madison,   “the   driving   force   behind   the   adoption   of   of privacy that society may reasonably recognize in various
the Amendment was widespread hostility among the former Colonists situations would necessarily have to be considered by the
to the issuance of writs of assistance empowering revenue officers to courts every now and then, as what might not be allowable
search suspected places for smuggled goods and general search now would become a matter of necessity at some other time,
warrants permitting the search of private houses, often to uncover under different circumstances
papers that might be used to convict persons of libel. (Boyd v. United
States, 116 U.S. 616, 625 – 626, [1886]) It is necessary to stress that unless the creeping interference of the
*Purpose for the Amendment was to protect the people of the United government in essentially private matters is moderated, it is likely to
States against arbitrary action by their own Government; it was never destroy that prized and peculiar virtue of the free society:
suggested to be intended to restrain the actions of the Federal individualism. Every member of society, while paying proper deference
Government against aliens outside of the United States. to the general welfare, must not be deprived of the right to be left
*Not every constitutional provision applies to governmental activity alone  or,  in  the  idiom  of  the  day,  “to  do  his  thing”.  As  long  as  he  does  
even where the United States has sovereign power. not prejudice others, his freedom as an individual must not ne unduly
*Congress   was   not   required   to   adopt   “a   system   of   laws   which   shall   curtailed.
include the right of trial by jury and that the Constitution does not
without legislation and its own force, carry such right to territory so
Ople v. Torres
situated.
293 SCRA 141 (1998)
Only  “fundamental”  constitutional  rights  are  guaranteed  to  inhabitants  

34 | P LATON
The instant petition prays for the invalidation of Administrative order cause indeed, to issue a warrant of arrest. The complainant and the
no.   308   entitled   “Adoption   of   a   National   Computerized   Identification   witnesses did not show up and because of that, the trial court judge
Reference  System”  on  constitutional  grounds – usurpation of the power dismiss the case. This was elevated to the SC.
of congress to legislate and violation of the right to privacy. A.O. no.
308 was issued by then President Fidel V. Ramos on 12 December 1996, HELD: it is not mandatory for the judge to conduct a hearing for the
was published in four newspaper of general circulation on 22 and 23 purpose of determining probable cause if there has already been a case
January 1997. filed before him and the records are sufficient to justify the finding of a
probable cause for the issuance of a warrant to arrest. In this case, the
Yes. Assuming, arguendo, that A.O. No. 308 need not be the subject of SC itself determined that the records are already enough to justify the
a law, still it cannot pass constitutional muster as an administrative issuance of a warrant of arrest, such that the act of the judge in
legislation because facially it violates the right to privacy. A.O. 308 is so requiring the complainant to show up together with her witnesses and
vague. The vagueness, the overbreadth of A.O. No. 308 which if eventually dismissing the case for their failure to show up was
implemented will put our people's right to privacy in clear and present considered as a grave abuse of discretion.
danger. There are no vital safeguards the indefiniteness of A.O. No.
308 can give the government the roving authority to store and retrieve When it comes to the kind of information that would justify a probable
information for a purpose other than the identification of the individual cause   there’s   of   course   the   requirement   that   those   appearing   before  
through his PRN the judge must have personal knowledge and not simply hearsay.
Because if its hearsay then the judge could obviously not carry on a
The data may be gathered for gainful and useful government purposes; follow up manner of questioning. The witness would simply say that is
but the existence of this vast reservoir of personal information what was told me so how could the judge now prove further and
constitutes a covert invitation to misuse, a temptation that may be too deeper. But if the person appearing before him has personal knowledge
great for some of our authorities to resist. then he could satisfy his curiosity of the judge whether he is really
telling the truth or not.
Even that hospitable assumption will not save A.O. No. 308 from
constitutional infirmity for again said order does not tell us in clear and D3. United States v. Grubbs
categorical terms how these information gathered shall be handled. It 547 U.S. 90 (2006)
does not provide who shall control and access the data, under what
circumstances and for what purpose. These factors are essential to FACTS: Jeffrey Grubbs purchased a videotape containing child
safeguard the privacy and guaranty the integrity of the information. pornography from a Website operated by an undercover postal
inspector. Officers from the Postal Inspection Service arranged a
It is plain and we hold that A.O. No. 308 falls short of assuring that controlled   delivery   of   a   package   containing   the   videotape   to   Grubb’s  
personal information which will be gathered about our people will only residence. A postal inspector submitted an "anticipatory" search
be processed for unequivocally specified purposes. warrant application to a Magistrate Judge for the Eastern District of
California, accompanied by an affidavit describing the proposed
They threaten the very abuses that the Bill of Rights seeks to prevent. operation in detail, explaining that the warrant would be executed only
upon the receipt of the parcel by a person(s) and has been physically
The petition is granted and A.O. no 308 declared null and void. taken into the residence (triggering condition). The warrant was issued.
Two days later, an undercover postal inspector delivered the package.
Additional Cases Grubb’s  wife   signed  for   it   and  took   the   unopened  package   inside.   The  
(D) Searches and Seizures [11-12] inspectors detained Grubbs as he left his home a few minutes later,
then entered the house and commenced the search. Roughly 30
D1. NBI – Microsoft Corp. v. Hwang minutes into the search, Grubbs was provided with a copy of the
warrant, which included both attachments but not to supporting
A former authorize distributor of Microsoft products was raided for affidavit that explained when the warrant would be executed. Grubbs
allegedly selling fake Microsoft products. During  the  raid,  installer  CD’s   consented to interrogation by the postal inspectors and admitted
were   found.   What   are   installer   CD’s?   They   lump   together   in   one   CD   ordering the videotape. He was placed under arrest, and various items
several programs so how could that be indicative of copyright were seized, including the videotape.
infringement. The DOJ however dismiss the case saying that there was
no probable cause because this person who was the subject of the A grand jury for the Eastern District of California indicted Grubbs on
search warrant was an authorize distributor in the past. one count of receiving a visual depiction of a minor engaged in sexually
explicit conduct. Grubbs moved to suppress the evidence seized during
HELD:  the  presence  of  installer  CD’s  is  indicative  of  a  probable  cause  of   the search of his residence, arguing as relevant here that the warrant
software infringement because Microsoft does not produce installer was invalid because it failed to list the triggering condition. The district
CD’s.   Microsoft   only   comes  up   with   CD’s  of   particular   programs  but   it   court denied the motion. The court of appeals for the ninth circuit
does   not   put   them   together   in   only   one   CD.   If   its   windows   it’s   just   reversed; it held that the particularity requirement of the Fourth
windows  if  its  office  it’s  just  office.  But  if  you  find  them  in  only  one  CD,   Amendment applies with full force to the conditions precedent to an
that’s  installer  CD,  and  then  obviously  those  are  fake  or  counterfeit. So anticipatory search warrant, because the postal inspectors failed to
the SC said that is indicative of counterfeiting. Therefore there is present the affidavit the warrant was inoperative, and the search was
probable cause to continue with the case. illegal.

ISSUE: Whether anticipatory search warrants are categorically


unconstitutional
D2. AAA v. Carbonell
DECISION: No. The judgment of the Court of Appeals is reversed, and
The names of victims in crimes involving violence against women and the case is remanded for further proceedings consistent with this
their children would now have to be identified through this manner opinion
AAA and so on in order to protect their privacy. In this case, there was a
charge for rape and the accused asked the judge for judicial REASON: The Fourth Amendment does not set forth some general
determination of probable cause. Instead of the judge simply issuing “particularity  requirement”,  it  specifies  only  two  matters  that  must  be  
the warrant as a result of the filing of the case, he was asked to particularly described in the warrant: (1) the place to be search and (2)
determine if there is really a probable cause for the purpose of issuing a the persons or things to be seized. The court defined an anticipatory
warrant. Acting on this motion, the judge required the complainant to search  warrant  as  “a  warrant  based  upon  an  affidavit  showing  probable  
appear together with her witnesses to determine if there was probable cause that at some future time (but not presently) certain evidence of

35 | P LATON
crime will be located at a specified place. The court further held that
the probable-cause requirement looks to whether evidence will be REASON: When the deputies ordered respondents from their bed, they
found when the search is conducted, all warrants are in a sense, had no way of knowing whether the African-American suspects were
anticipatory. Anticipatory warrants are, therefore, no different in elsewhere in the house. The presence of some Caucasians in the
principal from ordinary warrants. They require a magistrate to residence did not eliminate the possibility that the suspects lived there
determine (1) that it is now probable that (2) contraband, evidence of a as well. The deputies, who were searching a house where they believed
crime, or a fugitive will be on the described premises (3) when the a suspect, might be armed, possessed authority to secure the premises
warrant is executed. It should be noted, however, that where the before deciding whether to continue with the search. In executing a
anticipatory warrant places a condition (other than the mere passage of search warrant officers may take reasonable action to secure the
time) upon its execution, the first of these determinations goes not premises and to ensure their own safety and the efficacy of the search.
merely to what will probably be found if the condition is met. Rather,
the probability determination for a conditioned anticipatory warrant Unreasonable actions include the use of excessive force or restraints
looks also to the likelihood that the condition will occur, and thus that a that cause unnecessary pain or are imposed for a prolonged and
proper object of seizure will be on the described premises. Two pre- unnecessary period of time. The orders by the police to the occupants,
requisites of probability must be satisfied (1) It must be true that if the in the context of this lawful search, were permissible, and perhaps
triggering   condition   occurs  “there   is   a  fair   probability   that   contraband   necessary, to protect the safety of the deputies. Blankets and bedding
or   evidence   of   a   crime   will   be   found   in  a  particular   place”   (2)   there   is   can conceal a weapon, and one of the suspects was known to own a
probable cause to believe that the triggering condition will occur. firearm, factors which underscore this point. The Constitution does not
require an officer to ignore the possibility that an armed suspect may
D4. Los Angeles County v. Rettele sleep with a weapon within reach. The deputies needed a moment to
550 U.S. 90 (2006) secure the room and ensure that other persons were not close by or
did not present a danger. The Fourth Amendment allows warrants to
FACTS:   Los   Angeles   County   Sheriff’s   Department   Deputy   Dennis   issue on probable cause, a standard well short of absolute certainty.
Watters investigated a fraud and identity-theft crime ring. There were Valid warrants will issue to search the innocent, and people like Rettele
four suspects of the investigation. The four suspects were known to be and Sadler unfortunately bear the cost. When officers execute a valid
African-Americans. Watters obtained a search warrant for two houses warrant and act in a reasonable manner to protect themselves from
in Lancaster, California, where he believed he could find the suspects. harm, however, the Fourth Amendment is not violated.
The warrant authorized him to search the homes and three of the
suspects for documents and computer files. Watters briefed six other D5. Valeroso v. Ca
deputies in preparation for the search of the houses. Watters informed 598 SCRA 41 (2009)
them they would be searching for three African-American suspects.
However, Watters did not know that one of the houses (the first to be FACTS: For resolution is the Letter-Appeal of Senior Inspector Jerry
searched) had been sold to Max Rettele. He had purchased the home Valeroso praying that the Feb. 22, 2008 decision and June 30, 2008
and moved into it three months earlier with his girlfriend Judy Sadler resolution be set aside and a new one be entered acquitting him of the
and  Sadler’s  17-year-old son Chase Hall. All three, respondents here, crime of illegal possession of firearm and ammunition.
are Caucasians.
During trial there were two versions as to where Valeroso was arrested.
The   deputies’   announcement   awoke   Rettele   and   Sadler.The   deputies   Prosecution claims that Valeroso was arrested near the INP central
entered their bedroom with guns drawn and ordered them to get out Police Station in Culiat, Quezon City, while he was about to board a
of their bed and to show their hands. They protested that they were tricycle; after placing him under arrest, the arresting officers bodily
not wearing clothes. Rettele and Sadler were held at gunpoint for one searched him, and they found the subject firearms and ammunition. On
to two minutes before Rettele was allowed to retrieve a robe for the other hand, the defense insists that he was sleeping inside a room
Sadler. He was then permitted to dress. By that time the deputies in the boarding house of his children in Quezon City and was awakened
realized they had made a mistake, they apologized to Rettele and by four heavily armed men in civilian attire who pointed their guns at
Sadler. him and pulled him out of the room, tied his hands and placed him near
the faucet outside the room then went back inside, searched and
Rettele and Sadler, individually and as guardians ad litem for Hall, filed ransacked the room and forcibly opened a locked cabinet where they
this suit against Los Angeles County, the Los Angeles County   Sheriff’s   discovered the subject firearm.
Department,   Deputy   Watters,   and   other   members   of   the   sheriff’s  
department. Respondents alleged petitioners violated their Fourth The RTC, branch 97, QC, convicted Valeroso as charged. On appeal, the
Amendment rights by obtaining a warrant in reckless fashion and CA affirmed the RTC decision with modification to the penalty. On
conducting an unreasonable search and detention. The District Court petition for review, SC affirmed in full the CA decision. He then filed a
held that the warrant was obtained by proper procedures and the motion for reconsideration which was denied with finality on June 30,
search was reasonable. It concluded in the alternative that any Fourth 2008. The present letter-appeal focused on his breached constitutional
Amendment rights the deputies violated were not clearly established rights against unreasonable search and seizure.
and that, as a result, the deputies were entitled to qualified immunity.
OSG filed a manifestation in lieu of comment  recommending  Valeroso’s  
On appeal respondents did not challenge the validity of the warrant; acquittal, considering the testimonies of the witnesses for the defense
they did argue that the deputies had conducted the search in an more credible. The OSG agrees with Valeroso that the subject firearms
unreasonable manner. The Court of Appeals concluded that the search was obtained by the police officers in violation of his constitutional
and detention were   ‘unnecessarily   painful,   degrading,   or   prolonged,’   right against illegal search and seizure, and should thus be excluded
and   involved   ‘an   undue   invasion   of   privacy.’   Turning   to   whether   from the evidence for the prosecution.
respondents’   Fourth   Amendment   rights   were   clearly   established,   the  
majority held that a reasonable deputy should have known the search ISSUE: whether the warrantless search and seizure of the firearm and
and detention were unlawful. ammunition valid.

ISSUE: whether the act of the deputies constitute an unreasonable DECISION: No. The Feb. 22, 2008 decision and June 30, 2008 resolution
manner of conducting a search. are reconsidered and set aside. Sr. Insp. Jerry Valeroso is acquitted of
illegal possession of firearm and ammunition.
DECISION: No. The court held that the search was reasonable under the
circumstances. The judgment of the Court of Appeals is reversed, and REASON: Must give more credence to the version of the defense.
the case is remanded for further proceedings consistent with this Sec. 2 of Art. III of the Constitution , as a general rule, the procurement
opinion. of a warrant is required before a law enforcer can validly search or

36 | P LATON
seize the person, house, papers, or effects of any individual. cocaine possession. On the other hand, the Arizona Supreme Court
Furthermore,   Art.   III,   Sec.   3(2)   states   that   “any   evidence   obtained   in   concluded that the search   of   Gant’s   car   was   unreasonable   within   the  
violation of this or the preceding section shall be inadmissible in meaning of the Fourth Amendment.
evidence   for   any   purpose   in   any   proceeding”.   However,   this   is   not  
absolute, there are exceptions or instances where searches and ISSUE: whether the search conducted by police officers after
seizures are allowed even without a valid warrant among these are: (1) handcuffing the defendant and securing the scene a violation of the
warrantless search incidental to a lawful arrest and (2) seizure of Fourth Amendment's protection against unreasonable searches and
evidence  in  “plain  view”.   seizures.

The Court explained that when an arrest is made, it is reasonable for DECISION: Yes. The judgment of the State Supreme Court is affirmed.
the arresting officer to search the person arrested in order to remove
any weapon that the latter might use in order to resist arrest or effect REASON:   Police   may   search   a   vehicle   incident   to   a   recent   occupant’s  
his escape, or for the protection of the officer, as well as to prevent the arrest only if the arrestee is within reaching distance of the passenger
concealment  or  destruction  of  evidence  on  the  suspect’s  person.  A  valid   compartment at the time of the search or it is reasonable to believe the
arrest allows the seizure of evidence or dangerous weapon either on vehicle contains evidence of the offense of arrest. When these
the person of the one arrested or within the area of his immediate justifications   are   absent,   a   search   of   an   arrestee’s   vehicle   will   be  
control, within which he might gain possession of weapon or unreasonable unless police obtain a warrant or show that another
destructible evidence. exception to the warrant requirement applies. The Arizona Supreme
In the present case, the arresting officers served the warrant of arrest Court correctly held that this case involved an unreasonable search.
without any resistance from Valeroso, his hands were tied and he was
placed outside the room, and also the cabinet, which was locked, could The court observed that the search-incident-to-arrest exception to the
no   longer   be   considered   as   an   “area   within   his   immediate   control”   warrant requirement is justified by interests in officer safety and
because there was no way for him to take any weapon or to destroy evidence preservation. In this case, the justifications no longer exist
any evidence that could be used against him. The purpose of the because the scene is secure and the arrestee is handcuffed, secured in
exception is to protect the arresting officer from being harmed by the the back of a patrol car, and under the supervision of an officer, the
person arrested, based on the said circumstances, the search exceeded court  concluded,  a  “warrantless  search  of  the  arrestee’s  car  cannot  be  
the bounds of what may be considered as an incident to a lawful arrest. justified as necessary to protect the officers at the scene or prevent the
The warrantless search in this case cannot also be justified under the destruction  of  evidence.”  Accordingly,  the  court  held  that  the  search  of  
plain view doctrine because it may not be used to launch unbridled Gant’s  car  was  unreasonable.
searches and indiscriminate seizure or to extend a general exploratory
search  made  solely   to   find   evidence   of  defendant’s  guilt.  The  doctrine D7. Brigham city v. Stuart
usually applied where a police officer is not searching for evidence 547 U. S. 398 (2006)
against the accused, but nonetheless unintentionally comes across an
incriminating object. However, in this case, the police officers did not Facts: Four police officers responded to a call regarding a loud party at
just accidentally discover the subject firearm and ammunition; they a residence. Upon arriving at the house, they heard shouting from
actually searched for evidence against Valeroso. inside, and proceeded down the driveway to investigate. There, they
Clearly,   the   search   made   was   illegal,   a   violation   of   Valeroso’s   right   observed two juveniles drinking beer in the backyard. They entered the
against unreasonable search and seizure. Consequently, the evidence backyard, and saw—through a screen door and windows—an
obtained in violation of said right is inadmissible in evidence against altercation taking place in the kitchen of the home. Four adults were
him.   Without   the   illegally   seized   firearm,   Valeroso’s  conviction  cannot   attempting, with some difficulty, to restrain a juvenile. At this point, an
stand. officer opened  the  screen  door  and  announced  the  officers’  presence.      
The officers subsequently arrested respondents and charged them with
D6. Arizona v. Gant contributing to the delinquency of a minor, disorderly conduct, and
556 U.S. ___ (2009) intoxication. In the trial court, respondents filed a motion to suppress
all evidence obtained after the officers entered the home, arguing that
FACTS: On August 25, 1999, acting on an anonymous tip that the the warrantless entry violated the Fourth Amendment.
residence at 2524 North Walnut Avenue was being used to sell drugs,
Tucson police officers Griffith and Reed knocked on the front door and Issue: Whether police may enter a home without a warrant when they
asked to speak to the owner. Gant answered the door and, after have an objectively reasonable basis for believing that an occupant is
identifying himself, stated that he expected the owner to return later. seriously injured or imminently threatened with such injury?
The officers left the residence and conducted a records check, which
revealed  that  Gant’s  driver’s  license  had  been  suspended  and  there  was   Held:   It   is   a   “basic   principle   of   Fourth   Amendment   law   that   searches  
an outstanding warrant for his arrest for driving with a suspended and seizures inside a home without a warrant are presumptively
license. When the officers returned to the house that evening, the unreasonable.”   Groh   v.   Ramirez,   540   U.   S. 551, 559 (2004) (quoting
officers recognized his car as it entered the driveway Gant parked at Payton v. New York, 445 U. S. 573, 586 (1980) (some internal quotation
the end of the driveway, got out of his car, and shut the door. Griffith marks omitted)). Nevertheless, because the ultimate touchstone of the
immediately arrested Gant and handcuffed him. They locked Gant in Fourth   Amendment   is   “reasonableness,”   the   warrant   requirement   is  
the back-seat of their patrol car, after which two officers searched his subject to certain exceptions. Flippo v. West Virginia, 528 U. S. 11, 13
car: One of them found a gun, and the other discovered a bag of (1999) (per curiam); Katz v. United States, 389 U. S. 347, 357 (1967).
cocaine in the pocket of a jacket on the backseat. Gant was charged
with two offenses—possession of a narcotic drug for sale and The officers had an objectively reasonable basis for believing both that
possession of drug paraphernalia. the injured adult might need help and that the violence in the kitchen
was just beginning. Nothing in the Fourth Amendment required them
He moved to suppress the evidence seized from his car on the ground to  wait  until  another  blow  rendered  someone  “unconscious”  or  “semi-
that the warrantless search violated the Fourth Amendment's conscious”   or   worse   before   entering.   The   role   of   a   peace   officer  
prohibition of unreasonable searches and seizures. Among other things, includes preventing violence and restoring order, not simply rendering
Gant argued that it did not authorize the search of his vehicle because first aid to casualties; an officer is not like a boxing (or hockey) referee,
he posed no threat to the officers after he was handcuffed in the patrol poised to stop a bout only if it becomes too one-sided.
car and because he was arrested for a traffic offense for which no
evidence could be found in his vehicle. The trial court declined Gant's The  officer’s  announcement  of  his  presence  was  at  least  equivalent  to  a  
request, stating that the search was a direct result of Gant's lawful knock on the screen door. Indeed, it was probably the only option that
arrest and therefore an exception to the general Fourth Amendment had even a chance of rising above the din. Under these circumstances,
warrant requirement. The court convicted Gant on two counts of there   was   no   violation   of   the   Fourth   Amendment’s   knock-and-

37 | P LATON
announce rule. Furthermore, once the announcement was made, the individual’s  claim  to  security  against  the  government’s  intrusion  into  his  
officers were free to enter; it would serve no purpose to require them dwelling place. However, society can have the benefit of these interests
to stand dumbly at the door awaiting a response while those within without  relying  on  a  theory  of  consent  that  ignores  inhabitant’s  refusal  
brawled on, oblivious to their presence. to allow a warrantless search. The cotenant acting on his own initiative
may be able to deliver evidence to the police, and can tell the police
what he knows, for use before a magistrate in getting a warrant. Lastly,
D8. Georgia v. Randolph the court drew a fine line between other similar cases Matlock and
547 U.S. 103 (2006) Rodriguez by stating if a potential defendant with self-interest in
objecting is in fact at the door and objects, the co-tenant’s  permission  
Facts: Respondent Scott Randolph and his wife, Janet, separated in late does not suffice for a reasonable search, whereas the potential
May 2001, when she left the marital residence in Americus, Georgia, objector, nearby but not invited to take part in the threshold colloquy,
and went to stay with her parents in Canada, taking their son and some loses out.
belongings. In July, she returned to the Americus house with the child,
though the record does not reveal whether her object was Concurring: Justice Breyer who write a concurring opinion, stated that if
reconciliation or retrieval of remaining possessions. On the morning of Fourth Amendment law forced us to choose between two bright-line
July 6, she complained to the police that after a domestic dispute her rules;   (1)   a   rule   that   always   found   one   tenant’s   consent   sufficient   to  
husband took their son away, and when officers reached the house she justify a search without a warrant and (2) a rule that never did, I believe
told them that her husband was a cocaine user whose habit had caused we should choose the first. A rule permitting such searches can serve
financial troubles. She mentioned the marital problems and said that important law enforcement needs (for example, in domestic abuse
she and their son had only recently returned after a stay of several cases)  and  the  consenting  party’s  joint  tenancy diminishes the objecting
weeks with her parents. Shortly after the police arrived, Scott Randolph party’s   reasonable   expectation   of   privacy.   Furthermore,   he   stressed  
returned and explained that he had removed the child to a neighbor's that   the   court   should   take   a   ‘totality   of   the   circumstances’   approach  
house out of concern that his wife might take the boy out of the because if the circumstances change significantly, so should the result.
country again; he denied cocaine use, and countered that it was in fact
his wife who abused drugs and alcohol. One of the officers, Sergeant
Murray, went with Janet Randolph to reclaim the child, and when they D9. Dizon v. Lambino,
returned she not only renewed her complaints about her husband's 498 SCRA 233 (2006)
drug   use,   but   also   volunteered   that   there   were   “‘items   of   drug  
evidence’”   in   the   house. Sergeant Murray asked Scott Randolph for Facts: The killing during a rumble on December 8, 1994 of University of
permission to search the house, which he unequivocally refused. The the Philippines graduating student Dennis Venturina, the chairperson of
sergeant turned to Janet Randolph for consent to search, which she the UP College of Public Administration Student Council, drew the then
readily gave. She led the officer upstairs to a bedroom that she Chancellor of UP Diliman Roger Posadas to seek the assistance of the
identified as Scott's, where the sergeant noticed a section of a drinking National Bureau of Investigation (NBI).
straw with a powdery residue he suspected was cocaine. He then left
the house to get an evidence bag from his car and to call the district As two student-suspects in the killing, Francis Carlo Taparan and
attorney's office, which instructed him to stop the search and apply for Raymundo Narag, were at the time in the office of Col. Bentai, Atty.
a warrant. When Sergeant Murray returned to the house, Janet Marichu Lambino, Legal counsel of UP Diliman, who repaired to the
Randolph withdrew her consent. The police took the straw to the police Office  of  Col.  Bentain,  advised  against  Atty.  Dizon’s  move,  however,  he  
station, along with the Randolphs. After getting a search warrant, they not being armed with a warrant for their arrest.
returned to the house and seized further evidence of drug use, on the
basis of which Scott Randolph was indicted for possession of cocaine Chancellor Posadas and Vice Chancellor for students Rosarion Torres-
. Yu, who also repaired to the office of the colonel, joined Atty. Lambino
Procedural Posture: Trial court denied motion to suppress ruling that in opposing the turn-over of the suspects to Atty. Dizon, despite the
Janet Randolph had common authority to consent to the search. Court latter’s   claim   that   under   its   Charter   the   NBI   was   authorized   to   make  
of Appeals reversed. warrantless arrests.

Issue(s): Whether such an evidentiary seizure is likewise lawful with the Issues: (1) Whether the attempted arrest of the student suspects by the
permission of one occupant when the other, who later seeks to NBI could be validly made without a warrant; and (2) Whether there
suppress the evidence, is present at the scene and expressly refuses to was probable cause for prosecuting petitioner for violation of P.D. No.
consent. 1829.

Judgment/Disposition: Affirmed Held: Respecting the complaint against Atty. Dizon, this court, also in
Posadas v. Ombudsman,  held  that  “[f]or  the  failure  of  the  NBI  agents  to  
Holding: A physically present co-occupant’s   stated   refusal   to   permit   comply   with   the   constitutional   and   procedural   requirements,…   their  
entry prevails. attempt to arrest [the two student-suspects] without a warrant was
illegal.”
Reasoning: Justice Souter, who authored the majority opinion, began
by  providing  an  analogy  similar  to  this  case  by  stating  “To  begin  with,  it   The NBI Charter clearly qualifies the power to make arrests   to   be   “in  
is fair to say that a caller standing at the door of shared premises would accordance  with  existing  laws  and  rules.”
have   no   confidence   that   one   occupant’s   invitation   was   a   sufficiently  
good   reason   to   enter   when   a   fellow   tenant   stood   there   saying   ‘stay   Members of the investigation staff of the Bureau of Investigation shall
out’.   Without   some   very   good   reason,   no   sensible   person   would   go be peace officers, and as such have the following powers:
inside under those conditions. Fear for the safety of the occupant (a) To make arrests, searches and seizures in accordance
issuing the invitation, or of someone else inside, would be thought to with existing laws and rules. xxxx(Emphasis supplied)
justify entry, but the justification then would be the personal risk, the
threats to life or limb, nit the disputed invitation”.   He   further   states   D10. Social Justice Society (SJS) v. Dangerous Drugs Board
that there is no common understanding that one co-tenant generally 570 SCRA 410 (2008)
has a right or authority to prevail over the express wishes of another,
whether the issue is the color of the curtains or invitations to outsiders. Facts: In these kindred petitions, the constitutionality of Section 36 of
The court also applied the balancing test by stating that in the balancing Republic Act No. (RA) 9165, otherwise known as the Comprehensive
of competing individual and governmental interest entailed by the bar Dangerous Drugs Act of 2002, insofar as it requires mandatory drug
to   unreasonable   searches,   the   cooperative   occupant’s   invitation   adds   testing of candidates for public office, students of secondary and
nothing  to   the  government’s  side  to  counter  the  force of an objecting tertiary schools, officers and employees of public and private offices,

38 | P LATON
and persons charged before the prosecutor’s   office   with   certain   unit, and the inherent right of the employer to maintain discipline and
offenses, among other personalities, is put in issue. efficiency in the workplace. Their privacy expectation in a regulated
office environment is, in fine, reduced; and a degree of impingement
The Commission on Elections issued Resolution No. 6486, prescribing upon such privacy has been upheld.
the rules and regulations on the mandatory drug testing of candidates
for public office in connection with the May 10, 2004 synchronized The reduced expectation of privacy on the part of the employees, the
national and local elections. compelling state concern likely to be met by the search, and the well-
defined limits set forth in the law to properly guide authorities in the
Petitioner Aquilino Q. Pimentel, Jr., a senator of the Republic and a conduct of the random testing, we hold that the challenged drug test
candidate for re-election in the May 10, 2004 elections. He seeks (1) to requirement is, under the limited context of the case, reasonable and,
nullify Sec. 36(g) of RA 9165 and COMELEC Resolution No. 6486 dated ergo, constitutional.
December 23, 2003 for being unconstitutional in that they impose a
qualification for candidates for senators in addition to those already The situation is entirely different in the case of persons charged before
provided for in the 1987 Constitution; and (2) to enjoin the COMELEC the  public  prosecutor’s  office  with  criminal  offenses  punishable  with  six  
from implementing Resolution No. 6486. Pimentel Invokes Sec. 3, years and one day imprisonment. The operative concepts in the
Article VI of the Constitution. mandatory   drug   testing   are   “randomness”   and   “suspicionless.”   In   the  
Petitioner Social Justice Society (SJS), a registered political party, seeks case of persons charged with a crime before the prosecutor’s  office,  a  
to prohibit the Dangerous Drugs Board (DDB) and the Philippine Drug mandatory drug testing can never be random or suspicionless. To
Enforcement Agency (PDEA) from enforcing paragraphs (c),(d),(f), and impose mandatory drug testing on the accused is a blatant attempt to
(g) of Sec. 36 of RA 9165 on the ground that they are constitutionally harness a medical test as a tool for criminal prosecution, contrary to
infirm. For one, the provisions constitute undue delegation of the stated objectives of RA 9165. Drug testing in this case would violate
legislative power when they give unbridled discretion to schools and a   persons’   right   to   privacy   guaranteed   under   Sec.   w,   Art.   III   of   the  
employers to determine the manner of drug testing. Constitution. Worse still, the accused persons are veritably forced to
incriminate themselves.
Petitioner Atty. Manuel J. Laserna, Jr., seeks in his petition that Sec. 36
(c),(d),(f), and (g) of RA 9165 be struck down as unconstitutional for Sec. 36(g) of RA 9165 and COMELEC Resolution No. 6486 –
infringing on the constitutional right to privacy, the right against UNCONSTITUTIONAL
unreasonable search and seizure, and the right against self-
incrimination, and for being contrary to the due process and equal Sec. 36(c) and (d) of RA 9165 - CONSTITUTIONAL but declaring its Sec.
protection guarantees. 36(f) UNCONSTITUTIONAL

Issues: (1) Do Sec. 36(g) of RA 9165 and COMELEC Resolution No. 6486
impose an additional qualification for candidates for senator?
Corollarily, can Congress enact a law prescribing qualifications for
candidates for senator in addition to those laid down by the
Constitution? And (2) Are paragraphs (c),(d),(f), and (g) of Sec. 36, RA
9165 unconstitutional? Specifically, do these paragraphs violate the Chapter 5
right to privacy, the right against unreasonable searches and seizure, Privacy of Communications and Correspondence
and the equal protection clause? Or do they constitute undue
delegation of legislative power? “The   privacy   of   communication   and   correspondence   shall   be  
inviolable except upon lawful order of the court, or when public
Held: The congress cannot validly amend or otherwise modify these safety or order requires otherwise, as prescribed by law.
qualification standards, as it cannot disregard, evade, or weaken the
force of a constitutional mandate, or alter or enlarge the Constitution.
Any evidence obtained in violation of this or the preceding
Accordingly, Sec. 36(g) of RA 9165 should be declared unconstitutional. section shall be inadmissible for any purpose in any
8
The Constitution is the basic law to which all laws must conform; no act proceeding.”
shall be valid if it conflicts with the Constitution.
Part of the right to be let alone is necessarily the corresponding
The provisions of RA 9165 requiring mandatory, random, and freedom to communicate in confidence with some other
suspicionless drug testing of students are constitutional. The right to persons   of   one’s   choosing   without   the   contents   of   that  
enroll is not absolute; it is subject to fair, reasonable, and equitable
communication being disclosed to others, especially the
requirements. The Court can take judicial notice of the proliferation of
prohibited drugs in the country that threatens the will-being of the government
people, particularly the youth and school children who usually end-up
as victims. Accordingly, and until a more effective method is Privacy of Communications and Searches
conceptualized and put in motion, random drug testing of students in
secondary and tertiary schools is not only acceptable but may even be To ensure respect and observance of the guarantee, the
necessary if the safety and interest of the student population, Constitution requires that there be a court order, or some
doubtless a legitimate concern of the government, are to be promoted weighty, justifiable and substantial state interest, such as public
and protected.
safety or order, before interference with the privacy of
Just as in the case of secondary and tertiary level students, the communications and correspondence could be allowed
mandatory but random drug test prescribed by Sec. 36 of RA 9165 for
officers and employees of public and private offices is justifiable, albeit The guidelines are supposed to be set out in law. Finally, as
not  exactly  for  the  same  reason.  Reasonableness’  is  the  touchstone  of   further deterrence, it mandates that any evidence obtained in
the validity of a government search or intrusion. And whether a search violation of its proscriptions shall be useless--“inadmissible   for  
at issue hews to the reasonableness standard is judged by the balancing any proceeding
of the government-mandated   intrusion   on   the   individual’s   privacy  
interest against the promotion of some compelling interest. The
Katz v. US
employees’   privacy   interest   in   an   office   is   to   a   large   extent  
389 US 347, 19 L Ed 2d 576, 88 S Ct 507 (1967)
circumscribed  by  the  company’s  work  policies,  the  collective  bargaining
agreement, if any, entered into by management and the bargaining
8 CONSTITUTION, Art. III, § 3(1) and (2)
39 | P LATON
The aforestated provision clearly and unequivocally makes it illegal for
Petitioner was convicted under an indictment charging him with any person, not authorized by all the parties to any private
transmitting wagering information by telephone across state lines-- communication to secretly record such communication by means of a
from LA to Miami and Boston--in violation of a federal statute. Evidence tape recorder. The law makes no distinction as to whether the party
of  petitioner’s  end  of  the  conversations,  overheard  by  FBI  agents  who   sought to be penalized by the statute ought to be a party other than or
had attached an electronic listening and recording device to the different from those involved in the private communication
outside of the telephone booth from which the calls were made, was
introduced and admitted at the trial The word communicate comes from the latin word
communicare,  meaning   “to  share  or  to  impart.”  In   its  ordinary  
Held: No. The Government’s   activities   in   electronically   listening   and  
signification, communication connotes the act of sharing or
recording   the   petitioner’s   words   violated   the   privacy   upon   which   he  
justifiably relied while using the telephone booth, and thus constituted imparting, as in a conversation,   or   signifies   the   “process   by  
a  “search  and  seizure”  within  the  meaning  of  the  Fourth  Amendment which meanings or thoughts are shared between individuals
through a common system of symbols (as language signs or
Wherever a man may be, he is entitled to know that he will remain free gestures)”  these  definitions  are broad enough to include verbal
from unreasonable searches and seizures or non-verbal, written or expressive communications of
“meanings   or   thoughts”   which   are   likely   to   include   the  
“The  Fourth  Amendment  protects  people,  not  places” emotionally-charged exchange
There is a twofold requirement:
1. That a person have exhibited an actual )subjective) expectation of What the law prohibits is the overhearing, intercepting, or
privacy recording of private communications. Thus, a tape recording of
2. That the expectation be one that society is prepared to recognize as an altercation or verbal exchange between a policeman and a
“reasonable” radio reporter at a police station is admissible even if said
recording was done without the knowledge of the participants.
the   point   is   not   that   the   booth   is   “accessible   to   the   public”   at   other   And, as noted in Ramirez, the use of a telephone extension for
times, but that it is a temporarily private place whose momentary the purpose of overhearing does not violate RA 4200
occupants’   expectations   of   freedom   from   intrusion   are   recognized   as  
reasonable

Zulueta v. CA
Salcedo-Ortañez v. CA
253 SCRA 699 (1996)
235 SCRA 111 (1994)
Petitioner Cecilia Zulueta, wife of private respondent Alfredo Martin,
Private respondent Rafael S. Ortañez filed with the RTC of QC a
entered the clinic of her doctor husband, a doctor of medicine, and in
complaint for annulment of marriage with damages against petitioner
the   presence   of   her   mother,   a   driver   and   private   respondent’s  
Teresita Salcedo-Ortañez on grounds of lack of marriage license and/or
secretary,   forcibly   opened   the   drawers   and   cabinet   in   her   husband’s  
psychological incapacity of the petitioner. Among the exhibits offered
clinic and took 157 documents consisting of private correspondence
by private respondent were three (3) cassette tapes of alleged
between Dr. Martin and his alleged paramours, i.e. greeting cards,
telephone conversations between petitioner and unidentified persons
cancelled  checks,   diaries,   Dr.   Martin’s  passport,   and  photographs.   The  
documents and papers were seized for use in evidence in a case for
The trial court issued the assailed order admitting all of the evidence
legal separation and for disqualification from the practice of medicine
offered by private respondent, including tape recordings of telephone
which petitioner had filed against her husband
conversations of petitioner with unidentified persons. these tape
recordings were made and obtained when private respondent allowed
Held: No. The only exception to the prohibition in the Constitution is if
his friends from the military to wire tap his home telephone
there   is  a   “lawful   order   [from   a]   court   or   when  public   safety   or   order  
requires   otherwise,   as   prescribed   by   law.”   Any   violation   of   this  
Held:   No.   RA   4200   entitled   “An   Act   to   Prohibit   and   Penalize   Wire-
provision renders the evidence  obtained  inadmissible  “for  any  purpose  
Tapping and Other Related Violations of the Privacy of Communication,
in  any  proceeding”
and   for   other   purposes” expressly makes such tape recordings
inadmissible in evidence
Clearly, respondents trial court and Court of Appeals failed to consider Warrant Requirement
the afore-quoted provisions of the law in admitting in evidence the
cassette tapes in question. Absent a clear showing that both parties to Consistent with Section 2 of Article III, searches and seizures,
the telephone conversations allowed the recording of the same, the whether of tangible or intangible things, must be authorized by
inadmissibility of the subject tapes is mandatory under RA 4200 a warrant supported by probable cause and with a particularity
of description of what is sought to be searched or seized
The subject cassette tapes are declared inadmissible in evidence

Accordingly, what might only be possible is a reasonable


Ramirez v. CA
248 SCRA 590 (1995)
description of the persons whose communication is sought to
be intercepted, identification of the crime that might be
A civil case for damages was filed by petitioner Socorro D. Ramirez in committed by means of such communication, as well as a
the RTC of QC alleging that the private respondent, Ester S. Garcia, in a delimitation of the period of the allowable search and seizure
confrontation   in   the   latter’s   office,   allegedly   vexed,   insulted   and  
humiliated   her   in   a   “hostile   and   furious   mood”   and   in   a   manner   Additional Cases
offensive   to   petitioner’s   dignity   and   personality,   “contrary   to   morals,   (E) Privacy of Communications and Correspondence
good  customs  and  public  policy”--Garcia  essentially  belittled  Ramirez’s  
intelligence,   and   attributing   her   employment   to   Garcia’s   help.   In  
E1. City of Ontario, California, et al. v. Quon et al
support of her claim, petitioner produced a verbatim transcript of the
560 U. S. ____ (2010)
event   and  sought   moral   damages,   attorney’s   fees   and  other   expenses  
of litigation. the transcript on which the civil case was based was culled
Petitioner acquired alphanumeric pagers able to send and receive text
from a tape recording of the confrontation made by petitioner
messages. Its contract with its service provider, Arch Wireless, provided

40 | P LATON
for a monthly limit on the number of characters each pager could send that Arch Wireless violated the SCA by giving the City the transcript.
or receive, and specified that usage exceeding that number would Whether the audit was nonetheless reasonable, the court concluded,
result in an additional fee. The City issued the pagers to respondent turned on. Whether Scharf used it for the improper purpose of
Quon and other officers in its police department (OPD). When Quon determining if Quon was using his pager to waste time, or for the
and others exceeded their monthly character limits for several months legitimate purpose of determining the efficacy of existing character
running,   petitioner   Scharf,   OPD’s   chief,   sought   to   determine   whether   limits to ensure that officers were not paying hidden work-related
the existing limit was too low, i.e., whether the officers had to pay fees costs.      After  the  jury  concluded  that  Scharf’s  intent  was  legitimate,  the  
for sending work-related messages or, conversely, whether the court granted petitioners summary judgment on the ground they did
overages were for personal messages. After Arch Wireless provided not violate the Fourth Amendment. The Ninth Circuit reversed.
transcripts   of   Quon’s   and   another   employee’s   August   and   September   Although it agreed that Quon had a reasonable expectation of privacy
2002   text   messages,   it   was  discovered  that   many   of   Quon’s  messages   in his text messages, the appeals court concluded that the search was
were not work related, and   some   were   sexually   explicit.   McMahon’s   not reasonable even though it was conducted on a legitimate, work-
report noted that Quon sent or received 456 messages during work related rationale. The opinion pointed to a host of means less intrusive
hours in the month of August 2002, of which no more than 57 were than the audit that Scharf could have used. The court further
work related; he sent as many as 80 messages during a single day at concluded that Arch Wireless had violated the SCA by giving the City
work; and on an average workday, Quon sent or received 28 messages, the transcript.
of which only 3 were related to police business. The report concluded
that Quon had violated OPD rules. Quon was allegedly disciplined. Chapter 6
Freedom of Expression and Assembly
The employee contends that the privacy of the messages is protected
by   the   ban   on   “unreasonable   searches   and   seizures”   found   in   the  
Fourth Amendment to the United States Constitution “No   law shall be passed abridging the freedom of speech, of
expression, or of the press, or the right of the people peaceably
HELD:  No.  “[O]ffices  of  government  employees...are  [generally]  covered   to assemble and petition the government for redress of
9
by Fourth Amendment protections, but government searches to grievances.”
retrieve work-related materials or to investigate violations of workplace
rules—searches of the sort that are regarded as reasonable and normal Among the most cherished liberties in a free society, where
in the private-employer context—do  not  violate  the...Amendment” freedom of thought and conscience is a bedrock principle, one
that occupies a preferred and predominant status, is the right
[T]he correct analysis has two steps: First,   because   “some   to  freely  speak  one’s  mind
[government] offices may be so open...that no expectation of
privacy  is  reasonable,”  a  court  must  consider  “[t]he  operational   Freedom of expression is the matrix, the indispensable
realities   of   the   workplace”   to   determine   if   an   employee’s   condition of nearly every freedom. The guarantee of the
constitutional rights are implicated. Second, where an freedom   of   speech   has   been   defined   as   “the   instrument   and  
employee   has   a   legitimate   privacy   expectation,   an   employer’s   guarantee  and  the  bright  and  consummate  flower  of  all  liberty”
intrusion   on   that   expectation   “for   non   investigatory,   work-
related purposes, as well as for investigations of work-related [I]t was made part of the First Amendment to the American
misconduct, should be judged by the standard of Constitution
reasonableness under all the  circumstances.”
Under this guarantee, the people are to determine their own
E1. Ontario vs Quon
direction and chart their own destiny through the free
560 U.S. ___ (2010)
exchange of ideas and not through dictation from or coercion
Petitioner Ontario (hereinafter City) acquired alphanumeric pagers able of  the  government  or  anybody  else’s
to send and receive text messages. Its contract with its service
provider, Arch Wireless, provided for a monthly limit on the number of The theory of freedom of expression involves more than a
characters each pager could send or receive, and specified that usage technique for arriving at better social judgments through
exceeding that number would result in an additional fee. The City democratic procedures. It comprehends a vision of society, a
issued the pagers to respondent Quon and other officers in its police faith and a whole way of life
department (OPD), also a petitioner here. When Quon and others
exceeded their monthly character limits for several months running,
petitioner   Scharf,   OPD’s   chief,   sought   to   determine   whether   the  
Scope of Guarantee -- Prior Restraint, Subsequent Punishment
existing limit was too low, i.e., whether the officers had to pay fees for and Damages
sending work-related messages or, conversely, whether the overages
were for personal messages. After Arch Wireless provided transcripts The Freedom of Expression Clause is basically directed against
of   Quon’s   and   another   employee’s   August   and   September   2002   text   prior restraint or censorship and subsequent punishment
messages,   it   was   discovered   that   many   of   Quon’s   messages   were   not  
work related, and some were sexually explicit. Scharf referred the [I]t means that the people are kept free from any undue
matter  to  OPD’s  internal  affairs  division.    The  investigating  officer  used  
interference from the government in their thoughts and words
Quon’s   work   schedule   to   redact   from   his   transcript   any   messages   he  
sent while off duty, but the transcript showed that few of his on-duty
messages related to police business. Quon was disciplined for violating [T]he people should be allowed to see and discuss for
OPD rules. themselves what is best for them
The employee contends that the privacy of the messages is protected
by   the   ban   on   “unreasonable   searches   and   seizures”   found   in   the   The First   Amendment’s   guarantee   of   “the   freedom   of   speech,  
Fourth Amendment to the United States Constitution, made applicable or   of   the   press”   prohibits   a   wide   assortment   of   government  
to the States by the Due Process Clause of the Fourteenth restraints upon expression, but the core abuse against which it
Amendment. He and the other respondents each of whom had
was directed was the scheme of licensing laws implemented by
exchanged text messages with Quon during August and September—
filed this suit, alleging, inter alia, that petitioners violated their Fourth
Amendment rights and the federal Stored Communications Act (SCA) by
obtaining  and  reviewing  the   transcript  of  Quon’s  pager  messages,  and   9 CONSTITUTION, Art. III, § 4
41 | P LATON
the monarch and Parliament   to   contain   the   “evils”   of   the   General Considerations
th th
printing press in the 16 - and 17 -century England
To properly understand the value of the freedom of speech, of
It punished the publication of any book or pamphlet without a the press and of expression, it would be best to consider the
license and required that all works be submitted for approval to background, the history and the circumstances which called
a government official, who wielded broad authority to suppress forth such guarantee
works  that  he  found  to  be  “heretical  seditious,  schismatical,  or  
offensive” Near v. Minnesota
283 US 697, 75 L Ed 2d 1357, 51 S Ct 625 (1931)
The freedom has also expanded its coverage through the years.
A Minnesota statute declared that one who engages "in the business of
While it may have been primarily meant to assure the right to
regularly and customarily producing, publishing," etc., "a malicious,
speak  one’s  mind  on  matters   affecting government affairs and scandalous and defamatory newspaper, magazine or other periodical,"
politics, it has gradually encompass expressions which are of is guilty of a nuisance, and authorizes suits, in the name of the State, in
private and commercial concerns which such periodicals may be abated and their publishers enjoined
from future violations. In such a suit, malice may be inferred from the
The guarantee has also come to ensure that claims for damages fact of publication. The defendant is permitted to prove, as a defense,
arising from the utilization of the freedom be not so that his publications were true and published "with good motives and
unreasonable or exorbitant as to practically still or chill its for justifiable ends." Disobedience of an injunction is punishable as a
contempt.
exercise
Under said statute, the County Attorney of Hennepin County brought
[T]he right of free speech is not absolute at all times and under action to enjoin the publication of what was describe as a "malicious,
all circumstances scandalous and defamatory newspaper, magazine and periodical"
known as "The Saturday Press," published by the defendants in the city
These include the lewd and obscene, the profane, the libelous, of Minneapolis.
and  the  insulting  or  “fighting”  words--those which, by their very
utterance, inflict injury or tend to incite an immediate breach of Held: Yes. The statute is not directed at threatened libel, but at an
existing business which, generally speaking, involves more than libel"
the peace
The object of the statute is not punishment, in ordinary sense, but
The First Amendment protects the press from governmental suppression of the offending newspaper or periodical
interference; it confers no analogous protection on the
Government The statute not only operates to suppress the offending newspaper or
periodical, but to put the publisher under an effective censorship (in a
The Chilling Effect Principle -- The Danger of Self-Censorship way that, unless the owner or publisher is able and disposed to bring
competent evidence to satisfy the judge that the charges are true and
are published with good motives and for justifiable ends, his
If the freedom to speak is not adequately shielded from
newspaper or periodical is suppressed and further publication is made
overbearing and misdirected zeal to check or restrict it, people punishable as a contempt)
who might otherwise be minded to say something would rather
keep quiet than risk the danger of being prosecuted or For these reasons we hold the statute, so far as it authorized the
otherwise subjected to disagreeable and unwelcome proceedings in this action under clause (b) of section one, to be an
consequences infringement of the liberty of the press guaranteed by the Fourteenth
Amendment
This form of censorship is likewise something that the Free
Speech Clause seeks to prevent, for in having people keep quiet New York Times Co v. US
out of their own self-induced fears society suffers just as much 403 US 713, 29 L Ed 2d 822, 91 S Ct 2140 (1971)
as when their mouths are kept shut by the authorities
The United States brought these actions to enjoin publication in the
New York Times and in the Washington Post of the contents of a
The chilling effect need not emanate only from threats coming classified study entitled "History of the US Decision-Making Process on
from the government itself. It may also be effected by means of Viet Nam Policy"
the use of libel laws that may directly enforce silence through
the threat of financial ruin brought about by claims for Held: Any system of prior restraints of expression comes to this Court
damages or prosecution under criminal laws for defamation bearing a heavy presumption against its constitutional validity.

J. Black & J. Douglas: The amendments were offered to curtail and


Facial Challenges and the Overbreadth Doctrine
restrict the general powers granted to the Executive, Legislative and
the Judicial Branches two years before in the original Constitution.
The overbreadth doctrine permits a party to challenge the
validity of a statute even though as applied to him it is not In the case at bar, we are asked to hold that, despite the First
unconstitutional but it might be if applied to others not before Amendment's emphatic command, the Executive Brach, the Congress,
the Court whose activities are constitutionally protected and the Judiciary can make laws enjoining publication of current news
and abridging freedom of the press in the name of "national security"
Accordingly, considering the preferred and paramount position
The dominant purpose of the First Amendment was to prohibit the
of the freedom of speech and of the press, such extraordinary
widespread practice of governmental suppression of embarrassing
safeguards   as   the   “facial   challenge”   and   the   use   of   the   information.
overbreadth doctrine are allowed to be engaged in whenever
speech is threatened Babst v. National Intelligence Board
132 SCRA 316 (1984)

42 | P LATON
Just like any other liberty, the freedom of expression is not
The petitioners are journalists and columnists. On different dates in July absolute
1980, they were summoned by military authorities for interrogation
regarding their work, feelings, sentiments, beliefs, associations and
There are three basic standards--the   “dangerous   tendency
even private lives. In addition, one of them was charged with libel by a
General who sought to recover P10 million in damages. They brought
rule,”   the   “clear   and   present   danger”   test   and   the   “balancing-
an action for prohibition to stop the NIB from questioning them and of-interest”  test.  Of  the  three,  he  clear and present danger test
from filing libel suits on matters that had been the subject of inquiry by is the most liberal and latitudinarian
the NIB.
Under the dangerous tendency rule,   “if   the   words   uttered  
The petition has become moot and academic. Be that as it may, it is not create a dangerous tendency which the state has a right to
idle to note that, while ordinarily, an invitation to attend a hearing and prevent, ten such words are punishable. It is not necessary that
answer some questions is not illegal or constitutionally objectionable,
some definite or immediate acts of force, violence, or
under certain circumstances, however, such an invitation can easily
assume a different appearance as when it comes from a powerful
unlawfulness be advocated. It is sufficient that such acts be
group composed predominantly of ranking military officers and the advocated in general terms. Nor it is necessary that the
designate interrogation site is a military camp. language used be reasonably calculated to incite persons to
acts of force, violence, or unlawfulness. It is sufficient if the
Forms and Variations of the Freedom and Relativity of State natural tendency and probable effect of the utterance be to
Regulation bring about the substantive evil which the legislative body
seeks to prevent
Depending on the form and medium in which speech is being
exercised, to that extent may it also affect the extent of The clear and present danger test posits that the evil
governmental power expended consequence  of  the  comment  or  utterance  must  be  “extremely  
serious   and   the   degree   of   imminence   extremely   high”   before  
It is also true that speech has its own hierarchy, that is, some the utterance can be punished. The danger to be guarded
specie of speech are given more weight and importance, and against  is  the  “substantive  evil”  sought  to  be  prevented
thus a greater extent of protection, compared to others. One
could not equate, for instance, the right to speak on matters The question in every case is whether the words used are used
affecting public matters with the right to comment on some in such circumstances and are of such a nature as o create a
private concerns clear and present danger that they will bring about the
substantive evils that Congress has a right to prevent. It is a
Eastern Broadcasting Corporation v. Dans, Jr. question of proximity and degree
137 SCRA 628 (1985)
This [clear and present danger] test then as a limitation on
The SC held that radio broadcast also enjoys the protection of the freedom of expression is justified by the danger or evil of a
freedom of expression. If close down, the owner enjoys the rights to substantive character that the state has a right to prevent.
due process according to the standards set in Ang Tibay v. CIR.
Unlike the dangerous tendency doctrine, the danger must not
But radio deserves greater regulation than newspapers because it could
only be clear but also present. The term clear seems to point to
invade the privacy of everyone for no fee, and it is such that one is a causal connection with the danger of the substantive evil
likely to listen to what is being said. arising from the utterance questioned. Present refers to the
time element. It used to be identified with imminent and
The petitioner filed this action to compel respondent government immediate danger. The danger must not only be probable but
officials to allow the reopening of Radio Station DYRE after it had been very likely inevitable
closed for allegedly having been used to incite the people to sedition.
The petitioner contended that it was denied due process because no
The essential difference between the two doctrines related to
hearing was held and no proof was submitted to establish a factual
basis for the closure. However, before the Court could promulgate its
the degree of proximity of the apprehended danger which
decision the petitioner filed a motion to withdraw its action on the justified the restriction upon speech
ground that it had sold the radio station to Manuel Pastrana and that
the National telecommunications Commission had expressed its Dangerous Tendency Clear and Present Danger Rule
willingness to grant the requisite license. Doctrine
Permits the restrictions once Requires the Government to
The case has been moot and academic. However, for the guidance of
a rational connection defer application of restrictions
the inferior courts and administrative bodies, the following guidelines
must be observed: The cardinal primary requirements in administrative
between the speech until the apprehended danger
proceedings as laid down in Ang Tibay v. CIR should be followed before restrained and the danger was much more visible, until its
a broadcast station may be closed; All forms of communication are apprehended--the realization was imminent and
entitled to the broad protection of the freedom of expression clause. “tendency”   of   one   to   create   nigh at hand
Necessarily, the freedom of television and radio broadcasting is the other--was shown
somewhat lesser in scope than the freedom accorded to newspapers
and print media. This limitation derives from the fact the broadcast The latter rule was thus considerably more permissive of
media have a uniquely pervasive presence in the lives of all Filipinos;
speech than the former, in context for the testing of which they
The government has a right to be protected against broadcasts which
incite listeners to violently overthrow it; and Broadcast stations deserve were originally designed
the special protection given to all forms of media by the due process
and freedom of expression clauses of the Constitution. The third test, the balancing-of-interests”   test,   the   crucial  
question is: how much deference should be given to the
Standards for Regulations and Restrictions legislative judgment?

43 | P LATON
Factors in ascertaining the point or line of equilibrium:
1. The social value and importance of the specific aspect of the The defendants were indicted in three counts. The first charges a
particular freedom restricted by the legislation conspiracy to violet the Espionage Act by causing and attempting to
cause insubordination, in the military and naval forces of the United
2. The specific thrust of the restriction, i.e., whether the
States, and to obstruct the recruiting and enlistment service of the
restriction is direct or indirect, whether or not the persons United States. The second count alleges a conspiracy to commit an
affected are few offence against the United States. The third count charges an unlawful
3. The value and importance of the public interest sought to be use of the mails for the transmission of the same matter. The
secured by the legislation--the reference here is to the nature defendants were found guilty on all the counts.
and gravity of the evil which the Congress seeks to prevent
4. Whether the specific restrictions decreed by the Congress is But the character of every act depends upon the circumstances in
reasonably appropriate and necessary for the protection of which it is done.
such public interest
The question in every case is whether the words used are used in such
5. Whether the necessary safeguarding of the public interest circumstances and are of such a nature as to create a clear and present
involved may be achieved by some other measures less danger that they will bring about the substantive evils that Congress
restrictive of the protected freedom has a right to prevent. It is a question of proximity and degree.

Relevant also to any discussion of the balancing test would be If the act (speaking, or circulating a paper), its tendency, and the intent
the so-called O’Brien test,   as   well   as   the   “time,   place   and   with which it is done are the same, we perceive no ground for saying
manner”  restrictions that success alone warrants making the act a crime. (Goldman v. United
States, 245 U.S. 474, 477). Indeed, that case might be said to dispose of
the present contention if the precedent covers all media concludendi.
Under the O’Brien test, a government regulation is sufficiently
justified if:
Content-Based and Content-Neutral Regulations
1. It is within the constitutional power of the Government
2. It furthers an important or substantial government interest
Content-based are those which either approve or disapprove
3. The governmental interest is unrelated to the suppression of
based on contents of expression, such as favoring or disfavoring
free expression
some topics
4. The incident restriction on expression is no greater than is
essential to the furtherance of the interest
Content-based prohibitions, enforced by severe criminal
penalties, have the constant potential to be a repressive force
Under   the   reasonable   “time,   place   and   manner”   rule,   an  
in the lives and thoughts of a free people
expression, whether oral or written or symbolized by conduct,
is subject to reasonable time, place or manner restrictions.
Consistent with the idea that the government is not supposed to be
Restrictions of this kind are valid, provided that they are: dictating to the people what to think or believe in, what to read or see
1. Justified without reference o the content of the regulated or hear, regulations which are content based-those which either
speech approve or disapprove based on the contents of the expression, are
2. They are narrowly tailored to serve a significant subjected to strict scrutiny. To justify them by the presence of a
governmental interest compelling state interest and a showing of an absence of any other
3. They leave open ample alternative channels for means which the state objective could be attained.
communication of the information
“Content   based-prohibitions, have the constant potential to be a
repressive force in the lives and thoughts of the free people. The
And then again, one must take note of the differences between government through its public libraries, public television and public
a law or regulation, on one hand, and court injunction, on the funding for the arts, has broad discretion to make content-based
other judgments in deciding what private speech to make available to the
public.
An injunction, by its nature, applies only to a particular group
(or individuals) and regulates the activities, and perhaps the R.A.V. v. City of St. Paul
speech, of that group. It does so, however, because of the
group’s   past   actions   in   the   context   of   a   specific   dispute   In the predawn hours one day in June 1990, petitioner R.A.V. and
between real parties several other teenagers allegedly assembled a crudely made cross by
taping together broken chair legs which they then burned inside the
fenced yard of a black family that lived across the street from the house
Ordinances represent a legislative choice regarding the where petitioner was staying. Respondent city chose to charge
promotion of particular societal interests. Injunctions, by petitioner was the St. Paul Bias- Motivated Crime Ordinance, which
contrast, are remedies imposed for violations (or threatened prohibits the display of a symbol which one knows or has reason to
violations) of a legislative or judicial decree know   “arouses   anger,   alarm   or   resentment   in   others   on   the   basis   of  
race,   color,   creed,   religion   or   gender.”   The   trial   court   dismissed   this  
Injunctions also carry greater risks of censorship and charge on the ground that the ordinance was substantially overbroad
discriminatory application than do general ordinances and impermissibly content-based, but the State Supreme Court
reversed.
Injunctions, of course, have some advantages over generally The court also concluded that the ordinance was not impermissibly
applicable statutes in that they can be tailored by a trial judge content-based, because it was narrowly tailored to serve a compelling
to afford more precise relief than a statute where a violation of governmental interest in protecting the community against the bias-
the law has already occurred motivated threats to public safety and order.

Schenck v. United States Let there be no mistake about our belief that burning across in
39 S Ct 247 (1919) someone’s  front  yard  is  reprehensible.  But  St.  Paul  has  sufficient  means

44 | P LATON
at its disposal to prevent such behavior without adding the First apply   to   “public   figures”   who   sued in libel on the basis of alleged
Amendment to the fire. defamatory falsehoods.

The judgment of the Minnesota Supreme Court is reversed, and the If a matter is a subject of public or general interest, it cannot suddenly
case is remanded for proceedings not inconsistent with this opinion. become less so merely because a private individual is involved or
because   in   some   sense,   the   individual   did   not   “voluntarily’   choose   to  
Libel and the Deliberate Falsehood become   involved.   The   public’s   primary   interest   is   in   the   event,   the  
public focus is on the conduct of the participant and the content, effect
and  significance  of  the  conduct,  not  the  participant’s  prior  anonymity  or  
As noted earlier, one of the exceptions to the freedom of
notoriety. The present case illustrates the point.
expression would be libel or defamation. Freedom to speak
does not include the knowing lie or falsehood Petitioner’s   argument   that   the   Constitution   should   be   held   to   require  
that the private individual prove only that the publisher failed to
At the same time, he freedom also means that one may not be exercise   “reasonable   care”   in   publishing   defamatory   falsehoods  
accurate, pr may even be wrong, in the presentation of facts, or proceeds along two lines. First, he argues that the private individual,
might be guilty of a lot of exaggerations and overstatements in unlike the public figure, does not have access to the media to counter
the narration of events and other circumstances, but these the defamatory material and that the private individual, unlike the
public figure, has not assumed the risk of defamation by thrusting
would not be enough to take them out of the mantle of the
himself in to the public arena. Second, petitioner focuses on the
protection accorded by the guarantee important values served by the law of defamation in preventing and
redressing attacks upon reputation. We have recognized the force of
New York Times Co. v. Sullivan petitioner’s  arguments.
376 US 254, 11 L Ed 2d 686, 84 S Ct 710 (1964)
Borjal v. CA
ACTUAL MALICE TEST 301 SCRA 1 (1999)
Particularly applicable to public officials and public figures. Under this FACTS: During the congressional hearings on the transport crises in
test, it is not enough that what might have been published is false. 1988, the attendees agreed to organize the First National Conference
There must be an element of malice in it. Namely, that the one who on Land Transportation (FNCLT) to be participated by the private and
publish it knew very well that it was false or he did it with other government sector in order to find ways to solve the transportation
disregard of whether it was true or false. crises. The estimated cost of conference estimated at P 1.8M would be
funded thru solicitations. During its first organizational meeting, private
With regard to people who are not public officials but considered as respondent Francisco Wenceslao was elected as Executive Director and
public figures, they are also open to criticisms and comments he then undertook to solicit support for the conference from the
business sector. Between May and July 1989, a series of articles written
Rosenbloom v. Metromedia by petitioner Borjal was published in his column in the Philippine Star.
403 US 29, L ED 2d 296, 91 S Ct 1811 (1971) The  articles  dealt  with  the  alleged  anomalous  activities  of  an  “organizer
of   a  conference”   without   naming   private   respondent     nor   referring   to  
FACTS: Petitioner was a distributor of nudist magazines in the FNCLT. Private respondent reacted to the articles and refuted the
Philadelphia metropolitan area. During the fall of that year, in response matters adverted to by Borjal. Thereafter a complaint was lodged by
of citizen complaints, the Philadelphia Police arrested several private respondent with the National Press Club for unethical conduct.
newsstand operators, including the petitioner, in charge of selling This was followed by a criminal case for libel which was, however,
obscene material. Three days later, the police obtained a warrant to dismissed for insufficiency of evidence. A civil action for damages based
search  petitioner’s  home  and  the  rented  barn  he  used  as  a  warehouse   on libel was then filed against petitioners. Borjal and Soliven, publisher
and seized the inventory of magazines and books found there. Upon and chairman of the editorial board of Philippines today, owner of
learning of the seizure, the petitioner who had been released on bail Philippine Star. The trial court decided in favor of private respondent,
after his first arrest, surrendered to the police and was arrested for a which decision was affirmed by CA, although the amount of damages
second time. Following the second arrest, the police informed the was reduced. Hence, this petition.
respondent’s  radio   station  WIP  and  another   local  radio  station,  a  wire  
service  and  a  local  newspaper  of  the  raid  on  the  petitioner’s  home  and   HELD: The petition is impressed with merit. In order to maintain a libel
of his arrest. WIP reported   news   stories   of   the   petitioner’s   arrest   for   suit, it is essential that the victim be identifiable although it is not
possession of obscene literature and of the police seizure of the necessary that he be named. It is also not sufficient that the offended
obscene   books.   It   did   not   mention   petitioner’s   name   but   used   the   party recognized himself as the person attacked or defamed but it
terms   “smut   literature   racket”   and   “girlie   book   peddlers”.   Following   must be shown that at least, a third person could identify him as the
petitioner’s   acquittal   of   criminal   obscenity   charges,   he   filed   action   in   object of the libelous publication. Regrettably, these requirements
District  Court  seeking  damages  under  Pennsylvania’s  libel  law.  The  jury   have not been complied with in the case at bar. Identification is grossly
found  for  petitioner  but  CA  reversed  the  decision  holding  that  the  “fact   inadequate when even the alleged offended party is himself unsure
that plaintiff was not a public figure cannot be accorded decisive that he was the object of the verbal attack. It is well to note that the
significance.”   The   issue   here   is   whether,   because   he   is   not   a   “public   revelation of the identity of the person alluded to came not from
official”   or   a   “public   figure”,   but   a   private   individual,   those   limits   petitioner Borjal but from the private respondent himself when he
required that he prove that the falsehoods resulted from a failure of supplied the information through his June 4 letter to the editor. Had
the respondent to exercise reasonable care, or required that he prove private respondent not revealed that he was the organizer of the FNCLT
that the falsehoods were broadcast with knowledge of their falsity or referred to in the Borjal articles, the public would have remained in
with reckless disregard of whether they were false or not. blissful ignorance of his identity. It is therefore clear that on the
element of identifiability alone the case fails.
HELD: In the series of cases beginning with New York Times Co v.
Sullivan, 376 US 254 (964), the Court has considered the limitations Indisputably,   Borjal’s   articles   are   neither   private   communications   nor  
upon state libel laws imposed by the constitutional guarantees of fair and true report without any comments or remarks. However, this
freedom of speech and of the press. It held that in a civil libel action by does not necessarily mean that they are not privileged. To be sure, the
a pubic official against a newspaper those guarantees required clear enumeration on Art 354 of the Revised penal code (on the requirement
and convincing proof that a defamatory falsehood alleged as libel was for publicity of defamatory materials) is an exclusive list of qualifiedly
uttered with knowledge that it was false or with reckless disregard of privileged communications since fair commentaries on matters of
whether it was false or not. The same requirement was later held to public interest are likewise privileged. The rule on privileged

45 | P LATON
communication had its genesis not  in  the  nation’s  penal  code  but  in  the   the broadcasts are not privileged and remain libelous per se. FBNI is
Bill of Rights of the Constitution guaranteeing freedom of speech and of solidarily liable to pay for damages arising from the libelous broadcasts.
the press. An employer and employee are solidarily liable for defamatory
statement by the employee within the scope and course of his
Borjal did not act with malice, we find the petitioner Borjal to have employment, at least when the employer authorizes or ratifies the
acted in good faith. Moved by the civic duty and prodded by his defamation. Moreover, FBNI, as shown by circumstances (e.g. no clear
responsibility as a newspaperman, he proceeded to expose and and  convincing  evidence  shows  that  they  underwent  FBNI’s  regimented  
denounced what he perceived to be a public deception. Every citizen process of application and their deficiencies in their KBP accreditation)
has the right to enjoy a good name and reputation, but we do not lacked the diligence in selecting and supervising Rima and Alegre,
consider that petitioner Borjal has violated that right (of the Hence, FBNI is solidarily liable to pay damages together with Rima and
respondent’s)   nor   abused   his   press   freedom.   We   must   however   take   Alegre.
this opportunity to likewise remind media practitioners of the high
ethical standards attached to and demanded by their noble profession. Flor v. People
454 SCRA 440 (2005)
Filipinas Broadcasting Network Inc (FBNI) v. Ago Medicaland
Educational Center-Bicol Christian College of Medicine (AMEC-BCCM) FACTS: Petitioner Flor and Nick Ramos, managing editor and news
448 SCRA 413 (2005) correspondent respectively of the Bicol forum, a local weekly
newspaper circulated in the Bicol Region were charged with libel, for
FACTS:  Mel  Rima  and  Jun  Alegre,  host  of  radio  program  “Expose”  aired   having published an allegedly defamatory news article regarding
every morning over DZRC-AM owned by FBNI heard over Legaspi City. financial irregularities involving then minister of the Presidential
Rima and Alegre exposed various alleged complaints from students, Commission on Govt. Reorganization and concurrently Camarines Sur
teachers and parents against AMEC-BCCM. Alegre said among other Gov. Luis Villafuerte. The news article reported, inter alia that the
things that 1) AMEC students in Physical Theraphy complained that the official’s  denial  that  he  did  not  spend  government money for his recent
course is not recognized by DECS 2)students are required to take and trips to Japan and Israel failed to convince the people and that the
pay for he subject even if he subject does not have an instructor, people knew that the trips were purely junket. It also stated that about
commenting   such   greed   for   money   on   the   part   of   AMEC’s   P700,000 was collected by way of cash advances by ranking provincial
administration 3) the administrators of AMEC BCCM, AMEC Science HS officials, at the instance of the Governor and without resolution
and the AMEC Institute of Mass Communication in their effort to approving its release, was allegedly used for the two trips. It also
minimize expenses in terms of salary are absorbing or continues to reported that Villafuerte claimed that he spent his own money for the
reject making reference to many teachers in AMEC who were former trips. The petitioner admitted that he wrote the questioned news items
teachers of Aquinas University but were removed because of on the basis of a note given to him by a source whom he refused to
Immorality. In offering to prove that AMEC is a dumping ground, identify. Said source was allegedly connected with the Provincial
garbage, not merely of moral and physical misfits, mention was made Treasurer’s  office.   He   said  that   prior   to   writing   the   article,   he   went   to  
of the case of the Dean of Student Affairs of AMEC, Justita Lola, which his source to ask some clarificatory questions and was given
according to Alegre, as the family name implies she is too old to work, authenticated records of the cash advances. Villafuerte claimed that no
being an old woman. On the other hand, Rima echoed Alegre’s   one from Bicol Forum made any attempt to get his side of the story nor
allegations about Dean Lola. AMEC filed a complaint for damages confirm the veracity of the contents of the article from any source at
claiming that the broadcasts were defamatory, against FBNI, Rima and the provincial capitol. To him, the Bicol Forum seemed to be making a
Alegre. In reply, they claimed that they were plainly impelled by a sense mockery of his previous explanations regarding the cash advances and
of public duty to report the going on in AMEC, which is an institution his trips abroad and such a sweeping statement subjected him to public
imbued with public interest. The trial court rendered a decision finding ridicule and humiliation. The trial court rendered a decision convicting
FBNI and Alegre liable for libel and held that the broadcasts are libelous both accused which CA affirmed.
per se. In absolving Rima from the charge, the trial court ruled that
Rima’s only   participation   was   when   he   agreed   with   Alegre’s   expose.   HELD: The sole issue here is if the questioned news is libelous. We
The   parties   appealed   to   the   CA,   which   affirmed   the   trial   court’s   reverse. Clearly when confronted with libel cases involving publication
judgment with modification, but made Rima solidarily liable with FBNI which deal with public official and the discharge of their official
and Alegre. Hence, this petition. functions, this Court is not confined within the wordings of the libel
statute; rather, the case should likewise be examined under the
HELD: We deny the petition. There is no question that the broadcasts constitutional precept of the freedom of the press. A public official is
were made public and imputed to AMEC defects or circumstances barred from recovering damages in cases involving defamations. His
tending to cause it dishonor, discredit and contempt. We do not agree entitlement, however, is limited to instances when the defamatory
with FBNI contention that Rima and Alegre did not act with malicious statement was made with actual malice-that is with the knowledge
intent. Every defamatory imputation is presumed malicious. Rima and that it was false or with reckless disregard of whether it was false or
Alegre failed to show adequately their good intention and justifiable not.
motive in airing the supposed gripes of the students.
Vasquez v. CA
Some  US  courts  apply  the  privilege  of  “neutral  reportage” in libel cases 314 SCRA 460 ( 1999)
involving matter so of public interest or public figures. Under this
privilege, a republisher who accurately and disinterestedly reports FACTS: Petitioner Vasquez, a resident of Tondo Foreshore area,
certain defamatory statements made against public figures is shielded together with other families went to see then NHA General manager
from liability regardless of the republisher’s  subjective  awareness  of  the   Lito Atienza regarding their complaint against their Bgy. Chairman,
truth or falsity of the accusation. Rima and Alegre cannot invoke the Jaime Olmedo. After their meeting with Atienza and other NHA
privilege of neutral reportage because unfounded comments abound in officials, petitioner and his companions were interviewed by newspaper
the broadcasts. Moreover, there is no existing controversy involving reporters concerning their complaint. The following day, a news article
AMEC when the broadcasts were made. The privilege of neutral appeared in the newspaper Ang Tinig ng Masa saying that the families
reportage applies where the defamed person is a public figure who is of the Tondo Foreshore Area complained about their Bgy. Chairman
involved in an existing controversy and a party to that controversy who, in connivance with some of the project managers of NA, managed
makes the defamatory statement. to get for themselves some 14 lots in the said area. It was also reported
that Olmedo was involved in illegal gambling and theft of fighting cocks.
Had the comment been an expression of opinion based on established Based on the article, Olmedo filed a complaint for libel against
facts, it is immaterial that the opinion happened to be mistaken, as long petitioner   alleging   that   the   latter’s   statements   cast   aspersions   in   him  
as it might reasonably be inferred from the facts. However, the and his damaged reputation. The trail court found petitioner guilty of
comments of Rima and Alegre were not backed up by facts. Therefore, libel. The CA affirmed.

46 | P LATON
amnesty under PD 1740 and 1840. In both, petitioner did not recognize
HELD:The question for determination in this case is the liability for libel that his sale of land to AYALA was on cash basis. Reacting to the news
of a citizen who denounces a bgy official for misconduct in office. SC article, petitioner filed with the RTC an action for damages against BIR
held that the decision of CA must be reversed. Petitioner contends that for   extortion   and   malicious   publication   of   the   BIR’s   tax   audit   report,  
what he said was true and was made with good motives and for claiming that the filing of criminal complaints against him for violation
justifiable ends which the SC found merit. of tax laws were improper because he had already availed of 2 tax
amnesty decrees, The trial court decided in favor of the respondents
To find a person guilty of libel under Art 353 of the Revised Penal Code, and the CA affirmed. Before the SC, petitioner questions the propriety
the following elements must be proved: a) the allegation of a of awarding damages to Larin.
discreditable act or condition concerning another, b) publication of
the charge, and c) identity of the person defamed, and d) existence of
malice. In this case, the first 3 elements are present. The question is HELD: Moral damages may be recovered in cases involving acts referred
whether from the fact that the statements were defamatory, malice to in Art 21 of the Civil code. As a rule, a public official may not recover
can be presumed so that it was incumbent upon petitioner to damages for charges of falsehood related to his official conduct unless
overcome such presumption. Under Art. 361 of the Revised Penal code, he proves that the statement was made with actual malice.
if the defamatory statement is made against a public official with
respect to the discharge of his official duties and functions and the We   appreciate   petitioner’s  claim   that   he   filed  his  1972 return in good
truth of the allegation is shown, the accused will be entitled to an faith and he had honestly believed that the law allowed him to declare
acquittal even though he does not prove that the imputation was the sale of the land in installment. We can further grant that the
published with good motives and for justifiable ends. In this case, pertinent tax laws needed construction. The petitioner was offended by
contrary to the findings of the lower court, petitioner was able to prove the headlines alluding him as a tax evader is fully understandable,
the truth of his charges against the bgy official. however, all these, do not justify what amounted to a baseless
prosecution of respondent Larin. Petitioner presented no evidence to
It   was   error   for   the   lower   court   to   hold   that   petitioner   “only   tried   to   prove Larin extorted money from him. He even admitted that he never
prove that the complainant (bgy official) is guilty of the crimes alluded met nor talked to the   respondent.   On   Larin’s   instruction,   the   tax  
to; Accused, however, has not proven that the complainant committed assessment was reduced. Petitioner went on to file the extortion cases
the  crimes.  ”For  that  is  not  what  petitioner said as reported in the Ang against Larin in different fora and this is where actual malice could
Tinig ng Masa. The fact that charges had been filed against the bgy. attached  on  petitioner’s  part.  We  are  constrained  to  agree  that  there is
official, not the truth of such charges, was the issue. sufficient basis for the award of moral and exemplary damages in favor
of the respondent. He suffered anxiety and humiliation because of the
Privacy, Expression and Damages unfounded charges against him.

The award is in favor of a government official in connection with his


The question here is in determining to what extent such an
official function, it is with caution that we affirm granting moral
action for vindication of private rights or sense of self-esteem damages, for it might open the floodgates for government officials
and worth may be held consistent with the guarantees of the counter claiming damages in suits filed against them in connection with
liberty   to   speak   one’s   mind   and   publicize   what   he   may   have   their functions. Moreover, we must be careful lest the amounts
discovered or learned awarded make citizens hesitate to expose corruption in the
government, for fear of lawsuits from vindictive government officials.
Bañas, Jr. v. CA Hence, we reduce the moral and exemplary damages in this case.
325 SCRA 259 (2000)
Lopez vs. Court of Appeals
FACTS: In 1976 petitioner sold to Ayala Investment Corp. 28,265 sq. 34 SCRA 116 (1970)
mtrs. Located in Muntinlupa, for P 2.3M. The Deed of Sale provided
that upon signing of the contract, Ayala shall; pay 400K with the Sanitary inspector assigned to babuyan islands, Fidel Cruz , sent a
balance of 1.8M++ was to be paid in 4 equal consecutive annual distress signal that there are series of killings in the island, however
installments, with 12% interest pa on the outstanding balance. The when Philippine defense team got there, they found out that he only
periodic payment of 400K shall be payable starting on Feb 20 1977 and wanted transportation to manila. He was called “hoax  of  the  year”.  But  
every year thereafter or until Feb 20 1980. On the same day, however, Together with the article, The Manila Chronicle mistakenly published
petitioner discounted the promissory note with AYALA, for its face the photograph of Fidel G. Cruz,former mayor of bulacan. The
value of P 1.8M evidenced by a Deed of Assignment signed by the publishing was corrected by the MC. Cruz filed for damages, He was
petitioner and AYALA. The latter then issued 9 checks to petitioner, all awarded P11,000 by CA.
dated Feb. 20 1976, drawn against BPI with the uniform amount of
200K++. In his 1976 Income tax Return, petitioner reported the 400K++ SC: So long as it is done in good faith, newspapers have the legal right
initial payment as income from disposition of capital asset. In the to have and express opinions on legal questions. To deny them of that
succeeding years, until 1979, petitioner reported a uniform income of right would infringe upon the freedom of the press. The newspapers
200K++ as gain from sale of capital asset. In is 1980 income tax amnesty should be given leeway and tolerance to enable them to courageously
return, petitioner also reported the same amount of 200K++ as the and effectively perform their important role in our democracy. They
realized gain on disposition of capital asset for the year. In 1978, BIR should not be punished for honest mistakes and imperfect choice of
examined the books of petitioner for 1976. They discovered that he had words if done in good faith. However in this case, there was no
no outstanding receivable from the 1976 land sale, concluding that the pressure of a deadline to meet, no occasion to haste and they should
sale was cash and the entire profit should have been taxable in said have imposed reasonable care. The correction promptly made would
year since the income was wholly derived then instead of being spread reduce   the   damages   awarded.   The   practice   being,   “more   likely   to  
over 4 years. They assessed a tax deficiency of P 2.4M. After reviewing reduce  damages  for  libel  than  to  increase  them”
the    examiner’s  report,  Larin  as Regional Director of Manila Region IVA
of BIR directed the revision of the report in order to consider the land BULLETIN PUBLISHING CORPORATION V. NOEL
as capital asset, which resulted to the reduced tax deficiency of 900K++. 167 SCRA 255 (1988)
On Sept. 1980, petitioner acknowledged receipt of the BIR assessment
but insisted that the sale of his land to AYALA was on installment. In Bulletin publishing corp published an article claiming Lanao politics
1981, BIR charged the petitioner with tax evasion. It appeared in the being dominated by big royal families and the only time one who was
Evening News Express, Evening Post and Bulletin Today. All news items not of any royal house became a leader was during the American era
mentioned   petitioner’s   false   income   tax   return concerning the sale of when the late Amir Mindalano held some sway. Relatives of mindalano
land to AYALA. Meanwhile on July 2 1981, petitioner availed of tax filed a complaint for damages against petitioners for libel because

47 | P LATON
mandalanos belong to royal houses and he did not live with an Collective bargaining negotiations between chief negotiator(bartnicki)
American family. and president (kane) of union representing teachers were unlawfully
intercepted and recorded. Vopper, a radio commentator played it in his
SC: PETITION GRANTED. Libel has not been committed. Personal hurt or show. Yocum, who delivered the tape to vopper, said that he found it in
embarrassment is not automatically equivalent to defamation. The law his mailbox. The information was acquired by the respondents lawfully.
against   defamation   protects   one’s   interest   in   acquiring,   retaining,   and   According to district court, under statutory language, an individual
enjoying   a   reputation   “as   good   as   one’s   character and conduct violates the federal act by intentionally disclosing the contents of an
warrant”   in   the community and it is to community standards- not electronic communication when he knows or has reason to know that
personal or family standards-that a court must refer in evaluating a the information was obtained by illegal interception even if the
publication claimed to be defamatory. A newspaper should be free to individual was not involved in the interception. CA stated the statute
report on events and developments in which the Public has legitimate invalid because they dettered more speech than necessary to protect
interest, wherever they may take place within the nation and as well as the private interest at stake.
in the outside world, with minimum fear of being hauled to court so
long as newspapers keep within the standards of morality and civility Whether the punished publisher of information has obtained the
prevailing within the general community. Any other rule on defamation information in question in a manner lawful in itself but from a source
in a national community like ours with many, diverse cultural, social who has obtained it unlawfully, may the government punish the
religious and other groupings is likely to produce an unwholesome ensuing publication of that info based on the defect of the chain?
“chilling   effect”   upon   the   constitutional   protected   operations   of the
press. Privacy of communication is an important interest. Privacy concerns
give way when balanced against the interest in publishing matters of
Ayer Productions PTY. LTD. VS Capulong public importance.
160 SCRA 861 (1988)
SC   .   AFFIRMED.   Stranger’s   illegal   conduct   does   not   suffice   to   remove  
Petitioner proposed a motion picture to re enact the events that made the first amendment shield from speech about a matter of public
possible the EDSA revolution. Juan Ponce Enrile did not approve his and concern.
his   family’s   inclusion   in   the   docu-drama. He was deleted from the
script. Thereafter, he still filed to enjoin production of the film without Miller vs California
his consent and that it constitutes violation of his right to privacy. 413 US 15, 37 ED 2d 419, 93 S Ct 2607 (1973)
Petitioners claim they are exercising their freedom of speech and
expression. Appellant   conducted   mass   mailing   campaign   to   advertise   “adult  
materials”.   An   unsolicited   mail   containing   pictures   and   drawings  
The right of privacy like the right of free expression is not absolute. depicting men and women engaging in various sexual activities was
Limited intrusion is permissible where the person is a public received by a restaurant manager and his mother. He was convicted of
figure(anyone who has arrived at a position where public attention is misdemeanor.
focused upon him as a person) and the information is matters of public
character. The interest protected by the right of privacy is the right to The first amendment protects works which , taken as a whole, have
be free from unwarranted publicity, wrongful publicizing of private serious literary, artistic, political or scientific value regardless of
affairs and activities outside the realm of legitimate public concern. 1. whether the government or a majority of the people approve of the
Clear and present danger rule 2. Balancing of interest test are ideas these works represent. The protection given speech and press
limitations of freedom of speech and of the press. was fashioned to assure unfettered interchange of ideas for the
bringing about of political and social changes desired by the people. But
Such public figures have lost to some extent their right to privacy. The the portrayal of hardcore sexual conduct for its own sake and for
press had a privilege under the constitution to inform the public about commercial gain is a different matter. Roth holding that Obscene
those who have became legitimate matters of public interest. The material is not protected by the first amendment is affirmed. Obscenity
privilege of enlightening the public was not limited to the dissemination is   determined   by   applying   “contemporary   community   standards”   not  
of news in the sense of current events. It extended to information or “national  standards”
education or even entertainment and amusement, by books, articles,
pics, films and broadcasts concerning interesting phases of human Stanley vs. Georgia
activity in general, as well as the reproduction of the public scene in 394 US 557 , 22 L Ed 2d 524 (1969)
newsreels and travelogues.
An   investigation   of   appelant’s   bookmaking   activities   led   to   a   search  
The film itself limits portraying only the participation of Enrile that are warrant. While searching, obscene films were found and seized.
directly related to the public facts of Edsa. Such can be carried out
without license from him. However, there must be no reckless Won a statute imposing criminal sanctions upon mere possession of
disregard of truth in depicting of Enrile, no representation of private obscene matter is constitutional.
life.
No. First and 14th amendments prohibit making mere possession of
Arts, Letters and Obscenity obscene material as a crime. The states retain broad power to regulate
obscenity but that power does not extend to mere possession by the
The problem is in discerning to what extent it may be individual in the privacy of his home. If the first amendment means
anything it means that the state has no business telling a man what
considered art and allowed expression and display and where it
books he may read or what films he should watch. The case cannot be
crosses over to obscenity, an area that falls beyond the decided simply by citing roth. Publication and mere possessing does not
protection of freedom of expression impose same dangers.

The constant challenge to the courts then is how to provide New York v. Ferber
guidelines by which people may determine what is allowed 458 U.S. 747, 73 L Ed 2d 1113, 102 S Ct 3348 (1982)
form what is proscribed
Facts: A New York statute prohibits person from knowingly promoting a
Bartnicki vs. Vopper sexual performance by a child under age of 16 by distributing material
532 US 514, 149 Led 787, 121 S Ct 1753 (2001) which depicts such performance. Respondent sold to an undercover
police officer two films devoted almost exclusively to depicting young

48 | P LATON
boys masturbating.
As to what might be permissible regulations or restrictions
Issue: Whether or not New York criminal statute which prohibits beyond that is something that the courts may continue to
persons from knowingly promoting sexual performances b children
grapple with as cases are presented before the, questioning, for
under the age of 16 by distributing material which depicts such
performances is constitutional or overbroad.
instance, the classification that might have been accorded a
particular work, and so on
Held: No, a trier of fact need not find that the material appeals to the
prurient interest of the average person, it is also not required that Times Film Corporation v. City of Chicago
sexual conduct portrayed be done so in a offensive manner and the 365 U.S. 43, 5 L Ed 2d 403, 81 S Ct 391 (1961)
material at issue need not be considered as a whole. The law should
not be invalidated for overbreadth unless it reaches a substantial Facts: Municipal Code of Chicago requires submission of all motion
number of permissible application is hardly novel. Therefore it is pictures for examination or censorship prior to their public exhibition
considered as a paradigmatic case of a state statute whose legitimate and forbids their exhibition unless they meet certain standards. The
reach dwarf its arguably impermissible applications. film   known   as   “Don   Juan”   applied   for   a   permit   but   refused   to   submit  
the film for examination. Appropriate city officials refused to issue
Ashcroft v. Free Speech Coalition permit  for  the  reason  of  the  petitioner’s  denial  to  submit  the  film.  The  
535 U.S. 234, 152 L Ed 2d 403, 122 S Ct 1389 (2002) petitioner on the other hand brought a suit seeking injunctive relief
ordering the issuance of the permit. For the reason that the provision
Facts: Child Pornography Act of 1996 (CCPA) expanded the prohibition of the ordinance requiring submission of the film constitutes, on its face
on child pornography to include not only pornographic images made a prior restraint within the prohibition of the first and Fourteen
using actual children but also any visual depiction like photograph, film, amendments.
video, picture or computer or computer–generated image or picture
that depict a minor engage in sexual explicit conduct, also known as Issue: the precise question at issue here never having been specifically
virtual child pornography. Free speech coalition an adult entertainment decided.
trade   association   filed   a   suit   alleging   that   the   “appears   to   be”   and  
“conveys  the  impression”  provisions  are  overboard  and  vague,  chilling   Held: Motion picture censorship cases which reached the court
production of works protected by First amendment. involved  question  of  standards.  The  challenge  here  is  the  censor’s  basic  
authority; it does not go to any statutory standards employed by the
Issue: Whether or not the mentioned provision abridges the freedom of censor or procedural requirements as to the submission of the film.
speech.
Movie and Television Review and Classification Board (MTRCB) v. ABS –
Held: CPPA prohibits speech despite its serious literary, artistic, political CBN Broadcasting Corporation 448 SCRA 575 (2005)
or scientific value. The statute proscribes the visual depiction of an idea
teenager engaging in sexual activity that is a fact of modern society and Facts:   “The   Inside   Story”   a   public affairs show of the ABS – CBN
has been a theme in art and literature throughout the ages. Virtual network,   had   aired   “Prosti–tuition”   episode   which   the   main   topic   is  
child pornography is not intrinsically related to the sexual abuse of about students who enters into prostitution in order to earn money, to
children. The harm does not necessarily follow from the speech, but be use for their enrolment. It was alleged that the school the most of
depends upon some unquantified potential for subsequent criminal these students   came   from   is   from   Philippine   Women’s   University  
acts. Therefore provision is considered overbroad and (PWU). The reason was in the said episode PWU was the background.
unconstitutional. The parents and teacher association of PWU filed a complaint under
the MTRCB, alleging that the episode is a besmirched the name of
Pita v. Court of Appeals PWU. That the petitioner did not submit the episode to the petitioner
178 SCRA 362 (1989) for review and that they exhibit it without permission violating
presidential decree No. 1986.
Facts: Pursuant to an Anti – Smut Campaign, Western Police district,
INP of Metropolitan Police Force of Manila, seized and confiscated from Issue: whether or not the MTRCB has authority to review the above
dealers, distributors, newsstand owners and peddlers along Manila mentioned show.
sidewalks magazines, publications and other reading materials believed
to be obscene, pornographic and indecent and later burned the seized Held: Yes, there are only two exceptions where MTRCB cannot review a
materials. Among seized publication seized was Pinoy Playboy certain show; 1) that the television program imprinted or exhibited by
magazine. The company filed a case for injunction seeking to restrain the Philippine Government, its department or agencies 2) newsreels.
respondents   confiscating   petitioner’s   magazine   claiming   that   the   Newsreels are straight news reporting and therefore such program is
magazines are decent, artistic and educational magazine, which not considered to be one for it is more of a public affairs program which
protected by the constitution which guarantees of freedom of speech is in no contest is subjected to MTRCB review.
and of the press.
Freedman v. Maryland
Issue: Whether or not such confiscation is unconstitutional. 380 US 51, 13 L Ed 2d 649, 85 S Ct 734 (1965)

Held: The court rejected the argument that there is neither Facts: Appellant sought to challenge the constitutionality of the
constitutional nor legal provision which would free the accused from all Maryland motion picture censorship statute Md. Ann. Code, 1957, Art.
criminal responsibility because of the absence of the warrant. The 66A,   and   exhibited   the   film   “Revenge   at   Daybreak”   at   his   Baltimore  
reasons are first there is no accused to speak of and second would be theatre without first submitting the picture to the State Board of
the Mayor could have directly ordered the raid without search warrant Censors as required.
for the reason that the violation of the penal law was already
committed. Issue: Whether or not non issuance of license prior to the failure to
submit the picture is unconstitutional.
Films, Review, Classifications and Censorship
Held: Maryland does not satisfy the following criteria 1) once the
censor disapproves the film, the exhibitor must assume the burden of
An issue that also presents itself to the courts is the matter of instituting judicial proceedings and persuading the courts that the film
censorship or prior review of motion pictures or pre-recorded is protected expression 2) once the board has acted against a film,
broadcasts before exhibition exhibition is prohibited pending judicial review 3) Maryland statute

49 | P LATON
provide no assurance of prompt judicial determination. Maryland to potentially harmful speech, the CDA effectively suppresses a large
scheme fails to provide safeguards against inhibition of protected amount of speech that adults have a constitutional right to receive and
expression, and this renders the requirement of prior submission of to address to one another.
films to the Board an invalid previous restraint.
Where obscenity is involved, we have consistently held that the fact
Gonzalez v. Kalaw Katigbak that protected speech may be offensive to some does not justify its
137 SCRA 717 (1985) suppression. It is true that we have repeatedly recognized the
governmental interest in protecting children from harmful materials
Facts: Gonzalez president of the Malaya Films produced the movie but that interest does not justify an unnecessarily broad suppression of
“Kapit  sa  patalim.”  In  a  resolution  of  a  sub  – committee of respondent speech addressed to adults ( example: since ang gusto ng government
Board of Review for Motion Pictures and Television (BRMT) a permit para macheck kung minor ka is magswipe pa ng credit card, etc. E since
was exhibit but the film was under the classification “For  adults  only.”  A   yung ibang adult walang credit card, Boom!)
motion for reconsideration was filed by the petitioners stating that the
classification of the film was without legal basis. Moreover,   the   general,   undefined   terms   “indecent”   and   “patently  
offensive”   cover   large   amounts   of   nonpornographic   material   with  
Issue: Whether or not there was been a grave abuse of discretion by serious educational or other value.
the board in the light of the difficulty and travail undergone by
petitioners  before  Kapit  sa  Patalim  was  classified  as  for  “Adults  Only.”   United States v. Playboy Entertainment Group, Inc.
529 U.S. 803, 146 L Ed. 2d 865, 120 S Ct 1878 (2000)
Held: Obscene material is material which deals with sex in a manner
appealing to prurient interest. The perception of what constitutes Playboy Entertainment Group owns and prepares programs for adult
obscenity appears to be unduly restrictive. Light of the facts in this case television networks. Playboy transmits its programming to cable
should be construed in such fashion top avoid any taint of television operators, who retransmit it to their subscribers. Playboy
unconstitutionality. Court concludes then that there was an abuse in challenges 505 of the Television Communication Act of 1996, which
discretion nonetheless there is no enough votes to maintain that such requires   cable   television   operators   who   provide   channels   “primarily  
abuse can be considered grave. All that remains to be said is that the dedicated to sexually-oriented  programming”  either  to  “fully  scramble  
ruling is to be limited to the concept of obscenity applicable to motion or  otherwise  fully  block”  those  channels  or  to  limit  their  transmission  to  
pictures. hours when children are unlikely to be viewing, set by administrative
regulation as the time between 10 pm to 6am. Playboy contends that
Modern Amenities and State Regulations the stature is unnecessarily restrictive content-based legislation
violative of the First Amendment. Disctrict court ruled content based
restriction on speech violates the First Amendment because the
There would necessarily have to be adaptations and
Government might further its interests in less restrictive ways.
adjustments as some features of these advances may not
readily fit into the traditional molds in which the rights and Issue: Whether or not DC erred in its ruling that the legislation violated
regulations coexisted in the past the First Amendment

Reno v. American Civil Liberties Union Decision: Laws designed or intended to suppress or restrict the
521 U.S. 844, 138 L Ed 2d 874, 117 S Ct 2329 (1997) expression of specific speakers contradict the First Amendment
principles.   Section   505   limited   Playboy’s   market   as   a   penalty   for   its  
Two provisions of Communications Decency Act of 1996 seek to protect programming choice, though other channels capable of transmitting
minors from harmful material on the internet. 223 (a)(1)(b)(ii) like material are altogether exempt.
criminalizes   the   “knowing   transmission   of   “obscene   and   indecent”  
message to any recipient under 18 years of age. Section 223(d) 505 is a content based speech restriction, it can only stand of it satisfies
prohibits  the  “knowin[g]”.  Sending  or  displaying  to  person  under  18  of   strict scrutiny. If a statute regulates speech based on its content, it
any   message   “that,   in   context,   depicts,   describes,   in   terms   patently   must be narrowly tailored to promote compelling Government interest.
offensive as measured by contemporary community standards, sexual If a less restrictive alternative would serve  the  Government’s  purpose,  
or   excretory   activities   or   organs.”   A   number   of   plaintiff   files   suit the legislature must use that alternative.
challenging the constitutionality of 223(a)(1) and 223(d). Three judge
of District court convened and entered a preliminary injunction against As we consider a content-based regulation, the answer should be clear:
enforcement   of   both   challenged   provisions.   The   court’s   judgment   The standard is strict scrutiny. This case involves speech alone, and
enjoins the government from enforcing 223(a)(1)(B)’s   prohibitions   even where speech is indecent and enters the home, the objective of
insofar as they relate to indecent communications, but reserves the shielding children does not suffice to support a blanket ban if the
right of the Government to investigate and prosecute the obscenity or protection can be accomplished by a less restrictive alternative.
child pornography activities prohibited therein. The injunction against
223(3) is unqualified because that section contains no separate Cable systems have the capacity to block unwanted channels on a
reference to obscenity and child pornography. Government appealed household-by-household basis. Targeted blocking is less restrictive than
arguing the DC erred in its ruling. banning, and the Government cannot ban speech if targeted blocking is
a feasible and effective means of furthering its interests.
Issue: Whether or not the two provisions enacted to protect minors
from indecent and patently offensive communications on the internet In problems like these, the Government has the burden to formulate a
constitutional solution to address such problems. It must do so in way consistent
with First Amendment Principles. The Government has failed to show
Decision: Yes there is a violation. The Court upheld the decision of the that 505 is the least restrictive means for addressing a real problem.
District Court.
Fighting Words Doctrine
Rationale:  The  stature  abridges  “the  freedom  of  speech”  protected  by   -One of the exceptions to the guarantee of freedom of expression.
the First Amendment. -Those which, by their very utterance, inflict injury or tend to incite an
immediate breach of the peace.
The CDA is a content based regulation on speech.
Heckler’s  veto
The CDA lacks the precision that the First Amendment requires when a -The opposition of a rowdy or obstreperous crowd might as well drown
stature regulates the content of speech. In order to deny minors access out the voice of the one seeking to exercise the right to speak. In legal

50 | P LATON
context, that might come from in the form of regulations tending to is a valid limitation under the clear and present danger doctrine.
prevent the expression of an idea in view of the reaction that might be
engendered among those opposed to it. Once such situation is found to exist, there is no limit to the allowable
-This may be in the guise of a permit requirement in the holding of limitations on such constitutional rights. Evils substantial in character
rallies, parades or demonstrations conditioned on the payment of a fee taint the purity of the electoral process.
computed on the basis of the cost needed to keep order in view of the
expected opposition by persons holding contrary views. It is no exaggeration to state that violence and even death did
frequently occur because of the heat engendered by such political
activities. The opportunity for dishonesty and corruption, with the right
The  “Fighting  Words”  Doctrine  and  the  “Heckler’s  Veto” to suffrage being bartered, was further magnified. The legislative body
must have felt impelled to impose the foregoing restrictions. Without
such restrictions, the laudable purpose of R.A. 5880 would be
As noted earlier, one of the exceptions to the guarantee of
frustrated and nullified.
freedom of expression is the species of speech referred to as
“fighting   words”--those which, by their very utterance, inflict We give due recognition to the legislative concern to cleanse, and if
injury or tend to incite an immediate breach of the peace--since possible, render spotless, the electoral process.
they   are   “no   essential   part   if   any   exposition   of   ideas,   and   are  
such slight social value as a step to truth that any benefit that Sanidad v. Commission on Elections
may be derived from them is clearly outweighed by the social 181 SCRA 529 (1990)
interest in order and morality
In relation to the holding of a plebiscite for the ratification of the
Organic Act for Cordillera Autonomous Region, Resolution no. 2167
“Heckler’s   veto”   comes   in   the   form   of   regulations   tending   to  
prohibits columnists, commentators or announcers during the
prevent the expression of an idea in view of the reaction that plebiscite campaign period, on the day before and on plebiscite day,
might be engendered among those opposed to it. This may be from using their columns or radio or television times to campaign for or
in the guise of a permit requirement in the holding of rallies, against the plebiscite issues. Petitioner, who claims to be a newspaper
parades or demonstrations conditioned n the payment of a fee columnist   of   “overview”   fir   the   Baguio   Midland   Carrier,   alleges   that  
computed on the basis of the cost needed to keep order in view such provision is void. He maintains that unlike regular news reporter,
of the expected opposition by persons holding contrary views his column obviously and necessarily contains his opinions, views, and
beliefs on any issue. Comelec counters that the provision is a valid
implementation of their power to supervise and regulate media during
Governance, Elections and Speech
election or plebiscite period under Article IX-C, section 4 of the
Constitution Moreover; he may still express his views for or against the
At the core of the freedom of expression is speech directed at act through the Comelec space and airtime.
the political order
Issue: Whether or not Resolution 2167 is unconstitutional.
The   assertion   of   one’s   freedom   to   speak   or   to   express   one’s  
thoughts would have to be balanced against the other interests Decision: Unconstitutional.
of the state, be it in keeping peace and order, clean, honest and
Rationale: It is clear from Art. IX-C that the evil sought to be avoided is
credible elections, making equitable access to media for
the possibility that a franchise holder may favour or give any undue
publicity or in preventing visual blights, all of which may come advantage to a candidate in terms of advertising space or radio or
as an inevitable consequence of campaign activities, spending television time. This is also the reason why a columnist, commentator,
and posting election materials announcer or personality, who is a candidate for any elective office is
required to take a leave of absence from his work during the campaign
Gonzales v. Commission on Elections period. It shall not be construed to mean that the Comelec has also
27 SCRA 835 (1969) been granted the right to supervise and regulate the exercise by media
practitioners themselves of their right to expression during plebiscite
Petitioners, a private individual and a councillor and a candidate for periods. Media practitioners exercising their freedom of expression are
vice-mayor of Manila, challenged the validity of two sections in the neither the franchise holder nor the candidates; in fact there are no
Revised Election Code, under R.A. 4880, which prohibited the too early candidates involved in a plebiscite.
nomination of candidates and limiting the period of election campaign
or partisan political activity. The law provides however that simple Osmena v. Commission on Elections
expression of opinion and thoughts concerning the election shall not be 288 SCRA 447 (1998)
considered as part of an election campaign, and further provision that
nothing stated in the   Act   “shall   be   understood   to   prevent   any   person   In National Press Club v. Comelec, the court upheld the constitutionality
from expressing his views on current political problems or issues, or of Section 11(b) of R.A. no. 6646 which prohibits mass media from
from mentioning the candidates for public office whom he supports. selling or giving free of charge print space or air time for campaign or
other political purposes, except the COMELEC. Petitioners, candidates
Issue: Whether or not the enforcement of R.A. 4880 prejudice the basic for public office, seek a re-examination of the validity of aforecited
rights such as freedom of speech and assembly. provision, contending that events after the ruling in National Press Club
“have   called   into   question   the   validity   of   the   very   premises”   if   that  
Decision: Does not prejudice. R.A. is constitutional. decision.

Rationale: Freedom of speech or of the press involves the liberty to There is no case or controversy to decide, only an academic discussion
discuss publicly and truthfully any matter of public interest without to hold.
censorship or punishment. It means something more than the right to
approve existing political beliefs or economic arrangements, to lend Decision: Petition is dismissed.
support to official measures, to take refuge in the existing climate of
opinion on any matter of public consequence. Petitioners claim that the experience in the last five years since the
decision has shown undesirable effects of the law. However, petitioners
Freedom  of  speech  could  only  be  limited  if  there  through  the  “clear  and   do not complain of any harm suffered as a result of the operation of the
present  danger”  rule  and  the  “dangerous  tendency”  rule.  The  said  R.A.   law. What petitioners seek is not the adjudication of a case but simply

51 | P LATON
the holding of an academic exercise. conflict with official COMELEC count as well as the unofficial quick
count of NAMFREL, and ABS-CBN did not have any authorization nor
The  law’s  concern  is  not  with  the  message  or  content  of  the  ad  but  with   deputized by the Commission. Thus, the petition for certiorari,
ensuring  media  equality  between  candidates  with  “deep  pockets”  and   petitioner arguing that holding of exit polls and the nationwide
those with less resources. There is not total ban on political ads, much reporting of their results are valid exercises of freedoms of speech and
less restriction on the content of the speech. Given money could be of a of the press.
disadvantage of a poor candidate, there is a substantial or legitimate
governmental interest justifying exercise of the regulatory power of the Holding of exit polls and the dissemination of their results through mass
COMELEC. media constitute an essential part of the freedoms of speech and of the
press. The COMELEC cannot ban them totally in the guise of promoting
Puno, J. Concurring: clean, honest, orderly and credible elections.
The guaranty of freedom of speech should not be used to frustrate
legislative attempts to level the playing field in politics. R.A. 6646 does Social Weather Stations, Inc. v. Commission on Elections
not curtail speech as it no more than prevents the abusive wealth by 357 SCRA 496 (2001)
the rich to frustrate the poor  candidate’s  access  to  media.  If  we  allow  
money to monopolize the media, the political framework will cease to Social Weather Stations, Inc. (SWS) is a non-stock social research
be a market of ideas but a market for influence of the rich. institution which conducts surveys in various fields and thereafter
processes, analyzes, and publicly reports the results thereof, while
Blo Umpar Adiong v. COMELEC petitioner Kamahalan Publishing Corporation publishes the Manila
207 SCRA 712 (1992) Standard, a newspaper of general circulation. Petitioners bring action
for prohibition from implementing Sec. 5.4 of R.A. No. 9006, the Fair
Comelec promulgated Resolution no. 2347, regulating election Election Act, which the former claim unconstitutional. Sec. 5.4 provides:
propaganda. The said resolution allows pamphlets, decals, stickers, etc. “Surveys   affecting   national   candidates   shall   not   be   published   fifteen  
To be posted only in any of the authorized posting areas provided in the (15) days before an election and surveys affecting local candidates shall
same resolution and declares it unlawful to draw, paint, inscribe, post, not be published seven (7) days before an election. SWS, which wishes
display, or publicly exhibit, any election propaganda in any place, to conduct survey throughout the period of elections and to release to
whether public or private, mobile or stationary, except in Comelec the media the results as well as publish them directly, and Kamahalan
common posting area. Petitioner assails the resolution as regards the Publishing, which intends to publish election survey results to the last
prohibition  of  posting  of  decals  and  stickers  in  “mobile”  places  like  cars   day of election survey results to the last day of elections, assail the
and other moving vehicles. The posting of decals and stickers, according aforecited provision as equivalent to prior restraint without any
to him, shall be his last medium to inform the electorate that he is a justification.
senatorial candidate (neophyte sya) since there is already a ban on
radio, television, and print political advertisements. O’Brien   test   – used to determine the constitutional validity. A
government regulation is sufficiently justified: [1] if it is within the
Issue: Whether or not the COMELEC may prohibit the posting of decals constitutional power of the government; [2] if it furthers an important
and   stickers   on   “mobile”   places,   public   or   private   and   limit   their   or substantial government interest; [3] if the governmental interest is
location or publication to the authorized posting areas that it fixes. unrelated to the suppression of free expression.

Decision: Null and void. Sec. 5.4 of RA 9006 constitutes an unconstitutional abridgment of
freedom of speech, expression, and the press. It is invalid because: [1] it
Rationale:  First:  The  prohibition  unduly  infringes  on  the  citizen’s  right  of   imposes a prior restraint on the freedom of expression; [2] it is a direct
free speech. Considering the period of legitimate campaign activity is and total suppression of a category of expression even though such
limited, it becomes obvious that unduly restrictive regulations may suppression is only for a limited period, and [3] the governmental
prove unfair to affected parties and the electorate as well. interest sought to be promoted can be achieved by other means other
than the suppression of freedom of expression.
The posting of decals and stickers in mobile places like cars and other
moving vehicles does not endanger and substantial government Courts, Criticism and Contempt
interest. There is no clear public interest threatened by such activity so
as to justify the curtailment of such right. (Di siya pasok sa clear and
The courts may also express sensitivity at times to their delicate
present danger rule)
and important ole in society such that they feel the need to
Second: The restriction as to where the decals and stickers should be preserve the kind of respect and dignity that they should
posted  is  so  broad  that  it  encompasses  even  citizen’s  private  property,   deserve in order that they could effectively continue to
which in this case a privately-owned vehicle. (Violation ng property discharge their critical function of dispensing justice
without  due  process  of  law  na  ‘to.)
United States v. Bustos
There are many candidates whose names alone evoke qualifications, 37 Phil. 731 (1918)
platforms, programs and ideologies which voter may accept or reject.
When a person attaches a sticker with such a candidate’s  name  on  his   Numerous citizens of Pampanga assembled, prepared and signed a
car and bumper, he is pressing more than the name, he is espousing petition to the Executive Secretary charging Roman Punsalan, justice of
ideas. Our view of the validity of the challenged regulation includes its the peace and Macabebe and Masantol, Pampanga, with malfeasance
effects  in  today’s  particular  circumstances.  We  are  constrained  to  rule   in office and asked for his removal. It was contended that said justice of
against the Comelec prohibition. peace exacted money and property from a complainant, asked for
money in exchange of victory in litigation and paid a complaint in
ABS-CBN Broadcasting Corporation v. Commission on Elections another to justify the shelving of his case. Charges had been instituted
323 SCRA 811 (2000) and Punsalan was acquitted.

During the 1998 National Elections, ABS-CBN prepared a project to Criminal   action   for   libel   against   those   who   petitioned   for   Punsalan’s  
conduct radio-TV coverage of the elections and to make an exit survey removal was then instituted. Defendants contend that their petition for
of the vote during the elections for national officials particularly for the removal of the justice of the peace falls within the protection of the
President and the Vice President. COMELEC issued a Resolution which freedom of speech and right to assembly and to petition for the redress
approved the restraining order to stop ABS-CBN or any other groups of their grievances. Moreover, they contend that the content of their
from conducting such exit survey, believing that such project might petition is to be considered privileged communication and thus, cannot

52 | P LATON
be the basis for a libel case. front of the Justice Hall of Las Piñas was prohibited under the Supreme
Court’s  En Banc Resolution  dated  7  July  1998,  entitled,  “Re:  Guidelines  
The guaranties of a free speech and a press include the right to criticize on the Conduct of Demonstrations, Pickets, Rallies and Other Similar
judicial conduct. The administration of law is a matter of public Gatherings in the Vicinity of the Supreme Court and All Other Courts.
concern. The right to assemble10 and petition11 is the necessary They submit that the Supreme Court gravely abused its discretion
consequence of republican institutions and the complement of the and/or acted without or in excess of jurisdiction in promulgating those
right of free speech. A communication made bona fide upon any guidelines. Freedom of speech and expression despite its
subject-matter in which the party communicating has an interest, or in indispensability has its limitations. It has never been understood as the
reference to which he has a duty, is privileged. Qualified privilege is a absolute right to speak whenever, however, and wherever one pleases,
complaint made in good faith and without malice in regard to the for the manner, place, and time of public discussion can be
character or conduct of a public official when addressed to an officer or constitutionally controlled. As well put by our Justice Isagani Cruz, the
board having some interest on the duty or in the matter. better policy is not liberty untamed but liberty regulated by law where
every freedom is exercised in accordance with law and with due regard
In the case, it is not a case of direct and vicious accusations published in for the rights of others. Court reiterates that judicial independence and
the press, but of charges predicated on affidavits made to proper the fair and orderly administration of justice constitutes paramount
official and thus qualifiedly privileged. Although charges are probably governmental interests that can   justify   the   regulation   of   public’s   right  
not true as to the justice of peace but believed to be true by the of free speech and peaceful assembly in the vicinity of the courthouses.
petitioners. Good faith surrounded their action. Probable cause for Even in the United States, a prohibition against picketing and
them to think that malfeasance or misfeasance in office existed is demonstrating in or near courthouses has been ruled as valid
apparent. The manner in commenting on the conduct of justice of the constitutional.
peace was proper. And finally, charges and the petition were submitted
through reputable attorneys to proper functionary, the Executive Petitioners also claim that this Court committed an act of judicial
Secretary. legislation in promulgating the assailed resolution. They charged that
this Court amended the provisions of Batas Pambansa (B.P.) Blg. 880,
Nestle Philippines, Inc. v. Sanchez otherwise   known   as   “The   Public   Assembly   Act,”   by   converting   the  
154 SCRA 542 (1987) sidewalks and streets within a radius of two hundred (200) meters from
every   courthouse   from   a   public   forum   place   into   a   “no   rally”   zone.  
Union of Filiro Employees and Kimberly Independent Labor Union for Contrary   therefore   to   petitioners’   impression,   B.P.   Blg.   880   did   not  
Solidarity, activism and Nationalism-Olalia intensified the intermittent establish streets and sidewalks, among other places, as public fora. A
pickets they have been conducting in front of Padre Faura gate of the close look at the law will reveal that it in fact prescribes reasonable
Supreme Court building.  They  set  up  pickets’  quarters  on  the  pavement   time, place, and manner regulations. It requires a written permit for the
in front of the SC building, constructed provisional shelters along the holding of public assemblies in public places subject, even, to the right.
sidewalks, set up a kitchen and littered the place. They waved their red Existence of B.P. Blg. 880, however, does not preclude this Court from
streamers and placards with slogans, and took turns haranguing the promulgating rules regulating conduct of demonstrations in the vicinity
court all day using loudspeakers. These acts were done even after their of courts to assure our people of an impartial and orderly
leaders had been received by the SC Justices who were chairpersons of administration of justice as mandated by the Constitution.
the Division s where their cases are pending. Thus, thereafter, the SC en
banc issued a resolution giving the said unions the opportunity to SC is especially vested by the Constitution with the power to adopt
withdraw graciously and requiring the union leaders and their counsels measures essential to an orderly administration of justice. These rules
to show cause why they should not be held in contempt of court. The are designed to ensure the orderly and expeditious conduct of court
counsel for one of the unions apologized to the Court for the acts, business as well as to secure the rights of parties. These court-made
together with an assurance that they will not be repeated. rules have the force and effect of law.

The apologies offered by the respondents were accepted by the Court. Social Weather Stations, Inc. v. Asuncion
The individuals cited who are non-lawyers are not aware that even as 228 SCRA xi (1993)
the rights of free speech and of assembly are protected by the
Constitution, any attempt to pressure or influence courts of justice Manila  Standard  published  an  item  entitled  “Judiciary  worse  than  PNP”,  
through the exercise of either right amounts t an abuse thereof, is no which reported that, according to the opinion polls conducted by the
longer within the ambit of constitutional protection, nor did they Social Weather Station (SWS), the Judiciary had an even lower
realize any such efforts to influence the course of justice constitutes satisfaction rating than the PNP. The report prompted Judge
contempt of Court. The duty and responsibility of advising them, Maximiano Asuncion to initiate, motu proprio,  proceedings  entitled  “In  
therefore, rest primarily and heavily upon the shoulders of their the Matter of Findings of Social Weather Research Group Derogatory to
counsel of record. Atty. Jose Espinas, when his attention was called by the   Judiciary”.   He   then   ordered   the   President   of   SWS,   Prof.   Mahar  
the Court, did his best to demonstrate to the pickets the untenability of Mangahas, to explain why he should not be held in contempt for
their acts. It is their duty as officers of the court to properly apprise distributing to the general public without prior permission from any
their clients on matters of decorum and proper attitude toward courts court findings which tend to directly or indirectly degrade the
of justice. No demonstrations or pickets intended to pressure or administration of justice. An explanation was submitted to the court
influence courts of justice into acting one way or the other on pending and the contempt charge against SWS President was dismissed after
cases shall be allowed in the vicinity and/or within the premises of any finding the explanation submitted satisfactory. After some time, Prof.
and all courts. Mangahas addressed a letter to the Chief Justice intended as formal
complaint against Judge Asuncion for grave abuse of authority and
gross ignorance of the law, in connection with the contempt charge
In Re Petition to Annual En Banc Resolution A.M. No. 98-7-02-SC that was initiated by respondent judge.
296 SCRA xi (1998)
A publication which tends to impede, obstruct, embarrass or influence
Petitioner   Valmonte   applied   for   a   Mayor’s   permit   to   hold   a   rally   and   the courts in administering justice in a pending suit or proceeding,
camp out in front of the Justice Hall of Las Piñas. Office of the Mayor constitutes criminal contempt which is summarily punishable by courts.
refused to issue the permit on the ground that the holding of a rally in A publication which tends to degrade the courts and to destroy public
confidence in them or that which tends to bring them in any way into
disrepute, constitutes likewise criminal contempt, and is equally
10 Assembly – a right on the part of the citizens to meet peaceably for punishable by courts. In the first there is no contempt where there is no
consultation in respect to public affairs. decision which might in any way be influenced by newspaper
11 Petition – any person or group of persons can apply, without fear of penalty,
publication. In the second, the contempt exists, with or without a
to appropriate branch or office of the government for redress of grievances.
53 | P LATON
pending case, as what is sought to be protected is the court itself and the performance of his duties.
its dignity would lose their utility if public confidence is destroyed.
The contention that Professor Mangahas was improperly cited for [His] actuations, in the context in which they were done, demonstrate
contempt for acts or utterances not related to a pending action must, gross irresponsibility, and indifference to factual accuracy and the
therefore, be rejected. What was clearly implicit in the newspaper injury that he might cause to the name and reputation of those whom
report about the results of the SWS poll – in the words of Judge he wrote. They constitute contempt of court, directly tending as they
Asuncion,  “that  the  people  have  more  confidence  with  the  police  that   do to degrade or abase the administration of justice and the judges
with  the  judges”  – in light of the fact, of which judicial notice is taken, engaged in that function.
that said report came out at a time when there already was widespread
publicity adverse to the judiciary, there can be no doubt of its clear WHEREFORE, the Court declares Atty. Emil (Emiliano) P. Jurado guilty
tendency to degrade the administration of justice. Judge Asuncion can of contempt of court.
hardly be faulted for what, at a minimum, he must have felt duty-
bound to do in the circumstances. No question of prior restraint or Speech, Professions and Callings
violation of the guarantee of free speech arises here, what he did being,
in essence, merely to initiate an inquiry into the source and basis of the
While  the  right  to  freely  speak  one’s  mind  may  be  available  to  
derogatory news report. And he forthwith abated the proceedings
upon receiving an explanation he deemed satisfactory. the people, it does not necessarily mean that it is allowed to
the  same  extent  for  everyone.  Depending  on  one’s  occupation  
In Re Emil P. Jurado or calling, the extent to which he may exercise that right may
243 SCRA 299 (1995) be affected, and his behavior, including the manner by which
he may express himself, would have to be accordingly adjusted
Facts: Jurado is a journalist who writes a column entitled “Opinion” in a
newspaper of general circulation, the “Manila  Standard.” He describes Zaldivar v. Sandiganbayan
himself  as  a  columnist,  who  “incidentally  happens   to  be  a  lawyer.”  He   166 SCRA 316 (1988)
wrote about alleged improprieties and irregularities in the judiciary
over several months. In light of these abnormal developments, the Facts: Petitioner Zaldivar is one of the several defendants in criminal
Chief Justice took an extraordinary step. He issued Administrative Order cases pending before the Sandiganbayan for violation of the Anti-Graft
No. 11-93 dated 25 January 1993, “Creating   an   Ad   Hoc   Committee   to   and Corrupt Practices Act. Petitioner asserts that respondent Gonzales,
Investigate   Reports  of   Corruption   in   the   Judiciary,”   tasked to ascertain as Tanodbayan and under the provisions of the 1987 Constitution, was
the truth respecting said reports and statements, and to interview at no longer vested with power and authority independently to
closed-door sessions or otherwise, such persons as may appear to it to investigate and to institute criminal cases for graft and corruption
have some knowledge of the matter and who may be appealed to share against public officials and employees. Hence, the criminal informations
that knowledge with the court, and otherwise gather such evidence as filed were all null and void.
may be available.
The Court issued a Temporary Restraining Orders directing the
Material to the present inquiry are Jurado’s  published  statements  from   Tanodbayan from acting upon cases already filed and from filing
late 1992 to the middle of February, 1993 where he wrote of graft and additional criminal informations. Thereafter, petitioner Zaldivar filed
corruption in the judiciary. There were calls for impeachment of the with the Court a Motion to Cite in Contempt directed against Gonzales
justices, for resignation of judges. There were insistent and more alleging that Gonzales issued contemptuous statements to the media.
widespread reiterations of denunciations of incompetence and He either released his Motion for Reconsideration with facsimiles of
corruption in the judiciary. For shortly afterwards, on 10 February 1993, said notes to the press or repeated the same to the press. And, for the
Mr. Vicente R. Samson, First Vice-President of the PLDT addressed a next several days, the metropolitan papers carried long reports on
letter   to   the   chief   Justice   and   requesting   that   the   Court   “take   such   those statements and variations and embellishments thereof. The
action  as  may  be  appropriate”. Court issued a resolution ordering Gonzales to explain why he should
not be punished for contempt and/or subjected to administrative
Jurado moved for the termination of the proceeding on the following sanctions.
posited premises: 1) the court has no administrative supervision over
him as a member of the press or over his work as a journalist; 2) the Issue: Whether  or  not  respondent  Gonzales’  statements  are  covered  by  
present administrative matter is not a citation for (a) direct contempt Constitutional provision of freedom of speech?
as there is no pending case or proceeding out of which a direct
contempt charge against him may arise, (b) indirect contempt as no Held: His [Gonzales] defense is not that he did not make the statements
formal charge for the same has been laid before the court in ascribed to him but that those statements give rise to no liability on his
accordance with Section 3 (Rule 71) of the Rules of Court; and,, 3) his part, having been made in the exercise of his freedom of speech.
comments would be more relevant and helpful to the Court if taken
together with the other evidence and reports of other journalists “***  A  free  press  is  not  to  be  preferred  to  an  independent  judiciary,  nor  
gathered before the Ad Hoc Committee. He perceives no reason why an independent judiciary to a free press. Neither has primacy over the
his comments should be singled out and taken up in a separate other; both are indispensable to a free society.
administrative proceeding.
The freedom of the press in itself presupposes an independent judiciary
Issue:  Whether  or  not  Jurado’s  acts  are  protected  by  the  Constitutional   through which that freedom may, if necessary, be vindicated. And one
provision of Freedom of Expression? of the potent means for assuring judges their independence is a free
press.”
Held:  “There  is  no  constitutional  value  in  false  statements  of  fact,”  and  
“the   erroneous   statement   of   fact   is   not   worthy   of   constitutional   This constitutional right must be protected in its fullest extent. The
protection   (although)   **   nevertheless   inevitable   in   free   debate.”   Court has heretofore given evidence of its tolerant regard for charges
“Neither   the   intentional   lie   nor   the   careless   error,”   it   said,   “materially   under the Libel Law which come dangerously close to its violation. But
advances   society’s   interest   in   ‘unhibited,   robust,   and   wide-open’   license or abuse of liberty of the press and of the citizens should not be
debate on public issues. confused with liberty in its true sense. As important as is the
Jurado [failed] to undertake even the most cursory verification of their maintenance of unmuzzled press and the free exercise of the rights of
objective   truth;   the   abdication   of   the   journalist’s   duty   to   report   and   the citizens is the maintenance of the independence of the Judiciary.
interpret the news with scrupulous fairness; and  the  breach  of  the  law’s  
injunction that a person act with justice, give everyone his due and Some  courts  have  held,  persuasively  it  appears  to  us,  that  a  lawyer’s  
observe honesty and good faith both in the exercise of his rights and in right of free expression may have to be more limited than that of a

54 | P LATON
layman. Bates v. State Bar of Arizona
433 US 350, 53 L Ed 2d 810,97 S Ct 2691 (1977)
Respondent Gonzalez claims to be and he is, of course, entitled to
criticize the rulings of this court. But it is the cardinal condition of all Facts: The appellants, licensed attorneys and members of the Arizona
such criticism that it shall be bona fide, and shall not spill over the walls State   Bar,   placed   a   newspaper   advertisement   for   their   “legal   clinic,”  
of decency and propriety. Intemperate and unfair criticism is a gross listing their fees for certain routine services, namely, uncontested
violation of the duty of respect to courts. It is such a misconduct that divorces, uncontested adoptions, simple personal bankruptcies, and
subjects   a   lawyer   to   disciplinary   action.   The   lawyer’s   duty   to   render   changes of name. Because of this they were charged in a complaint
respectful subordination to the courts is essential to the orderly filed   by   the   State   Bar’s   president   with   violating   the   State   Supreme  
administration of justice Court’s   disciplinary   rule,   which   prohibits   attorneys   from   advertising   in  
newspapers or other media. The Arizona Supreme Court upheld the
The Court concludes that respondent Gonzales is of guilty both of conclusion of a bar committee that appellants had violated the rule,
contempt court in facie curiae and of gross misconduct as an officer of rejecting,   among   others,   appellants’   claim   that   the   rule   infringed  
the court and member of the Bar. appellants’  First  Amendment  rights.

Resolution on Motion for Reconsideration Issue: Whether or not the State may prevent the publication in a
170 SCRA 1 (1989) newspaper   of   appellants’   truthful   advertisement   concerning   the  
availability and terms of routine legal services?
Facts:   In   his   point   D,   respondent’s   [Gonzales]   counsel   urges   that it is
error  “for  this  Court  to  apply  the  ‘visible  tendency’  rule  rather  than  the   HELD: We rule simply that the flow of such information may not be
‘clear  and  present  danger’  rule  in  disciplinary  and  contempt  charges. restrained, and we therefore hold the present application of the
disciplinary rule against appellants to be violative of the First
Held:   The   “clear   and   present   danger”   doctrine   has   been   an   accepted   Amendment.
method for marking out the appropriate limits of freedom of speech
and of assembly in certain contexts. It is not, however, the only test
which has been recognized and applied by courts. Symbolic   Speech,   Expressive   Conduct   and   the   “Public   Forum”  
Doctrine
‘From   the   language   of   the   specific   constitutional   provision,   it   would  
appear that the right is not susceptible of any limitation. No law may be
Speech is not confined to words. It can also be expressed
passed abridging the freedom of speech and of the press. The realities
of life in a complex society preclude however, a literal interpretation.
through   a   person’s   conduct   or   other   symbolic manner of
Freedom of expression is not an absolute. It would be too much to delivering a message, either alone or in collaboration with
insist that all times and under all circumstances it should remain other. The Constitution itself guarantees as part of the freedom
unfettered and unrestrained. There are other societal values that press of   expression   the   right   of   the   people   “peaceably   to   assemble  
for  recognition.’ and petition the Government for redress of grievances

The prevailing doctrine is that the clear and present danger rule is such Public Forum Doctrine posits that public areas such as streets
a limitation. Another criterion for permissible limitation on freedom of
and parks are traditionally considered as proper venues for the
speech  and  of  the  press  is  the  ‘balancing  of  interests  test’.  The  principle  
‘requires   a   court   to   take   conscious   and   detailed   consideration   of   the  
free articulation of ideas and views and that, therefore, the
interplay of interests observable in a given situation or type of exercise of such freedom should not be unnecessarily restricted
situation’. or impaired

ACCORDINGLY, the Court Resolved to DENY the Motion for The   “public   forum”   doctrine   holds   that   restrictions   on   speech  
Reconsideration for lack of merit. The denial is FINAL. should be subject to higher scrutiny when, all other things
being equal, that speech occurs in areas playing a vital role in
communication--such as in those places historically associated
Commercial Speech with first amendment activities, such as streets, sidewalks, and
parks--especially because of how indispensable communication
As noted earlier, at the core of the freedom of speech and of in these places is to people who lack access to more elaborate
the press is political expression, or those in connection with the (and more costly) channels
people’s  relationship  with  their  government  and  politics.  It  was  
only subsequently that the freedom was also considered to United  States  v.  O’Brien
encompass commercial speech 391 U.S. 367, 20 L Ed 2d 672, 88 S Ct 1673 (1968)

For commercial speech to come within that provision, it at least Facts:   O’Brien   and   three   companions   burned   their   Selective   Service  
must concern lawful activity and not be misleading. Next, we certificates on the steps of the South Boston Courthouse. A sizeable
ask whether the asserted governmental interest is substantial. crowd, including several agents of the Federal Bureau of Investigation,
witnessed the event and immediately after the burning members of the
If both inquiries yield positive answers, we must determine
crowd   began   attacking   O’Brien   and   his   companions.   O’Brien   stated   to  
whether the regulation directly advances the governmental FBI agents that he had burned his registration certificate because of his
interest asserted, and whether it is not more extensive than is beliefs, knowing that he was violating federal law. He stated in
necessary to serve that interest argument to the jury that he burned the certificate publicly to influence
others to adopt his antiwar beliefs. He also argued that the 1965
Intrinsically related to commercial speech is the advertising Amendment prohibiting the knowing destruction or mutilation of
industry, whose power to influence could either be for good or certificates was unconstitutional because it was enacted to abridge free
bad. Accordingly, the Constitution provides for its regulation by speech, and because it serve no legitimate legislative purpose, which
arguments the District Court rejected.
holding  that  “The  advertising  industry  is  impressed  with  public  
interest, and shall be regulated by law for the protection of Issue: Whether or not the 1965 Amendment prohibiting the knowing
consumers and the promotion of the general welfare destruction or mutilation of certificates was unconstitutional because it
was enacted to abridge free speech?

55 | P LATON
public streets and parks, is examined under strict scrutiny. Regulation
Held:   This   court   has   held   that,   when   “speech”   and   “non-speech”   of speech on property that the Government has expressly dedicated to
elements are combined in the same course of conduct, a sufficiently speech activity is also examined under strict scrutiny. But regulation of
important governmental interest in regulating the non-speech element speech activity where the Government has not dedicated its property
can justify incidental limitations on First Amendment freedoms. to First Amendment activity is examined only for reasonableness.

Whatever impression inheres in these terms, we think it clear that a Regulation of speech activity n governmental property that has
government regulation is sufficiently justified if it is within the been traditionally open to the public for expressive activity,
constitutional power of the Government; if it furthers an important or
such as public streets and parks, is examined under strict
substantial governmental interest; if the governmental interest is
unrelated to the suppression of free expression; and if the incidental scrutiny
restriction on alleged First Amendment freedoms is no greater than is
essential to the furtherance of that interest. Regulation of speech on property that the Government has
expressly dedicated to speech activity is also examined under
The governmental interest and the scope of the 1965 Amendment are strict scrutiny
limited to preventing harm to the smooth and efficient functioning of
the   Selective   Service   System.   When   O’Brien   deliberately   rendered   But regulation of speech activity where the Government has
unavailable his registration certificate, he willfully frustrated this
not dedicated its property to first Amendment activity is
governmental interest. For this non-communicative impact of his
conduct, and for nothing else, he was convicted. examined only for reasonableness

Since the 1965 Amendment to § 12(b)(3) of the Universal Military Anonymous Speech
Training and Service Act is constitutional as enacted and as applied.
Accordingly, we vacate the judgment of the Court of Appeals, and The speaker need not always have to identify himself in order
reinstate the judgment and sentence of the District Court. This that he may avail of the protection of the guarantee of freedom
disposition  makes  unnecessary  consideration  of  O’Brien’s  claim  that  the   of   speech.   Anonymity   need   not   result   in   loss   of   one’s   right   to  
Court of Appeals erred in affirming his conviction on the basis of the
speak freely
non-possession regulation.

Clark v. Community for Creative Nonviolence


The anonymity of an author is not ordinarily a sufficient reason
468 U.S. 288, 82 L Ed 2d 221, 104 S Ct 3065 (1984) to exclude her work product from the protections of the First
Amendment
Facts: In 1982, the National Park Service issued a renewable permit to
respondent Community for Creative Non-Violence (CCNV) to conduct a The freedom to publish anonymously extends beyond the
wintertime demonstration in Lafayette Park and Mall, which are literary realm
National Parks in the heart of Washington, D.C., for the purpose of
demonstrating the plight of the homeless. The permit authorized the
If the State can show a compelling interest to be subserved by a
erection of two symbolic tent cities. However, the Park Service, relying
on its regulations – particularly   on  that  permits  “camping”  (defined  as  
regulation which requires identification of the speaker,
the use of park land for living accommodation purposes such as however, then the same may be upheld, but again it would
sleeping activities, or making preparations to sleep) only in designated have to be subjected to an exacting scrutiny which is the usual
campgrounds, no campgrounds having ever been designated in standard in relation to speech-related restrictions
Lafayette Park or the Mall – denied  CCNV’s  request  that  demonstrators  
be permitted to sleep in the symbolic tents. CCNV and the individual Freedom of Assembly and Right to Petition
respondents then filed an action in the District Court, alleging, inter
alia, that application of the regulations to prevent sleeping in the tents
The   expression   of   one’s   opinion   or   plaints need not only be
violated the First Amendment. The District Court granted summary
judgment for the Park Service, but the Court of Appeals reversed.
limited to pure words. An individual with a grievance that needs
to be articulated may find a more effective manner of having
Issue: Whether or not a National Park Service regulation prohibiting his message get across by associating and joining with others
camping in certain parks violates the First Amendment when applied to and expressing themselves in a collaborative manner and
prohibit demonstrators from sleeping in Lafayette Park and the Mall in through a collective voice
connection with a demonstration intended to call attention to the
plight of the homeless? As stated in Reyes v. Bagatsing,   “it   was   not   by   accident   or  
coincidence that the rights to freedom of speech and of the
HELD: Expression, whether oral or written or symbolized by conduct, is
subject to reasonable time, place, or manner restrictions. Symbolic
press were coupled in a single guarantee with the rights of the
expression of this kind may be forbidden or regulated if the conduct people peaceably to assemble and to petition the government
itself may constitutionally be regulated, if the regulation is narrowly for redress of grievances. All these rights, while not identical,
drawn to further a substantial governmental interest, and if the interest are inseparable
is unrelated to the suppression of free speech.
In relation to regulation of activities or conduct on Reyes v. Bagatsing
government property, a distinction would have to be made on whether 125 SCRA 553 (1983)
such   property   is   traditionally   considered   as   “public   forums”   – streets
and parks – or not. Facts: Petitioner, retired Justice J.B.L. Reyes, on behalf of the Anti-Bases
The   Government’s   ownership   of   property   does   not   Coalition, sought a permit from the City of Manila to hold a peaceful
automatically open that property to the public. It is a long-settled march and rally on 26 October 1983, starting from Luneta, a public
principle that governmental actions are subject to a lower level of First park, to the gates of the United States Embassy, hardly two blocks
Amendment   scrutiny   when   “the   government   function   operating…   [is]   away. It turned out that on 19 October such permit was denied.
not   the   power   to   regulate   or   license,   as   lawmaker…   but,   rather,   as   Petitioner was unaware of such a fact as the denial was sent by
proprietor, to manage [its] internal operation[s]…” ordinary  mail.  The  reason  for  refusing  a  permit  was  due  to  “persistent  
Regulation of speech activity on governmental property that intelligence reports affirm[ing] the plans of subversive/criminal
has been traditionally open to the public for expressive activity, such as elements to infiltrate and/or disrupt and assembly or congregations

56 | P LATON
where   a   large   number   of   people   is   expected   to   attend.”   On   the   same   individual or a group. There can be no legal objection, absent the
day the Court heard the parties on oral argument and in the afternoon, existence of a clear and present danger of a substantive evil, on the
a minute resolution was issued by the Court granting the mandatory choice of Luneta as the place where the peace rally would start. Time
injunction prayed for on the ground that there was no showing of the immemorial Luneta has been used for purposes of assembly,
existence of a clear and present danger of a substantive evil that could communicating thoughts between citizens, and discussing public
justify the denial of a permit. questions.

Issue: Whether or not the mayor of the city of Manila acted in lack or Such use of the public places has from ancient times, been a part of the
excess of jurisdiction in denying the rally permit to petitioner and if privileges, immunities, rights, and liberties of citizens.
such decision abridge their right of expression and assembly? With regard to the ordinance, there was no showing that there was
violation and even if it could be shown that such a condition is satisfied
Held: It is true that the licensing official, here respondent Mayor, is not it does not follow that respondent could legally act the way he did. The
devoid of discretion in determining whether or not a permit would be validity of his denial of the permit sought could still be challenged.
granted. It is not, however, unfettered discretion. While prudence
requires that there be a realistic appraisal not of what may possibly A summary of the application for permit for rally: The applicants for a
occur but of what may probably occur, given all the relevant permit to hold an assembly should inform the licensing authority of the
circumstances, still the assumption – especially so where the assembly date, the public place where and the time when it will take place. If it
is scheduled for a specific public place – is that the permit must be for were a private place, only the consent of the owner or the one entitled
the assembly being held there. to its legal possession is required. Such application should be filed well
ahead in time to enable the public official concerned to appraise
While the general rule is that a permit should recognize the right of the whether there may be valid objections to the grant of the permit or to
applicants to hold their assembly at a public place of their choice, its grant but at another public place. It is an indispensable condition to
another place may be designated by the licensing authority if it be such refusal or modification that the clear and present danger tests be
shown that there is a clear and present danger of a substantive evil if the standard for the decision reached. Notice is given to applicants for
no such change were made. Hence the discretion reached by the Court. the denial.
The mere assertion that subversives may infiltrate the ranks of the
demonstrators does not suffice. Academic Freedom
Ordinarily, the remedy in cases of this character is to set aside the
The   Constitution   guarantees   that   “academic   freedom   shall   be  
denial or the modification of the permit sought and order the
respondent official to grant it. Nonetheless, as there was urgency in this enjoyed   in   all   institutions   of   higher   learning.”   This   freedom  
case, the proposed march and rally being scheduled for the next day essentially involves the right of such institutions of learning to
after the hearing, this Court, in the exercise of its conceded authority, determine what to teach, how to teach them, who may teach
granted the mandatory injunction in the resolution of October 25, them, and who to admit to study therein
1983.
The  classroom  is  peculiarly  the  “marketplace  of  ideas”
Reyes v. Bagatsing
125 SCRA 553 (1983) “Academic   freedom,”   the   term   as   it   evolved   to   describe   the  
emerging rights related to intellectual liberty, has traditionally
Facts: Petitioner sought a permit from the City of Manila to hold a
peaceful march and rally on October 26, 1983 from 2:00 to 5:00 in the
been associated with freedom of thought, speech, expression,
afternoon, starting from the Luneta to the gates of the United States and the press; in other words, the right of individuals in
Embassy. Once there, and in an open space of public property, a short university communities, such as professors, researchers and
program would be held. The march would be attended by the local and administrators, to investigate, pursue, discuss and, in the
foreign participants of such conference. That would be followed by the immortal words of Socrates,  “to  follow  the  argument  wherever  
handing over of a petition based on the resolution adopted at the it   may   lead,”   free   from   internal   and   external   interference   or  
closing session of the Anti-Bases Coalition. There was likewise an pressure
assurance in the petition that in the exercise of the constitutional rights
to free speech and assembly, all the necessary steps would be taken by
it "to ensure a peaceful march and rally. However the request was
The component of this aspect of academic freedom have been
denied. Reference was made to persistent intelligence reports affirming categorized under the areas of: (1) who may teach and (2) how
the plans of subversive/criminal elements to infiltrate or disrupt any to teach
assembly or congregations where a large number of people are
expected to attend. Respondent suggested that a permit may be issued As corporate entities, educational institutions of higher learning
if it is to be held at the Rizal Coliseum or any other enclosed area where are inherently endowed with the right to establish their
the safety of the participants themselves and the general public may be policies, academic and otherwise, unhampered by external
ensured. An oral argument was heard and the mandatory injunction
controls or pressure. In the Frankfurter formulation, this is
was granted on the ground that there was no showing of the existence
of a clear and present danger of a substantive evil that could justify the
articulated in the areas of: (1) what shall be taught, e.g., the
denial of a permit. However Justice Aquino dissented that the rally is curriculum and (2) who may be admitted to study
violative of Ordinance No. 7295 of the City of Manila prohibiting the
holding of rallies within a radius of five hundred (500) feet from any Admission to an institution of higher learning is discretionary
foreign mission or chancery and for other purposes. Hence the Court upon a school, the same being a privilege on the part of the
resolves. student rather than a right
Issue: Whether or not the freedom of expression and the right to The   Constitution  guarantees  that   “academic   freedom   shall   be   enjoyed  
peaceably assemble violated. in  all   institutions  of  higher   learning.”  This  freedom  essentially   involves  
the right of such institutions of learning to determine what to teach,
Ruling: Yes. The invocation of the right to freedom of peaceable how to teach them, who may teach them, and who to admit to study
assembly carries with it the implication that the right to free speech has therein. It has also been held that if, pursuant to its academic freedom,
likewise been disregarded. It is settled law that as to public places, such institutions of higher learning can decide who can and who cannot
especially so as to parks and streets, there is freedom of access. Nor is study in it, it certainly can also determine on whom it can confer the
their use dependent on who is the applicant for the permit, whether an honor and distinction of being its graduates, including the revocation or
57 | P LATON
withdrawal of the honor or distinction it may have conferred where it is the special characteristics of the school environment.
subsequently shown that the same was obtained through fraud.
The power of the school to investigate is an adjunct of its power to
Garcia v. Faculty Admission Committee, Loyola School of Theology suspend or expel. It is a necessary corollary to the environment
68 SCRA 277 (1975) conducive to learning: that power, like the power to suspend or expel,
is an inherent part of the academic freedom of institutions of higher
Facts: This is a mandamus case compelling the respondent to allow the learning guaranteed by the Constitution. Therefore, the Miriam College
petitioner to continue studying there. The petitioner contends that she has the authority to her and decide.
was denied re – admission for the reason given by the respondent
school,   namely,   that   “her   frequent questions and difficulties were not Among   the   reasons   for   the   indispensability   of   the   people’s  
always pertinent and had the effect of slowing down the progress of freedom of speech and of assembly to democracy are that
the class, is not a valid ground for her expulsion. While the respondent freedom of expression is essential as a means of assuring
contends that the school has discretion to admit or continue admitting
individual fulfillment and that it is an essential process for
in said school any particular student, considering not only academic or
intellectual standards but also other students, space limitations,
advancing knowledge and discovering
facilities, professors and optimum classroom size.
Additional Cases
Issue: Whether or not the petitioner is deemed possessed of such a (F) Freedom of Expression, Right of Assembly
right that has to be respected. and Academic Freedom [3-8]

Ruling: The petition is deemed not only on general principle, but also in Chavez vs. Gonzales
view of the character of the particular educational institution involved. 545 SCRA 441 (2008)
It is a seminary. Also, there is the autonomy recognized by the
Constitution  in  this  explicit  language:  “All  institutions  of  higher  learning   The case originates from events that occurred a year after the 2004
shall  enjoy  academic  freedom.”  In  connection  to  this,  the  four  essential   elections. On June 5, 2005 Press Secretary Ignacio Bunye told reporters
freedoms of a university are to determine for itself on academic that the opposition is planning to destabilize the administration by
grounds who may teach, what may be taught, how it shall be taught, releasing an audiotape of a mobile phone conversation allegedly
and who may be admitted to study. between Pres. GMA and a high-ranking official of the Comelec
(Garcillano) through wire-tapping. He produced two kinds of tape a
Justice Makasiar dissenting: What is involved here is not merely complete  version  and  a  “doctored”  version.    That  which  instructed the
academic freedom of the higher institutions of learning as guaranteed comelec   official   to   manipulate   the   election   results   in   the   president’s  
by Section 8 (2) of Art. XV of the 1973 Constitutions. The issue here favor. There were also conversations of President, First gentleman,
strikes at the broader freedom of expression of the individual – the very Garci and the late senator Barbers.
core of human liberty.
On the June, 2005 respondent DOJ secretary Gonzales warned
Miriam College Foundation, Inc. v. CA reporters that who had copies of the CD or those who will publish the
348 SCRA 265 (2000) contents will be liable under the Anti-wiretapping Act because of the
continuing the offense that had the personal knowledge of the crime
Facts: The petitioner took disciplinary action against some of its that included Sec. Bunye and Atty. Paguia.
students who were members of the   editorial   boards   of   the   college’s  
school paper (Chi – Rho) and magazine (Ang Magazing Pampanitikan Ng Because of the incident the NTC gives fair warning to radio and
Chi – Rho) for contents therein that were allegedly described by some television owners/operators to observe anti-wire-tapping law and
members   of   the   Miriam   College   community   as   “obscene”,   “vulgar”,   pertinent circulars on program standards. It has been subsequently
“indecent”,   “gross”,   “sexually   explicit”,   “injurious   to   young   readers”,   established that the said tapes are false and/or fraudulent after a
and  “devoid  of  all  moral  values.”  The  students  then  filed  a  petition  for   prosecution or appropriate investigation, the concerned radio and
prohibition and certiorari with preliminary injunction/restraining order television companies are hereby warned that their broadcast/airing of
before the RTC of Quezon City questioning the jurisdiction of the such false information and/or willful misrepresentation shall be just
Discipline Board of Miriam College over them. The RTC eventually cause for the suspension, revocation and/or cancellation of the licenses
granted the writ of preliminary injunction but subsequently dismissed or authorizations issued to the said companies.
the petition on the ground raised by both parties that it is the DECS
which has jurisdiction. Eight of the eleven students elevated to the SC, On June 14, 2005, NTC held a dialogue with the Board of Directors of
but the court referred to the C.A. in due time the appellate court the Kapisanan ng mga Brodkaster sa Pilipinas (KBP). NTC allegedly
granted the petition of the students and declared the RTC order, as well assured the KBP that the press release did not violate the constitutional
as  the  students’  supervision  and  dismissed,  void. freedom of speech, of expression, and of the press, and the right to
information
Issue: Who has the jurisdiction over the case?
Petitioner Chavez filed a petition under Rule 65 of the Rules of Court
Ruling: Sec. 5(2), Article XIV of the Constitution guarantees all against   respondents  Secretary   Gonzales  and  the   NTC,   “praying   for   the  
institutions of higher learning academic freedom. The essential issuance of the writs of certiorari and prohibition, as extraordinary legal
freedoms  subsumed  in  the  term  “academic  freedom”  encompasses  the   remedies, to annul void proceedings, and to prevent the unlawful,
freedom to determine for itself an academic grounds: unconstitutional and oppressive exercise of authority by the
(1) Who may teach, respondents.”   Alleging   that   the   acts   of   respondents   are   violations   of  
(2) What may be taught, the freedom on expression and of the press, and the right of the people
(3) How it shall be taught, and to information on matters of public concern.
(4) Who may be admitted to study.
We rule that not every violation of a law will justify straitjacketing the
The right of the school to discipline its students is at once apparent in exercise of freedom of speech and of the press. Our laws are of
the  third  freedom,  i.e.,  “how  it  shall  be  taught.”  Accordingly,  the  right   different kindsand doubtless, some of them provide norms of conduct
to discipline   the   student   likewise   finds   basis   in   the   freedom   “what   to   which  even  if  violated  have  only  an  adverse  effect  on  a  person’s  private  
teach”. comfort but do not endanger national security. There are laws of great
significance but their violation, by itself and without more, cannot
The right of the students to free speech in school premises, however, is support suppression of free speech and free press. In fine, violation of
not absolute. The right to free speech must always be applied in light of law is just a factor, a vital one to be sure, which should be

58 | P LATON
weighed in adjudging whether to restrain freedom of speech and of the 1986, creating the MTRCB, in relation to Sec. 3, Chapter XIII of the 2004
press. The totality of the injurious effects of the violation to private and Implementing Rules and Regulations (IRR) of PD 1986 and Sec. 7, Rule
public interest must be calibrated in light of the preferred status VII of the MTRCB Rules of Procedure.5The same order also set the case
accorded by the Constitution and by related international covenants for preliminary investigation. The following day after the suspension
protecting freedom of speech and of the press. In calling for a careful the petitioner sought for reconsideration of the preventive suspension
and calibrated measurement of the circumference of all these factors and Laguardia ( MTRCB chair person) recuse from hearing the case.
to determine compliance with the clear and present danger test, the After the case was heard in this court it was given 3 months suspension
Court should not be misinterpreted as devaluing violations of law. By all of the program Ang Dating Daan.
means, violations of law should be vigorously prosecuted by the After which they again filed for petition for certiorari and prohibition
State for they breed their own evil consequence. But to repeat, the with prayer for injunctive relief with the following issues: (A) BY
need to prevent their violation cannot per se trump the exercise of REASON THAT THE [IRR] IS INVALID INSOFAR AS IT PROVIDES FOR THE
free speech and free press, a preferred right whose breach can ISSUANCE OF PREVENTIVE SUSPENSION ORDERS; (B) BY REASON OF
lead to greater evils. For this failure of the respondents alone to offer LACK OF DUE HEARING IN THE CASE AT BENCH; (C) FOR BEING
proof to satisfy the clear and present danger test, the Court has no VIOLATIVE OF EQUAL PROTECTION UNDER THE LAW; (D) FOR BEING
option but to uphold the exercise of free speech and free press. There VIOLATIVE OF FREEDOM OF RELIGION; AND (E) FOR BEING VIOLATIVE
is no showing that the feared violation of the anti-wiretapping law OF FREEDOM OF SPEECH AND EXPRESSION.
clearly endangers the national security of the State. In ending, what petitioner obviously advocates is an unrestricted
speech paradigm in which absolute permissiveness is the norm.
This is not all the faultline in the stance of the respondents. We slide to Petitioner’s  flawed  belief  that  he  may  simply  utter   gutter profanity on
the issue of whether the mere press statements of the Secretary of television without adverse consequences, under the guise of free
Justice and of the NTC in question constitute a form of content-based speech, does not lend itself to acceptance in this jurisdiction. We
prior restraint that has transgressed the Constitution. In repeat: freedoms of speech and expression are not absolute freedoms.
resolving this issue, we hold that it is not decisive that the press To say "any act that restrains speech should be greeted with furrowed
statements made by respondents were not reduced in or followed up brows" is not to say that any act that restrains or regulates speech or
with formal orders or circulars. It is sufficient that the press statements expression is per se invalid. This only recognizes the importance of
were made by respondents while in the exercise of their official freedoms of speech and expression, and indicates the necessity to
functions. Undoubtedly, respondent Gonzales made his statements as carefully scrutinize acts that may restrain or regulate speech.
Secretary of Justice, while the NTC issued its statement as the WHEREFORE, the decision of the MTRCB in Adm. Case No. 01-04 dated
regulatory body of media. Any act done, such as a speech uttered, for September 27, 2004 is hereby AFFIRMED with the MODIFICATION of
and on behalf of the government in an official capacity is covered by limiting the suspension to the program Ang Dating Daan. As thus
the rule on prior restraint. The  concept  of  an  “act”  does  not  limit  itself   modified, the fallo of the MTRCB shall read as follows:
to acts already converted to a formal order or official WHEREFORE, in view of all the foregoing, a Decision is hereby
circular. Otherwise, the non formalization of an act into an official rendered, imposing a penalty of THREE (3) MONTHS SUSPENSION on
order or circular will result in the easy circumvention of the prohibition the television program, Ang Dating Daan, subject of the instant
on prior restraint. The press statements at bar are acts that should be petition.
struck down as they constitute impermissible forms of prior restraints Co-respondents Joselito Mallari, Luzviminda Cruz, and UNTV Channel
on the right to free speech and press. 37 and its owner, PBC, are hereby exonerated for lack of evidence.Costs
against petitioner.
There is enough evidence of chilling effect of the complained acts on
record. The warnings given to media came from no less the NTC, a Soriano v. Laguardia
regulatory agency that can cancel the Certificate of Authority of the (2009)
radio and broadcast media. They also came from the Secretary of
Justice, the alter ego of the Executive, who wields the awesome power Ang Dating Daan host Eliseo S. Soriano uttered the following
to prosecute those perceived to be violating the laws of the land. After statements in his TV program against Michael Sandoval (Iglesia ni
the warnings, the KBP inexplicably joined the NTC in issuing an Cristo’s   minister   and   regular   host   of   the   TV   program   Ang   Tamang  
ambivalent Joint Press Statement. After the warnings, petitioner Chavez Daan):
was left alone to fight this battle for freedom of speech and of the Lehitimong anak ng demonyo[!] Sinungaling [!]
press. This silence on the sidelines on the part of some media Gago ka talaga[,] Michael[!] [M]asahol ka pa sa putang babae[,] o di
practitioners is too deafening to be the subject of misinterpretation. ba[?]   [‘]Yung   putang   babae[,]   ang   gumagana   lang   doon[,]   [‘]yung  
ibaba, dito kay Michael[,] ang gumagana ang itaas, o di ba? O, masahol
In VIEW WHEREOF, the petition is GRANTED. The writs of certiorari and pa   sa   putang   babae   [‘]yan.   Sobra   ang   kasinungalingan   ng   mga  
prohibition are hereby issued, nullifying the official statements made by demonyong ito.
respondents on June 8, and 11, 2005 warning the media on airing the As   a   result,   The   MTRCB   initially   slapped   Soriano’s   Ang   Dating Daan,
alleged wiretapped conversation between the President and other which  was  earlier  given  a  “G”  rating  for  general  viewership,  with  a  20-
personalities, for constituting unconstitutional prior restraint on the day preventive suspension after a preliminary conference. Later, in a
exercise of freedom of speech and of the press decision, it found him liable for his utterances, and was imposed a
three-month suspension from his TV program Ang Dating Daan. Soriano
Soriano v. Laguardia challenged the order of the MTRCB.
(2009)
HELD:   The   SC   ruled   that   “Soriano’s   statement   can   be   treated   as  
This is a petition regarding Eliseo Soriano who seeks to nullify and set obscene,   at   least   with   respect   to   the   average   child,”   and   thus   his  
aside an order and decision of the MTRCB in connection with certain utterances cannot be considered as protected speech. Citing decisions
utterances he made in the television show, Ang Dating Daan. from the US Supreme Court, the High Court said that the analysis
On August 10, 2004, at 10pm, the petitioner, as host of the television should   be   “context   based”   and   found   the   utterances   to   be   obscene  
show Ang Dating Daan made following remarks: after considering the use of television broadcasting as a medium, the
Lehitimong anak ng demonyo, sinungaling; time  of  the  show,  and  the  “G”  rating  of  the show, which are all factors
Gago ka talaga Michael, masahol ka pa sa putang babae o dib ba. that made the utterances susceptible to children viewers. The Court
Yung putang babae ang gumagana lang doon ung ibaba, ditto kay emphasized on how the uttered words could be easily understood by a
Michael ang gumagana ang itaas. O dib a! o masahol pa sa putang child  literally  rather  than  in  the  context  that  they  were  used.”
babae yan. Sabi ng lola ko masahol pa sa putang babae yan sora ang The  SC  also  said  “that  the suspension is not a prior restraint, but rather
kasinungalingan ng mga demonyong ito. a   “form   of   permissible   administrative   sanction   or   subsequent  
Because of that the MTRCB ordered the suspension of the program for punishment.”  In  affirming  the  power  of  the  MTRCB  to  issue  an  order  of  
20 days in accordance with Section 3(d) of Presidential Decree No. (PD) suspension,  the  majority  said  that  “it  is  a  sanction  that  the  MTRCB  may  

59 | P LATON
validly impose under its charter without running afoul of the free and criticism. His irresponsible and baseless statements, his
speech   clause.”   visit   fellester.blogspot.com   The   Court   said   that   the   unrepentant stance and smug insistence of his malicious and
suspension   “is   not   a   prior   restraint   on   the   right   of   petitioner   to   unfounded accusation against Justice Tinga have sullied the dignity and
continue with the broadcast of Ang Dating Daan as a permit was authority of this Court. Beyond question, therefore, De La Serna's
already   issued   to   him   by   MTRCB,”   rather,   it   was   a   sanction   for   “the   culpability for indirect contempt warrants the penalty of a fine not
indecent   contents   of   his   utterances   in   a   “G”   rated   TV   program.”   exceeding P30,000.00 or imprisonment not exceeding six (6) months or
(Soriano v. Laguardia; GR No. 165636, April 29, 2009) both under the Rules.

Dissenting Opinion: Chief Justice Reynato S. Puno, in a separate The power to declare a person in contempt of court and in dealing with
dissenting opinion, said that a single government action could be both a him accordingly is a means to protect and preserve the dignity of the
penalty and a prior restraint. The Chief Magistrate pointed out that the court, the solemnity of the proceedings therein and the administration
three month suspension takes such form because it also acts as a of justice from callous misbehavior and offensive personalities. Respect
restraint   to   petitioner’s   future   speech   and thus deserves a higher for the courts guarantees the stability of the judicial institution.
scrutiny   than  the   “context   based”  approach  that   the   majority   applied.   Without such guarantee, the institution would be resting on a very
In  voting  to  grant  Soriano’s  petition,  the  Chief  Justice  said  that  “in  the   shaky foundation. The Court will not hesitate to wield this inherent
absence of proof and reason, he [Soriano] should not be penalized with power to preserve its honor and dignity and safeguard the morals and
a three-month suspension that works as a prior restraint on his ethics of the legal profession.
speech.”
F11. Pleasant Grove City v. Summum
F9. In Re: Amado P. Macasaet 555 U.S. __ (2009)
561 SCRA 395 (2008)
Summum, a religious organization, sent a letter to the mayor of
The case stemmed from certain articles that appeared in the "Business Pleasant Grove, Utah asking to place a monument in one of the city's
Circuit" column of Amado P. Macasaet in the Malaya, a newspaper of parks. Although the park already housed a monument to the Ten
general circulation of which he is the publisher. The articles, containing Commandments, the mayor denied Summum's request because the
statements and innuendoes about an alleged bribery incident in the monument did not "directly relate to the history of Pleasant Grove."
Supreme Court, came out in four (4) issues of the newspaper on Summum filed suit against the city in federal court citing, among other
September 18, 19, 20 and 21, 2007. things, a violation of its First Amendment free speech rights. The U.S.
Upon evaluation of the columns "Business Circuit" of Amado P. District Court for the District of Utah denied Summum's request for a
Macasaet in the September 18, 19, 20, and 21, 2007 issues of the preliminary injunction.
Malaya, it appears that certain statements and innuendoes therein
tend, directly or indirectly, to impede, obstruct, or degrade the The U.S. Court of Appeals for the Tenth Circuit reversed the district
administration of justice, within the purview of Section 3(d), Rule 71 of court and granted Summum's injunction request. The Tenth Circuit held
the 1997 Rules of Civil Procedure. Amado P. Macasaet is ordered to that the park was in fact a "public" forum, not a non-public forum as
explain why no sanction should be imposed on him for indirect the district court had held. Furthermore, Summum demonstrated that
contempt of court it would suffer irreparable harm if the injunction were to be denied,
and the interests of the city did not outweigh this potential harm. The
The High Court created an investigating committee composed of retired injunction, according to the court, was also not against the public
Supreme Court justices. The Investigating Committee held hearings and interest.
gathered affidavits and testimonies from the parties concerned
Does a city's refusal to place a religious organization's monument in a
The Court declares respondent Amado P. Macasaet guilty of indirect public park violate that organization's First Amendment free speech
contempt of court. Unfortunately, the published articles of respondent rights when the park already contains a monument from a different
Macasaet are not of this genre. On the contrary, he has crossed the religious group?
line, as his are baseless scurrilous attacks which demonstrate nothing
but an abuse of press freedom. They leave no redeeming value in No. The Supreme Court reversed the Tenth Circuit holding that the
furtherance of freedom of the press. They do nothing but damage the placement of a monument in a public park is a form of government
integrity of the High Court, undermine the faith and confidence of the speech and therefore not subject to scrutiny under the Free Speech
people in the judiciary, and threaten the doctrine of judicial Clause of the First Amendment. The Court reasoned that since Pleasant
independence Grove City had retained final authority over which monuments were
displayed, the monuments represented an expression of the city's
He published his highly speculative articles that bribery occurred in the viewpoints and thus government speech. Although a park is traditional
High Court, based on specious information, without any regard for the public forum for speeches and other transitory expressive acts, the
injury such would cause to the reputation of the judiciary and the display of a permanent monument in a public park is not a form of
effective administration of justice. Nor did he give any thought to the expression to which forum analysis applies.
undue, irreparable damage such false accusations and thinly veiled
allusions would have on a member of the Court. F12. PHCAP v Duque III
535 SCRA 265 (2007)
F10. Fudot v. Cattleya Land, Inc.
570 SCRA 86 (2008) Facts: Before the Court is a petition for certiorari under Rule 65 of the
Rules of Court, seeking to nullify A.O No. 2006-0012 entitled, “Revised
Mr. Chan had already bought the interest of Cattleya Land, Inc. Implementing Rules and Regulations of Executive Order No 51,
(Cattleya) over a property adjacent to the property subject of the case Otherwise   known   as   the   "Milk   Code,”,   Relevant   International  
and that he was interested in putting up a resort/hotel in the property. Agreements, Penalizing Violations Thereof and for Other Purposes
He wanted to purchase Carmelita Fudot's interest in the property as (RIRR)”.   Petitioner   posits   that   the   RIRR   is   not   valid   as   it   contains
well to put an end to the litigation. On 9 November 2007, the Court provisions that are not constitutional and go beyond the law it is
received from De La Serna a request for the inhibition of Associate supposed to implement.
Justice Dante O. Tinga, claiming that Justice Tinga received P10 Million
from Mr. Johnny Chan (Mr. Chan) in exchange for a favorable decision Issue: WON RIRR is valid and constitutional
in the instant case.
Held: Only Sections 4(f), 11 and 46 of A.O. No. 2006-0012 are declared
After hearing the explanation of both parties, the court holds that Atty. as beyond the authority of the DOH and are therefore null and void.
De La Serna has transcended the permissible bounds of fair comment The rest of the Code is in consonance with the objective, purpose and

60 | P LATON
intent of the Milk Code since it constitutes reasonable regulation of an same day, she issued G.O. No. 5 setting the standards which the AFP
industry which affects public health and welfare. Thus, the rest of RIRR and the PNP should follow in the suppression and prevention of acts of
do not constitute illegal restraint of trade nor are they violative of the lawless violence. Prof. Randolf David and others were then arrested
due process clause of the Constitution. without warrant while they were exercising their right to peaceful
assembly. The authorities also raided the office of the newspaper
Concurring Opinion of Chief Justice Puno: Tribune, threatened the media, imposed censorship and threatened
Another reason why the absolute ban on advertising and promotion of take over public utilities. Petitioners were charged with the crime of
breast milk substitutes found under Sections 4(f) and 11 of A.O. No. inciting  to  sedition  and  violation  of  BP  880,  “The  Public  Assembly Act of
2006-0012 should be struck down: 1985”.

The advertising and promotion of breast milk substitutes is considered Issue: WON PP 1017 and G.O. No. 5 are constitutional
a commercial speech which is a kind of speech that proposes an
economic transaction. It is a separate category of speech which is not Held: PP 1017 is constitutional on the ground that it constitutes a call
accorded the same level of protection as that given to other by the President for the AFP to prevent pr suppress lawless violence.
constitutionally guaranteed forms of expression but is nonetheless But its provisions regarding the President’s   power   to   issue   decrees,  
entitled to protection. Commercial speech must concern lawful activity direct the AFP to enforce obedience to all laws even those which are
and not be misleading if it is to be protected under the First unrelated to lawless violence and to impose standards on media or any
Amendment. The asserted governmental interest must be substantial. for, or prior restraint on the press are unconstitutional.
If both of these requirements are met, it must next be determined
whether the state regulation directly advances the government interest G.O. No. 5 is valid since it is as order issued by the President acting as
asserted, and whether it is not more extensive than is necessary to the Commander-in –Chief addressed to the AFP to carry out the
serve the interest. provision of PP 1017.

The absolute ban on advertising prescribed under Sections 4 (f) and 11 However, the warrantless arrest of the petitioners, the dispersal of
of the RIRR is unduly restrictive and is more than necessary to further rallies and warrantless arrest of others and the imposition of standards
the avowed governmental interest of promoting the health of infants on media or any prior restraint on the press, and the warrantless search
and young children. It must be self-evident, for instance that the of the Tribune offices and the seizures of some articles for publication
advertisement of such products which are strictly informative cuts too are neither authorized by the Constitution nor by the provisions of PP
deep on free speech. The laudable concern of the respondent for the 1017 and G.O. No. 5.
promotion of the health of infants and young children cannot justify the
absolute, overarching ban. *Assembly is a right of the citizens to meet peaceably for consultation
regarding public affairs. It is a necessary consequence of our republican
F13. Bayan v. Ermita institution and complements the right of speech. In connection with
488 SCRA 226 (2006) the freedom of expression, such right is not limited or denied except on
a showing of a clear and present danger of a substantive evil that
Facts:   Petitioners   come   in   three   groups   and   they   assail   BP   880   “The   Congress has a right to prevent. The right to assemble, like other rights
Public   Assembly   Act   of   1985”   ,   some   of   them   in   toto   and   others   only   of freedom of expression, is not subject to previous restraint or
Sections 4, 5, 6, 12, 13(a), and 14(a), as well as the policy of Calibrated censorship. It may not also be conditioned upon the prior issuance of a
Preemptive Response (CPR). They also seek to stop violent dispersals of permit or authorization from the government authorities except if the
rallies under the "no permit, no rally" policy and the CPR policy recently assembly is intended to be held in a public place, a permit for the use of
announced. the place and not the assembly itself may be validly required.

Issues: (1) WON the Calibrated Preemptive Response policy is valid In the case at bar, the petitioners were not notified and heard on the
(2) WON BP 880 is valid and constitutional revocation of their permits. The first time they learned of it was at the
time of the dispersal. Thus, such absence of notice is a fatal defect.
Held: The right to peaceably assemble and petition for redress of
grievances is, together with freedom of speech, of expression, and of *The search conducted in the office of Tribune, even though it has
the press, a right that enjoys primacy in the realm of the constitutional anti-government sentiments, is considered as illegal as it violated
protection. Since these rights constitute the very basis of a functional petitioners’   freedom   of   the   press.   It   must   be   noted   that   freedom   to  
democratic polity, without which all the other rights would be comment on public affairs is essential to the vitality of a representative
meaningless and unprotected. However, such right is not absolute. democracy.
Thus, BP 880 provides restriction that simply regulates the time, place
and manner of the assemblies. It does not impose an absolute ban on *(1) The overbreadth doctrine is an analytical tool developed for testing
public assemblies. ‘’on   their   faces’’   statutes   in   free   speech   cases   known   under   the  
American laws First Amendment Cases. In connection to that, a plain
(1) The Calibrated Preemptive Response (CPR) policy has no place in reading of PP 1017 shows that it is not primarily directed to speech or
our legal firmament and must be struck down. It serves no valid even a speech-related conduct. Also, it is not intended for testing the
purpose if   it   is  also  means  “maximum   tolerance”   under   BP   880   and  is   validity of a law that reflects legitimate interest in maintaining
illegal if it means something else. comprehensive control over harmful constitutionally unprotected
conduct. The claims of facial overbreadth are entertained in cases
(2) BP 880 is valid and constitutional since it does not curtail or unduly involving  statutes  which,  by  their  terms,  seek  to  regulate  only  ‘’spoken  
restrict freedom. It merely regulates the use of public places as to the words  and  again  that  “overbreathed  claims,  if  entertained  at  all  ,  have  
time, place and manner of assemblies.   Moreover,   “maximum   been curtailed when invoked against ordinary criminal laws that are
tolerance" is for the benefit of rallyists, not the government. The sought to be applied to protected conduct.”   Therefore,   PP   1017  
delegation to the mayors of the power to issue rally "permits" is pertains to a spectrum of conduct, not free speech, which is manifestly
likewise valid because it is subject to the constitutionally-sound "clear subject to state regulation.
and present danger" standard.
(2)   Facial   invalidation   of   laws   is   considered   as   “manifestly   strong  
medicine”   to   be   used   “sparingly   and   only   as   a   last   resort” and is
F14. David v. Macapagal-Arroyo
“generally  disfavored”  for  the  reason  that  a  person  to  whom  a  law  may  
486 SCRA 160 (2006)
be applied will not be heard to challenge a law on the ground that it
may be conceivably be applied unconstitutionally to others, for
Facts: As part of the commemoration of EDSA I, Pres. GMA
example, in situations not before the Court. Thus, a facial challenge
promulgated PP 1017 declaring the state of national emergency. On the

61 | P LATON
using the overbreadth doctrine will require the Court to examine pp Although such issuance and sale may be inseparably linked with
1017 and pinpoint its flaws and defects not on the basis of its actual theRoman Catholic Church, any benefit and propaganda incidentally
operation to petitioners but on the assumption that its very existence resulting from it was no the aim or purpose of the Government.
may cause others not before the Court to refrain from constitutionally
protected speech or expression. Defining and Divining Religion
(3) A facial challenge on the ground of overbreadth is the most difficult
In having to decide issues that implicate freedom of religion,
challenge to mount successfully because the challenger must establish
that there can be no instance when the assailed law may be valid. In courts must necessarily have to define what religion is
the case at bar, petitioners did not even attempt to show whether this
situation exists. Defining religion is a difficult task for even theologians,
philosophers and moralists cannot agree on a comprehensive
Chapter 7 definition
Freedom of Religion
Religion is derived from the Middle English religioun, from Old
“No  law  shall  be  made  respecting  an  establishment  of  religion,   French religion, from Latin religio,  vaguely  referring  to  a  ‘bond  
or prohibiting the free exercise thereof. The free exercise and between  man  and  the  gods’
enjoyment of religious profession and worship, without
discrimination or preference, shall forever be allowed. No The  term  ‘religion’  has  reference  to  one’s  views  of  his  relations  
religious test shall be required for the exercise of civil or to his Creator, and to the obligations they impose of reverence
12
political  rights.” for his being and character, and of obedience to his will

A   man’s   faith   and   belief are his alone and the State has no The  test  of  belief  “in  a  relation  to  s  Supreme  Being”  is  whether  
business interfering with that a given belief that is sincere and meaningful occupies a place in
the life of its possessor parallel to that filled by the orthodox
The   Constitution   also   provides   in   emphatic   terms”   “The   belief in God of one who clearly qualifies for the exemption
separation  of  Church  and  State  shall  be  inviolable”
Four criteria to qualify as religion under the First Amendment:
The assurance of religious freedom under the Constitution 1. There must be belief in God or some parallel belief that
principally consists of two guarantees, embodied in the so- occupies  a  central  place  in  the  believer’s  life
called  “Establishment  Clause”  and  “Free  Exercise  Clause” 2. The religion must involve a moral code transcending
individual belief, i.e., it cannot be purely subjective
Aglipay v. Ruiz 3. A demonstrable sincerity in belief is necessary, but the court
64 Phil. 201 (1937) must not inquire into the truth or reasonableness of the belief
4. There must be some associational ties, although there is also
Facts: Petitioner seeks the issuance of a writ of prohibition against a view that religious beliefs held by a single person rather than
respondent Director of Posts from issuing and selling postage stamps
being part of the teachings of any kind of group or sect are
commemorative of the 33rd International Eucharistic Congress.
Petitioner contends that such act is a violation of the Constitutional
entitled to the protection of the Free Exercise Clause
provision stating that no public funds shall be appropriated or used in
the benefit of any church, system of religion, etc. This provision is a In Aglipay v. Ruiz, religion has been described as that
result of the principle of the separation of church and state, for the “profession  of  faith  to  an  active  power  that  binds  and  elevates  
purpose of avoiding the occasion wherein the state will use the church, man  to  his  Creator”
or vice versa, as a weapon to further their ends and aims. Respondent
contends that such issuance is in accordance to Act No. 4052, providing American Bible Society v. City of Manila also spoke of religion as
for the appropriation funds to respondent for the production and
having   “reference   to   one’s   view   of   his   relations   to   His   Creator  
issuance of postage stamps as would be advantageous to the
government.
and to the obligations they impose of reverence to His being
and character, and obedience to His Will
Issue: Whether or not there was a violation of the freedom to religion.
The Establishment Clause
Ruling: What is guaranteed by our Constitution is religious freedom and
not mere religious toleration. It is however not an inhibition of This clause principally prohibits the state from sponsoring any
profound reverence for religion and is not a denial of its influence in religion, or favouring any religion as against other religions. It
human affairs. Religion as a profession of faith to an active power that
mandates a strict neutrality in affairs among religious groups
binds and elevates man to his Creator is recognized. And in so far as it
instills into the minds the purest principles of morality, its influence is
deeply felt and highly appreciated. The phrase in Act No. 4052 The state may not favor religion to the extent of enforcing
“advantageous  to the  government”  does  not  authorize  violation  of  the   people to believe in a god or any other supernatural being.
Constitution. The issuance of the stamps was not inspired by any With the freedom to believe also comes the liberty not to
feeling to favor a particular church or religious denomination. They believe in any other being
were not sold for the benefit of the Roman Catholic Church. The
postage stamps, instead of showing a Catholic chalice as originally In the Philippine jurisdiction, there us substantial agreement on
planned, contains a map of the Philippines and the location of Manila,
the values sought to be protected by the Establishment Clause,
with   the   words   “Seat   XXXIII   International   Eucharistic   Congress.”   The  
focus of the stamps was not the Eucharistic Congress but the city of
namely voluntarism and insulation of the political process from
Manila,   being   the   seat   of   that   congress.   This   was   to   “to advertise the interfaith dissension
Philippines   and   attract   more   tourists,”   the   officials   merely   took  
advantage of an event considered of international importance. Religion requires voluntarism because compulsory faith lacks
religious efficacy
12 CONSTITUTION, Art. III, § 5
62 | P LATON
faith at all. This is not to say, however that religion has been so
Such voluntarism cannot be achieved unless the political identified with our history and government that religious freedom is
process is insulated from religion and unless religion is not likewise as strongly imbedded in our public and private life. Nothing
but the most telling of personal experiences in religious persecution
insulated from politics
could have implanted such belief.
School District of Abington Township v. Schempp
374 U.S. 203, 10 L Ed 2d 844, 83 S Ct 1560 (1963) Marsh v. Chambers
463 U.S. 783, 77 L Ed 2d 1019, 103 S Ct 3330 (1983)
Facts:
Applicable Amendments: Facts: The Nebraska Legislature begins each of its sessions with a prayer
First Amendment, Establishment Clause: Congress shall make no law by a chaplain paid by the State with the legislature's approval.
respecting an establishment of religion, or prohibiting the free exercise Respondent member of the Nebraska Legislature brought an action in
thereof. Federal District Court, claiming that the legislature's chaplaincy practice
violates the Establishment Clause of the First Amendment, and seeking
-No. 142 – The Commonwealth of Pennsylvania requires that at least injunctive relief. The District Court held that the Establishment Clause
ten verses from the Holy Bible shall be read without comment, at the was not breached by the prayer but was violated by paying the chaplain
opening of each Public school on each school day. Any child shall be from public funds, and accordingly enjoined the use of such funds to
excused from such Bible reading, or attending such Bible reading, upon pay the chaplain. The Court of Appeals held that the whole chaplaincy
the request of his parent or guardian. The exercises are broadcast into practice violated the Establishment Clause, and accordingly prohibited
each room in the building through an intercom system. This is followed the State from engaging in any aspect of the practice.
by   the   recitation   of   the   Lord’s   Prayer. Participation in the open
exercises was considered voluntary. The student reading the bible must Issue: Whether or not the legislature's chaplaincy practice violates the
select the passages and read any form or version he chases. (King James Establishment Clause of the First Amendment.
version, Douay or the Revised Standard versions as well as the Jewish
Holy Scriptures) Ruling: By a 6-3 vote the Supreme Court permitted the practice of
beginning a legislative session with a prayer delivered by a publicly
-The constitutionality of the said statute was assailed by Edward funded chaplain, with Chief Justice Warren Burger writing the majority
Schempp, a member of the Unitarian faith who, along with his wife and opinion.
children, questioned the validity of the statute, contending that his The Court relied almost entirely on historical practice and tradition.
rights have been violated, under the 14th of table and to the Congress had paid a chaplain and opened sessions with prayers for
Constitution of the United States. almost 200 years. Indeed, the fact that Congress had continued the
practice after considering constitutional objections in the Court's view
-The children study in Abington Senior High School strengthened rather than weakened the historical argument.

-Schempp testified that he at first refused to exercise his prerogative of The opening of sessions of legislative and other deliberative public
excusing his children from the morning exercises upon fear that his bodies with prayer is deeply embedded in the history and tradition of
children would be labeled as odd balls. Their classmates would be liable this country. From colonial times through the founding of the Republic
for lumping religious differences and objections as atheism with and ever since, the practice of legislative prayer has coexisted with the
immoral and un-patriotic overtones. principles of disestablishment and religious freedom. In the very
courtrooms in which the United States District Judge and later three
-Doctor Solomon Grayzel (witness for the appellees): The reading of Circuit Judges heard and decided this case, the proceedings opened
such with an announcement that concluded, "God save the United States
verses without explanation may be psychologically harmful to the and this Honorable Court." The same invocation occurs at all sessions of
children and may cause a divisive force in the social media of the this Court.
school.
In light of the unambiguous and unbroken history of more than 200
-Doctor Luther A. Weigle (witness for the defense): The Bible is a non- years, there can be no doubt that the practice of opening legislative
sectarian piece of literature within among the Christian faiths. The sessions with prayer has become part of the fabric of our society. To
exclusion of the New Testament would be in itself a sectarian practice. invoke Divine guidance on a public body entrusted with making laws is
-The trial court struck down the practices and the statute requiring not, in these circumstances, an "establishment" or a step toward
them after making the specific findings of fact that attendance to establishment; it is simply a tolerable acknowledgement of beliefs
Abington and undergoing the practices were compulsory. The court widely held among the people of this country.
further found that the reading of the verses without comment would
constitute in effect a religious observance. Basically, the decision argued that both the Supreme Court and
Congress have traditionally begun their sessions with prayers. Since
-The   court   rejected   the   defence’s   argument   that   the   children   were   individual states do not have to abide by more stringent First
allowed  to  excuse  themselves  via  their  parents’  request, saying that it Amendment limits than the federal government, then they, too, are
did not mitigate the obligatory nature of the ceremony. This was still in permitted to use prayers. The "Establishment Clause does not always
violation of the establishment clause in that it threatens religious bar a state from regulating conduct simply because it harmonizes with
liberty by putting a premium upon belief as opposed to non-belief, religious concerns."
rendering sinister, alien, and suspect the beliefs, ideals, and even
morality of the petitioners. Marsh vs. chambers
463 us 783 (1983)
Issue: WON rule 142 of the Commonwealth of Pennsylvania is
unconstitutional under the violation of the Establishment Clause under Facts: Nebraska Legislature begins its sessions with a prayer offered by
the Fourteenth Amendment. a chaplain who is chosen biennially by the Executive Board of
Legislative Council and paid out of public funds. Respondent, claims
Ruling: Yes. It is true that religion has been closely identified with that   the   Legislature’s   chaplaincy   practice   violates   the   Establishment  
American history and government. This background is evidence today Clause of the First Amendment.
in our public life through the continuance in our oaths of office from
the   Presidency   to   the   Alderman   of   the   final   supplication,   “So   help  me   Issue: whether or not the prayer offered upon the start of every session
God.”  Indeed,  only  last  year,  an  official  survey  of  the  country  indicated   of the Nebraska Legislature and the payment of public funds thereof
that 64% of our people have church membership while 3% profess no constitutes a violation of the Establishment Clause of the First

63 | P LATON
Ammendment? Held: the assistance was unconstitutional. There are three criteria that
should be used to assess legislation: "First, the statute must have a
Held: the prayer offered by the chaplain and the funds paid thereon are secular legislative purpose; second, its principal or primary effect must
not a violation of the Establishment Clause. be one that neither advances or inhibits religion; finally, the statute
must not foster and excessive government Entanglement with religion."
The offering of prayer in the opening of sessions is deeply rooted in the The two statutes in question violate the third of these criteria. The
history and tradition of this country. The practice has coexisted with teachers whose salaries are being partially paid by the State are
the principles of disestablishment and religious freedom. It has religious agents who work under the control of religious officials. There
continued without interruption ever since that early session of is an inherent conflict in this situation of which the state should remain
Congress. However, standing alone, historical patterns cannot justify clear. To ensure that teachers play a non-ideological role would require
contemporary violations of constitutional guarantees. In this context, the state to become entangled with the church. Allowing this
historical evidence shed light not only on what the draftsmen intended relationship could lead to political problems in areas in which a large
the Establishment Clause mean, but also on how they thought that number of students attend religious schools.
Clause applied to the practice authorized by the First Congress- their
actions reveal their intent. Tilton v Richardson
403 us 672 (1971)
BOARD OF EDUCATION VS. ALLEN
392 US 236 (1968) facts: the higher education facilities act was passed in response to a
strong nationwide demand for the expansion of college and university
Facts:   Section   701   of   New   York’s   Education   law   requires   local   public   facilities to meet the sharply rising number of young people demanding
school authorities to lend textbooks free of charge to all students in higher education. Act provides federal construction grants and loan for
grade  …  including  those  in  private  and  parochial  schools.    The  appellant   college   and   university   facilities,   excluding   “any   facility   used   or   to   be  
school board sought a declaration that the statutory requirement was used   for   sectarian   instruction   or   as   a   place   of   religious   worship,   or   …  
invalid as violative of the State and Federal Constitutions, an order primarily in connection with any part of the program of a school or
barring the appellee Commissioner of Education from removing department of divinity.”    The  act  stipulated  that  after  twenty  years,  the  
appellant’s   members   from   office   for   failing   to   comply   with   it,   and   an   school could use the facilities for whatever purpose they chose.
order preventing the use of state funds for the purchase of textbooks
to be lent to parochial students. Held:

Held: the express purpose of the law is the furtherance of the the Court decided that the grants for non-religious school facilities did
educational opportunities available to the young. There is nothing that not violate the Establishment Clause. it decided that the provision
shows about the necessary effects of the statute that is contrary to its limiting   the   statutes’   interest   to   twenty   years   was   unconstitutional.
stated purpose. The law merely makes available to all children the
benefits of a general program to lend school books free of charge. The primary effect of the Higher Education Facility Act was not to aid
religious institutes. The objective was to encourage education among
WALZ VS. TAX COMMISSION OF THE CITY OF NEW YORK the   country’s   youth.   In   an   earlier case (Bradfield v. Rob) the Court
397 US 664 (1970) decided that not all of financial aid to church-sponsored activities
violates the religious clauses of the constitution. The beneficiaries of
facts: appellant imsiccessfully sought an injunction in the New York the act are secondary schools in which children are not as susceptible
courts to prevent the NY Tax Commission fom granting property tax to religious coercion and in which religious instruction is not as central
exemptions to religious organizations for properties used solely for to  the  curriculum.  Because  the  State’s  interest  in  the  structure  remains  
religious worship, as authorized by the state constitution and the after twenty years, the provision giving the schools the ability to use
implementing statue. The appellant contends that the grant of tax the facility for religious purposes is unconstitutional. This finding does
exemptions to church property indirectly requires the appellant to not require the invalidating of the entire act because it was not
make a contribution to religious bodies, and thereby violates the essential to the whole law. The HEFA(higher education financial aid) did
Establishment and Free Exercise Clause. not lead to excessive entanglement because the aid was aimed at
religiously neutral facilities. Also, the aid was non-ideological and was a
Held: the legislative purpose of the property tax exemptions is neither one-time, single-purpose program. The taxpayers� rights were not
the advancement nor the inhibition of religion; it is neither sponsorship violated by the act because there was no coercion directed at the
nor hostility. NY has determined thath certain entities that exist in practice or exercise of their religious beliefs.
harmonious relationship to the community at large, and that foster its
“moral   and   mental   improvement”   should   not   be   inhibited   in   their  
activities by property taxation or the hazard of loss of its property for In making this decision the Court did not discuss whether the assistance
nonpayment of taxes. Nothing in this national attitude towards to the religious schools for non-religious purposes would enhance their
religious tolerance and two centuries of uninterrupted freedom from ability to further their religious instruction. Taxpayers, whose money
taxation has given the remotest sign of leading to an established church was given to religious institutions, were not harmed provided their own
or religion, it has operated affirmatively to guarantee the free exercise religious practices were not affected.
of all forms of religious beliefs.
Agostini v. Felton
Lemon v. Kurtzman 521 US 203 (1997)
403 US 602 (1971)
Facts: A New York parochial school board challenged the District Court's
Facts: a Pennsylvania and Rhode Island statutes providing state aid to upholding of a twelve year-old decision in Aguilar v. Felton which
church-related elementary and secondary schools. Both statutes are prohibited public school teachers from teaching in parochial schools.
challenged as violative of the Establishment and Free Exercise Clause. The current proposal offered help to needy students in private schools
The Pennsylvanian statutes reimburse the cost of teachers’   salaries,   by sending public school teachers to tutor them after school. New York
textbooks, and instructional materials in specified secular subjects. was forced to offer remedial help to students through 'local educational
Rhode Island pays directly to teachers in nonpublic elementary shools a agencies'. Students did not need to attend public schools in order to be
supplement of 15% of their salaries. A federal court upheld the eligible for the assistance. Those who were to receive tutoring were
Pennsylvania law while a District Court ruled that the Rhode Island law students who a) reside in low income areas or b) failed or were at risk
fostered 'excessive entanglement'. of failing the state's student performance standards.

64 | P LATON
Held: the Court allowed public school teachers to tutor private school the individual from obedience to a general law not aimed at the
students in their private schools. Because of the Supreme Court's ruling promotion or restriction of religious beliefs. The mere possession of
in Aguilar, the State had been forced to tutor students in either public religious convictions which contradict the relevant concerns of a
schools or mobile units outside of the parochial schools. As a result, the political society does not relieve the citizen from the discharge of
school board was forced to deduct $7.9 million dollars from their political responsibilities." Finally, compulsion is not a legitimate means
budget for transportation and establishing the mobile units. An earlier for creating national unity.
ruling in Zobrest disavowed a ban of placing all public employees in
sectarian schools (a sign language interpreter had been provided for a Gerona v. Secretary of Education
deaf student). Additionally, not all government aid that directly assists 106 Phil. 2 (1959)
in the educational function of religious schools is invalid. The location of
the classroom (either in public or religious schools) should not matter. Facts: In accordance to Sec. 2 of R.A. 1265 of June 11, 1995, authorizing
Furthermore, there is little difference between providing a sign and directing the Sec. of Education to issue or cause to be issued rules
language interpreter, which the Court already allowed, and a tutor. The and regulations for the proper conduct of the flag ceremony, he issued
interaction that would result between the state and church is allowable Department Order No. 8 series of 1995 making it compulsory for public
because a relationship between the two is inevitable. This decision and private schools to have daily flag ceremonies. Petitioners,
reinforces the belief that the state can conduct public programs in members of the religious group known as   Jehovah’s   Witnesses,   have  
religious schools without becoming excessively entangled with the children attending a school in Masbate and said children refused to
religion. This is contrary to the earlier attitude that there must be an salute the flag, sing the national anthem and recite the patriotic pledge.
absolute wall between public and religious schools. As a result, they were expelled from school. They wrote to the
Secretary of Education petitioning that their children be allowed to
The Free Exercise Clause remain silent, stand at attention and that they be exempted from
saluting the flag, singing the national anthem and reciting the patriotic
Under this part of the religious freedom guarantee, the state is pledge.   Their   ground   is   their   belief   as   Jehovah’s   witnesses   that   they
must  not  “bow  down  nor  serve”  any  image  (Ex  20:  4-5), and the flag, for
prohibited from unduly interfering with the outward
them, is considered as an image of the State. However, the Secretary
manifestations  of  one’s  beliefs  and  faith denied such petition. Petitioners filed a complaint against them and
prayed for the declaration of Department Order No. 8 as invalid and
With this free exercise guarantee, the individual is assured that contrary to the Bill of Rights, but the trial court dismissed such
his religion would not be confined to thought only but also may complaint.
find   expression   through   action.   “The   Free   Exercise   Clause  
accords absolute protection to individual religious convictions Issue: WON Department Order No. 8, which made it compulsory for
and beliefs and proscribes government from questioning a public and private schools to have daily flag ceremonies, is valid and
constitutional
person’s   beliefs   or   imposing   penalties   or   disabilities   based  
solely on those beliefs. The Clause extends protection to both Held: The Filipino flag is not an image which requires religious
beliefs and unbelief” veneration but it is a symbol of the Republic of the Philippines. Also, the
flag salute is not a religious ceremony but an act of profession of love
Further, while freedom to believe may be absolute, the liberty and allegiance and pledge of loyalty to the fatherland which the flag
to act on said belief is not so. The same may be subject to valid stands for. By the authority of the legislature, the Secretary of
state regulations designed to promote the general welfare and Education was duly authorized to promulgate Department Order No. 8
the interests of society which does not violate the Constitutional provision about freedom
and religion and exercise of religion. And since compliance with the
non-discriminatory and reasonable rules and regulations and school
The government must be able to show a compelling state discipline, which includes the observance of the flag ceremony, is a pre-
interest in order to justify any regulation that may impair requisite to attendance in public schools, the failure and refusal of the
religious exercise petitioners to participate in the flag ceremony led to their proper
exclusion and dismissal.
The Free Exercise Clause principally guarantees voluntarism,
although the Establishment Clause also assures voluntarism by Ebralinag v. Division of Superintendent of schools of Cebu
placing the burden of advancement of religious groups on their 219 SCRA 256 (1993)
intrinsic merits and not on the support of the state
Facts: The petitioners, assisted by their parents, are high school and
West Virginia State Board of Education v. Barnette grade school students  who   belong   to   the   religious   group   of   Jehovah’s  
319 US 624 (1943) Witnesses. They were expelled from school by the public authorities in
Cebu because they refuse to salute the flag, sing the national anthem,
Facts: s part of instituting a required curriculum teaching American and recite the patriotic pledge as required by R.A. No. 1265 of July 11,
values, the state of West Virginia forced students and teachers to 1995 and by Department Order No. 8 of July 21, 1995 of DECS which
participate in saluting the flag. Failure to comply with this resulted in made the flag ceremony compulsory in all educational institutions.
expulsion and the student was considered illegally absent until Jehovah’s   Witnesses   admittedly   teach   their   children   not   to   salute   the  
readmitted. A group of Jehovah's Witnesses refused to salute the flag flag, sing the national anthem and recite the patristic pledge for they
because it represented a graven image that was not to be recognized. consider  such  acts  as  ‘’acts  of  worship’’  which  should  be  only  given  to  
God.   They   feel   bound   by   the   Bible’s   command   to   ‘’guard   themselves  
Held: the Court ruled that the school district violated the rights of against  idols’’  (1  John  5:21)  as  they  also  consider  the  flag  as  an  image  or  
students by forcing them to salute the American flag. The refusal of the idol representing the State.
students to say the pledge did not infringe on the rights of other
students. The flag salute required students to declare a belief that was Issue: WON the school children who are members of a religious sect
contrary to their faiths. The state did not claim that a clear and present known   as   Jehovah’s   Witnesses   may   be   expelled   from   school   for  
danger would be created if the students remained passive during the refusing, on account of their beliefs, to take part in the flag ceremony,
pledge. Unlike the decision in Gobitis, this Court does not believe that which includes playing by a band or singing the Philippine national
allowing an individual's rights to be supported over government anthem, saluting the Philippine flag, and reciting the patriotic pledge
authority is a sign of a weak government. "Conscientious scruples have
not, in the course of the long struggle for religious toleration, relieved Held: Although the petitioners do not take part in the compulsory flag
ceremony,  they  do  not,  however,  engage  in  ‘’external  acts’’  or  behavior  

65 | P LATON
that would offend their countrymen who express their love of country not a regulation. If a certificate is procured, solicitation is permitted
through the flag ceremony. They quietly stand at attention during the without restraint, but in the absence of which, solicitation is altogether
flag ceremony to show respect for the right of those who participate in prohibited. However, they argue that requiring them to procure a
the solemn proceedings. Since they do not engage in disruptive certificate would be a restraint on the exercise of their religion. The
behavior, there is no warrant for their expulsion. conviction of Jesse must be reversed since there was no evidence that
his deportment was noisy, truculent, overbearing or offensive. He
The sole justification for a prior restraint or limitation on the exercise of actually had not invaded a right or interest of the public or of the men
religious freedom is the existence of a grave and present danger of a accosted. It likewise did not amount to a breach of the peace. The
character both grave and imminent, of a serious evil to public safety, petitioner’s   communication   raised   no   such   clear   and   present   danger  
public morals, public health, or any other legitimate public interest, to public peace and order.
that the State has a right and duty to prevent. Without such threat to
public safety, the expulsion of the petitioners from school is not Iglesia Ni Cristo v. CA
justified. 259 SCRA 529 (1996)

Also, the  expulsion  of  the  members  Jehovah’s  Witnesses  from  schools   Facts:   Petitioners   were   members   of   the   “Iglesia   ni   Cristo”   religious  
where they are enrolled will violate their right as citizens of the group. As they have TV programs aired during Saturdays and Sundays,
Philippines, under the 1987 constitution, to receive free education, about the propagation of their religion, they submitted to the Board of
because  it  is  the  duty  of  the  State  to  “protect  and  promote the right of Review for Moving Pictures and Television the VTR series of their
all citizens to quality education... and to make such education program.   However,   the   Board   classified   such   series   as   “X”   or   not   for  
accessible  to  all”  Sec.  1,  Art.  XIV. public viewing since they offend and constitute an attack against other
religions which is expressly prohibited by law. Petitioner filed two
Resolution on Motion for Reconsideration courses of action against the Board and appealed to the Office of the
251 SCRA 569 (1995) President which reversed the decision of the respondent Board and
allowed the series for public viewing. Petitioner also filed an case
Facts: The State moves for a reconsideration of the decision on March against the respondent Board with the RTC of Quezon City on the
1, 1993  which  granted  private   respondents’   petition  for   certiorari   and   ground that the Board acted without jurisdiction or with grave abuse of
prohibition and annulled the expulsion orders issued by said discretion in requiring them to submit their VTR tapes and x-rating
respondents on the ground that the decision created and exemption in them. The RTC ordered the Board to grant the petitioner the necessary
favor  of  the  Jehovah  Witnesses’  religious  group  members  ,  in violation permit for public viewing but it directed the petitioner to refrain from
of  the  “Establishment  Clause”  of  the  Constitution.    Although  they  refuse attacking  or  offending  other  religions.  Upon  the  petitioner’s  motion  for  
to salute the flag, they are willing to stand quietly and peacefully at reconsideration, the directive to refrain from attacking other religions
attention in order not to disrupt the ceremony or to disturb those who was deleted and the Board was prohibited from requiring the petitioner
believe differently. to submit a review of their VTR of their program. The CA however
reversed the RTC decision and held that the Board has the jurisdiction
Issue: WON the compulsory flag salute is valid and  power  to  review  the  petitioner’s  TV  program  and  that  it  did  not  act  
with grave abuse of discretion. The CA likewise found the series
Held:   The   refusal   of   the   Jehovah’s   Witnesses   in   saluting   the   flag   is   “indecent,  contrary  to  law  and  contrary  to  good  customs”.
based on their religious belief which is shared by their entire
community. As the petitioners were expelled because of their religious Issues:  (1)  WON  respondent  Board  has  the  power  to  review  petitioner’s  
belief, such action, as stated by the Court, was against religious TV program
practice. It is likewise apparent that the said orders and memoranda (2) assuming it has power, WON it gravely abused its discretion when it
would gravely endanger the free exercise of the religious beliefs of the prohibited   the   airing   of   petitioner’s   religious   program   series   on   the  
members of their sects. The refusal to salute the flag is not the same as ground that they constitute an attack against other religions and that
the refusal to pay taxes and to submit to compulsory vaccination since they are indecent, contrary to law and good customs.
the former has no threat to the life or health of the State. Thus, there is
no reason for compulsory or coercive flag salute. Although the Held: (1) PD No. 1986 gives the Board the power to screen, review,
Constitution provides for a national flag, it does not give the State the examine   all   “television   programs,   and   to   “approve,   delete,   and/or  
power to compel a salute to the flag. prohibit   the  exhibition  and/or  television  broadcast  of  TV  programs”.  It  
also directs the Board to apply contemporary Filipino cultural values as
Cantwell v. Connecticut standard   to   “determine those which are objectionable for being
310 U.S. 296, 84 L Ed 1213, 60 S Ct 900 (1940) immoral, indecent, contrary to law and/or good customs, injurious to
the prestige of the Republic of the Philippines and its people, or with a
Facts: Newton Cantwell and his 2 sons, Jesse and Russell, are members dangerous tendency to encourage the commission of violence or of a
of  the  religious  group  Jehovah’s  Witnesses.  They  were  arrested  because wrong   or   a   crime.”   Thus,   it   has   jurisdiction   to   review   petitioner’s   TV  
they were engaged in selling books and soliciting in a neighborhood program  entitled  “Ang  Iglesia  ni  Cristo”.
street which is thickly populated by 90% Roman Catholics. Jesse *The right to religious profession and worship has a 2-fold aspect:
Cantwell, asked two men to listen to a phonograph record which freedom   to   believe   and   freedom   to   act   on   one’s   belief.   The   first   is  
attacked their religion as they were Catholics. They were tempted to absolute as long as the belief is confined within the realm of thought
strike Cantwell unless he went away. Thus, Cantwell went away and while the second is subject to regulation where the belief is translated
there was no evidence that he was personally offensive or had entered into external acts that affect the public welfare.
into any argument with them. However, the Cantwells were arrested in (2) However, the Court reversed the ruling of the CA since the claim of
violation of the General Statutes of Connecticut which prohibited the Board that such TV programs attack the Catholic faith, has no basis.
solicitation of money, services, subscriptions, or any valuable thing for The CA has not even viewed the tapes as they were not presented as
any alleged religious, charitable or philanthropic cause unless such evidence but it considered them as indecent, contrary to law and good
cause is approved by the secretary of public welfare council. They were customs. Such   ruling   suppresses   petitioner’s freedom of speech and
also convicted of the common law offense of inciting breach of peace. interferes with its right to free exercise of religion. The respondents
cannot   also   rely   on   the   ground   “attacks   another   religion””   in   x-rating
Issue: WON the method adoption by Connecticut to that end the religious program of the petitioner since such claim was merely
transgresses the liberty safeguarded by the Constitution added by the Board to its Rules. They have also failed o apply the clear
and present danger rule since there is no showing of the type of harm
Held: The statute deprives the appellants of their liberty without due that the tapes would bring about especially the gravity ad imminence of
process of law in contravention of the 14th Amendment for which they the threatened harm.
were arrested. They were also right in their insistence that the Act is

66 | P LATON
US v. Ballard
322 U.S. 78, 88 L Ed 1148, 64 S Ct 882 (1944) It is a flat license tax levied and collected as a condition to the pursuit of
activities whose enjoyment is guaranteed by the constitutional liberties
Facts:   Respondents   belong   to   the   “I   Am”   movement   which   promotes   of press and religion and inevitably tends to suppress their exercise.
the religious belief and doctrines that a certain Guy W. Ballard, now
deceased, alias Saint Germain, had been selected and designated by Ordinance No. 3000 does not impose any charge upon the enjoyment
alleged   “ascertained   masters”   as   a divine messenger. They also of a right granted by the Constitution, nor tax the exercise of religious
believed   that   the   messages   of   such   “ascertained   masters”   and   of   the   practice.
alleged divine entity, Saint Germain, would be transmitted to mankind
through Guy W. Ballard. Edna W. Ballard and Donald Ballard were also In Re Summers
allegedly selected as messengers just like Guy. They also believed that 325 US 561, 89 L Ed 1795, 65 S Ct 1307 (1945)
they have supernatural powers which enable them to heal persons with
ailments. They were indicted and convicted for using and conspiring to FACTS:  Petitioner’s  prayer  for  admission  to  the  practice  of  law  in  Illinois  
use mails to defraud in the organization and promotion of their was  denied  by  the  State’s  Supreme  Court  allegedly  “on  the  sole  ground  
movement.   They   were   also   charged   since   they   “well   knew”   that   what   that   he   is   a   conscientious   objector   to   war,”   or,   “because   of   his  
they believed in was false. conscientious scruples against participation in  war”.  From  the  record,  it  
appears that Clyde Wilson Summers has complied with all the
Issues:   WON   in   respondents’   good   faith,   i.e.   whether   they   did   not   prerequisites for admission to the bar of Illinois except that he has not
believe those things, that Jesus or Saint Germain came down and obtained the certificate of the Committee on Character and fitness,
dictated, or those things that they wrote, and preached, they used the which committee declined to sign a favourable certificate on the
mail for the purpose of getting money ground that petitioner was a conscientious objector to war.

Held: On appeal, the Circuit CA reversed, holding that the decision of THE   COURT’S   RULING:   The   decision   of   the   Supreme   Court   of   Illinois  
the   District   Court   in   restricting   the   jury   to   the   issue   of   respondents’   denying  the  petitioner’s  admission  to  the  practice  of  law  in  Illinois  was  
good faith was error and granted a new trial. The SC granted certiorari AFFIRMED.
because of the importance of the question presented. The Circuit CA
held that the question of the truth of the representations concerning REASON: A conscientious belief in nonviolence to the extent that the
respondents’  religious  beliefs  or  doctrines  should  have  been  submitted   believer will not use force to prevent wrong, no matter how
to the jury and thus, it remanded the case for a new trial. aggravated, and so cannot swear in good faith to support the Illinois
Constitution, must disqualify such a believer for admission.
We   do   not   agree   that   the   truth   or   verity   of   respondents’   religious  
doctrines or beliefs should have been submitted to the jury. The The responsibility for choice as to the personnel of its bar rests with
District court ruled properly when it withheld from the jury all Illinois. Only a decision which violated a federal right secured by the
questions concerning the truth or falsity of the religious beliefs or Fourteenth Amendment would authorize our intervention...We cannot
doctrines   of   respondents.   “The   law   knows   no   heresy,   and   is   say that any such purpose to discriminate motivated the action of the
committed  to  the  support  of  no  dogma,  the  establishment  of  no  sect” Illinois Supreme Court.
(Watson v Jones, 13 Wall. 679, 728). The First Amendment does not
only   “forestall   compulsion   by   law   of   the   acceptance   of   any   creed   or   It is impossible for us to conclude that the insistence of Illinois that an
the  practice  of  any  form  of  worship”  but  it  also  “safeguards  the  free   officer who is charged with the administration of justice must take an
exercise of the chosen form of religion”   (Cantwell   v   Connecticut,   310   oath  to  support  the  Constitution  of  Illinois  and  Illinois’  interpretation of
U.S. 296, 303). It also does not select any one group or any type of that oath to require a willingness to perform military service violates
religion for preferred treatment. the principles of religious freedom.

American Bible Society v. City of Manila *However, Mr. Justice Black filed a DISSENT stating among other things
101 Phil. 386 (1957) that,  “Under  our  (US)  Constitution,  men  are  punished  for  what they do
or  fail  to  do,  and  not  for  what  they  think  and  believe.”
FACTS: The American Bible Society is a foreign, non-stock, non-profit,
religious, missionary corporation doing business in the Philippines Wisconsin v. Yoder
through its Philippine agency established in manila in 1898. In the 406 US 205, 32 L Ed 2d 15, 92 S Ct 1526 (1972)
course of its ministry, the Society has been distributing and selling
bibles and/or gospel portions thereof. Sometime in 1953, the acting FACTS: Respondents were members of the Amish religion. They and
City treasurer of manila required the Society to secure the their families are residents of Green County, Wisconsin.   Wisconsin’s  
corresponding   Mayor’s   permit   and   license   fees.   The   Society   paid   the   compulsory school attendance law required them to cause their
accumulated permit and license fees (P5,891.45) under protest and children to attend public or private school until reaching age 16, but the
then filed a complaint in court to question the constitutionality and respondents declined to send their children. They believed that, by
legality of the ordinances under which the said fees were being sending their children to high school, they would not only expose
collected. themselves to the danger of censure of the church community, but also
endanger their own salvation and that of their children. The Wisconsin
ISSUE:  “Whether  or  not  Ordinance  2529  (requiring  every  entity  engaged   Supreme Court sustained respondents claim under the Free Exercise
in business to pay a license fee based on gross sales) and ordinance Clause of the First Amendment, and reversed the convictions.
3000 (requiring that a municipal permit to be first obtained before
engaging in business) could be made applicable to the American Bible THE  COURT’S   RULING:  The   First   and  Fourteenth  Amendments  prevent  
Society.” the State from compelling respondents to cause their children to attend
formal high school to age 16.
THE   COURT’S   RULING:   Defendant   was   sentenced   to   return   to   the  
plaintiff the sum of P5,891.45 unduly collected from it. REASON: Old Order Amish communities today are characterized by a
fundamental belief that salvation requires life in a church community
REASON: The Constitutional guarantee of the free exercise and separate and apart from the world and worldly influence. In the Amish
enjoyment of religious profession and worship carries with it the right belief, higher learning tends to develop values they reject as influences
to disseminate religious information. Any restraint of such right can that alienate man from God.
only be justified like other restraints of freedom of expression on the
grounds that there is a CLEAR AND PRESENT DANGER OF ANY As the record strongly shows, the values and programs of the modern
SUBSTANTIVE EVIL which the State has the right to prevent. secondary school are in sharp conflict with the fundamental mode of

67 | P LATON
life mandated by the Amish religion. familiar inscribed yellow T-shirts, they started to march down said
street with raised clenched fists and shouts of government invectives.
The   enforcement   of   the   State’s   requirement   of   compulsory   formal   However, they were barred from proceeding any further, on the ground
education after the eighth grade would gravely endanger, if not that St. Jude Chapel was located within the Malacañang security area.
destroy,  the  free  exercise  of  respondents’  religious  beliefs. However, because of the alleged warning given them by respondent
Major Lariosa that any similar attempt by petitioners to enter the
The Amish alternative to formal secondary school education has church in the future would likewise be prevented, petitioners took this
enabled them to function effectively in their day-to-day life under self- present recourse.
imposed limitations on relations with the world, and to survive and
prosper in contemporary society as a separate, sharply identifiable and THE  COURT’S  RULING:  The  instant  petition  is  hereby  dismissed.
highly self-sufficient community for more than 200 years in this
country. REASON: Undoubtedly, the yellow T-shirts worn by some of the
marchers, their raised clenched fists, and chants of anti-government
slogans strongly   tend   to   substantiate   the   respondents’   allegation   that  
*To have the protection of the religion Clauses, the claims must be the petitioners intend to conduct an anti-government demonstration.
rooted in religious belief.
While it is beyond debate that every citizen has the undeniable and
Victoriano  v.  Elizalde  Rope  Workers’  Union inviolable right to religious freedom, the exercise thereof, and of all
59 SCRA 54 (1974) fundamental rights for that matter, must be done in good faith.

FACTS:  Appellee,  a  member  of  the  religious  sect   known  as  the  “Iglesia   The restriction imposed is necessary to maintain the smooth
ni   Cristo”   had   been   in   the   employ   of   the   Elizalde   Rope   Factory,   Inc. functioning of the executive branch of the government, which
since 1958. As such employee, he was a member of the Elizalde Rope petitioners’  mass  action  would  certainly  disrupt.
Workers’   Union.   A   closed-shop provision was contained in the
collective bargaining agreement which required membership in the DOCTRINE   LAID   DOWN   IN   THIS   CASE:   “If   the   exercise   of   said   religious  
Union as a condition of employment for all permanent employees. belief clashes with the established institutions of society and with the
However,  RA  3350  was  enacted,  providing  that,  “such  agreement  shall   law,  then  the  former  must  yield  and  give  way  to  the  latter.”
not cover members of any religious sects which prohibit affiliation of
their members in any such labor organization. Thereupon, the Union However, several justices filed separate opinions:
wrote a formal letter to the Company asking the latter to separate
appellee from the service in view of the fact that he was resigning from FERNANDO, C.J. –concurs in the result but dissents insofar as the
the Union as a member. The management of the company informed opinion fails to declare that the freedom of exercise of religious
appellee that unless he could achieve a satisfactory arrangement with profession and worship could only be limited by the existence of a clear
the Union, the company would be constrained to dismiss him from the and present danger of a substantive evil.
service.
ABAD SANTOS, J. (dissenting) –“It  is  highly  presumptuous  for  both  the  
THE   COURT’S   RULING:   The   decision   of   the   Court   of   First   Instance   of   respondents and this Court to attribute unstated and unadmitted
Manila enjoining the Company from dismissing the appellee and motives to the petitioners. I fail to perceive the presence of any clear
sentencing   the   Union   to   pay   Appellee   P500   for   attorney’s   fees   and   danger to the security of Malacañang due to the action of petitioners.”
costs of the action is AFFIRMED.
MELENCIO-HERRERA, J. (dissenting) –“The   location   of   the   St.   Jude  
REASON: It is clear that the right to join a union includes the right to Chapel within the perimeter of the Malacañang security area is not, to
abstain from joining any union. my  mind,  sufficient   reason  for  a  prior  restraint  on  petitioner’s  right  to  
freedom of religious worship.
RA 3350 merely excludes ipso jure from the application and coverage of
the closed shop agreement the employees belonging to any religious RELOVA, J. (Separate vote and statement) –“The   fact   that   petitioners  
sects which prohibit affiliation of their members with any labor chose a Tuesday to hear mass and/or pray for their special intention
organization. In spite of any closed shop agreement, members of said negates the suspicion that they were out to stage a demonstration.
religious sects cannot be refused employment or dismissed from their Respondents should have allowed petitioners to hear mass and/or pray
jobs on the sole ground that they are not members of the collective and,  thereafter,  see  what  they  would  do.”
bargaining union.
Estrada v. Escritor
The free exercise of religious profession or belief is superior to contract 408 SCRA 1 (2003)
rights. It is only where unavoidably necessary to prevent an IMMEDIATE
AND GRAVE DANGER to the security and welfare of the community that FACTS: Compalinant Alejandro Estrada wrote to the presiding judge of
infringement of religious freedom may be justified, and only to the Branch 253, Regional Trial Court of Las Piñas City, requesting for an
smallest extent necessary to avoid the danger. investigation of rumors that respondent Soleded Escritor, court
interpreter in said court, is living with a man not her husband. They
The exemption from the effects of the closed shop agreement does not allegedly have a child of eighteen to twenty years old. She admitted
directly advance, or diminish, the interests of any particular religion. that she has been living with Luciano Quilapio Jr., without the benefit of
The benefit upon the religious sects is merely incidental and indirect. marriage for twenty years and that they have a son. Quilapio was
likewise married at that time, but had been separated in fact from his
*A statute in order to withstand the strictures of constitutional wife.   But   as   a   member   of   the   religious   sect   known   as   the   Jehovah’s  
prohibition, must have a secular legislative purpose and a primary Witnesses and the Watch Tower and Bible Tract Society, their conjugal
effect that neither advances nor inhibits religion. arrangement is in conformity with their religious beliefs. In fact, the
respondent has executed a Declaration of Pledging Faithfulness that is
German v. Barangan recognized as giving the parties the right to marital relationship even if
135 SCRA 514 (1985) not recognized by civil authorities.

FACTS: Petitioners –composed of about 50 businessman, students and PRINCIPAL  ISSUE:   “Whether   or   not   respondent   should   be   found  guilty  
office employees—converged at J.P. laurel Street, Manila, for the of  the  administrative  charge  of  gross  and  immoral  conduct”.
ostensible purpose of hearing mass at the St. Jude Chapel which adjoins
the Malacañang grounds located in the same street. Wearing the SUB-ISSUE:   “Whether   or   not   respondent’s   right   to   religious   freedom  

68 | P LATON
should carve out an exception from the prevailing jurisprudence on in carefully balancing the interests and values implicated in
illicit relations for which government employees are held each case that may be brought before them
administratively  liable.”
Religious Affairs, Intramural Disputes and Secular Jurisdiction
THE   COURT’S   RULING:     The   case   is   REMANDED   to   the   Office   of   the  
Court Administrator.
While generally it should be no business of the government
The Solicitor General is ordered to intervene in the case where it will be trying to meddle in problems among members of the same
given the opportunity: (a) to examine the sincerity and centrality of sectarian group, the same would hold true only on matters of
respondent’s   claimed   religious   belief   and   practice;   (b)   to   present   purely religious dogma and discipline. It cannot, withhold its
evidence  on  the  state’s  “compelling  interest”  to  respondent’s  religious   judicial or administrative machineries when the dispute spills
belief and practice and (c) that the means the state adopts in pursuing over to the secular concerns of society
its  interest  is  the  least  restrictive  to  respondent’s  religious  freedom.

REASON: The jurisdiction of the Court extends only to public and


When the problem is about more mundane matters, however,
secular morality. such as when they involve conflicting claims to the same
property, or when they are concerned with civil aspects of
Should   the   Court   declare   respondent’s   conduct   as   immoral and held membership in society, such as employment, then the courts
her administratively liable, the Court will be holding that in the realm of and secular authorities would have to interpose their power
public morality, her conduct is reprehensible or there are state
interests overriding her religious freedom. But in so ruling, the Court In matters purely ecclesiastical the decisions of the proper
does not and cannot say that her conduct should be made
church tribunals are conclusive upon the civil tribunals
reprehensible in the realm of her church where it is presently
sanctioned and that she is answerable for her immorality to Jehovah
God nor that other religions prohibiting her conduct are correct. Austria v. NLRC
312 SCRA 410 (1999)
In any event, even if the court  deems  sufficient  respondent’s  evidence  
on the sincerity of her religious belief and centrality to her faith, the Petitioner was a pastor of private respondent Central Philippine Union
case  at  bar  cannot  still  be  decided  using  the  “compelling  state  interest”   Mission Corporation of the Seventh Day Adventist, a religious
test. corporation duly organized and existing under Philippine Law.
Petitioner worked with the SDA for 28 years. Petitioner one day
The government should be given the opportunity to demonstrate the received a letter of dismissal citing misappropriation of funds, willful
compelling state interest it seeks to uphold in opposing the breach of trust, serious misconduct, gross and habitual neglect of duties
respondent’s  stance  that  her  conjugal  arrangement  is  not  immoral  and   and the like as grounds for the termination of his services. Petitioner
punishable as it comes within the scope of the free exercise protection. then filed a complaint against SDA and its officers before the Labor
Arbiter for illegal dismissal and ruled in his favor. On appeal, the NLRC
*The benevolent neutrality approach is used in interpreting the vacated  the  arbiter’s  decision  and  dismissed  the  case  for  want  of  merit.  
religious clauses in the Philippine Constitution. The private respondents filed a motion for reconsideration and ruled
*Benevolent Neutrality Approach –looks further than the secular that it has no jurisdiction over the matter because it involves
purposes of government action and examines the effect of these ecclesiastical affair to which state cannot interfere.
actions on religious exercise.
*Tests must be applied to draw the line between permissible and HELD: The separation of church and state finds no application in this
forbidden religious exercise: case. What is involved is the relationship of the church as an employer
and  the  minister  as  an  employee.  The  grounds  invoked  for  petitioner’s  
CLEAR AND PRESENT DANGER TEST; IMMEDIATE AND GRAVE DANGER dismissal are all based on Article 282 of the Labor code which
TEST –involves religious speech as this test is often used in cases of enumerates just causes for termination of employment. By this alone, it
freedom of expression. is  palpable  that  the  reason  for  petitioner’s  dismissal from the service is
not religious in nature. And it is clear that when the SDA dismissed the
COMPELING STATE INTEREST TEST –is proper where conduct is petitioner, it was merely exercising its management prerogative to fire
involved. an employee which it believes to be unfit for the job.

In a vain attempt to support their claim of breach of trust, private


The Tension Between the Establishment Clause and the Free respondents pin on the petitioner the alleged non-remittance of the
Exercise Clause tithes collected by his wife. As proven by convincing and substantial
evidence it was the wife of the petitioner who collected the tithes and
The two subclauses of the provision on religion are meant to failed to remit the same and not the petitioner. There being no
ensure the guarantee that they were meant to vivify conspiracy or collusion, the petitioner cannot be held accountable for
the alleged infraction of his wife.
The First Amendment forbids both the abridgment of the free
Wherefore, the petition for certiorari is granted. The challenged
exercise   of   religion   and   the   enactment   of   laws   ‘respecting   an  
resolution of the public respondent NLRC, is nullified and set aside. The
establishment  of  religion’ decision of the Labor Arbiter is reinstated and hereby affirmed.

Even as they might have common purpose, the two clauses Islamic  Da’wah  Council  Of  The  Phils.,  Inc  v.  Office  of  the  Executive  
may somehow come into conflict if they were allowed to be Secretary
extended to their logical end 405 SCRA 497 (2003)

There is a natural antagonism between a command not to Petitioner is a non-governmental organization that extends voluntary
establish religion and a command not to inhibit its practice; this services to the Filipino, especially to Muslim communities. It claims to
be a federation of national Islamic organization and an active member
tension between the religion clauses often leaves the courts
of international organizations such as RISEAP which had accredited the
with a choice between competing values in religion cases. petitioner to issue halal certifications in the Philippines. However, EO
Resolving the tension, of course, would again involve the courts 46 was issued creating the Philippine Halal Certification Scheme and

69 | P LATON
designing the Office of the Muslim Affairs to oversee its state’s  limited  constitutional   convention  deprived   appellant   McDaniel,  
implementation. OMA sent letters to food manufacturers asking them of the right to the free exercise of religion guaranteed by First and
to secure the halal certification only from it lest they violate EO 46 and Fourteenth Amendment.
RA 4109. As a result, petitioner lost revenues after food manufacturers
stopped securing certification from it. Petitioner filed instant petition Tennessee has failed to demonstrate that its views of the dangers of
for prohibition, contending that EO 46 violates the constitutional clergy participation in the political process have not lost whatever
provision on separation of the church and state since the halal validity they may once have enjoyed. The essence of the rationale
certification scheme is a function only of religious organization. restriction on ministers is that, if elected to public office, they will
necessary exercise their powers and influence and promote the
HELD: The petition is granted. OMA deals with the societal, legal, interests of one sect or thwart the interest of another, thus pitting one
political and economic concerns of the Muslim community as a against the others, contrary to the anti-establishment principle with its
“national   cultural   community”   and   not   as   a   religious   group.   Without   command of neutrality. However, the American experience provides no
doubt, classifying food product as halal is a religious function because persuasive support for the fear that clergymen in the public office will
the   standards   used  are   drawn  from   the   Qu’ran   and  Islamic   beliefs.  By   be less careful of anti-establishments interests or less faithful to their
giving OMA the exclusive power to classify food products as halal, EO oaths of civil office than their unordained counterparts.
46 encroached on the religious freedom of Muslim organizations to
interpret for Filipino Muslims what food product are fit for Muslim We  hold  that  S4  of  ch.  848  violates  McDaniel’s  First  Amendment  right  
consumption. The halal certifications issued by petitioner and similar to free exercise of his religion made applicable to the states by the
organizations come forward as the religious approval of a food product Fourteenth Amendment. Accordingly, the judgment of Tennessee SC is
fit for Muslim consumption. reversed, and the case is remanded to that court for further
proceedings.
Wherefore, the petition is granted. EO 46 is hereby declared null and
void. Consequently, respondents are prohibited from enforcing the Religious Leaders and Participation in Political Affairs
same.
While on one side may be the claim that as members of society,
Conscientious Objectors as citizens themselves, such religious leaders and their flock are
still entitled to participate in political affairs and help chart the
The right of conscientious objectors is based more on statutory course of the country, on the other side is the need to keep
provisions than the constitutional freedom of religion. It is a those same sectarian principals from intruding into purely
governmental accommodation extended to those, who out of secular concerns thereby limiting them to their spiritual
religious scruples, may find themselves opposed to war and the province
concomitant taking of lives
Additional Cases
In order to qualify for classification as a conscientious objector, (G) Freedom of Religion [1-2]
a registrant must satisfy three basic tests:
1. He must show that he is conscientiously oppose to war in any Chapter 8
form Liberty of Abode and Freedom of Movement
2. He must show that his opposition is based upon religious
training and belief, as the term has been construed in our “The   liberty   of   abode   and   of   changing   the   same   within   the  
decisions limits prescribed by law shall not be impaired except upon
3. He must show that this objection is sincere lawful order of the court. Neither shall the right to travel be
impaired except in the interest of national security, public
Religious Test and Exercise of Political and Civil Rights safety,  or  public  health,  as  may  be  provided  by  law.”
13

The last sentence of the religion clause declares that no The assurance of the right to live and stay where [one] wants
religious test shall be required for the exercise of civil and within legal bounds. Thus, unless there be important societal
political rights. This should follow if the State is really to be free considerations   and   interests   that   are   implicated   by   a   person’s  
from meddling into religious affairs decision to stay anywhere, he should be given complete
freedom of choice as to where he may want to dwell and set up
This prohibition against religious test has reference to the his abode. This right, pursuant to the Constitution, may only be
historically and constitutionally discredited policy of probing impaired by lawful order of the court
religious beliefs by test oaths or limiting public offices to
persons who have, or perhaps more properly profess to have, a Corollary  to  this  liberty  to  establish  one’s  abode  is  the  freedom  
belief in some particular kind of religious concept of locomotion, of moving about, travelling from place to place
as an individual may so deem appropriate or desirable
McDaniel v. Paty
435 US 618, 55 L Ed 2d 593, 98 S Ct 1322 (1978)
If there are no countervailing national security, public safety or
Paty, a candidate for delegate to the Tennessee constitutional public health interests that are implicated, a person should be
convention, sued for a declaratory judgment that McDaniel, an free to go where he pleases
opponent who was Baptist minister, was disqualified as serving as
delegate. That court held that the statutory provision violated the First Also, another limitation on freedom of locomotion involves
and Fourteenth Amendments. After the election, the Tennessee SC those applicable to people who might have pending criminal
reversed, holding that the clergy disqualification imposed no burden on cases, in which event their movement or choice of abode may
“religious   belief,”   and   restricted   religious   action   .   .   .   [only]   in   the   law
be restricted by the courts as part of the condition of their
making process of the government – where religious action is
absolutely prohibited by the establishment clause.
being out on bail

WON, Minister or priest is barred from serving as delegates to the


13 CONSTITUTION, Art. III, § 6
70 | P LATON
Rubi v. Provincial Board of Mindoro
39 Phil. 660 (1919) FACTS: After the US had broken diplomatic relations with Cuba and the
Department of State had eliminated Cuba from the area for which
Resolution No. 25 of the Provincial Board of Mindoro creates for a passports were not required, Louis Zemel applied to have his passport
permanent settlement of the Manguianes. Pursuant to Resolution No. validated  for   travel   to   Cuba  “to   satisfy   [his]   curiosity . . . and to make
25 and Section 2145 of the Administrative Code of 1917, the Governor [him]   a  better   informed  citizen”.  His   request   was  denied,   and  he   filed  
directed the Mangyanes in the province to take up their habitation on suit seeking a judgment declaring that:
Sitio of Tigbao on Lake Naujan. Any mangyan who refuses to comply 1. He was entitled under the Constitution and the laws of the
with the order shall, upon conviction, be sentenced to imprisonment. US to travel to Cuba and to have his passport validated for
Petitioners challenge the constitutionality of Section 2145 of the AC on that purpose;
among others, due process and equal protection grounds. 2. The  Secretary  of  State’s  travel  restrictions  were  invalid;  and  
that
The reasons for the action of the Resolution of the Provincial board are 3. The Passport Act of 1926 and Section 215 of the Immigration
as follows: (1) the failure of former attempts for the advancement of and Nationality Act of 1952 were unconstitutional.
non-Christian people of the province; (2) the only successful method for
educating the Mangyans was to oblige them to live in a permanent ISSUE: WON the Secretary of State is statutorily authorized to refuse to
settlement; (3) the protection of the Mangyans; (4) the protection of validate the passports of US citizens for travel to Cuba, and, if he is,
the public forests in which they roam; and (5) the necessity of WON the exercise of that authority is constitutionally permissible.
introducing civilized customs among the Mangyans. What the
government wished to do by bringing them into a reservation was to RULING: YES. The court thinks that the Passport Act of 1926 embodies a
gather the children for educational purposes, and to improve the health grant of authority to the Executive to refuse to validate the passports of
and ,orals – was in fine, to begin the process of civilization. US  citizens  for  travel  to  Cuba.  That  Act  provides,  in  pertinent  part:  “The  
The Mangyans, in order to fulfill this governmental policy, must be Secretary of State may grant and issue passports . . . under such rules as
confined for a time, for their own good and good of the country. the President shall designate and prescribe for and on behalf of the
United   States.   .   .   .”   The   right   to   travel   within   the   United   States   is,   of  
The action pursuant to section 2145 of the AC does not deprive a course, also constitutionally protected. But that freedom does not
person of hid liberty without due process of law and does not deny to mean that areas ravaged by flood, fire or pestilence cannot be
him the equal protection of laws, and that confinement in reservation quarantined when it can be demonstrated that unlimited travel to the
in accordance with said section does not constitute slavery and area would directly and materially interfere with the safety and welfare
involuntary servitude. Further, section 2145 of the AC is a legitimate of the area or the nation as a whole. So it is with international travel (as
exertion of the police power. Section 2145 of the AC of 1917 is is the case here).
constitutional.
Marcos v. Manglapuz
Petitioners are not lawfully imprisoned or restrained of their liberty. 177 SCRA 668(1989)
Habeas corpus can, therefore, not issue.
FACTS: In February 1986, Ferdinand E. Marcos was deposed from the
Edwards v. California presidency via the non-violent   “people   power”   revolution   and   forced  
62 S Ct 164 (1941) into exile in Hawaii. Corazon Aquino was declared President of the
Republic. Soon, the Marcos family sought to be allowed to return to the
FACTS: Edwards, a citizen of the United States and a resident of country but President Aquino barred their return. The Marcos family
California, bought to California from Texas his indigent brother in law, filed the instant petition for mandamus and prohibition asking the
likewise a citizen of the US and a resident of Texas. He was Court to order the respondents to issue travel documents to Mr.
subsequently charged with and convicted of violating a California law Marcos and the immediate members of his family and to enjoin the
which penalizes one who knowingly brings into the State an indigent implementation  of  the  President’s  decision  to  bar  their  return  from  the  
person who is not a resident of the State, knowing him to be such an Philippines.
indigent person.
ISSUE: WON, in the exercise of the powers granted by the Constitution,
ISSUE: WON the prohibition embodied in said California law against the the President may prohibit the Marcoses from returning to the
“bringing”  or  transportation  of  indigent  persons  into  California  is  within   Philippines.
the police power of that State.
RULING: YES. The President did not act arbitrarily or with grave abuse
RULING: NO. It is not a valid exercise of the police power of California, it of discretion in determining that the return of former President Marcos
imposes an unconstitutional burden upon interstate commerce, and and his family at the present time and under present circumstances
thus the conviction under it cannot be sustained. Article 1, 8 of the poses a serious threat to national interest and welfare and in
Constitution delegates to the Congress the authority to regulate prohibiting their return to the Philippines. That the President has such
interstate commerce. And it is settled beyond question that the power under the Constitution has been recognized by members of the
transportation   of   persons   is   “commerce”   within   the   meaning   of   that   Legislature. The request or demand of the Marcoses to be allowed to
provision. But this does not mean that there are no boundaries to the return to the Philippines cannot be considered in the light solely of the
permissible area of State legislative authority. There are. And none is constitutional provisions guaranteeing liberty of abode and the right to
more certain than the prohibition against attempts on the part of any travel, subject to certain exceptions, or of case law which clearly never
single state to isolate itself from difficulties common to all of them by contemplated situations even remotely similar to the present one. It
restraining the transportation of persons and property across its must be treated as a matter that is appropriately addressed to those
borders. The California law in question is squarely in conflict with this as residual unstated powers of the President which are implicit in and
its express purpose and inevitable effect is to prohibit the correlative to the paramount duty residing in that office to safeguard
transportation of indigent persons across the California border. and protect general welfare. In that context, such request or demand
should submit to the exercise of a broader discretion on the part of the
CONCURRING OPINION: The right to move freely from State to State is President to determine whether it must be granted or denied. The
an incident of national citizenship protected by the privileges and petition was dismissed.
immunities clause of the Fourteenth Amendment against state
interference. DISSENTING   OPINION:   It   is   of   the   dissenters’   belief   that   Marcos,   as   a
citizen of the Philippines, is entitled to return to and live – and die – in
Zemel v. Rusk his own country. The government failed dismally to show that the
85 S Ct 1271 (1965) return of Marcos dead or alive would pose a threat to the national

71 | P LATON
security as it had alleged. Publication should be a prerequisite to effectivity

Resolution on the Motion for Reconsideration In so far as publication is concerned, it would be preferable that
178 SCRA 760 (1989) the same be done through the means which is more readily
accessible to the people, i.e., newspapers of general circulation,
FACTS: The SC, in its decision on Marcos v. Manglapuz, dismissed said
in addition to the traditional manner of doing through the
petition (close fight, voting 8 to 7). On September 28, 1989, Marcos
died in Honolulu, Hawaii. President Aquino then declared that his
Official Gazette
remains will not be allowed to be brought to the Philippines until such
time as the government shall otherwise decide, and thus the Tañada v. Tuvera
petitioners filed their motion for reconsideration. 136 SCRA 27 (1985)

RULING: The death of Marcos has not changed the factual scenario FACTS: Invoking the  people’s  right  to  be  informed  on  matters  of  public  
under which   the   Court’s   decision   was   rendered.   The   threats   to   the   concern (Sec. 6, Art. IV of the 1973 Constitution), petitioners seek a writ
government have not been shown to have ceased and on the contrary, of mandamus to compel respondent public officials to publish or cause
instead of erasing fears as to the destabilization that will be caused by the publication in the Official Gazette of various presidential decrees,
the return of the Marcoses, Mrs. Marcos reinforced the basis for the letters of instruction, letters of implementation, GOs, EOs,
decision  to  bar  their  return  when  she  called  President  Aquino  “illegal”,   proclamations and administrative orders. The respondents seek the
claiming  that  Mr.  Marcos  is  the  “legal”  President  of  the  Philippines,  and   outright dismissal of the petition on the ground that petitioners have
declared   that   the   matter   “should   be   brought   to   all   the   courts   of   the   no legal personality or standing to bring said petition, as there is no
world”.  President  Aquino’s decision to bar the return of the Marcoses showing that petitioners are personally and directly affected by the
and  subsequently,  Mr.  Marcos’  remains  at  the  present  time  and  under   alleged non-publication of the presidential issuances concerned.
the present circumstances is in compliance with her bounden duty to
protect and promote the interest of the people. In the absence of a ISSUE: WON publication in the Official Gazette is a sine qua non
clear showing that she had acted with arbitrariness or with grave abuse requirement for the effectivity of laws where the laws themselves
of discretion in arriving at this decision, the Court will not enjoin its provide for their own effectivity dates, and WON petitioners have legal
implementation. The motion was thus denied for lack of merit. standing to bring the petition in the first place.

RULING: YES. The SC held that Article 2 of the Civil Code does not
Additional Cases
preclude the requirement of publication in the Official Gazette, even if
(H) Liberty of Abode and Freedom of Movement [1] the law itself provides for the date of its effectivity. Publication of all
presidential  issuances  “of  a  public  nature”  or  “of  general  applicability”  
is mandated by law. Presidential issuances of general application which
have not been published shall have no force and effect. This is covered
Chapter 9 by Section 1 of Commonwealth Act 638 which object is to give the
Right to Information general public adequate notice of the various laws which are to
regulate their actions and conduct as citizens. Without such notice and
publication, there would be no basis for the application of the maxim
“The   right   of   the   people   to   information   on   matters   of   public  
“ignorantia   legis   non   excusat”. As for the second issue, when the
concern shall be recognized. Access to official records, and to question is one of public right and the object of the petition is to
documents and papers pertaining to official acts, transactions, procure the enforcement of a public duty, the people are regarded as
or decisions, as well as to government research data used as the real party in interest and need not show that he has any legal or
basis for policy development, shall be afforded the citizen, special interest in the result, it being sufficient to show that he is a
14
subject  to  such  limitations  as  may  be  provided  by  law.” citizen and as such interested in the execution of the laws.

If the people are really and effectively to participate in charting Resolution on the Motion for Reconsideration
their own destiny and that of their government, then they must 146 SCRA 446 (1986)
be given sufficient data and information upon which to base
FACTS: Petitioners move for clarification in regard to the following:
any intelligent and meaningful decisions
a. Meaning  of  “law  of  public  nature”  or  “general  applicability”;
b. Whether distinction must be made between laws of general
The incorporation of the right to information in the applicability and laws which are not;
Constitution is a recognition of the fundamental role of free c. Meaning  of  “publication”;
exchange of information in a democracy--there can be no d. Where the publication is to be made; and
realistic  perception  by  the  public  of  the  nation’s  problems,  nor   e. When the publication is to be made.
a meaningful democratic decision-making if they are denied
access to information of general interest RULING:  The  SC  held  that  clause  “unless  it  is  otherwise  provided”  refers  
to the date of effectivity and not to the requirement of publication
itself, which cannot in any event be omitted. Petition is indispensable in
A person could not be presumed to know what was not open to any case, but the legislature may in its discretion provide that the usual
view, much more so if deliberately kept from sight 15-day  period  be  shortened  or  extended.  The  term  “laws”  should  refer  
to all laws and not only to those of general application, for strictly
Publication and Effectivity speaking all laws relate to the people in general albeit there are some
that do not apply to them directly. As such, all statutes, including ones
The  Civil  Code  proclaim:  “Ignorance  of  the  law  excuses  no  one   of local application and private laws, shall be published as condition for
from  compliance  therewith.”  But  it  would  hardly  be  fair  for  the   their effectivity. Interpretative regulations and those merely internal in
nature need not be published, and so are letters of instruction issued
law to presume knowledge if the government does not
by administrative superiors concerning rules and guidelines to be
disseminate the laws and rules that govern the norms of followed by their subordinates in the performance of duties.
conduct that it expects from its citizens Publication should be done in the Official Gazette (Art. 2 of the Civil
Code was not yet amended to its present form at the time of this
decision). Publication must be made forthwith, or at least as soon as
possible, to give effect to the law pursuant to said Art. 2.
14 CONSTITUTION, Art. III, § 7
72 | P LATON
Bengzon v. Drilon Relative to the case of Bengzon, timely publication of laws
208 SCRA 133 (1992) would also be a manner of giving flesh to the principle of
accountability of public officers
FACTS: The petitioners are retired Justices of the Supreme Court and
Court of Appeals who are currently receiving monthly pensions under
Umali v. Estanislao
R.A. 910 as amended by R.A. 1797.
209 SCRA 446 (1992)
R.A. 910 provides for the retirement pension of Justices of the Supreme
FACTS: In 1991, Congress enacted R.A. 7167 providing for upward
Court and Court of Appeals, it was amended in 1957 by R.A. 1797
adjustments in basic personal and additional exemptions allowable to
providing in Section 3-A thereof for automatic adjustments of pensions
individuals for income tax purposes. Its effectivity clause provided:
if salaries of justices were increased or decreased. Identical retirement
“This  Act   shall   take   effect   upon  its  approval”.   The   Act   was  signed  and  
benefits were also given, by President Marcos, to the members of the
approved by the President on Dec. 19, 1991 and was published on Jan.
constitutional commissions under R.A. 3595 and to the members of the
14, 1992 in a newspaper of general circulation. On Dec. 1992
Armed Forces under P.D. 578. Two months later, he issued P.D. 644
respondents promulgated Revenue Regulations No. 1-92, providing that
repealing Section 3-A of the previous R.As and P.Ds. Subsequently, the
said regulations shall take effect on compensation income from Jan. 1,
president decreed the restoration of the automatic readjustment of the
1992. Two petitions were then filed by several taxpayers to compel the
retirement pension of officers and enlisted men. A later decree also
respondents to implement R.A. 7167 with respect to taxable income
issued providing for the automatic readjustment of the pensions of
earned or received in 1991.
members of the Armed Forces. On the other hand, the same was not
restored for the retired Justices of the Supreme Court and Court of
ISSUE: (1) Whether or not R.A. 7167 took effect upon its approval by
Appeals.
the President on Dec. 1991, or on Jan. 30, 1992, after 15days following
its publication on Jan. 14, 1992 in a newspaper of general circulation.
In 1990, Congress, realizing the unfairness of the discrimination,
(2) Whether or not, assuming it took effect on Jan. 30, 1992, the said
approved a bill for the reenactment of the repealed provisions of R.A.
law nonetheless covers or applies to compensation income earned or
1797 and R.A. 3595. However, the president vetoed the bill, stating that
received during calendar year 1991.
“it   would   erode   the   very   foundation   of   the   government’s   collective  
effort to adhere faithfully to and enforce strictly the policy on
DECISION: (1) The court rules that R.A. 7167 took effect on Jan. 30,
standardization of compensation as articulated in the Compensation
1992, which is after 15 days following its publication on Jan. 14. (2) Sec.
and   Position     Classification   Act   of   1989”.   The   following   year,   retired  
1, 3, and 5 of Revenue Regulations No. 1-92 which provide that the
Justices of the Court of Appeals filed a letter/petition asking the court
regulations shall take effect on compensation income earned or
for a readjustment of their monthly pensions in accordance with R.A.
received from Jan. 1 1992 are hereby SET ASIDE. They should take
1797 claiming that P.D. 644 repealing R.A. 1797 did not become law as
effect on compensation income earned or received from Jan. 1, 1991.
there was no valid publication, it only appeared for the first time in the
supplemental issue of the Official Gazette. Since, P.D. 644 has no
REASON:  (1)  The  clause  ‘unless  it  is  otherwise  provided’,  in  Article  2  of  
binding force and effect of law, it therefore did not repeal R.A. 1797.
the Civil Code, refers to the date of effectivity and not to the
The court, in its resolution, acted favorably on the request and
requirement of publication itself, which cannot in any event be
pursuant to the resolution, Congress included in the General
omitted. Publication is indispensable in every case, but the legislature
Appropriations Bill for the Fiscal Year 1992 certain appropriations for
may in its discretion provide that the usual 15 day period shall be
the judiciary intended for the payment of the adjusted pension rates
shortened or extended.
due to them. The President vetoed provisions of the bill related to
(2) The personal exemptions as increased by R.A. 7167 cannot be
aforesaid appropriations, reiterating the earlier reasons for vetoing the
regarded as available only in respect of the compensation income
former house bill.
received during 1992. Revenue Regulations No. 1-92 would in effect
postpone the availability of the increased exemptions to Jan 1 – April
ISSUE: (1) Whether or not the veto by the President (cory) of certain
15, 1993, and thus literally defer the effectivity of R.A. 7167 to Jan. 1,
provisions in the general appropriations act for the fiscal year 1992
1993. Thus, the implementing regulations collide frontally with Sec. 3 of
relating to the payment of the adjusted pensions of retired justices of
the   said   Act   which   states   that   the   statute   “shall   take   effect   upon   its  
the aforementioned courts constitutional.
approval”.
(2) Whether or not P.D. 644 became law.

DECISION: Petition is GRANTED. (1) The court declared the questioned An   effectivity   clause   which   provides   that   the   law   “shall   take  
veto invalid, set aside as illegal and unconstitutional. (2) The court effect  immediately  upon  its  approval”  is  defective which, while
agreed that P.D. 644 never became a law. not invalidating the whole laws, means that, pursuant to Article
2 of the Civil Code, the enactment takes effect fifteen days after
REASON: (1) It is invalid for being in contravention of the constitutional its publication
guidelines in the exercise of the same, i.e., that it vetoed provisions and
not items. And also what were really vetoed are the R.A. 1797 of 1957
People v. Veridiano II
and the Resolution of the Supreme Court in 1991, thus no President
132 SCRA 523 (1984)
may veto the provisions of a law enacted 35 yrs. before his or her term
of office and neither may the President set aside or reverse a final and
FACTS: Benito Go Bio, Jr. was charged with violation of BP Blg. 22 for a
executory judgment of the Supreme Court through the exercise of the
check he issued in the second week of May 1979. Bio filed a motion to
veto power.
quash the information on the ground that it failed to charge an offense,
(2) P.D. 644 which purportedly repealed R.A. 1797 never achieved that
pointing out that on the date of the alleged commission of the offense
purpose because it was not properly published, it never became a law
BP. Blg. 22 had not yet taken effect. Although BP. Blg. 22 was published
and consequently, it did not have the effect of repealing R.A. 1797. As
in April 9, 1979 issue of the Official Gazette, the said issue was only
justified   in   the   case   of   Tañada   v.   Tuvera   “all   laws   shall   immediately  
released on June 14, 1979 and, considering that the questioned check
upon their approval or as soon thereafter as possible, be published in
was issued about the second week of May 1979, then he could not
full in the Official Gazette, to become effective only after 15days from
have violated the said statute because it was not yet released for
their publication, or on another date specified by the legislature, in
circulation at that time.
accordance  with  Article  2  of  the  Civil  Code”.  P.D.  644  was  promulgated  
by President Marcos in 1975, but was not immediately or soon
Petitioner contends that B.P. Blg. 22 was published in April 9, 1979
thereafter published. It took more than 8 years to publish the decree
issue of the Official Gazette and 15 days therefrom would be April 24,
after its promulgation.
1979, or several days before respondent Go Bio, Jr. issued the
73 | P LATON
questioned check around the second week of May 1979 and that violation of Central Bank Foreign Exchange Restrictions. These cases
respondent judge should not have taken into account the date of had been filed by the complainant prosecutors against Mrs. Imelda
release of the Gazette for circulation because Sec. 11 of the Revised Marcos. The prosecutors alleged that respondent judge issued his
Administrative Code provides that for the purpose of ascertaining the Order dated Aug. 13, 1992 solely on the basis of newspaper reports
date   of   effectivity   of   a   law   that   needed   publication,   “the   Gazette   is   concerning the announcement on Aug. 10, 1992 by the President of the
conclusively presumed to be published on the day indicated therein as lifting by the government of all foreign exchange restrictions. The
the  date  of  issue”. judge, taking judicial notice of what allegedly were of public knowledge,
dismissed the cases even before the Central Bank Circular or Monetary
ISSUE: whether or not Go Bio, Jr. violated B.P. Blg. 22 when he issued a Board Resolution had been officially issued.
check around the second week of May 1979.
Respondent judge contended that there was no need to await
DECISION: No. The accused cannot be held liable for bouncing checks publication of the Central Bank Circular repealing the existing law on
prior to the effectivity of B.P. Blg. 22 although the check may have foreign exchange controls for the simple reason that the public
matured after the effectivity of the said law. announcement made by the President in several newspapers of general
circulation lifting foreign exchange controls was total,, absolute,
REASON: The Solicitor General admitted the certification issued by Ms. without qualification, and was immediately effective, and that having
Charito A. Mangubat stating – “This  is  to  certify  that  Vol.  75,  No.  15,  of   acted only on the basis of such announcement, he cannot be blamed
the April 9, 1979 issue of the Official Gazette was officially released for for relying on the erroneous statement of the President that the new
circulation on June 14, 1979”.   It   is   therefore,   certain   that   the   penal   foreign exchange rules rendered moot and academic the cases filed
statute in question was made public only on June 14, 1979 and not on against Mrs. Marcos, which erroneous statement was corrected only on
the printed date April 9, 1979. Therefore, June 14, 1979 was the date of Aug. 17, 1992 but published in the newspapers on Aug. 18, 1992, and
publication of B.P. Blg. 22, before the public may be bound by its only after he had issued his order of dismissal dated Aug. 13, 1992.
contents especially its penal provisions, the law must be published and
the people officially informed of its contents and/or its penalties. For, if ISSUE: whether or not respondent judge committed gross ignorance of
a statue had not been published before its violation, then in the eyes of the law in dismissing the eleven cases solely on the basis of newspaper
the law there was no such law to be violated and, consequently, the reports concerning the announcement by the President of the lifting of
accused could not have committed the alleged crime. When Go Bio, Jr. all foreign exchange restrictions.
committed the act complained of in May 1979 there was then no law
penalizing such act. DECISION: Yes. The court finds respondent judge guilty of gross
ignorance of the law and is dismissed from the service.
De Roy v. Court of Appeals
157 SCRA 757 (1988) REASON: The doctrine of judicial notice rests on the wisdom and
discretion of the courts. Matters of judicial notice have three material
FACTS: On Aug. 17, 1987, the Court of Appeals promulgated a decision requisites: (1) the matter must be one of common and general
affirming a lower court judgment adverse to the petitioners. Said knowledge; (2) it must be well and authoritatively settled and not
decision of the appellate court was received by petitioners on Aug. 25, doubtful or uncertain; and (3) it must be known to be within the limits
1987. On Sept. 9, 1987, the last day of the 15-day period to file an of the jurisdiction   of   the   court.   Things   of   “common   knowledge”   are  
appeal, petitioners filed a motion for extension of the time to file a matters coming to the knowledge of men generally in the course of the
motion for reconsideration, which was eventually denied by the ordinary experience of life, or mattes which are generally accepted by
appellate court. The petitioners herein filed this instant petitione for mankind as true and are capable of ready and unquestioned
certiorari. The appellate court applied the rule laid down in Habaluyas demonstration. Respondent judge, in the guise of exercising discretion
Enterprises, Inc. v. Japzon, that the 15-day period for appealing or for and on the basis of a mere newspaper account which is sometimes
filing a motion for reconsideration cannot be extended. This ruling has even referred to as hearsay evidence twice removed, took judicial
been reiterated in several cases relating to such. Petitioners contend notice of the supposed lifting of foreign exchange controls, a matter
that the rule enunciated in the Habaluyas case should not be made to which was not and cannot be considered of common knowledge or of
apply to the case at bar owing to the non-publication of the Habaluyas general notoriety. Worse, he took cognizance of an administrative
decision in the Official Gazette as of the time the subject decision of the regulation which was not yet in force when the order of dismissal was
Court of Appeals was promulgated. issued. Jurisprudence dictates that judicial notice cannot be taken of a
statute before it becomes effective.
ISSUE: whether or not the Court of Appeals committed grave abuse of
discretion  when  it   denied  petitioners’   motion  for   extension  of   time   to   The assertion of respondent judge that there was no need to await
file a motion for reconsideration, directed entry of judgment and publication of the circular for the reason that the public announcement
denied their motion for reconsideration. made by the President in several newspapers of general circulation is
beyond comprehension. As a judge of the RTC of Manila he is supposed
DECISION: This court finds that the Court of Appeals did not commit a to be well-versed in the elementary legal mandates on the publication
grave abuse of discretion. of laws before they take effect. The very act of respondent judge in
altogether dismissing the eleven criminal cases without even a motion
REASON: The CA correctly applied the rule laid down in Habulayas case. to quash having been filed by the accused, and without at least giving
Also,   contrary   to   petitioners’   view,   there   is   no   law   requiring   the   the prosecution the basic opportunity or an oral argument, is not only a
publication of Supreme Court decisions in the Official Gazette before blatant denial or elementary due process to the Government but is
they can be binding and as a condition to their becoming effective. It is palpably indicative of bad faith and partiality. In order that bias may not
the bounded duty of counsel as lawyer in active law practice to keep be imputed to a judge, he should have the patience and circumspection
abreast of decisions of the Supreme Court particularly where issues to give the opposing party a chance to present his evidence even if he
have been clarified, consistently reiterated, and published in the thinks  that  the  oppositor’s  proof  might not be adequate to overthrow
advance reports of Supreme Court decisions and in such publications as the case for the other party. At the very least, respondent judge acted
the Supreme Court Reports Annotated and law journals. injudiciously and with unjustified haste in the outright dismissal of the
eleven cases, and thereby rendered his actuation highly dubious.
STATE PROSECUTORS v. MURO
236 SCRA 505 (1994) Nature of the Guarantee and Matters of Public Concern

FACTS: Respondent Judge Manuel Muro was charged with ignorance of


the law, grave misconduct and violation of Rules 2.01, 3.01 and 3.02 of
the Code of Judicial Conduct, for having dismissed eleven cases for

74 | P LATON
The guarantee is a self-executory provision under both the and submission of names of nominees under the party-list system of
1973 and 1987 Constitution. It can be invoked by any citizen representation in connection with the May 14, 2007 elections. Pursuant
before the courts thereto, a number of organized groups filed the necessary
manifestations. Some were ostensibly subsequently accredited by the
Comelec to participate in the 2007 elections
The   “information”   and   the   “transactions”   referred   to   in   the  
subject provision of the Constitution have as yet no defined Two consolidated petitions for certiorari and mandamus to nullify and
scope and extent set aside certain issuances of the Commission on Elections (Comelec)
respecting party-list groups which have manifested their intention to
Recognized restrictions: participate in the party-list elections on May 14, 2007.
1. National security matters and intelligence information
2. Trade secrets and banking transactions In the first petition, petitioners Bantay Republic Act (BA-RA 7941, for
short) and the Urban Poor for Legal Reforms (UP-LR, for short) assail
3. Criminal matters
the various Comelec resolutions accrediting private respondents
4. Other confidential information Biyaheng Pinoy et al., to participate in the forthcoming party-list
elections on May 14, 2007 without simultaneously determining
whether or not their respective nominees possess the requisite
qualifications defined in Republic Act (R.A.) No. 7941, or the "Party-List
Confidential Matters System Act" and belong to the marginalized and underrepresented
sector each seeks to represent.
While people have the general right to know most everything, it
does not mean, however, that they have an unfettered access In the second, petitioners Loreta Ann P. Rosales, Kilosbayan Foundation
and Bantay Katarungan Foundation impugn Comelec Resolution 07-
to everything in the possession of the government. In the very
0724 dated April 3, 2007 effectively denying their request for the
nature of things, some matters have to be kept confidential if release or disclosure of the names of the nominees of the fourteen (14)
the government is to be effective. One such instance is in accredited participating party-list groups mentioned in petitioner
diplomatic negotiations Rosales’  previous  letter-request.

The nature of diplomacy requires centralization of authority While both petitions commonly seek to compel the Comelec to disclose
and expedition of decision which are inherent in executive or publish the names of the nominees of the various party-list groups
action. Another essential characteristic of diplomacy is its named in the petitions, the first have the following additional prayers:
1) that the 33 private respondents named therein be "declare[d] as
confidential nature
unqualified to participate in the party-list elections as sectoral
organizations, parties or coalition for failure to comply with the
Indeed, while the Constitution guarantees a right to guidelines prescribed by the [Court] in [Ang Bagong Bayani v. Comelec]"
information  on  “matters  of  public  concern,”  for  which  purpose   and, 2) correspondingly, that the Comelec be enjoined from allowing
access to official records, documents, and papers pertaining to respondent groups from participating in the May 2007 elections.
official acts, transactions, or decisions, as well as to government
research data used as basis for policy development, shall be Issue
afforded the citizens, it states at the same time that the same 1. Whether respondent Comelec, by refusing to reveal the names of the
nominees of the various party-list groups, has violated the right to
shall  be  “subject  to  such  limitations  as  may  be  provided  by  law”
information and free access to documents as guaranteed by the
Constitution; and
What is sought to be had then is the grant of enough 2. Whether respondent Comelec is mandated by the Constitution to
information to enable the citizens to exercise their political disclose to the public the names of said nominees.
rights as the ultimate source of power without sacrificing the Comelec based its refusal to disclose the names of the nominees of
need for the government to keep sacrosanct and confidential subject party-list groups on Section 7 of R.A. 7941.
those that need to be kept secret. Where the demarcation line
might be between what should be publicly accessible and what SEC. 7. Certified List of Registered Parties.- The COMELEC shall, not
later than sixty (60) days before election, prepare a certified list of
is to be kept sub rosa is a question that the courts may have to
national, regional, or sectoral parties, organizations or coalitions which
visit every now and then have applied or who have manifested their desire to participate under
the party-list system and distribute copies thereof to all precincts for
Privacy Interests posting in the polling places on election day. The names of the party-list
nominees shall not be shown on the certified list.
The people also have the right to keep matters to themselves,
and having certain data and information about them which had COMELEC   Chairman   Benjamin   S.   Abalos,   Sr.   …   said   he   and   [the   other  
been gathered and stored by the Government does not five COMELEC] Commissioners --- believe that the party list elections
must not be personality oriented.Abalos   said   under   [R.A.]   7941   …,  
authorize others, pursuant to their right to information, to have
the people are to vote for sectoral parties, organizations, or coalitions,
access to the same either for their personal consumption or for not for their nominees.He said there is nothing in R.A. 7941 that
publication requires the Comelec to disclose the names of nominees. xxx (Words in
brackets and emphasis added)Insofar as the disclosure issue is
Additional Cases concerned, the petitions are impressed with merit.
(I) Right to Information [1]
Assayed against the non-disclosure stance of the Comelec and the given
rationale therefor is the right to information enshrined in the self-
executory Section 7, Article III of the Constitution, viz:
I2. Bantay vs. COMELEC
Sec.7. The right of the people to information on matters of public
G.R. No. 177271 May 4, 2007
concern shall be recognized. Access to official records, and to
documents, and papers pertaining to official acts, transactions, or
The Comelec issued Resolution No. 7804 prescribing rules and
decisions, as well to government research data used as basis for policy
regulations to govern the filing of manifestation of intent to participate
75 | P LATON
development, shall be afforded the citizen, subject to such limitations advanced in their petition. The exercise would require the Court to
as may be provided by law. make a factual determination, a matter which is outside the office of
judicial review by way of special civil action for certiorari.
Complementing and going hand in hand with the right to information is In certiorari proceedings, the Court is not called upon to decide factual
another constitutional provision enunciating the policy of full disclosure issues and the case must be decided on the undisputed facts on record.
and transparency in Government. We refer to Section 28, Article II of The sole function of a writ of certiorari is to address issues of want of
the Constitution reading: jurisdiction or grave abuse of discretion and does not include a review
of  the  tribunal’s  evaluation  of  the  evidence.  (note  that  nowhere  in  R.A.  
Sec. 28. Subject to reasonable conditions prescribed by law, the State No. 7941 is there a requirement that the qualification of a party-list
adopts and implements a policy of full public disclosure of all its nominee be determined simultaneously with the accreditation of an
transactions involving public interest. organization. )

..The right to information is a public right and for every right of the I3. Senate of the Philippines v. Ermita
people recognized as fundamental lies a corresponding duty on the part 488 SCRA 1 (2006)
of those who govern to respect and protect that right. However, the
right to information and its companion right of access to official records The people as well as their representatives are entitled to know what
are not absolute and is limited to "matters of public concern" and is are happening around them especially those of public concern involving
further subject to such limitation as may be provided by law. the use of public funds, as well as other deals entered into by those
who are in temporary ascendancy in the government. Also, there are
The terms "public concerns" and "public interest" have eluded precise things that should be kept confidential for the proper and efficient
definition. But both terms embrace, to borrow from Legaspi, a broad functioning of the Government.
spectrum of subjects which the public may want to know, either
because these directly affect their lives, or simply because such matters “  a  transparent  government  is  one  of  the  hallmarks  of  a  truly  republican  
naturally whet the interest of an ordinary citizen. At the end of the day, state however, even in the early history of republican thought, it has
it is for the courts to determine, on a case to case basis, whether or not been recognized that the head may keep certain information
at issue is of interest or importance to the public. confidential in pursuit of the public interest.

Here.. no national security or like concerns is involved in the disclosure EO 464 (does it ring a bell? Buset.. minumulto ako ng poli bat gnun?)
of the names of the nominees of the party-list groups in question.
Doubtless, the Comelec committed grave abuse of discretion in refusing .. for the purpose of ensuring the observance of the separation of
the legitimate demands of the petitioners for a list of the nominees of powers
the party-list groups subject of their respective petitions. Mandamus, Executive privilege and power of legislative inquiries in aid of
therefore, lies. legislation.

The last sentence of Section 7 of R.A. 7941 is certainly not a justifying EO 64 practically sealed the lips of everyone in the executive
card for the Comelec to deny the requested disclosure. department. Not only department heads but also minor functionaries
are prevented as the executive privilege was also bestowed on them-
The prohibition imposed on the Comelec under said Section 7 is limited meaning no appearance without clearance.
in scope and duration, meaning, that it extends only to the certified list
which the same provision requires to be posted in the polling places on Inquiry in aid of legislation..
election day. To stretch the coverage of the prohibition to the absolute
is to read into the law something that is not intended. If the info is necessary for wise legislation, Congress has the right to
compel disclosure thereof.
The Comelec obviously misread the limited non-disclosure aspect of the
provision as an absolute bar to public disclosure before the May 2007 Executive privilege- (Schwartz) the power of the government to
elections. The interpretation thus given by the Comelec virtually tacks withhold information from the public the courts and the congress.
an unconstitutional dimension on the last sentence of Section 7 of R.A.
No. 7941. (Rozell) the right of the president and high level officials to withhold
information from Congress, the courts, and ultimately the public
The  Comelec’s  reasoning  that  a  party-list election is not an election of
personalities is valid to a point. It cannot be taken, however, to justify (tribe) 3 distinct kinds
its assailed non-disclosure stance which comes, as it were, with a 1. State secrets privilege- (on the ground that
weighty presumption of invalidity, impinging, as it does, on a the information is of such nature that
fundamental right to information, While the vote cast in a party-list disclosure would subvert crucial military or
elections is a vote for a party, such vote, in the end, would be a vote for diplomatic objectives)
its nominees, who, in appropriate cases, would eventually sit in the 2. Informers privilege- (not to disclose the
House of Representatives. identity of persons who furnish info of
violations of law to officers charged with
DENIED insofar as it seeks to nullify the accreditation of the the enforcement of that law)
respondents named therein. However, insofar as it seeks to compel the 3. Generic privilege (for internal deliberations
Comelec to disclose or publish the names of the nominees of party-list attaching to intragovernmental documents
groups, sectors or organizations accredited to participate in the May reflecting advisory opinions,
14, 2007 elections, the same petition and the petition in G.R. No. recommendations and deliberations
177314 are GRANTED. comprising part of a process by which
governmental decisions and policies are
Comelec is hereby ORDERED to immediately disclose and release the formulated.
names of the nominees of the party-list groups, sectors or organizations
accredited to participate in the May 14, 2007 party-list elections. The A president and those who assist him must be free to explore
Comelec is further DIRECTED to submit to the Court its compliance alternatives in the process of shaping policies and making decisions and
herewith within five (5) days from notice hereof. to do so in a way many would be unwilling to express except privately.
The privilege is fundamental to the operation of government and
The Court is unable to grant the desired plea of petitioners BA-RA 7941 inextricably rooted in the separation of powers under the Constitution.
and UP-LR for cancellation of accreditation on the grounds thus

76 | P LATON
Recognized only in certain types of information of a sensitive character. legislation is not only intended to benefit Congress but also the
citizenry. The people are equally concerned with this proceeding and
A claim therefore maybe valid or invalid. Noticeably absent is the have the right to participate therein in order to protect their interests.
recognition that executive officials are exempt from the duty to disclose The extent of their participation will largely depend on the information
information by the mere fact of being executive officials. The gathered and made known to them. In other words, the right to
extraordinary character of the exemptions indicates that the information really goes hand-in-hand with the constitutional policies of
presumption inclines heavily against executive secrecy and in favor of full public disclosure and honesty in the public service. It is meant to
disclosure. enhance the widening role of the citizenry in governmental decision-
making as well as in checking abuse in the government." They also
Sec 2(a) of EO 464, the reference to persons being covered by the mentioned that some of our standing jurisprudence, such as the case of
executive privilege may be read as an abbreviated way of saying that Tañada v. Tuvera, "have recognized a citizen's interest and personality
the person is in possession of info which is in the judgment of the head to enforce a public duty and to bring an action to compel public officials
office concerned, privileged. Executive privilege refers to information and employees to perform that duty.)
and not to persons. The claim therefore under section 3 of EO 464 in
relation to section 2(b)is thus invalid per se because, it is not asserted I5. Air Philippines Corporation v. Pennswell, Inc.
but merely implied instead of providing precise and certain reasons for 540 SCRA 215 (2007)
the claim, it merely invokes eo464 coupled with the announcement
that the President has not given consent. It severely frustrates the FACTS: Air Philippines Corporation (APC) is a domestic corporation
power of inquiry of Congress. In Fine, Section 3 and 2 (b) must but engaged in air transportation services. APC purchased chemical
invalidated. lubricants from Pennswell, a company that manufactures and sells
The president may not authorize her subordinates to exercise such industrial chemicals. In the course of events, APC alleged that
power. Pennswell defrauded it: APC insisted that Pennswell committed to
deliver lubricants of the type belonging to a new line instead of what it
Right of people to information v the right of Congress to information. had previously delivered to them, which were apparently lubricants of
second-grade quality not fit for APC's specific use. The second batch of
The demand of a citizen has no same obligatory force as that of a lubricants demanded for, though, turned out to be of the same type as
subpoena duces tecum. that of the first delivery. APC demanded return of its money, but
Pennswell ignored the demand and billed APC instead for its
Also,  it  doesn’t  give  an  individual  to  exact  a  testimony  from  government   outstanding debts. APC now seeks to convince the Supreme Court that
officials. it has a right to obtain the chemical composition and ingredients of
respondent’s   products   to   conduct   a   comparative   analysis   of its
It does not follow in every sense that when the congress exercises the products.
power of inquiry, that the people are exercising their right to info.
ISSUE: Can chemical components of Pennswell products be accessed
To the extent that investigations in aid of legislation are generally pursuant to right to information under Sec. 7, Art III?
conducted in public, however, any executive issuance tending to unduly
limit disclosures of information in such investigations necessarily HELD: No. Jurisprudence has consistently acknowledged the private
deprives the people of information which being presumed to be in aid character of trade secrets, as there is a privilege not to disclose them.
of legislation, is presumed to be a matter of public concern. The citizens The Supreme Court has declared that trade secrets and banking
are thereby denied access to information which they can use transactions are among the recognized restrictions to the right of the
formulating their own opinions on the matter before congress- people to information as embodied in the Constitution.
opinions which they can then communicate to their representatives
and other government officials through the various legal means
I6. Antolin vs. Domondon15
allowed by their freedom of expression.
G.R. No. 165036; 5 July 2010
Moreover, even as EO 464 applies only to officials of the executive
Facts: Hazel Ma. C. Antolin (Petitioner) failed the Certified Public
branch, it does not follow that the same is exempt from publication.
Accountant (CPA) Licensure Exam she took in October 1997. Convinced
Since this concerns public interest, the challenged order must be
she deserved to pass the Exam, she wrote to the Board of Accountancy
covered by the publication requirement.
(Board), requesting that her answer sheets be re-corrected. She was
shown her answer sheets but since these showed only shaded marks,
I4. Sabio v. Gordon she was unable to determine why she failed the Exam. Consequently,
504 SCRA 704 (2006) she asked the Board for copies of the questionnaire, her answer sheets,
the answer keys and an explanation of the grading system (collectively,
FACTS: Then-President Corazon Aquino issued E.O. No. 1, creating the the Examination Papers). Her request was denied on two grounds: (1)
PCGG, a body tasked with recovering the ill-gotten wealth obtained by Section 36, Article III of the Rules and Regulations Governing the
deposed President Ferdinand Marcos and his close associates. Section 4 Regulation and Practice of Professionals, as amended by Professional
(b)  of  said  E.O.  provides  that  “[n]o  member  or  staff  of  the  Commission   Regulation Commission (PRC) Resolution No. 332, series of 1994, only
shall be required to testify or produce evidence in any judicial, allowed access to her answer sheets, and reconsideration of the result
legislative or administrative proceeding concerning matters within its of her examination can be made only on grounds of mechanical error in
official   cognizance.”   This   was   to   ensure   the   PCGG's   unhampered   the grading of the answer sheets, or malfeasance; and (2) the Board
performance of its duties. 20 years later, the Senate invited PCGG was precluded from releasing the Examination Papers (other than the
Chairman Camilo Sabio to be one of the resource persons in a Senate answer sheets) by Section 20, Article IV of PRC Resolution No. 338,
investigation, who declined the invitation invoking Section 4 (b) of E.O. series of 1994. The Board later informed her that her exam was
No. 1. investigated and no mechanical error was
found in the grading.
ISSUE: May Section 4 (b) of E.O. No. 1 be invoked by Chairman Sabio to
justify non-appearance on legislative investigations? Issues: Whether or not petitioner has the constitutional right to have
access to the Examination papers.
HELD: No. Section 4 (b) of E.O. No. 1 has been repealed by the 1987
Constitution, particularly by Sec. 21, Art. VI (power of legislative Held: Like all the constitutional guarantees, the right to information is
inquiry). not  absolute;  it  is  limited  to  “matters  of  public  concern”  and  is  further  

(The Supreme Court said, "[T]he conduct of inquirires in aid of 15 Source: Lexforiphilippines.com
77 | P LATON
“subject   to   such   limitations   as   may   be   provided   by   law”   (Section 7, It operates through chartered affiliates which are independent
Article III, 1987 Constitution). Similarly, the   State’s   policy   of   full   unincorporated associations with membership therein equivalent to
disclosure   is   limited   to   “transactions   involving   public   interest,”   and   is   petitioner.
“subject   to   reasonable   conditions   prescribed   by   law”   (Sec. 28, Art. II,
1987 Constitution). National board examinations such as the CPA Board Had local affiliates in Alabama and operated there without complying
Exams are matters of public concern. The populace in general, and the with an Alabama statute that requires a foreign corporation to qualify
examinees in particular, would understandably be interested in the fair before doing business there by filing its corporate charter with the
and competent administration of these exams in order to ensure that secretary of the state and designating a place of business and an agent
only those The Court, nonetheless, realizes that there may be valid to receive service of process.
reasons to limit access to the Examination Papers in order to properly
administer the exam. That there exist inherent difficulties in the The association has never complied with the statute from which it
preparation, generation, encoding, administration, and checking of considered itself exempt.
these multiple choice exams that require that the questions and
answers remain confidential for a limited duration. The PRC had not Alleging   the   petitioner’s   activities- such as opening a regional office,
been given an opportunity to explain the reasons behind their organizing organized various activities in Alabama, recruiting members
regulations for keeping the Examination Papers confidential. The Court and soliciting contributions within the state, giving financial support
deemed it best to remand the case to the RTC for further proceedings. and furnishing of legal assistance to Negro boycott of the bus lines in
Montgomery to compel the seating of passengers without regard to
Chapter 10 race-were causing irreparable injury to the citizens of the state for
Right of Association which criminal prosecution and civil actions at law afforded no
adequate relief.
“The  right  of  the  people,  including  those  employed  in  the  public   The Attorney general of Alabama brought an equity suit to enjoin the
and private sectors, to form unions, associations, or societies petitioner from conducting further activities in and taking steps to
16
for  purposes  not  contrary  to  law  shall  not  be  abridged.” further activities in the state.

Man being gregarious by nature would find it natural to Court issued a restraining order.
associate with others, whether for intimate companionship or
for social political and other reasons Petitioner demurred to the allegations of the bill and moved to dissolve
the restraining order. It contended that what the state sought to
accomplish by its suit would violate rights to freedom of speech and
An individual may join or organize whatever associations and assembly guaranteed by the 14th amendment to the constitution of the
societies he and kindred minds and hearts like him might want US.
to form
The state moved for the production of a large number of records and
Just like any other rights, however, the right to associate is still papers, including bank statements, lease, deeds, and records containing
subject to limitation that it must not be for purposes contrary the names of all Alabama members and agents of the association.
to law
Petitioner produced all except its membership list and was adjudged in
contempt for that and was fined $100000.
Unlike the cases of other guarantees, which are mostly
American in origin, this particular freedom has an indigenous Whether Alabama, consistently with the due process clause of the 14th
cast. It can trace its origin to the Malolos Constitution amendment, can compel petitioner to reveal the names and addresses
of the members and agents without regard to their positions and
General Considerations functions in the association.

The right to association may involve such intimate and personal Petitioner’s  claim  is  that  the  order,  in  the  circumstances  shown  by  this
record violated rights assured to petitioner and its members under the
relations as friendship or marriage to the more impersonal
constitution.
groupings as those of unions and societies where objectives
might range from purely social to economic and political. The Can the association validly assert the right of its members? Yes
first one may be considered as the freedom of intimate because it and its members are in the practical sense identical.
association and the latter the freedom of expressive association
The rights are personal to the members, who are not immediately
The right to form associations also include the liberty not to before the court, may be asserted by the association on their behalf for
join at all. A person can not be compelled to join a group that the right could not be effectively vindicated except through an
appropriate party before the court.
he does not want to associate with. The constitutionally
guaranteed freedom of association includes the freedom not to Petitioner argues that the effect of compelled disclosure of
associate. The right to choose with whom one will associate membership list will be to abridge the rights of their rank-and-file
oneself is the very foundation and essence of that partnership. members to engage in lawful association in support of their common
It should be noted that the provision guarantees the right to beliefs.
form an association. It does not include the right to compel
others to form or join one The fact that Alabama has taken no direct action to restrict the right of
the  petitioner’s  members to associate freely, does not end inquiry into
the effect of the production order.
National Association for the Advancement of Colored People v.
Alabama ex rel. Patterson
Inviolability of privacy in group association may in many circumstances
357 US 449, 2 L Ed 2d 1488, 78 S Ct 1163 (1958)
be indispensable to preservation of freedom of association, particularly
when the group espouses dissident beliefs.
Petitioner is a nonprofit membership corporation organized under the
laws of New York for the purpose of advancing the welfare of Negroes.
It is not sufficient to answer that whatever repressive effect
compulsory   disclosure   of   names   of   petitioner’s   members,   may   have  
16 CONSTITUTION, Art. III, § 8
78 | P LATON
upon  participation  by  Alabama  citizens  in  petitioner’s  activities  follows   2. That compelling organizations to register and list their
not from private community pressures. The crucial factor here is the members on a showing merely that they are foreign-
interplay of governmental and private action, for it is only after the denominated and operate primarily to advance the
initial exertion of state power represented by the production order that objectives of the world Communist movement constitutes a
private action takes hold. restraint on freedom of expression and association in
violation of the 1st amendment
Whether Alabama has demonstrated an interest in obtaining the 3. That requiring Party officers to file registration statements
disclosures it seeks from petitioner which is sufficient to justify the for the Party subjects them to self-incrimination forbidden
deterrent effect which we have concluded these disclosures may well by the 5th amendment
have   on   the   free   exercise   by   petitioner’s   members   of   their   4. That the act violates due process by legislative
constitutionally protected right of association. predetermination of facts essential to the communist party
within the definition of a communist action organization,
The exclusive purpose was to determine if the association is conducting and that the evidentiary elements prescribed for
intrastate business in violation of the Alabama foreign corporation consideration by the board bear no rational relation to that
registration statute. definition
5. That, in several aspects the act is unconstitutionally vague
The court is unable to perceive the substantial bearing of the 6. That the subversive activities control board is so necessarily
membership list. As matters stand in the state court that the petitioner biased against the communist party as to deprive it of fair
has (1) admitted its presence and conduct of activities in Alabama since hearing
1918 (2) has offered to comply in all respects with the qualification
statute, although preserving its contention that the statute does not in threats to public safety, the congress meets the threat by the
apply to it, and (3) has apparently complied satisfactorily with the requirement of registration or disclosure.
production order except from its membership list.
The congress may entail restraints on speech and association to require
Whatever interest the state may have in obtaining the names of the publicity demanded by rational interests high in scale of national
ordinary members has not been shown to be sufficient to overcome concern.
petitioner’s  constitutional  objections  to  the  production  order.
The requirement that officers or members at anytime during the year
The court held that the immunity from state scrutiny of membership preceding the registration must be listed is a reasonable means of
lists which the association claims on behalf of its members is here so assuring that the obligation to list present members and officers may
related to the right of its members to pursue their private interests not be evaded. Also to include their aliases must be sustained.
privately and to associate freely with others in doing so as to come
within the protection of the 14th amendment. The judgment of civil Disclosure of the financial transactions and of the identity of the organs
contempt and the $100000 fine must fall. of publication which it controls is necessary to bring foreign-
denominated organizations out into the open, where the public can
Communist Party v. Subversive Activities Control Board evaluate their activities informedly against the revealed background of
367 US 1, 6 L Ed 2d 625, 81 S Ct 1357 (1961) the character, nature, and connections. The obligation to identify
presses without more and as applied to foreign-denominated
The subversive activities control act, as amended, and based on organizations, does not fetter constitutionally protected free
legislative findings from evidence adduced before various congressional expression.
committees that among others, there exists a world communist
movement, which is a worldwide revolutionary movement whose The subversive activities control act applies only to foreign
purpose it is, by treachery, deceit, infiltration into other groups, denominated organizations which work primarily to advance the
espionage, sabotage, terrorism, and any other means deemed objectives of a world movement controlled by the government of a
necessary, to establish a communist totalitarian dictatorship in the foreign country. There is no attempt here to impose stifling obligations
countries throughout the world through the medium of a worldwide upon the proponents of a particular political creed as such, or even to
communist organization, required the registration with the Attorney check the importation of particular political ideas abroad for
General of Communist action organization and Communist front propagation here. Organizations are subject to it only when shown
organizations including information about their officers. the front after administrative hearing subject to judicial review, to be dominated
organizations need not list their non-officer members, however. The by the foreign power or its organs and to operate primarily to advance
Act also provides that whatever the Attorney General has reason to its purposes. The registration requirement therefore on its face and as
believe that an organization which has not registered is an organization here applied, does not violate the 1st amendment.
of a kind required to register, or that any individual who has not
registered is required to register, he shall petition the Subversive People v Ferrer
Activities Control Board for an order that the organization or individual 48 SCRA 382 (1972)
register.
Petitioners Feliciano Co and Nilo Tayag, together with 5 others, were
After the registration, the act then subjects the organization or its separately charged with violation of the anti subversion act. They
members to certain prohibitions, restrictions, disabilities and moved to quash, assailing the constitutionality of the act on the
disqualifications in regard to their communications and grounds that, among others, it is a bill of attainder, it is vague and it
correspondence, as well as, employment, naturalization and their denies them the equal protection of the laws. Resolving the
foreign travel, i.e. their application for US passport. constitutional issues raised, the trial court declared the statute void on
the grounds that it is a bill of attainder and that it is vague and over-
The attorney general petitioned the subversive activities control board broad, and dismissed the information against the two accused.
for an order to require that the communist party register as a
communist action organization. After several hearings, the board found ..due process..
the party to a communist action organization within the meaning of the
act and ordered it to register as such. Before enacting the statute in question, congress conducted careful
investigations and stated the requirements in their preamble..
The constitutional contentions raised by the party with respect to the
registration requirement are: The constitutionality of the act would be open to question if, instead of
1. That the requirement, in the context of the act , in effect making these findings in enacting the statute, congress omitted to do
outlaws the Party and is in the nature of a bill of attainder so.

79 | P LATON
Co and Tayag both filed motions for reconsideration
In saying that by means of the act, congress has assumed judicial
magistracy, the trial court failed to take proper account of the Co filed a reiteration of his earlier arguments
distinction between legislative fact and adjudicative fact.
Tayag seeks the inclusion in the guidelines set forth in the decision of a
Legislative fact- those facts which are relevant to the legislative requirement that in prosecutions under the act, the state must prove
judgment. that the defendant joined or remained a member of the CPP or of the
subversive organization, knowing its subversive character and with
Adjudicative fact- those which tie the legislative enactments to the specific intent to further its basic objectives as shown by direct
litigants. participation in the  organization’s  unlawful  activities

The test formulated in Nebbia v New York is that laws are seen to have This submission would nullify the legislative policy embodied in the Act
a reasonable relation to a proper legislative purpose, and are neither and frustrate prosecutions under it.
arbitrary nor discriminatory; the requirements of due process are met.
The government has a right to protect itself against subversion. To require proof of direct participation would render the conspiracy
device ineffective in Penal Law.
Even though the purpose is legitimate, it cannot be achieved by means
that broadly stifle fundamental personal liberties be more narrowly Arresto Mayor – joins
achieved. The requirement of knowing membership as distinguished
from nominal membership has been held as sufficient basis for Prision Mayor to death – if as well takes arm against the government.
penalizing membership in a subversive organization. (one who knows
that the organization is of an unlawful purpose and accepts or retains It would also run counter to another established principle regarding
membership conspiracy that the act of one is the act of all.

The act is not overbroad. The argument was that the use of the word The requirement that membership is proved by overt acts is enough.
overthrow is broad for it could be achieved by peaceful means. Shown by taking oath of membership or signing affiliation m\papers in
a subversive organization, knowing its illegal purposes. Also may be
It is not. The first where as clause says overthrow not only by force and inferred through concert of action. May also be satisfied by non
violence but also by deceit, subversion and other illegal means. Also criminal and relatively minor acts such as signing membership papers,
Section 4 mentions overt acts. Knowingly, willfully and by overt acts. attending meetings and the like.
Also the word overthrow connotes violent and illegal means.
The function of the overt act in conspiracy prosecution is simply to
..the act and the guarantee of free expression. manifest that the conspiracy is at work and neither a project still resting
solely in the minds of the conspirators nor a fully completed operation
The act is aimed against conspiracies to overthrow the government. no longer in existence. The existence of the conspiracy is itself danger
Whatever interest in the freedom of speech and freedom of association to   national   security.   “………………..if   the   ingredients   are   present,   we  
is infringed by the prohibition against knowing membership in the cannot  bind  the  Government  to  wait  until  the  catalyst  is  added”
communist party of the Philippines, is so indirect and so insubstantial as
to be clearly and heavily outweighed by the overriding considerations In the first place, there is no reason why one who actively and
of national security and the preservation of democratic institutions in knowingly works in the ranks of the organization, intending to
this country. contribute to the success of its specific illegal activities should be any
more immune from prosecution than one to whom the organization
.. Conclusion and guidelines has assigned or entrusted the task of carrying out the substantial
criminal acts.
Upheld the validity of the act
In the second place, the requirement of proof of specific intent
Guidelines to be observed in the prosecution of the act. precisely limits the operation of the statute only to illegal conduct.

Elements of the crime The statute provides that a defendant must be proven to have
knowledge of the proscribed advocacy before he may be convicted.
In case of subversive organizations other than the communist party of Thus the member whom the organization is a vehicle for the
the Philippines advancement of legitimate aims and policies does not fall within the
a. Purpose of the organization is to overthrow the ban of the statute; he lacks the requisite of specific  intent  ‘to  overthrow  
present government of the Philippines and to establish of  the  government  as  speedily  as  circumstances  would  permit.’  Such  a  
in this country a totalitarian regime under the person may be foolish, deluded, or perhaps merely optimistic, but he is
denomination of foreign power not by this statute made a criminal.
b. That the accused joined such organization
c. That he did so knowingly, willfully, and by overt acts The 2 motions for reconsiderations are denied. Our decision of Dec 27,
1972 is hereby declared final and executory.
In case of the communist party of the Philippines
a. That the CPP continues to pursue the objectives which National Association for the Advancement of Colored People v Button
led Congress in 1957 to declare it to be an organized 371US415, (L Ed 2d 405, 83 S Ct 328 (1963)
conspiracy for the overthrow of the Government by
illegal means for the purpose of placing the country ….Chapter  33  of  the  Virginia  Acts  of  Assembly  forbidding  solicitation  of  
under the control of a foreign power. legal business by a runner or capper includes in the definition of runner
b. That the accused joined CPP or capper an agent for an individual or organization which retains a
lawyer in connection with an action to which it is not a party and in
That he did so knowingly, willfully, and by overt acts which  it  has  no  pecuniary  right  or  liability…..  

Resolution on Motions for Reconsiderations The case originated in companion suits by the NAACP which seeks to
56 SCRA 793 (1974) restrain Chapters 31,32,33,35,and 36 of the Virginia Acts of Assembly

Three-judge court struck down 31,32 and 35 but abstained in 33 and 36

80 | P LATON
Circuit court held the chapters both constitutional.
Grisworld v Connecticut
Supreme Court of Appeals reversed as to 36 and sustains 33 381 US 497, 14 L Ed 2d 510, 85 S Ct 1678 (1965)
..
A statute makes it a crime for any person to use any drug or article to
The NAACP is not a conventional political party, but the litigation it prevent conception. Appellants, the Executive Director and the Medical
assists, while serving to vindicate the legal rights of members of the Director of the Planned Parenthood League of Connecticut, were found
American Negro community, at the same time and perhaps more guilty as accessories for giving married people information and medical
importantly, makes possible the distinctive contribution of a minority advice on how to prevent conception and prescribing a contraceptive
group to the ideas and beliefs of our society. For such a group, device or material for the wife’s  use.  They  were  fined  $100  each.  They  
association for litigation may be most effective form of political claim that the accessory statute, as so applied violated the 14th
association. amendment.

Chapter 33, as authoritatively construed by the Supreme Court of The appellants have standing to raise the constitutional rights of the
Appeals, a person who advises another that his legal rights have been married people with whom they had a professional relationship.
infringed and refers to him to a particular atty or group of attys for Certainly the accessory should have standing to assert the offense
assistance has committed a crime, as the atty who knowingly renders which he is charged is assisting is not, or cannot constitutionally be a
assistance under such circumstances. There thus inheres in the statute crime.
the gravest danger of smothering all discussion looking to the eventual
institution of litigation on behalf of the rights of members of unpopular The right to association is more than the right to attend a meeting; it
minority. includes   the   right   to   express   one’s   attitudes or philosophies by
membership in a group or by affiliation with it or by other lawful
It is enough that a vague and broad statute lends itself to selective means. The present case, then, concerns a relationship lying within the
enforcement against unpopular cases. We cannot close our eyes to the zone of privacy created by several fundamental constitutional
fact that the militant Negro civil rights movement has engendered the guarantees. And it concerns a law which, in forbidding the use of
intense resentment and opposition of the politically dominant contraceptives rather than regulating their manufacture or sale, seeks
community of Virginia; litigation assisted by the NAACP has been to achieve its goals by means having a maximum destructive impact
bitterly fought. In such circumstances, a statute broadly curtailing upon  that  relationship.  “A  governmental  purpose  to  control  or  prevent  
group activity leading to litigation may easily become a weapon of activities constitutionally subject to state regulation may not be
oppression, however evenhanded its terms appear. Its mere existence achieved by means which sweep unnecessarily broadly and thereby
could freeze out all such activity on behalf of the civil rights of Negro invade  the  area  of  protected  freedoms.”  NAACP  v  Alabama.  Would  we  
citizens. allow the police to search the sacred precincts of marital bedrooms for
telltale signs of use of contraceptives? The very idea is repulsive to the
The association and its members are advocating lawful means of notions of privacy surrounding the marital relationship.
vindicating legal rights.
Marriage is an association that promotes a way of life, not causes; a
We hold that Chapter 33 violates the 14th amendment by unduly harmony in living, not political faiths; a bilateral loyalty, not commercial
inhibiting protected freedom of expression and association. As or social projects. Yet it is an association for as noble a purpose as any
construed by the court, Chap 33, at least potentially, prohibits every involved in our prior decisions.
cooperative activity that would make advocacy of litigation meaningful. ..
Precision of regulation must be the touchstone in an area so closely
touching our precious freedoms. Mr. Justice Black with whom Mr. Justice Stewarts joins dissenting..

A state may not, under the guise of prohibiting professional The right of privacy as a comprehensive substitute against
misconduct, ignore constitutional rights. unreasonable   search   and   seizures…   Privacy   is   broad,   abstract   and  
ambiguous concept which can easily be shrunken in meaning but which
Malicious intent was of the essence of the common law offenses of can also, on the other hand, easily be interpreted as a constitutional
fomenting or stirring up litigation. The exercise in our own, as in this ban against many things other than searches and seizures.
case, of 1st amendment rights to enforce constitutional rights through
litigation, as a matter of law, cannot be deemed malicious. There has The government has a right to invade the right to privacy unless
been no showing of a serious danger of professionally reprehensible prohibited by some specific constitutional provision.
conflicts of interests which rules against solicitation frequently seek to
prevent. This is so partly because no monetary stakes are involved, and There is no provision in the constitution that gives the court blanket
so there is no danger that the atty will desert or subvert the paramount power to exercise supervisory veto over the wisdom and value of
interests of his client to enrich himself or an outside sponsor. And the legislative policies and to hold unconstitutional those laws which they
aims and interests of NAACP have not been shown to conflict with believe is unwise or dangerous. The use of federal courts of such a
those of its members and nonmember Negro litigants. formula or doctrine or whatnot to veto federal laws simply takes away
the power to make laws from congress and transfers the power to this
The state failed to advance any substantial regulatory interest, in the court for ultimate determination.
form  of  substantive  evils  flowing  from  petitioner’s  activities,  which  can  
justify the broad prohibitions which it had imposed. It is not too much to say that there is no legislative body ever does pass
laws without believing that they will accomplish a sane, rational, wise
Intimate Associations and justifiable purpose.

Marriage and family and other personal relationships could Associations, Conformity and Dissension
very well illustrate the freedom of intimate association
It is generally the rule that the collective might of like-minded
The Court has long recognized hat, because the Bill of Rights is persons could achieve much more than any force or pressure
designed to secure individual liberty, it must afford the contributed by each member acting on his own. Like any other
formation and preservation of certain kinds of highly personal right, this is also subject to limitations, one of which applies to
relationships a substantial measure of sanctuary from those in the government service
unjustified interference by the state
81 | P LATON
The right of government employees to organized is limited to In contrast to other fundamental powers, the power of eminent
the formation of unions or associations only, without including domain may be exercised not only by the government itself but
the right to strike even by private corporations, such as public utilities involved in
supplying electricity, water, telecommunications, rail and air
While members may exercise their right to speak and disagree transportation. The authority to exercise the power is indicated
within the organization, they may also exercise their right to in their franchises
dissociate if they could no longer reconcile their own personal
feelings, principles or objectives with those of the group Public Use

Be that as it may, however, there may also be instances where, The  “public  use”  requirement  for  a  valid  exercise  of  the  power  
even if one desires to exercise his right not to associate with of eminent domain is a flexible and evolving concept influence
others, he may still be compelled to contribute his share for the by changing conditions
upkeep of an organization whose existence is required by
public interest When land has been acquired for public use in fee simple
unconditionally, either by the exercise of eminent domain or by
purchase, the former owner retains no rights in the land, and
the public use may be abandoned, or the land may be devoted
to a different use, without any impairment of the estate or title
acquired, or any reversion to the former owner
Expressive Association
Heirs of Juancho Ardona v. Reyes
This right to group together people with similar ideas or views 125 SCRA 220 (1983)
on life and its varied aspects may mean also the right to choose
whom to accept into, or reject from, the group. To that extent, Philippine Tourism Authority (PTA) expropriated certain properties in
Cebu to be converted to a Golf Course in a Sports Complex (basketball
therefore, the right to associate may be considered as an
courts, tennis courts, volleyball courts, track and field, baseball and soft
aspect of the right to express oneself ball  diamonds,  and  swimming  pools),  clubhouse,  children’s  playground  
and a nature area for picnics, horseback riding for the use of the public.
The right to associate for expressive purposes is not, however,
absolute. Infringements on that right may be justified by The heirs said that it is not for public use, for that taking was not
regulations adopted to serve compelling state interests, impressed with public use.
unrelated to the suppression of ideas, that cannot be achieved
through means significantly less restrictive of associational No specific constitutional provision allowing the taking for tourism
purposes.
freedoms
What the petitioners want the court to adopt was a strict construction
of PUBLIC USE.

Eminent domain is inherent in sovereignty and exists in a sovereign


state without any recognition in the constitution
Additional Cases
(J) Right of Association [1] The restrictive view may be appropriate for a Nation which
circumscribes the scope of the government activities and public
concern and which concerns big and correctly located public lands that
obviate the need to take private property for public purposes. Neither
applies to the Philippines. We have never been a laissez faire country.
Chapter 11 And the necessities which impel the exertion of sovereign power are all
Eminent Domain too often found in areas of scarce public land or limited government
resources.
“Private  property  shall  not  be  taken  for  public  use  without  just  
17 The constitutional restraints are public use and just compensation
compensation.”
HELD: public use can now be understood as for public interest or
It requires that if the government were to take any private welfare (Those that advance the interest of society)
property, it may do so provided it is for public use and that
there be payment for it, which compensation must be one that The public respondents have stressed that the development of the 808
is just hectares includes plans that would give the petitioners and other
displaced persons productive employment, higher incomes, decent
It is only where the owner is unwilling to sell, or cannot accept housing, water and electric facilities, and better living standards. Our
the price or other conditions offered by the vendee, that the dismissing this petition is, in part, predicated on those assurances. The
right of PTA to proceed with the expropriation of the 282 hectares
power of eminent domain will come into play to assert the
already identified as fit for the establishment of a sports complex to
paramount authority of the State over the interests of the promote tourism is, therefore, sustained.
property owner. Private rights must then yield to the irresistible
demands of the public interest in the time-honored Manosca v. CA
justification, as in the case of the police power, that the welfare 252 SCRA 412 (1996)
of the people is the supreme law
A piece of land located at P. Burgos St. Calzada Taguig, Metro Manila,
the birth place of Felix Manalo (founder of IglesiaNiCristo), is
expropriated to be converted to a National Historical Landmark.
17 CONSTITUTION, Art. III, § 9
82 | P LATON
declared by the owner or as determined by the assessor, whichever is
Public use= expansive meaning; public advantage; general benefit lower.

Violative of Religion Clause? NO. Prior to the promulgation of P.D. Nos. 76, 464, 794 and 1533, this Court
has interpreted the eminent domain provisions of the Constitution and
This attempt to give some religious perspective to the case deserves established the meaning, under the fundamental law, of just
little consideration, for what should be significant is the principal compensation and who has the power to determine it. Thus, in the
objective of, not the casual consequences that might follow from the following cases, wherein the filing of the expropriation proceedings
exercise of the police power. were all commenced prior to the promulgation of the aforementioned
decrees, we laid down the doctrine on just compensation:
The purpose was to recognize the distinctive contribution of the late F.
Manalo to the culture of the Phil., rather than to commemorate his Municipality of Daet v. Court of Appeals (93 SCRA 503, 516) xxx xxx xxx
pounding of INC. And in the case of J.M. Tuason & Co., Inc. v. Land Tenure
Administration, 31 SCRA 413, the Court, speaking thru now Chief justice
The practical reality that greater benefit may be derived by the Fernando, reiterated the 'well-settled (rule) that just compensation
members of INC is merely secondary/ incidental. That the fact that only means the equivalent for the value of the property at the time of its
a few would actually benefit does not necessarily diminish the essence taking. Anything beyond that is more and anything short of that is less,
and character of public use. than just compensation. It means a fair and fun equivalent for the loss
sustained, which is the measure of the indemnity, not whatever gain
Local governments have the authority to seize private land and would accrue to the expropriating entity
turn the property over to private developers for economic
Garcia v. Court ofappeals (102 SCRA 597, 608) xxx xxx xxx Hence, in
development
estimating the market value, afl the capabilities of the property and all
the uses to which it may be applied or for which it is adapted are to be
Just Compensation considered and not merely the condition it is in the time and the use to
which it is then applied by the owner. And the facts as to the condition
The   word   “just”   is   used   to   intensify   the   meaning   of   the   word   of the property and its surroundings, its improvements and capabilities
“compensation”   to   convey   the   idea   that   the   equivalent   to   be   may be shown and considered in estimating its value.
rendered for the property to be taken shall be real, substantial,
full, or ample Republic v. Santos (141 SCRA 30, 35-36), according to section 8 of Rule
67, the court is not bound by the commissioners' report. It may make
such order or render such judgment as shall secure to the plaintiff the
To compensate is to render something which is equal in value property essential to the exercise of his right of condemnation, and to
to that taken or received the defendant just compensation for the property expropriated. This
Court may substitute its own estimate of the value as gathered from
It is well-settled that just compensation means the equivalent the record (Manila Railroad Company v. Velasquez, 32 Phil. 286).
for the value of the property at the time of its taking. Anything
beyond that is more, and anything short of that is less, than just However, the promulgation of the aforementioned decrees practically
compensation set aside the above and many other precedents hammered out in the
course of evidence-laden, well argued, fully heard, studiously
deliberated, and judiciously considered court proceedings. The decrees
The market value of the land taken is the just compensation to categorically and peremptorily limited the definition of just
which the owner of condemned property is entitled, the market compensation thus:
value being that sum of money which a person desirous, but not
compelled to buy, and an owner, willing, but not compelled to P.D. No. 76: For purposes of just compensation in cases of private
sell, would agree on as a price to be given and received for such property acquired by the government for public use, the basis shall be
property the current and fair market value declared by the owner or
administrator, or such market value as determined by the Assessor,
whichever is lower.
The property owner is entitled to compensation only for what
he actually loses, and what he loses is only the actual value of P.D. No. 464, 794: Section 92. Basis for payment of just compensation in
the property at the time of taking expropriation proceedings. — In determining just compensation which
private property is acquired by the government for public use, the basis
Judicial Determination shall be the market value declared by the owner or administrator or
anyone having legal interest in the property, or such market value as
Just compensation would be determined in accordance with determined by the assessor, whichever is lower.
the market value declared by the owner or administrator or
P.D. No. 1533: Section 1. In determining just compensation for private
anyone having legal interest in the property, or such market
property acquired through eminent domain proceedings, the
value as determined by the assessor, whichever is lower compensation to be paid shall not exceed the value declared by the
owner or administrator or anyone having legal interest in the property
Determination   of   just   compensation   is   not   as   simple   as   child’s   or determined by the assessor, pursuant to the Real Property Tax Code,
arithmetic--it is a judicial function that requires the judicious whichever value is lower, prior to the recommendation or decision of
minds of men and women! the appropriate Government office to acquire the property.

Export Processing Zone Authority V. Dulay Held: PD 1533 which eliminates the court's discretion to appoint
149 SCRA 305 (1987) commissioners pursuant to Rule 67 of the Rules of Court, is
unconstitutional and void.
The question raised in this petition is whether or not Presidential
Decrees Numbered 76, 464, 794 and 1533 have repealed and We are convinced and so rule that the trial court correctly stated that
superseded Sections 5 to 8 of Rule 67 of the Revised Rules of Court, the valuation in the decree may only serve as a guiding principle or one
such that in determining the just compensation of property in an of the factors in determining just compensation but it may not
expropriation case, the only basis should be its market value as substitute the court's own judgment as to what amount should be

83 | P LATON
awarded and how to arrive at such amount. Would article 1250 of NCC apply regarding extraordinary inflation?
Peso to dollar rate at the time of hearing was P6.775=$1
It is violative of due process to deny to the owner the opportunity to
prove that the valuation in the tax documents is unfair or wrong. NO. 1. There is no contractual obligation 1250 applies if there was
contract or agreement. Obviously there can be no agreement to the
The determination of "just compensation" in eminent domain cases is a contrary to speak of because the obligation sought by the government
judicial function. The executive department or the legislature may did not originate from contract but from law, which is generally not
make the initial determinations but when a party claims a violation of subject to the will of the parties.
the guarantee in the Bill of Rights that private property may not be Also the unusual delay in bringing up the present action (a period of
taken for public use without just compensation, no statute, decree, or almost 25 years) bars her claim.
executive order can mandate that its own determination shag prevail
over the court's findings. Much less can the courts be precluded from Therefore, the correct amount to be paid was P14615 (based on the
looking into the "just-ness" of the decreed compensation time of taking in 1924, and not adjusted to P49459.34)

Timely and Prompt Payment Interest of 145410.44 at the rate 0f 6% from 1924 should be reduced.
The solicitor general said that the award of damages should be from
1924 –(in the decision in Feb 29 1972) citing the case of raymunda s.
Determination of just compensation would have to be
digsan v auditor general, et al ruling that it should be from the time
reckoned from the value of the property either at the time of when the claim for compensation was filed. Whether the ruling in this
the taking or the filing of the complaint for condemnation, case cited is still the prevailing doctrine, what was said in the decision
whichever comes first of this court in the above-cited case involving the same on the instant
matter has become the law of the case. No motion for reconsideration
Property cannot simply be taken in the meantime and payment having been filed by the solicitor general before the decision has
to follow some years or decades later. It is not fair that the become final.
owner, while immediately divested of his property, is made to
Wherefore, judgment appealed from was reversed; value of the land
wait for the equivalent value for some time
should be from the time of taking P14615 (at P2.37 per sqm) with
interest at 6% per annum from 1924 plus Atty fees of 5000
Without prompt payment, compensation cannot be considered
“just”   inasmuch   as   the   property   owner   is   made   to   suffer   the  
consequences of being immediately deprived of his land while
being made to wait for a decade or more before actually Form or Mode of Compensation
receiving the amount necessary to cope with his loss. Payment
of just compensation should follow as a matter of right GR: To satisfy the requirement of just compensation payment
immediately after the order of expropriation is issued. Any for property taken should be in the form of cash
delay in payment must be counted from said order. However,
the delay to constitute a violation of due process must be XPN: If the exercise of such power would entail expansive
unreasonable and inexcusable; it must be deliberately done by acquisitions to carry out an important public policy, such as
a party in order to defeat the ends of justice agrarian reform

Where the payment of compensation does not accompany the In relation to the agrarian reform program, the Government
taking of property for public use but is postponed to a later has provided for the payment partly in cash and partly in
date, the owner of the property is ordinarily entitled to the government financial instruments, as well as payment by
award of an additional sum which will compensate for delay or means of shares of stock in government-owned or controlled
which will, in other words, produce the full equivalent of the corporations, Land Bank of the Philippines preferred shares,
value of the property paid contemporaneously with the taking physical assets or other qualified investments, tax credits, and
Land Bank bonds
Where private property is acquired by the Government and all
that  remains  is  the  payment  of  the  price,  the  owner’s  action  to   Parties Entitled to Compensation
collect the price must be brought within ten years otherwise it
would be barred by the statute of limitations. However, where In  the  American  jurisdiction,  the  term  “owner”  when  employed  
private property is taken by the Government for public use in statutes relating to eminent domain to designate the persons
without first acquiring title thereto either through who are to be made parties to the proceeding, refers, as is the
expropriation  or  negotiated  sale,  the  owner’s  action  to  recover   rule in respect to those entitled to compensation, to all those
the land or the value thereof does not prescribe who have lawful interest in the property to be condemned,
including a mortgagee, a lessee and a vendee in possession
Commissioner of public highways v burgos under an executory contract
96 SCRA 831 (1980)

Gov’t  took  land  for  right-of-way purpose. Year 1924 Government Grants, Easement and Compensation

Petitioner wanted to recover, Respondent says, it was either sold or It may happen that a property that is now held private used to
donated, cannot be recovered. be government-owned and that it was just granted to citizens
by way of free patent, homestead or any other mode. The issue
Dismissed by trial court 1959. that might be raised is whether the government would have to
pay if it takes back part of it for use as an easement. The
Appealed to SC. Reversed. And the case was remanded to the court of
answer to this is to be determined in accordance with whatever
origin  for  the  compensation  plus  attorney’s  fees  plus  the  determination  
of just compensation. reservations there might have been in the grant itself

84 | P LATON
GR: Non-payment of just compensation does not entitle the
private landowners to recover possession of their expropriated
lots

Elements  of  “Taking” XPN: In cases where the government failed to pay just
compensation within five (5) years from the finality of the
(1) Entry by the expropriator into a private property judgment in the expropriation proceedings, the owners
(2) The entrance into private property must be for more than a concerned shall have the right to recover possession of their
momentary period property. This is in consonance   with   the   principle   that   “the  
(3) Such entry should be under warrant or color of legal government cannot keep the property and dishonour the
authority judgment”
(4) The property must be devoted to public use or otherwise
informally appropriated or injuriously affected Expropriation, Agrarian Reform and Socialized Housing
(5) The utilization of the property for public use must be in such
a way as to oust the owner and deprive him of all beneficial While the powers of the government have to march together
enjoyment of the property harmoniously, like in implementing the agrarian reform
program, there may be instances, however, where the power of
expropriation may come into conflict with such program. In the
event of a conflict, the latter must give way to the former
Questions of Necessity and Judicial Oversight
Province of Camarines Sur v. Court of Appeals
While basically the determination of which property to be 222 SCRA 173 (1993)
expropriated is left to the discretion of the authority seeking to
exercise the power, the issue of necessity and arbitrariness The Province of Camarines Sur filed two separate cases for
expropriation against San Joaquin and was subsequently authorized to
may, however, come to the fore, especially if it is a local
take possession of the property to (1) establish a pilot farm for non-
authority or other agency that is trying to exercise such food and non-traditional agricultural crops and (2) a housing project
delegated power. This would then be for the courts to look for provincial government employees. The San Joaquins filed a motion
into, determining from the facts and the circumstances of each to relief and a motion to admit an amended motion to dismiss, both
case whether the power sought to be utilized indeed exists, and were denied by the trial court. They then assailed before the Court of
whether it is being applied in a manner consistent with the Appeals. The CA ordered the suspension of expropriation proceedings
authority given until after the province shall have secured the approval of the
Department of Agrarian Reform of the plan to expropriate the lands of
petitioners for the use of housing project.
The authority to condemn is to be strictly construed in favor of
the owner and against the condemnor. When the power is There  has  been  a  shift  from  the  literal  to  broad  interpretation  of  “public  
granted, the extent to which it may be exercised is limited to purpose”  or  “public  use”  for  which  the  power  of  eminent  domain  may  
the express terms or clear implication of the statute in which be exercised. Under the new concept, “public   use”   means   public  
the grant is contained advantage, convenience or benefit, which tends to contribute to the
general welfare and the prosperity of the whole community. The
expropriation of the property in question is for public purpose. The
establishment of the pilot development center would make available to
the community the invaluable information and technology on
agriculture, fishery, and cottage industry. The housing project also
satisfies the public purpose requirement of the Constitution. As held in
Sumulong v. Guerrero (154SCRA461),  “Housing is a basic human need.
Shortage in housing is a state concern since it directly and significantly
Stages or Phases of Expropriation Proceedings affects  the  general  welfare.”

The first one determines the authority of the condemning Sec. 9 of BP Blg. 337 does not intimate in the least that local
agency and also of the propriety of its exercise of the power in government units must first secure the approval of DAR for the
the context of the facts obtaining in the suit. The second phase conversion of lands from agricultural to non-agricultural use, before
they can institute the expropriation proceedings. Likewise, there is no
is the determination of the amount of just compensation
provision in the Comprehensive Agrarian Reform Law which expressly
subjects the expropriation of agricultural lands by the local government
The second stage which involves the issue of just compensation units to the DAR. In Heirs of Juancho Ardana v. Reyes (125SCRA220),
is as important, if not more, than the first stage which refers to the power of expropriation is superior to the power to distribute lands
the issue of “public  purpose” under the land reform program.

Parenthetically, while entry and possession in favor of the Acquisition and Transfers of Municipal Properties by National
expropriator may be effected at once with the filing of the Government
complaint and the deposit of the assessed value of the
property, transfer f ownership would only be made after full Local government may have properties of their own. Depending
payment of the just compensation on the nature of property, whether held as patrimonial or
private property or one that is communal, the national
government may take the same either for consideration or not.
In the former, since the property is really privately owned, then
Non-Payment of Just Compensation and Recovery of Property the national government if it wants it would have to pay for it.
In the latter, the property may be considered as held in trust by

85 | P LATON
the local government for the benefit of the people and the military aircrafts used the airport. They frequently came close to
national authority respondent’s   property;   the   noise   was   startling,   and   the   glare   of   the  
landing lights lighted the place up brightly at night. As a result of the
City of Baguio v. National Waterworks and Sewerage Authority noise, respondents had to give up their chicken business. The result
106 Phil. 144 (1959) was destruction of the use of property as a commercial chicken farm.
Respondents were also frequently deprived of sleep, and the family had
RA 1383 created NAWASA, a public corporation, the purpose of which become nervous and frightened. They sued the in the Court of Claims
is to consolidate and centralize all waterworks, sewerage, and to recover for an alleged taking of their property and for damages to
drainage systems in the Philippines under one control, direction, and their poultry business.
general supervision. The law also provided that all existing government
owned waterworks and sewerage systems in the Philippines are The Court of claims held that there was a taking. The damages were
transferred to NAWASA, and that the net book value of the properties not merely consequential. They were product of a direct invasion of
and assets of said entities shall be received by the Authority in payment the   respondents’   domain. Flights over private lands are not taking,
for an equal value of the assets of NAWASA. unless they are so low and so frequent as to be a direct and immediate
interference with the enjoyment and the use of the land. The findings
A complaint was filed contending that RA 1383 does not include the of the court of claims established that there was a diminution in value
Baguio Waterworks System. Also, assuming that it does, said Act is of the property, and that the frequent low-level flights were the direct
unconstitutional because it has the effect of depriving the plaintiff the and immediate cause.
ownership, control, and operation of said waterworks system without
compensation and without due process of law. NAWASA contended Dissenting Opinion: When flights are made within the navigable
that RA 1383 is a proper exercise of police power of the State and airspace without any physical invasion of the property of the
assuming that the said act contemplates an act of expropriation, it is landowners, there has been no taking of property. It says that, at most,
still a constitutional exercise of the power of eminent domain and that there was merely incidental damage occurring as a consequence of
at  any  rate   BWS  is  not  a  private  property  but  “public  works  for  public   authorized air navigation.
service”  over  which  the  Legislature  has  control.
Republic v. PLDT
The contention that RA 1383 constitutes valid exercise of police power 26 SCRA 620 (1969)
rather than a directive to expropriate by the exercise of power of
eminent domain cannot be entertained. The Act does not confiscate, In 1947, soon after its creation, the Bureau of Telecommunications set
destroy nor appropriate property. It merely directs that all waterworks up its own Government Telephone System by utilizing its own
belonging to cities, municipalities and municipal districts in the appropriation and equipment and by renting trunk lines of PLDT, to
Philippines be transferred to the NAWASA. enable government offices to call private parties

Baguio Waterworks system is not a property held in trust by a In March 1958, the plaintiff, through the Director of
municipal corporation for the benefit of the public but it is rather a Telecommunications, entered into an agreement with RCA
property owned in a proprietary character. The property held by a Communications, Inc. (RCA), for a joint overseas telephone service
municipal corporation in its private capacity is not subject to whereby the Bureau would convey -radio-telephone overseas calls
unrestricted control of the legislature, and the municipality cannot be received   by   RCA’s   station   to   and   from local residents. Thereafter, in
deprived of such property against its will, except by the exercise of April 1958, PLDT complained to the Bureau of Telecommunications that
eminent domain with payment of full compensation. said bureau was violating the conditions under which Private Branch
Exchange   (PBX)   is   interconnected   with   the   PLDT’s   facilities   for   the  
Salas v. Jarencio bureau had used the trunk lines not only for the use of government
46 SCRA 734 (1972) offices but even to serve private persons of the general public, in
competition with the business of the PLDT. It gave notice that if said
RA 4118 was never intended to expropriate the property involved but violations were not stopped, it would sever the telephone connections
merely to confirm its character as a communal land of the State and to
make it available for disposition by the National Government. The They did not arrive at mutually acceptable terms, however
subdivision of the land and the conveyance of the resulting subdivision
lots to the occupants by Congressional authorization do not operate as After trial, the lower court rendered judgment that it could not compel
an exercise of the power of eminent domain without just compensation the PLDT to enter into an agreement with the Bureau because the
in violation of the Constitution, but simply as a manifestation of its right parties were not in agreement
and power to deal with state property.
Held: Yes. The court a quo has apparently overlooked that while the
Republic may not compel the PLDT to celebrate a contract with it, the
Constructive Expropriation and Consequential Damages Republic may, in the exercise of the sovereign power of eminent
domain, require the telephone company to permit interconnection of
While taking ordinarily connotes physical taking, the same need the government telephone system and that of the PLDT, as the needs
not always be. There could be taking even if there is no physical of the government service may require, subject to the payment of just
entry or dispossession of the owner. In this instance of compensation to be determined by the court
constructive taking, considering that the owner suffers some
loss,   he   ought   to   be   compensated.   “Taking   occurs   not   only   The Bureau of telecommunications, under Section 78(b) of Executive
Order No. 94, may operate and maintain wire telephone or radio
when the government actually deprives or dispossesses the
telephone communications throughout the Philippines by utilizing
property owner of his property or of its ordinary use, but also existing facilities in cities, towns, and provinces under such terms and
when there is a practical destruction or material impairment of conditions or arrangement with present owners or operators as may be
the  value  of  his  property.” agreed upon to the satisfaction of all concerned; but there is nothing in
this Section that would exclude resort of condemnation proceedings
United Sates v. Causby where unreasonable or unjust terms and conditions are exacted, to the
328 U.S. 256, 90 L Ed 1206, 66 S Ct 1062 (1946) extent of crippling or seriously hampering the operations of said bureau

Respondents owned 2.8 acres near an airport outside of Greensboro, The concept of public use is no longer confined in the
North Carolina. It has on it a dwelling house, and also various traditional notion of use by the public, but held synonymous
outbuildings which were mainly used for raising chickens. Various

86 | P LATON
with public interest, public benefit, public welfare, and public television stations is the sale of air time to advertisers and that to
convenience require these stations to provide free air time is to authorize a taking
which  is  not   “a   de minimis temporary limitation or restraint upon the
use  of  private  property”
Elections and the Power of Eminent Domain
Held: No. All broadcasting, whether radio or by television stations, is
The Constitution directs the Commission on Elections during licensed by the government
the election period to supervise and regulate the enjoyment or
utilization of all franchise or permits for the operation of media They are merely given temporary privilege of using them. Since a
of communication or information aimed at ensuring equal franchise is a mere privilege, the exercise of the privilege may
opportunity, time and space, and the right to reply, including reasonably be burdened with the performance by the grantee of
reasonable, equal rates therefor, for public information some form of public service
campaigns and forums among candidates in connection with
Under §92 of B.P. Blg. 881, the COMELEC does not take over the
the objective of holding free, orderly, honest, peaceful, and operation of radio and television stations but only the allocation of air
credible elections. In implementation of this mandate, the time to the candidates for the purpose of ensuring, among other things,
COMELEC was directed by law to procure from the print media equal opportunity, time, and the right to reply as mandated by the
the so-called COMELEC Space, and from the broadcast media, Constitution
the COMELEC Time. The issue of compensation came up in two
cases, with contrasting results This right of the people is paramount to the autonomy of broadcast
media. To affirm the validity of §92, therefore, is likewise to uphold the
Philippine Press Institute, Inc. v. COMELEC people’s  right  to  information  on  matters  of  public  concern
244 SCRA 272 (1995)
Public-Interest Business, Takeovers and Compensation
On March 2 1995, the COMELEC promulgated Resolution No. 2772,
which   provided   in   part   as   follows:   “Sec. 2. Comelec Space. — The With regard to public utilities, businesses imbued with public
Commission shall procure free print space of not less than one half (1/2) interest, or vital industries in which exigencies may at times
page in at least one newspaper of general circulation in every province dictate that the Government take over either momentarily or in
or city for use as "Comelec Space" from March 6, 1995 in the case of
a more permanent manner, the Constitution provides:
candidates for senator and from March 21, 1995 until May 12, 1995. In
the absence of said newspaper, "Comelec Space" shall be obtained
from any magazine or periodical of said province or city. Section 17. In times of national emergency, when the public interest so
requires, the State may, during the emergency and under reasonable
In the instant Petition for Certiorari and Prohibition, PPI asks the Court terms prescribed by it, temporarily take over or direct the operation of
to declare Comelec Resolution No. 2772 unconstitutional and void on any privately-owned public utility or business affected with public
the ground that it violates the prohibition against the taking of private interest.
property for public use without just compensation
Section 18. The State may, in the interest of national welfare or
Held: Yes. To compel print media companies to donate “Comelec   defense, establish and operate vital industries and, upon payment of
space”  of  the  dimensions  specified  in  Section  2  of  resolution  No.  2772   just compensation, transfer to public ownership utilities and other
(not less than one-half   Page),   amounts  to   “taking”   of   private   personal   private enterprises to be operated by the Government.
property for public use or purposes
While the provision on transfer of utilities is explicit that there
The threshold requisite for a lawful taking of private property for be payment of just compensation, that on temporary takeover
public use need to be examined here: one is the necessity for the does not say so
taking; another is the legal authority to effect the taking. The element
of necessity for the taking has not been shown by respondent Comelec The temporary takeover by the government extends only to the
operation of the business and not to the ownership thereof. As
Section 2 of Resolution No. 2772 does not, however, provide a
constitutional basis for compelling publishers, against their will, in the such the government is not required to compensate the private
kind of factual context here present, to provide free space for Comelec entity-owner of the said business as there is not transfer of
purposes. Section 2 does not constitute a valid exercise of the power of ownership, whether permanent or temporary
eminent domain
Miscellaneous Considerations
Section 2 of Resolution No. 2772 is a blunt and heavy instrument that
purports, without a showing of existence of a national emergency or Under the so-called   concept   of   “inverse   condemnation,”   it   is  
other imperious public necessity, indiscriminately and without regard
the owner who brings suit to recover the compensation on
to the individual business condition of particular newspaper or
magazines located in different parts of the country, to take private account of physical intrusion into his property by the
property of newspaper and magazine publishers authorities without initiating the appropriate condemnation
proceeding.   This   may   be   referred   to   as   “regulatory   taking,”  
Telecommunications and Broadcast Attorneys of the Philippines, Inc. such as when, by certain regulations affecting private property
v. COMELEC 289 SCRA 337 (1998) like land, there is effectively put in place a permanent
restriction or prohibition on its beneficial use. In such an
Petitioners challenge the validity of §92 of B.P. Blg. 881 (Omnibus instance,   the   same   might   come   under   the   concept   of   “taking”  
Election Code) on the ground, among others, that it takes property for which the owner may claim and be entitled to
without due process of law and without just compensation
compensation
It will be noted that while §90 of B.P. Blg. 881 requires the COMELEC to
procure print space which as we have held, should be paid for, § 92 When there is only an incidental taking as a result of certain
states that air time shall be procured by the COMELEC free of charge public works, such as those caused by the construction of
roads, bridges, underpass or flyovers, then the impairment of
Petitioners claim that the primary source of revenue of the radio and the   value   of   one’s   property   may   not   be   considered  
87 | P LATON
compensable. It is just one of the things one would have to live 582 scra 583 (2009)
up in an ever changing and dynamic society *from Constitutional Law Updates and Teasers (bar 2010) by Atty
Gorospe
The provision (Article III, Section 9) is the most important
Here   the   court   spoke   of   “cleansing   effect”   of   the   expropriation  
protection of property rights in the Constitution. This is a proceedings.   “The   fact   of   expropriation   is   extremely   significant,   for  
restriction on the general power of the government to take titles acquired by the state by way of expropriation are deemed
property. The constitutional provision is about ensuring that cleansed of whatever previous flaws may have attended these titles . . .
the government does not confiscate the property of some to .  ‘in  an  in  rem  proceeding,  condemnation  acts  upon  the  property.  After  
give it to others. condemnation, the paramount title is in the public under a new and
independent title; thus, by giving notice to all claimants to a disputed
There are two different kinds of taking that can be identified. A title, condemnation proceedings provide a judicial process for securing
better title against all the world than may be obtained by voluntary
”possessory”  taking  occurs  when  the  government  confiscates or
conveyance’”
physically  occupies  property.  A  “regulatory”  taking  occurs  when  
the  government’s  regulation  leaves  n   reasonable   economically   In this particular case, the court  noted  that  “  in  annulling  the  manotok  
viable use of the property titles, focus was laid on the alleged defects of TCT No. 4211 issued in
September of 1918. However, TCT No. 4211 was issued decades before
A regulation which denies all economically beneficial or the property was expropriated. Thus, any and all defects that may have
productive use of land will require compensation under the attended that particular title would have been purged when the
taking clause property covered by it was subsequently acquired by the State through
Eminent  Domain.”  
Any arm of the State that exercises the delegated power of SPOUSES CIRIACO and ARMINDA ORTEGA vs CITY OF CEBU
eminent domain must wield that power with circumspection In this case, the sangguniang panlunsod of the City of Cebu enacted
and utmost regard for procedural requirements. A government ordinance  1519  for  the  expropriation  of  the  land  of  the  spouses’Oiriaco  
instrumentality that fails to observe the constitutional and Armada Ortega also appropriating the sum of P3,284,400.00. The
guarantees of just compensation and due process abuses the city filed a case in court for the expropriation. The court fixed the value
authority delegated to it, and is liable to the property owner for of the land to 11000 per square meter or the sum of P31,416,000.00.
damages this decision of the rtc became final and executory because of Cebu
City’s   failure   to   appeal   and   a   writ   of   execution   was   issued   to   enforce  
the   court’s   judgment.   The   appropriated   amount   was   now   subject   for  
Familara v. JM Tuason Co., Inc
execution or garnishment for the same is no longer exempt from
49 SCRA 338 (1973)
execution.
The city of Cebu filed an Omnibus Motion to Stay Execution,
Petitioner, barrio captain of Barrio Tatalon, seeks to enjoin the
Modification of Judgment and Withdrawal of the Case, contending that
respondent  Corporations  “from  bulldozing  and  fencing  any  portion”  of  
the price set by the RTC as just compensation is way beyond the reach
the  Tatalon  Estate  “and/or  from  selling”  any  of  the  lots  therein,  “and/or  
of its intended beneficiaries for its socialized housing program. The
from   filing   ejectment   cases”   against   the   “bona-fide”   occupants   of   the  
motion was denied and the Motion for Reconsideration was likewise
estate...Reliance is thus placed by petitioner upon the provisions of
denied.
Section 4 of Republic Act No. 2616
Pursuant to the order by the RTC, Sheriff Benigno B. Reas served a
Notice of Garnishment to Philippine Postal Bank, P. del Rosario and
Held: This Court has also ruled that Section 4 thereof, which places a
Junquera   Branch   Cebu   City,   garnishing   [Cebu   City’s]   bank   deposit  
restraint upon the exercise and enjoyment by the owner of certain
therein.
rights over its property, is justifiable only if the government takes
Hence, [Cebu City] filed the instant Petition for Certiorari before the CA
possession of the land and is in a position to make a coetaneous
During its pendency, [Cebu City] filed before the [RTC] a Motion to
payment of just compensation to its owner
Dissolve, Quash or Recall the Writ of Garnishment, contending that
Account No. 101-8918-334 mentioned in Ordinance No. 1519 is not
For it must be realized that in a condemnation case, it is from the
actually an existing bank account and that the garnishment of [Cebu
condemnor’s   taking   possession   of   the   property   that   the owner is
City’s]   bank   account   with   Philippine   Postal   Bank   was   illegal,   because
deprived of the benefits of ownership, such as possession,
government funds and properties may not be seized under writ of
management and disposition thereof. Before that time, the proprietary
execution or garnishment to satisfy such judgment, on obvious reason
right of the owner over his property must be recognized
of   public   policy.   The   RTC   denied  said  motion.  [Cebu   City’s]   Motion  for  
Reconsideration was also denied.
It was definitively ruled that until the proceedings for condemnation
[The Spouses Ortega] filed an Ex-Parte Motion to Direct the New
have been fully instituted and possession of the property is taken
Manager of Philippine Postal Bank to Release to the Sheriff the
over by the condemnor, the enforcement of final decrees of eviction
Garnished Amount, which was granted by the [RTC]. [Cebu City] filed a
may not be lawfully suspended
Motion for Reconsideration, but the same was denied.
Hence, [Cebu City] filed another Petition for Certiorari (CA-G.R. SP NO.
In the acquisition of private property, the Government may 00147) [with the Court of Appeals].
either act through persuasion or negotiation, but if all else fails -Partially Granted. The assailed Orders of the [RTC] [Assailed Orders
and it is rendered helpless on that score, it may then resort to dated March 11, 2002 and July 2, 2003, respectively, in CA-G.R SP NO.
its coercive power of expropriation, subject always, of course, 80187] are hereby ANNULLED AND SET ASIDE insofar as they denied
to the constitutional guidelines that the property be for public [Cebu  City’s]  Motion  to  Stay  Execution,  but  they  are  hereby  AFFIRMED  
use and that there be payment of just compensation. Thus, it insofar   as   they   denied   [Cebu   City’s]   Motion   to   Modify   Judgment   and  
Withdraw from the Expropriation Proceedings. Furthermore, the
could be seen that the tension and accommodation between
assailed Orders of the [RTC dated March 8, 2004 in CA-G.R. SP NO.
private rights and public interests continue to play on 00147] are hereby ANNULLED AND SET ASIDE. Let the Decision of the
[RTC] be executed in a manner prescribed by applicable law and
Additional Cases jurisprudence.
(K) Eminent Domain [9] Hence, these consolidated appeals by petitioners Cebu City and the
Spouses Ortega positing the following issues:
K1. Manotok Realty Inc v CLT realty development corporation 1.   Whether   the   CA   erred   in   affirming   the   RTC’s   denial   of   Cebu   City’s  

88 | P LATON
Omnibus Motion to Modify Judgment and to be Allowed to Withdraw Where the expropriation case had long been final and executory, both
from the Expropriation Proceedings. the order of expropriation and the order fixing just compensation can
2. Whether the deposit of Cebu City with the Philippine Postal Bank, no longer be modified-the expropriator can no longer withdraw from
appropriated for a different purpose by its Sangguniang Panglungsod, the expropriation proceedings.
can be subject to garnishment as payment for the expropriated lot
covered by City Ordinance No. 1519. Also, the expropriation court cannot, by itself, order the expropriating
We deny both petitions. local government to enact an appropriation ordinance in order to
On the first issue, the CA did not  err  in  affirming  the  RTC’s  Order  that   satisfy its judgment--the land owner must file a separate mandamus
the expropriation case had long been final and executory. case for that purpose.
Consequently, both the Order of expropriation and the Order fixing just
compensation by the RTC can no longer be modified. In short, Cebu City
cannot withdraw from the expropriation proceedings.
Section 4, Rule 67 of the Rules of Court on Expropriation provides:
SEC. 4. Order of expropriation. – If the objections to and the defenses
against the right of the plaintiff to expropriate the property are
overruled, or when no party appears to defend as required by this Rule,
the court may issue an order of expropriation declaring that the
plaintiff has a lawful right to take the property sought to be
expropriated, for the public use or purpose described in the complaint,
upon the payment of just compensation to be determined as of the K7. Masikip v. City of Pasig
date of the taking of the property or the filing of the complaint, 479 SCRA 391 (2006)
whichever came first.
A final order sustaining the right to expropriate the property may be Lourdes Dela Paz Masikip is the registered owner of a parcel of land,
appealed by any party aggrieved thereby. Such appeal, however, shall which the City of Pasig sought to expropriate a portion thereof for the
not prevent the court from determining the just compensation to be “sports development and recreational activities”   of   the   residents   of  
paid. Barangay Caniogan. Masikip refused.
An order of expropriation puts an end to any ambiguity regarding the
right   of   the   petitioner   to   condemn   the   respondents’   properties.   W/N there was genuine necessity to expropriate the property.
Because an order of expropriation merely determines the authority to
exercise the power of eminent domain and the propriety of such Judicial review of the exercise of eminent domain is limited to the ff.
exercise, its issuance does not hinge on the payment of just areas of concern: (a) adequacy of just compensation; (b) necessity of
compensation. After all, there would be no point in determining just the taking; (c) public use character of the purpose of the taking.
compensation  if,  in  the  first  place,  the  plaintiff’s  right  to  expropriate  the  
property was not first clearly established. There is already an established sports development and recreational
Conversely, as is evident from the foregoing, an order by the trial court activity center at Rainforest Park in Pasig City. Evidently, there is no
fixing just compensation does not affect a prior order of expropriation. “genuine  necessity”  to  justify  the  expropriation.  The  records  show  that  
As applied to the case at bar, Cebu City can no longer ask for the Certification issued by the Caniogan Barangay Council which
modification of the judgment, much less, withdraw its complaint, after became the basis for the passage of Ordinance No. 4, authorizing the
it failed to appeal even the first stage of the expropriation proceedings. expropriation, indicates that the intended beneficiary is the
As regards the second issue raised by the Spouses Ortega, we quote Melendres   Compound     Homeowner’s     Association,     a     private,     non-
with  favor  the  CA’s  disquisition  thereon,  to  wit: profit organization, not the residents of Caniogan. Where the taking by
While the claim of [the Spouses Ortega] against [Cebu City] is valid, the the State of a private property is done for the benefit of small
[RTC] cannot, by itself, order the City Council of [Cebu City] to enact an community, such taking cannot be considered for public use.
appropriation ordinance in order to satisfy its judgment.
The proper remedy of [the Spouses Ortega] is to file a mandamus case The right to own and possess property is one of the most cherished
against [Cebu City] in order to compel its Sangguniang Panglungsod to rights of men. Unless the genuine necessity for the expropriations is
enact an appropriation ordinance for the satisfaction of [the Spouses clearly established, it shall be the duty of the courts to protect the
Ortega’s]  claim. This remedy is provided in the case of Municipality of rights of individuals to their private property.
Makati v. Court of Appeals, which provides:
Nevertheless, this is not to say that private respondent and PSB are left K8. National Power Coprporation v. Tiangco
with no legal recourse. Where a municipality fails or refuses, without 514 SCRA 674 (2007)
justifiable reason[s], to effect payment of a final money judgment
rendered against it, the claimant may avail of the remedy of mandamus Respondents are owners of a parcel of land with an area of 152,187
in order to compel the enactment and approval of the necessary square meters at Barangay Sampaloc, Tanay, Rizal. NPC requires 19,423
appropriation ordinance, and the corresponding disbursement of square   meters   of   the   respondents’   aforementioned property, across
municipal funds therefor which its 500Kv Kalayaan-San Jose Transmission Line Project will
It is a settled rule that government funds and properties may not be traverse.   NPC’s   Segregation   Plan   for   the   purpose   shows   that   the  
seized under writs of execution or garnishment to satisfy judgments, desired right-of-way  will  cut  through  the  respondents’  land.  Within the
based on obvious consideration of public policy. Disbursements of portion sought to be expropriated stand fruit-bearing trees, such as
public funds must be covered by the corresponding appropriation as mango, avocado, jackfruit, casuy, santol, calamansi, sintones and
required by law. The functions and public services rendered by the coconut trees. After repeated unsuccessful negotiations, NPC filed an
State cannot be allowed to be paralyzed or disrupted by the diversion expropriation complaint against the land of the respondent in the RTC
of public funds from their legitimate and specific objects, as of Tanay, Rizal. The RTC issued a writ of possession in favor of NPC after
appropriated by law. paying the deposit requirement. The trial court rendered its decision on
WHEREFORE, the petitions in G.R. Nos. 181562-63 and 181583-84 are the value of the property using the 1984 tax declaration. (which is
hereby DENIED. The Decision of the Court of Appeals in CA-G.R. SP Nos. incorrect as stated in the decision of the supreme court) The
80187 and 00147 is AFFIRMED. No pronouncement as to costs. respondents filed a motion for reconsideration but it was denied by
RTC. So they filed an appeal and the CA gave merit to the contention of
K2. Ortega v. City of Cebu the respondents and made its revised valuation using the 1993 tax
602 scra 601 (2009) declaration (increasing the value of the property). The case went up to
*from Constitutional Law Updates and Teasers (bar 2010) by Atty the SC.
Gorospe.. )
W/N the property should be valued using the 1984 or the 1993 tax

89 | P LATON
declarations. limit the general right to legislate on the subject on
divorce...Nor are judgments, though rendered upon contracts,
W/N Sec. 3-A of R.A. No. 6395, as amended by P.D. 938 will apply. deemed to be within the provision...Nor does a general law,
giving consent of a State to be sued, constitute a contract
In eminent domain cases, the time of taking is the filing of the
complaint, if there was no actual taking prior thereto. Hence, in this
case, the value of the property at the time of the filing of the complaint According to Black, any statute which introduces a change into
on November 20, 1990 should be considered in determining the just the express terms of the contract, or its legal instruction, or its
compensation due the respondents. Normally, the time of taking validity, or its discharge, or the remedy for its enforcement,
coincides with the filing of complaint for expropriation as ruled in the impairs the contract. The extent of the change is not material. It
case of Power Corporation v. Court of Appeals, et al. The expropriation is not a question of degree or manner or cause, but of
proceedings in this case having been initiated by NPC on November 20, encroaching in any respect on its obligation or dispensing with
1990, property values on such month and year should lay the basis for
any part of its force
the proper determination of just compensation.

It should not apply in the case at bar, the acquisition of such easement It should not be overlooked, however, that the prohibition to
is not gratis. The limitations on the use of the property taken for an impair the obligation of contracts is not absolute and
indefinite period would deprive its owner of the normal use thereof. unqualified. The prohibition is general, affording a broad
For this reason, the latter is entitled to payment of a just compensation, outline and requiring construction to fill in the details
which must be neither more nor less than the monetary equivalent of
the land taken. The policy of protecting contracts against impairment
presupposes the maintenance of a government by virtue of
which contractual relations are worthwhile--a government
which retains adequate authority to secure the peace and good
Chapter 12 order of society. The contract clause of the Constitution must,
Contract Clause therefore, be not only in harmony with, but also in
18
subordination to, in appropriate instances, the reserved power
“No  law impairing  the  obligation  of  contracts  shall  be  passed” of the State to safeguard the vital interests of the people

The clause, according to Corwin, is lately of negligible Indeed, since contracts of employment are impressed with
importance, and might be well stricken from the Constitution. public interest, the provision of applicable statutes are deemed
For most practical purposes, in fact, it has been written into such contracts and the parties are not at liberty to
insulate themselves and their relationships from the impact of
It has to give way to the superior and legitimate exercise by the labor laws and regulations by simply contracting with each
State of the police power to promote the health, morals, peace, other
education, good order, safety, and general welfare of the
people, especially so as statutes in the exercise of valid police Related to the foregoing, the Constitution also expressly
power must be read into every contract provides,  in  what  is  referred  to  as  “Reservation  Clause,”  that  no  
franchise or right for the operation of a public utility shall be
The impairment clause is no longer inviolate; in fact, there are granted  “except  under  the  condition  that  it  shall  be  subject  to  
many who now believe it is an anachronism in the present-day amendment, alteration, or repeal by the Congress when the
society common good  so  requires”

In truth, the Contract Clause has never been thought as a As  to  the  phrase  “obligations and contracts,”  the  same  “imports  
limitation  on  the  exercise  of  the  State’s  power  of  taxation save a legal duty to perform the specified obligation of that contract,
only where a tax exemption has been granted for a valid not to substitute and perform, against the will of one of the
consideration parties, a different, albeit equally valuable  obligation”

Contractual tax exemptions, in real sense of the term and What the guarantee prohibits is the passage of a law which
where the non-impairment clause of the Constitution can enlarges, abridges or in any manner changes the intention of
rightly be invoked, are those agreed to by the taxing authority the contracting parties
in contracts, such as those contained in government bonds or
debentures, lawfully entered into by them under enabling laws As to intrusion by private persons into contractual obligations,
in which the government, acting in its private capacity, sheds its the Court has said that this is governed by statutory
cloak of authority and waives its governmental immunity. Truly, enactments not by impairment clause of the Constitution. The
tax exemptions of this kind may not be revoked without sole purpose of this provision is to safeguard the integrity of
impairing the obligations of contracts. These contractual tax valid contractual agreements against unwarranted interference
exemptions, however, are not to be confused with tax by the State in the form of laws
exemptions granted under franchises. A franchise partakes the
nature of a grant which is beyond the purview f the non- HOME BUILDING & LOAN ASSOCIATION V. BLAISDELL
impairment clause of the Constitution 290 US 398, 78 L Ed 413, 54 S Ct 231 (1934)

With   regard   to   “contracts”   covered,   it   has   been   said   that   the   FACTS: Appellant contests the validity of the Minnesota Mortgage
term  embraces  “those  that  are  executed,  that  is,  grants,  as  well   Moratorium Law, as being t to the contract clause and the due process
as those that are executory...They embrace the charters of and equal protection clauses of the Fourteenth Amendment of the
Federal Constitution. Invoking the relevant provisions of the statue,
private corporations...But not the marriage contract, so as to
appellees applied to the District Court for an order extending the
period of redemption from a foreclosure sale. Their petition stated that
18 CONSTITUTION, Art. III, § 10
90 | P LATON
they owned a lot in Minneapolis which they had mortgaged to more so if the credits are unsecured. And the injustice is more patent
appellant; by reason of their default, the mortgaged had been when the debtor is not even required to pay interest during the
foreclosed and sold to appellant in May 1932; that, because of the operation of the relief, unlike similar statutes in the United States.
economic depression appellees had been unable to obtain a new loan Official pronouncements of the Chief Executive states that the
or to redeem, and that, unless the period of redemption were nation has already recovered from the war, and that the peso is one of
extended, the property would be irretrievably lost, and that the the most stable currencies of today.
reasonable value of the property greatly exceeded the amount due on
the mortgage. The District Court of Minnesota entered judgment *Moratorium- postponement of fulfilment of obligations decreed by
extending the period of redemption, subject to the condition that the the state through the medium of courts or the legislature.
appellees should pay the appellant $40 a month through the extended
period. The SC of the State sustained the judgment, upholding the PHILIPPINE VETERANS BANK EMPLOYEES UNION-NUBE V.PHILIPPINE
statute as an emergency measure, namely, the public economic VETERANS BANK
emergency. 189 SCRA 14 (1990)

ISSUE: Whether or not the Minnesota Mortgage Moratorium Law FACTS: The respondent bank, which was created by a special law (RA
violates the Contract Clause of the Federal Constitution. 3518), was placed under receivership by the Monetary board of the
Central Bank. Subsequently, the monetary board ordered the
THE  COURT’S  RULING:  The  Minnesota  statute  as  here  applied,  does  not   liquidation of the Bank after finding that the Bank had incurred an
violate the contract clause of the Federal Constitution. outstanding liability of P540, 835, 860.79. The petitioners claim that as
the bank was created by a special law, a contractual relationship now
REASON: exists between the Government and the stockholders of the bank that
The conditions upon which the period of redemption is extended do cannot be disturbed without violation of the impairment clause.
not appear to be unreasonable. The integrity of the mortgage
indebtedness is not impaired; interest continues to run; the validity of ISSUE: Whether or not the liquidation of Veterans Bank would
the sale and the right of a mortgagee-purchaser to title or to obtain a constitute a violation of the impairment clause.
deficiency judgment if the mortgagor fails to redeem within the
extended period are maintained, and the conditions of redemption HELD:
stand as they were under the prior law. Even if it be conceded that the charter of the Bank constitutes a
The mortgagor, during the extended period, is not ousted from contract between the government and the stockholders of the bank, it
possession, but he must pay the rental value of the premises as would not follow that the relationship cannot be altered without
ascertained in judicial proceedings, and this amount is applied to the violating the impairment clause.
carrying of the property and to interest upon indebtedness. The contract may be altered validly if it involves the public interest,
An emergency existed in Minnesota which furnished a proper to   which   private   interests   must   yield   “as   a   postulate   of   the   existing  
occasion for the exercise of the reserved power of the state to protect social  order”.
the vital interests of the community. The need in the case at bar is no less compelling, to wit, the
The legislation was addressed to a legitimate end, the legislation was preservation of the integrity and stability of our banking system. It is
not for the mere advantage of particular individuals, but for the the duty of the Central bank in such an event to step in and salvage the
protection of a basic interest of society. remaining resources of the bank so that they may not continue to be
dissipated or plundered by those entrusted with their management.
*Not only are existing laws read into contracts in order to fix the
obligation as between the parties, but the reservation of ESSENTIAL
Additional Cases
ATTRIBUTES OF SOVEREIGN POWER is also read into contracts as a
postulate of the legal order. (L) Contract Clause [1]

RUTTER V. ESTEBAN
93 Phil. 68 (18 May 1953)
Chapter 13
FACTS: Rutter sold to Esteban two parcels of land situated in the City of Poverty and Legal Protection
Manila for the sum of P9,600, of which P4,800 was paid outright and
the balance was made payable as follows: P2,400 on or before August “Free   access   to   the   courts   and   quasi-judicial bodies and
7, 1942 and P2,400 on or before August 27, 1943, with interest at rate adequate legal assistance shall not be denied to any person by
of 7 percent per annum. Esteban failed to pay the two instalments as 19
reason  of  his  poverty”
agreed upon, as well as the interest that had accrued thereon, and so
Rutter instituted on 1949 an action to recover the balance, the interest
due  thereon  and  the  attorney’s  fees  stipulated  in  the  contract.  Esteban   The right to litigate us an escape valve to relieve the pressures
set up as a defense the moratorium clause embodied in RA 342. He of personal disagreements that might otherwise explode into
claims that this is a pre-war obligation and as a war sufferer, payment physical confrontation
of his obligation cannot be enforced until after the lapse of eight years
from the settlement of his claim by the Philippine War Damage While authorities are not required to relieve the accused of his
Commission, and this period has not yet expired. poverty, they have the obligation not to take advantage of
indigence in the administration of justice
THE   COURT’S   RULE:   Judgment   is   hereby   rendered   ordering   the  
defendant to pay the plaintiff the sum of P4,800 with interest thereon
at the rate of 7 percent per annum from August 27, 1942, until its full Collective activity undertaken to obtain meaningful access to
payment,   plus   12   percent   as   attorney’s   fees.   Failure   to   pay   this   the courts is the fundamental right within the protection of the
judgement as stated, the properties mortgaged will be sold at public First Amendment. However, the right would be a hollow
auction and the proceeds applied to its payment in accordance with promise if courts could deny associations of workers or others
law. The continued operation and enforcement of RA 342 is the means of enabling their members to meet the costs of legal
unreasonable and oppressive, hence, NULL and VOID. representation
REASON:
The period in the case at bar seems to us unreasonable, if not
oppressive. The hope to effect collection becomes extremely remote, 19 CONSTITUTION, Art. III, § 11
91 | P LATON
It is not enough to say that all pauper litigants should be can appoint a counsel de oficio to   prosecute   the   latter’s   appeal  
assured of legal representation. They deserve quality pursuant to Rule 122 of the Rules of Court (the rules governing appeal)
representation as well and Art. III, Sec. 11.

Let’s   have   a   little   backgrounder   (based   on   Mr.   Justice   Malcolm’s  


Obviously, the constitutional guaranty of access to the courts writings):
refers only to courts with appropriate jurisdiction as defined by
law. It does not mean that a person can go to any court for Two of the basic privileges of the accused in a criminal prosecution are
redress of his grievances regardless of the nature or value of his (1) the right to the assistance of counsel and (2) the right to a
claim. If the petitioner is barred from filing his complaint before preliminary examination. Pres. McKinley made the first right a part of
our courts, it is because they are not vested with the the  US  Organic  Law  by  imposing  the  inviolable  rule  that  “[the  accused]  
appropriate jurisdiction under the Warsaw Convention, which is shall  enjoy  the  right  to  have  assistance  of  counsel  for  the  defense.”  Said  
right,   described   by   Judge   Cooley   as   “perhaps   the   privilege   most
part of the law of our land
important   to   the   person   accused   of   crime”   is   now   enshrined   in   our  
Constitution as Sec. 11 of Art. III.
PEOPLE V. HASSAN
157 SCRA 261 (1988) In criminal cases there can be no fair hearing unless the accused be
given an opportunity to be heard by counsel. The right to be heard
FACTS: Usman Hassan was accused of murder for stabbing to death would be of little meaning if it does not include the right to be heard by
Ramon Pichel, in Zamboanga City on July 23, 1981. The lone eyewitness counsel.
for the prosecution, Jose Samson, in a confrontation at La Merced
Funeral Homes, positively identified Hassan as the very person who The right to counsel de oficio does not cease upon the conviction of an
stabbed the victim. The Supreme Court, in ACQUITTING the accused accused by a trial court. It continues, even during appeal, such that the
based on insufficiency of evidence, also delved on the fact of his duty of the court to assign a counsel de oficio persists where an accused
poverty. interposes intent to appeal. Even in a case where the accused has
signified his intent to withdraw his appeal, the court is required to
THE   COURT’S   RULE:   Wherefore,   the   decision   is   hereby   reversed,   and   inquire into the reason for the withdrawal. Where it finds the sole
the accused Usman Hassan is ACQUITTED of the crime charged. His reason for the withdrawal to be poverty, as in this case, the court must
release from confinement is here by ordered, unless he i held for assign a counsel de oficio, for despite such withdrawal, the duty to
another legal cause. protect the rights of the accused subsists and perhaps, with greater
reason.
REASON:
The element of doubt, if reasonable in this case, must operate In this spirit, the Court ordered the appointment of a counsel de oficio
against the inference of guilt the prosecution would draw from its for Rio. (It did not turn out well.)
evidence.
Both Samson and the accused testified clearly and unequivocably
that Usman was alone when presented to Samson by Carpio (police Chapter 14
officer). The confrontation arranged by the police investigator between Rights of Suspects
the self-proclaimed eyewitness and accused did violence to the right of
the latter to counsel in all stages of the investigation into the “Any   person   under   investigation   for   the   commission   of   an  
commission of a crime. As it turned out, the method of identification offense shall have the right to be informed of his right to
became just a confrontation. remain silent and to have competent and independent counsel
The rest of the investigation of the crime and the preparation of the preferably of his own choice. If the person cannot afford the
evidence for prosecution were done haphazardly, perfunctorily and
services of counsel, he must be provided with one. These rights
superficially.
The records of the case do not show any attempt on the part of cannot be waived except in writing and in the presence of
Corporal Carpio, or any other police officer, to investigate or question counsel.
Benhar Isa, (a notorious and a deadly police character in Zamboanga
City, with a long record of arrests), in connection with the killing of “No   torture,   force,   violence,   threat,   intimidation,   or   any   other  
Pichel jr. means which vitiate the free will shall be used against him.
The trial judge did not propound any single question to the accused, Secret detention places, solitary, incommunicado, or other
and only three to his mother. Taking into account their poverty and similar forms of detention are prohibited.
illiteracy, the mother and son needed as much, if not more, help, than
the trial judge extended to the prosecution witnesses during their
examination by asking them clarificatory and mostly leading questions. “Any   confession   or   admission obtained in violation of this or
In that sense and to that extent, the ACCUSED was DISADVANTAGED. Section 17 hereof shall be inadmissible in evidence against him.

PEOPLE V. RIO “The   law   shall   provide   for   penal   and   civil   sanctions   for  
201 SCRA 702 (1991) violations of this section as well as compensation to the
rehabilitation of victims of torture or similar practices, and their
FACTS: Ricardo Rio was convicted of rape and sentenced to reclusion 20
families.”
perpetua by the trial court. He filed an appeal and as such, the records
of the case were forwarded to the Court of Appeals who then promptly
As now guaranteed by the Bill of Rights, a suspect, or one under
sent it to the Supreme Court in view of the penalty imposed upon him.
In the course of events, Rio filed a motion to withdraw his appeal due
custodial investigation, is accorded the following basic rights,
to his poverty. The Court denied Rio's motion and appointed a counsel viz: (a) right to remain silent; (b) right to have competent and
de oficio for him. independent counsel, preferably of his own choice; and, (c)
right to be informed of such rights. He is also entitled to have
ISSUE: This is a rape case. The main issue here is obviously irrelevant to counsel appointed for him if he could not afford one. And, to
our discussion. Instead, we focus on the constitutional mandate set make his rights more meaningful and not simply forfeited
forth in Article III, Sec. 11. through ignorance or inattention, the Constitution throws in
RULING: The Court opined that Rio seemed unaware that the former
20 CONSTITUTION, Art. III, § 12 (1), (2), (3), and (4)
92 | P LATON
the additional safeguard that waiver of such rights may only be rape, a decision affirmed on appeal.
had if it is in writing and in the presence of counsel
ISSUE: WON the privilege set in the Fifth Amendment is fully applicable
during a period of custodial investigation.
The Miranda Rights -- Background and Development
RULING: Due to the coercive nature of the custodial interrogation by
The Fifth Amendment of the U.S. Constitution guarantees the police (Mr. Chief Justice Warren cited several police training manuals
privilege against self-incrimination. The Sixth Amendment, on which had not been provided in the arguments), no confession could be
the other hand, assures the assistance of counsel in criminal admissible under the Fifth Amendment self-incrimination clause and
prosecutions. These were understood basically as referring to Sixth Amendment right to an attorney unless a suspect had been made
proceedings already instituted in court, not to custodial aware of his/her rights and the suspect had then waived them.
investigations
The person in custody must, prior to interrogation, be clearly
informed that he has the right to remain silent, and that anything he
ESCOBEDO V. ILLINOIS
says will be used against him in court; he must be clearly informed
378 U.S. 478, 12 L Ed 2d 977, S4 S Ct 1758 (1974)
that he has the right to consult with a lawyer and to have the lawyer
with him during interrogation, and that, if he is indigent, a lawyer will
FACTS: Danny Escobedo's brother-in-law, Manuel, a convict from
be appointed to represent him.
Chicago, was shot and killed on the night of January 19, 1960. Danny
Escobedo was arrested without warrant early the next morning and
If the individual indicates in any manner, at any time prior to or during
interrogated. However, Escobedo made no statement to the police and
questioning, that he wishes to remain silent, the interrogation must
was subsequently released that afternoon. Subsequently, Benedict
cease ... If the individual states that he wants an attorney, the
DiGerlando, who was in custody and considered another suspect told
interrogation must cease until an attorney is present. At that time, the
the police that indeed Escobedo fired the fatal shots because the victim
individual must have an opportunity to confer with the attorney and to
had mistreated Escobedo's sister. On January 30, the police arrested
have him present during any subsequent questioning.
Escobedo again – this time with his sister, Grace. While transporting
them to the police station, the police explained that DiGerlando had
implicated Escobedo, and urged him and Grace to confess. Escobedo DICKERSON V. UNITED STATES
again declined. Escobedo asked to speak to his attorney, but the police 530 U.S. 428, 152 L Ed 2d 1069, 122 S Ct 2315 (2000)
refused, explaining that although he was not formally charged as of yet,
he was in custody and could not leave. His attorney went to the police FACTS: In the wake of the ruling on Miranda v. Arizona, Congress
station and repeatedly asked to see his client, but was repeatedly enacted  a  law   which  in  essence   makes  the   admissibility   of   a   suspect’s  
refused access, saying that Escobedo did not want to see him. Police statements on a custodial interrogation turn solely on whether they
and prosecutors proceeded to interrogate Escobedo for fourteen-and- were made voluntarily. Dickerson was indicted for bank robbery,
a-half hours and refused his request to speak with his attorney with the conspiracy to commit bank robbery, and using a firearm in the course
same consistency throughout that time. While being interrogated, of committing a crime of violence. He moved to suppress a statement
Escobedo made statements implicating his knowledge of the crime. he had made to the FBI, on the ground that he had not received
After conviction for murder, Escobedo appealed on the basis of being “Miranda   warnings”   before   being   interrogated.   The   District   Court  
denied the right to counsel. granted his petition, and the Government took an interlocutory appeal;
the Fourth Circuit reversed, acknowledging that even though Dickerson
Escobedo appealed to the Illinois Supreme Court, which initially held had not received Miranda rights, the law was satisfied because his
the confession inadmissible and reversed the conviction. Illinois statement was voluntary – concluding that Miranda was not a
petitioned for rehearing and the court then affirmed the conviction. constitutional holding and that Congress could by statute have the final
Escobedo appealed to the U.S. Supreme Court. say on the question of admissibility.

ISSUE: WON, under the circumstances, the refusal by the police to ISSUE: WON the Miranda court announced a constitutional rule or
honor  petitioner’s  request  to  consult  with  his  lawyer  during  the  course   merely exercised its supervisory authority to regulate evidence in the
of an interrogation constitutes a  denial  of  “the  Assistance  of  Counsel”   absence of congressional direction.
in violation of the Sixth Amendment.
HELD: A little backgrounder: in the past, a suspect's confession had
RULING: YES. When Escobedo requested, and was denied, an always been inadmissible if it had been the result of coercion, or
opportunity to consult with his lawyer, the investigation had ceased to otherwise given involuntarily. This was true in England, and American
be   a   general   investigation   of   an   “unsolved   crime”. Escobedo had law inherited that rule from England.
become  the  accused,  and  the  purpose  of  the  interrogation  was  to  “get  
him”  to  confess  his  guilt  despite  his  constitutional  right  to  do  so.  In  such   However, as time went on, the US Supreme Court recognized that the
a scenario, with Escobedo requesting and being denied an opportunity Fifth Amendment was an independent source of protection for
to consult with his lawyer, and with the police not having effectively statements made by criminal defendants in the course of police
warned him of his absolute constitutional right to remain silent, the interrogation. Custodial police interrogation by its very nature "isolates
Supreme  Court  ruled  that  Escobedo  has  been  denied  “the  Assistance  of   and pressures the individual" so that he might eventually be worn
Counsel”   in   violation   of   the   Sixth   Amendment   as   “made   obligatory   down and confess to crimes he did not commit in order to end the
upon the  States  by  the  Fourteenth  Amendment”  (as  stated  in  Gideon v. ordeal. In Miranda, the Court had adopted the now-famous four
Wainwright). As such, no statement elicited by the police during the warnings to protect against this particular evil.
interrogation may be used against him in a criminal trial.
Congress, in response, enacted §3501. That statute clearly was
NOTE: The above ruling was later implicitly overruled by the landmark designed to overrule Miranda because it expressly focused solely on
case, Miranda v. Arizona. voluntariness of the confession as a touchstone for admissibility. As
regards Congress' authority to pass such a law: on the one hand, the
Court's power to craft non-constitutional supervisory rules over the
MIRANDA V. ARIZONA
federal courts exists only in the absence of a specific statute passed by
384 U.S. 436, 16 L Ed e2 692, 86 S Ct 1602 (1966)
Congress. But if on the other hand the Miranda rule was constitutional,
Congress could not overrule it, because the Court alone is the final
FACTS: Police arrested Miranda and took him to a special interrogation
arbiter of what the Constitution requires. As evidence of the fact that
room where they secured a confession. The inculpatory admission was
the Miranda rule was constitutional in nature, the Court pointed out
admitted at trial. In the end, Miranda was convicted for kidnapping and
that many of its subsequent decisions applying and limiting the

93 | P LATON
requirement arose in decisions from state courts, over which the Court In Illinois v. Perkins, the U.S. Supreme Court said that the
lacked the power to impose supervisory non-constitutional rules. Miranda warnings are not required when the suspect is
Furthermore, although the Court had previously invited legislative unaware that he is speaking to a law enforcement officer, and
involvement in the effort to devise prophylactic measures for
gives a voluntary statement which incriminates himself. In said
protecting criminal defendants against overbearing tactics of the police,
it had consistently held that these measures must not take away from
case an undercover government agent was placed in the cell of
the protections Miranda had afforded. Perkins, who was incarcerated on charges unrelated to the
subject   of   the   agent’s   investigation.   He   made   statements   that  
In conclusion, the Court held that Miranda announced a constitutional implicated him in the crime that the agent sought to solve
rule that Congress may not supersede legislatively.
Miranda forbids coercion, not mere strategic deception by
Miranda Rights, Adoption and Adaptation in Philippine Milieu taking   advantage   of   a   suspect’s   misplaced   trust   in   one   he  
supposes to be a fellow prisoner...Ploys to mislead a suspect or
A perusal of the Philippine provision on rights of suspects lull him into a false sense of security that do not rise to the level
would readily show that it has accorded more rights than found of compulsion or coercion to speak are not within Miranda’s
in Miranda itself concerns

The present Charter has broadened the guarantee in these The tactic employed here to elicit a voluntary confession from a
respects: (1) the right to counsel means not just any counsel, suspect does not violate the Self-Incrimination Clause
but  a  “competent  and  independent  counsel,  preferably  his  own  
choice”;   (2)   the   right   to   remain   silent   and   to   counsel   can   only   People vs. Marra
be waived in writing and in the presence of counsel; and (3) the
rule on inadmissibility expressly includes admission, not just SPO3 De Vera with three other policemen went in a crime scene. Upon
confessions arriving the said victim Tandoc was already brought to the hospital and
they went to investigate. As they were investigating, the person that
the  witnesses  was  telling  was  pointing  to  a  security  guard  of  a  “linda’s  
People v. Mahinay ihaw-ihaw”  who  is  Samuel  Marra. The policeman saw him eating in a
nearby eatery and asked him series of questions. And even went to
Mahinay, a houseboy was accused of rape with homicide. In the police their house to get the issued gun for investigation and upon seeing that
station he executed a extrajudicial confession with a counsel on how there was a missing bullet he asked Marra point-blank why he shot
the crime happened. After trial he was convicted and sentenced to Tandoc. At first he denied the accusation then when a witness pointed
death. Upon automatic review of the Supreme court, he claims, among at him he said it was a self-defense. De Vera picked up Din to and
others, that his extrajudicial confession was secured under duress and brought him to the police station and identified Marra as assailant.
that he was assisted by the counsel only when he was forced to sign the
same. He contends that militates against appellant is his constitutional The was held that under Article III, Sec.12(1) provides that  “any  person  
right to counsel. But his contention was belied by the records as well as under investigation for the commission of an offense shall have the
the testimony of the lawyer who assisted, warned and explained to him right to be informed of his right to remain silent and to have a
his constitutionally guaranteed pre-interrogatory and custodial rights. competent   and   independent   counsel   preferably   his   choice…”   the  
inquiry here is whether or not Marra was under custodial investigation
The court held that the heavy penalty of death is to be ensured that when he admitted the killing but invoked self-defense. The court
such evidence is obtained by lawful means. The court, as guardian of believed that it was not so situated.
the rights of the people lays down the procedure, guidelines and duties
which arresting, detaining, inviting, or investigating officer must do and He was not under custodial investigation when he made the admission.
observe at the time of making arrest and again at and during the time There was no coercion whatsoever to compel him to make such
of the custodial interrogation in accordance with constitution, statement. He could have refused to answer the questions from the
jurisprudence and republic act no. 7438. Miranda rights must be very  start…
updated by the court because of its insufficiency in the light of legal
developments.
People vs. Bravo

Investigation, Custody and Interrogations Appellant was the last person seen with the victim, a nine year old
child, before her disappearance and the subsequent decomposing
What was said on Escobado might be an appropriate starting body. She was found to be raped. Alexander Mico, Chief of the
yardstick, i.e., where the investigation has developed from a intelligence section of the Santiago Police Department was the person
general inquiry into an unsolved crime to a focused inquiry in-charge of the investigation came up with the investigation that
directed at a particular person as a suspect, then the trigger Bravo(appellant) was the suspect. Mico found the suspect at his work
for the operation of the guaranteed rights must already be place. And brought him to the police station were the appellant
admitted he was with the girl and was drunk and does not remember
deemed to have been activated. As the Court said in People v.
anything that happened. Mico admitted that he did not inform the
Bravo,   “[t]he   mantle   of   protection   under   this   constitutional   appellant   of   his   constitutional   right   to   remain   silent…Mico   said   it was
provisions covers the period from the time a person is taken an informal interviewing when he made the custodial interrogation
into custody for investigation of his possible participation in the proper.
commission of a crime or from the time he is singled out as a
suspect in the commission of a crime although not yet in The court held to acquit Bravo.
custody...Courts are not allowed to distinguish between
preliminary questioning and custodial investigation proper
21 The exclusionary rule will apply, sprang from the recognition that police
interrogatory procedures lay fertile grounds for coercion, physical, and
when applying the exclusionary rule
psychological, of the suspect to admit responsibility for the crime under
investigation. It was not intended as a deterrent to the accused from
confessing guilt, if he voluntarily and intelligently so desires but to
21Custodial investigation has been defined as any questioning initiated by law
protect the accused from admitting what he is coerced to admit
enforcement officers after a person has been taken into custody or otherwise
deprived of his freedom of action in any significant way. (Sebastian, Sr. v. although untrue. Any statement allegedly made by him pertaining to
Garchitorena, 343 SCRA 463 [2000]) his possible complicity in the crime without prior notification of his
94 | P LATON
constitutional right is inadmissible in evidence.
Counsel Competent, Effective and Vigilant
People vs. Sequino
We have constitutionalized the right to counsel because of our
Eugenio Godinez, overseer of the Hacienda Jose Ancajas in Medellin, hostility against the use of duress and other undue influence in
Cebu, went to the Medellin Rural Bank to withdraw 50,557.17 to pay
extracting confessions from a suspect. Force and fraud tarnish
the   wages   for   the   workers   in   the   hacienda.     The   bank’s   cashier  
instructed Jimmy the janitor and motorcycle driver of the bank to drive
confessions and render them inadmissible
Godinez and Broniola behind him. As they were nearing the hacienda
the accused, armed with gun block their way and as they past the The Constitution has not simply limited itself to guaranteeing
accused there was a gunshot and from the motorcycle Broniola has right to counsel. It has taken pains to qualify the kind of lawyer
fallen. The accused was noticed by Godinez because they were that  should  be  assigned  to  a  suspect,  namely,  a  “competent  and  
employees before of the hacienda. independent  counsel  preferably  of  his  own  choice”
SPO Luna,who was in the headquarters, received of the report and
It is noteworthy that the modifiers competent and independent
investigated. Upon investigating he saw a piece of paper in the bushes
were the incident happened ad it pointed out in a name Melvida,
were terms absent in all organic laws previous to the 1987
Elpidio. In the house  Melvida’s  they  saw  the  father  and  ask  where  was   Constitution. Their addition in the fundamental law of 1997 was
Elpidio  and  he  told  that  he  wsa  in  his  brother’s  house.    Then  they  go  the   meant to stress the primacy accorded to the voluntariness of
that house and found Elpidio and brought him to the police station the choice, under the uniquely stressful conditions of a
because of the absence of the barangay captain. Melvida was not custodial investigation, by according the accused, deprived of
assisted by a counsel and investigation was not reduced to writing. In normal conditions guaranteeing individual autonomy, and
the   course   of   Luna’s   investigation,   melvida   admitted   the   he   kept   his   informed judgment based on the choices given to him by a
part of the share of loot. After which he also identified his companions
competent and independent lawyer
place.

The court held that the evidence was inadmissible to the court as the assistance of counsel must be independent and competent,
evidence as a violation of the constitutional rights of the accused. that is, providing full protection to the constitutional rights of
the accused
Meaningful Communication of Rights
As a corollary to having a lawyer who is competent, he must
It is one thing to simply read. It is an entirely different thing to also be one who is effective and vigilant. He must be one
comprehend what is read. In the same manner, it is not enough devoted   to   his   client’s   cause   in   a   manner   that   really   protects  
that the Miranda rights are read to the suspect or even him and not one who throws in only a lackadaisical effort. His
explained to him. It must be done in a manner that enables the assistance must be continuous, from beginning to end
recipient of the advice to understand what is being sought to
be conveyed. It contemplates the transmission of meaningful To be an effective counsel, a lawyer need not challenge all the
information rather that just ceremonial and perfunctory questions being propounded to his client. The presence of a
recitation of an abstract constitutional principle lawyer is not intended to stop an accused from saying anything
which might incriminate him but, rather, it was adopted in our
The rights are meant to be availed of, not simply tendered as a Constitution to preclude the slightest coercion as would lead
formality the accused to admit something false

When the Constitution requires a person under investigation Parenthetically, the counsel who is to help a suspect in the
“to  be  informed”  of  his  right  to  remain  silent  and  to  counsel,  it   execution of an extrajudicial confession must be one whose
must be presumed to contemplate the transmission of assistance is not limited to the written waiver only. As to one
meaningful information rather the just the ceremonial and whose assistance would be needed in order that the right to
perfunctory recitation of an abstract constitutional principle counsel may be waived, he must be a real lawyer, i.e., a
member of the bar
In  other  words,  the  right  of  a  person  under  interrogation  “to  be  
informed”   implies   a   correlative   obligation   on   the   part   of   the   What all of these boil down to is this: an extrajudicial
police investigator to explain, and contemplates an effective confession, even if gospel truth, but executed by a suspect who
communication that results in understanding what is conveyed. was assisted by a lawyer who failed to meet the exacting
Short of this, there is a denial of his rights, as it cannot truly be standards of an independent and competent counsel is deemed
said  that  the  person  has  been  “informed”  of  his  rights an uncounselled confession and, therefore, inadmissible in
evidence
In people v. Compil, the Court found it highly improbable for a
CLAO lawyer to have fully explained in less than ten (10) Post-Facto Validation of Confession
minutes to the suspect who did not even finish Grade One the
latter’s   constitutional   rights   and   the   consequences   of   In People v. Rous, the Third Division of this Court held that an
subscribing to an extrajudicial confession. In People v. Suela, an extrajudicial confession may be admitted in evidence even if
interview of the suspect for five minutes by his lawyer before obtained without the assistance of counsel provided that it was
the former gave his extrajudicial statement was considered read and explained to confessant by counsel before it was
insufficient. In another case, the Court also expressed signed
misgivings on whether a farmer who only reached the fourth
grade, and who only understand Ilocano, could have ***At the time a person is arrested, it shall be the duty of the
understood his constitutional rights conveyed to him by arresting officer to inform him of the reason for the arrest and
counsel in English and Tagalog he must be shown the warrant of arrest, if any. He shall be

95 | P LATON
informed of his constitutional rights to remain silent and to (2) those which are given without the benefit of Miranda
counsel, and that any statement he might make could be used warnings, which are the subject of paragraph 1 of the same §12
against him. The person arrested shall have the right to
communicate with his lawyer, a relative or anyone he chooses Now, under the first paragraph of this provision, it is required
by the most expedient means--by telephone if possible--or by that the suspect in custodial investigation must be given the
letter or messenger. It shall be the responsibility of the following warnings: (1) He must be informed of his right to
arresting officer to see to it that this is accomplished. No remain silent; (2) he must be warned that anything he says can
custodial investigation shall be conducted unless it be in the and will be used against him; and (3) he must be told that he
presence of counsel engaged by the person arrested, by any has a right to counsel, and that if he is indigent, a lawyer will be
person on his behalf, or appointed by the court upon petition appointed to represent him
either of the detainee himself or by anyone on his
behalf.***Any statement obtained in violation of the procedure Voluntary and Spontaneous Admissions and Confessions to
herein laid down, whether exculpatory or inculpatory, in whole Private Persons
or in part, shall be inadmissible in evidence
The guarantee is basically designed to prevent an involuntary
Fruit of the Poisonous Tree--According to this rule, once the and uninformed admission or confession hat might otherwise
primary source (the   ‘tree’)   is   shown   to   have   been   unlawfully   be obtained by law enforcers either through the application of
obtained, any secondary or derivative evidence (the   ‘fruit’)   force, threats, coercion and other underhanded means.
derived from it is also inadmissible Accordingly, even if the Miranda warnings have not been given
to the suspect but he voluntarily made any admissions or
Independent Counsel of Choice confessions without any prodding on the part of the police,
then the same would be admissible
The Constitution likewise requires that in order for counsel to
be really of service to the needs of the suspect for which People v. Cayago
counsel is guaranteed, he must be one whose interest do not 312 SCRA 623 (1999)
run counter to the intended representation
Facts: Accused-appellant Rolando Cayago reported to the Pasig City
police that he found the decomposing body of his wife in an abandoned
The following have been considered not independent counsel: barangay hall.
municipal mayor, City Legal Officer, or a Municipal Attorney, a
barangay captain, a police station commander, a lawyer When Cayago was giving his statement at the police headquarters, the
a[plying for a job at the National Bureau of Investigation, or, an police noticed certain inconcistencies, hence, they advised him to
associate of the private prosecutor in the case in which the undergo a polygraph test. When Cayago was about to be brought to
suspect is implicated Camp Crame for polygraph test, he requested to go to a nearby church,
accompanied by SPO2 Delos Reyes. Thereat, Cayago admitted to Delos
Reyes that he killed his wife and that he was willing to give his
And, as further assurance that the counsel is really
statement relative to said killing.
independent, the person under investigation is given the
opportunity to make the choice--i.e., the lawyer to be Atty. Campanilla conferred with Cayago at the Office of the
“preferably  of  his  own  choice” Investigation Division. After apprising him of his constitutional rights,
Cayago admitted that he killed his wife. Atty Campanilla then advised
While the initial choice of the lawyer in cases where a person Cayago to personally write down his confession.
under custodial investigation cannot afford the services of a
lawyer is naturally lodged in the police investigators, the He was indicted for parricide and at trial his extrajudicial confession
was admitted. He was found guilty. On appeal, Cayago contends that
accused really has the final choice as he may reject the counsel
his extrajudicial confession should be inadmissible because the same
chosen for him and ask for another one was given without affording him the right to counsel guaranteed by the
Constitution.
Confessions and Admissions
Issue:  Whether  or  not  Cayago’s  confession  constitute  as  an  admissible  
In contrast to the 1973 Constitution, the 1987 Constitution as evidence.
explicitly includes in its guarantee extrajudicial confessions and
admissions. In People v. Agustin, the Court called attention to Held: In the case at bar, it is clear that appellant strangulated his wife
resulting  to  her  death.  This  is  supported  by  appellant’s  own  testimony,  
the difference between the two:
his confession to the police and the medical findings corroborating that
she died of asphyxia by strangulation.
In a confession, there is an acknowledgment of guilt. Admission
is usually applied in criminal cases to statements of fact by the Appellant’s   contention   that   the   statement   he   gave   to   the   police   is  
accused which do not directly involve an acknowledgment of inadmissible in evidence because it was given without affording him the
guilt of the accused or of the criminal intent to commit the right to counsel guaranteed by the Constitution has no merit. The right
offense with which he is charged to counsel is afforded by Section 12(1), Article III of the 1987
Constitution  only  to  “person(s)  under  investigation  for  the  commission  
of  an  offense.”  Custodial  rights  of  a  person  are  not  available  whenever  
Kinds of Coerced Confessions
he volunteers statements without being asked. He was not investigated
by the authorities. In fact, after appellant admitted to the police officer
In People v. Obrero, the Courts spoke of two kinds of coerced that he killed his wife, the officer told him that he will be provided with
confessions, i.e., (1) those which are the product of third a lawyer to assist him, namely, Atty. Campanilla. At the, trial, the latter
degree methods such as torture, force, violence, threat, of testified that he talked to appellant, advised him of his constitutional
intimidation, and which are dealt with in paragraph 2 §12, and rights and was present when the latter wrote his extrajudicial
statement admitting that he killed his wife.

96 | P LATON
Those in the media are always in search of what is news, a
People v. Maqueda substantial amount of which is about crimes. They try to report
242 SCRA 565 (1995) on   news   while   still   hot,   or   otherwise   look   for   “scoops”   which  
would interest their viewers, listeners, or readers. And they
Facts: Accused-appellant Maqueda was charge with robbery with
would come across suspects, especially in sensational cases,
homicide and serious physical injuries in relation to the slaying of
Horace William Barker and the infliction of serious injuries on his wife,
invariably they end up interviewing them. In these instances,
Teresita Mendoza, on the occasion of a robbery on 27 August 1991. they may get answers which are downright incriminating.
Because media practitioners are private persons and they are
Maqueda was then fetched from Quezon and brought to the Benguet not in law enforcement, any admissions or confessions made to
Provincial Jail. Its commanding officer directed on his men to get them by unsuspecting suspects are deemed admissible even if
Maqueda’s   statement.   He   did   so   and   according   to   him,   he   informed   the Miranda warnings were not given before the incriminating
Maqueda of his rights under the Constitution. Meanwhile, in April 1992, answers were elicited
while he was under detention, Maqueda filed a Motion to Grant Bail,
stating   therein   that   “he   is   willing   and   volunteering   to   be   a     State  
PEOPLE V. ANDAN
witness in the above entitled case, it appearing that he is the least
269 SCRA 95 (1997)
guilty  among  the  accused  in  this  case.”
The Facts: Accused-appellant Pablito-Andan was indicted for the rape –
In the meantime, Ray Dean Salvosa arrived at the Office of Prosecutor
slaying of Marianne Guevarra, a 20-year-old nursing student, in Baliuag,
Zarate and obtained permission from the latter to talk to Maqueda.
Bulacan. Her gruesome death drew public attention and prompted the
Salvosa then led Maqueda toward the balcony. Maqueda narrated to
town mayor to form a crack team of police officers to look for the
Salvosa that Salvamante then brought him to the Barker house and it
criminal. The police investigated the scene of the crime and the
was only when they were at the vicinity thereof that Salvamante
surrounding   areas,   including   appellant’s   nearby   house   where   they  
revealed to him that his real purpose in going to Baguio City was to rob
found bloodstains on the wall of the pigpen in the backyard. A police
the Barkers.
team   led   by   the   town   mayor   himself   traced   appellant   in   his   parents’  
house. They took him aboard a patrol jeep and brought him to the
He also told other details of the incident. After trial, appellant was
police headquarters where he was interrogated. Initially, appellant
convicted. Although the trial court had doubts on the identification of
denied  any  knowledge  of  Marriane’s  death.
Maqueda by prosecution witnesses and thus disregarded their
testimonies   on   this   matter,   it   decreed   a   conviction   “based   on   the  
By this time, people and media representatives were already gathered
confession and the proof of corpus delicti”  as  well  as  on  circumstantial  
at the police headquarters awaiting the results of the investigation. The
evidence.
mayor arrived and proceeded to the investigation room. Upon seeing
the mayor, appellant approached him and whispered a request that
Issue:  Whether  or  not  Maqueda’s  admission  constitute  as  an admissible
they talk privately. The mayor led appellant to the office of the Chief of
as evidence.
Police and there, appellant broke  down  and  said  “Mayor,  patawarin  mo  
ako!   I   will   tell   you   the   truth.   I   am   the   one   who   killed   Marianne.”   The  
Held: The exercise of the rights to remain silent and to counsel and to
mayor opened the door of the room to let the public and media
be informed thereof under Section (12), Article III of the Constitution
representatives witness the confession. The mayor first asked for a
are not confined to that period prior to the filing of a criminal complaint
lawyer to assist appellant but since no lawyer was available he ordered
or information  but  are  available  at  that  stage  when  a  person  is  “under  
the proceedings photographed and videotaped. In the presence of the
investigation  for  the  commission  of  an  offense.”
mayor,   the   police,   representatives   of   the   media   and   appellant’s   own  
wife and son, appellant confessed his guilt. His confession was captured
It was therefore, wrong for the trial court to hold that Section 12(1),
on videotape and covered by the media nationwide. He was found
Article III of the Constitution is strictly limited to custodial investigation
guilty and sentenced to death. On automatic review to the Supreme
and that it does not apply to a person against whom a criminal
Court, he raises as one of the issues the admission of his extrajudicial
complaint or information has already been filed because after its filing
confessions.
he loses his right to remain silent and to counsel.
The Issue: Whether or not the admission of accused-appellant
The Sinumpaang Salaysay of Maqueda taken by SPO2 Mollena after the
constitute an admissible evidence.
former’s   arrest   was   taken   in   palpable   violation   of   his   rights   under  
Section 12(1), Article III of the Constitution. As disclosed by a reading
Held: When appellant talked with the mayor as a confidant and not as a
thereof, Maqueda was not even told of any of his constitutional rights
law enforcement officer, his uncounselled confession to him did not
under the said section. The statement was also taken in the absence so
violate   his   constitutional   rights.   Appellant’s   confessions were made in
counsel. Such uncounselled Sinumpaang Salaysay is wholly inadmissible
response to questions by news reporters, not by police or any other
pursuant to paragraph 3, Section 12, Article III of the Constitution.
investigating officer. We have held that statements spontaneously
made by a suspect to news reporters on a televised interview are
However, the extrajudicial admissions of Maqueda to prosecutor Zarate
deemed voluntary and are admissible in evidence.
and to Ray Dean Salvosa stand on a different footing. These are not
governed by the exclusionary rules under the Bill of Rights. Maqueda
We   rule   that   appellant’s   verbal   confessions   to   the   newsmen   are   not  
voluntarily and freely made them to Prosecutor Zarate not in the
covered by Section 12(1) and (3) of Article III of the Constitution. The
course  of  an  investigation,  but  in  connection  with  Maqueda’s  plea to be
Bill of Rights does not concern itself with the relation between a private
utilized as a state witness; and as to the other admission, it was given
individual and another individual. It governs the relationship between
to a private person.
the individual and the State. The prohibitions therein are primarily
addressed to the State and its agents.
In the light of his admission to Prosecutor Zarate and Ray Dean Salvosa
and  his  willingness  to  be  a  state  witness,  Maqueda’s  participation  in  the  
IN VIEW WHEREOF, the decision of the Regional Trial Court, Branch 15,
commission of the crime charged was established beyond moral
Malolos, Bulacan in Criminal Case No. 1109-M-94 is affirmed and
certainty. His guilt was, as correctly ruled by the trial court, established
accused-appellant Pablito Andan y Hernandez is found guilty of the
beyond  doubt  by  circumstantial    evidence….
special complex crime of rape with homicide.

Confessions and Admissions to the Media


Even as admissions or confessions made to media men may be
considered admissible, the Court has cautioned against

97 | P LATON
indiscriminate acceptance of the same, however, in view of the assisted by counsel.
possibility of abuse
Under the 1973 and 1987 Philippine Constitutions, the right to counsel
attaches at the start of investigation against a respondent and,
Because of the inherent danger in the use of television as a
therefore, even before adversary judicial proceedings against the
medium   for   admitting   one’s   guilt,   and   the   recurrence   of   this   accused have begun.
phenomenon in several cases, it is prudent that trial courts are
reminded that extreme caution must be taken in further Given the clear constitutional intent in the 1973 and 1987 Constitutions
admitting similar confessions to extend those under police investigation the right to counsel, this
occasion may be better than any to remind police investigators that,
Reenactments, Police Line-ups and Showups while the Court finds no real need to afford a suspect the services of
counsel during a police line-up, the moment there is a move or even an
urge of said investigators to elicit admissions or confessions or even
The counsel requirement also applies to situations where the
plain information which may appear innocent or innocuous at the time,
suspect may be in a situation where his privilege against self- from said suspect, he should then and there be assisted by counsel,
incrimination would otherwise be compromised. This would be unless he waives the right, but the waiver shall be made in writing and
illustrated in the case of a reenactment. By showing how the in the presence of counsel.
crime was allegedly committed, the suspect practically
confesses his guilt. Thus, evidence or admissions obtained Receipts and Samples
through uncounselled reenactments, such as pictures taken on
said occasion, would be inadmissible The need for counsel would also be implicated where a person
may be asked to sign receipts whereby he acknowledges
in so far as the police line-ups are concerned, however, the certain items, which are likely objects or fruits of the crime,
general and prevailing rule is that no counsel is required since taken from him by the police. Again, this is similar to confessing
there is no custodial investigation or interrogation conducted one’s   guilt.   thus,   if   the   suspect   signed   them   in   the   absence   of  
yet. Instead, if there is anybody who is subjected to counsel, it would be inadmissible
questioning, it is the witness who is called to identify who
among those in the line-up might be the culprit

In resolving the admissibility of and relying on out-of-court


identification of suspects, courts have adopted the totality of
circumstances test where they consider the following factors, Exclusionary Rule
viz:  (1)  the  witness’  opportunity  to  view  the  criminal  at  the  time  
of  the  crime;  (2)  the  witness’  degree  of  attention  at  that  time; Just like the sanction for violation of the proscription against
(3) the accuracy of the prior description given by the witness; unreasonable search and seizures, violation of the rule on
(4) the level of certainty demonstrated by the witness at the Miranda warnings also carries the penalty of exclusion.
identification; (5) the length of time between the crime and the However, there is one difference. In the former, the
identification; and (6) the suggestiveness of the identification Constitution says that evidence obtained in violation of the said
procedure provision   “shall   be   inadmissible   for any purpose in any
proceeding.”   In   the   latter,   a   confession   or   admission   obtained  
In regard to the suggestive identification, Justice Brennan spoke in violation of the Miranda safeguards  shall  be  “inadmissible  in  
of  the  “freezing  effect”  of  a  pre-trial identification: evidence  against  him.”

Due   to   the   “freezing   effect”   recognized   in   United States v. Under the Constitution and existing law and jurisprudence, a
Wade, once suggestion has tainted the identification, its mark is confession to be admissible must satisfy the following
virtually indelible. For once a witness has made a mistaken requirements: 1) confession must be voluntary; 2) the
identification,  “he  is  not  likely  to  go  back  on  his  word  later  on.”   confession must be made with the assistance of competent and
As   a   result,   any   effort   of   the   accused   to   “duplicate”   the   initial   independent counsel; 3) the confession must be express; and 4)
photographic display will almost necessarily lead to a the confession must be in writing. Thus, if the confession or
reaffirmation of the initial misidentification admission does not comply with said requisites, it will have to
be excluded
GAMBOA V. CRUZ
162 SCRA 642 (1988) Failure to administer Miranda warnings creates a presumption
of compulsion
The Facts: The petitioner was arrested for vagrancy without a warrant
of arrest. The following day, during the lineup of five (5) detainees,
including petitioner, complainant Erlinda B. Bernal pointed to petitioner
and  said,   “that   one   is  a   companion.”  On  23   July   1979,   an   information  
for robbery was filed against the petitioner. After the prosecution had
rested its case, petitioner, instead of presenting his defense, filed a
Motion to Acquit or Demurrer to Evidence, predicated on the ground Waiver
that the conduct of the line-up, without notice to, and in the absence
of, his counsel violated his constitutional rights to counsel and to due
Related to the voluntary admission and the exclusionary rule is
process.
the rule on waiver. If the suspect knowingly, voluntarily and
The Issue: Whether or not there has   been   a   violation   of   petitioners’   intelligently waives his right to silence and counsel, then that is
constitutional rights to counsel and to due process. his own lookout

Held: Any person under investigation must among other things, be

98 | P LATON
In People v. Obrero,   the   Court   declared:   “It   does   not   matter   Protecting the privacy expectations and liberty interests of
that accused-appellant failed to object to the introduction of individuals need not necessarily be antagonistic to the goal of a
these constitutionally proscribed evidence. The lack of well-ordered society under the Rule of Law
objection did not satisfy the heavy burden of proof which
rested  on  the  prosecution” Additional Cases
(M) Rights of Suspects [3]
Lately, however, the Court has come to the doctrine that if no
timely objection is made to the introduction of such tainted
confessions or admissions, the same would be admissible on
the ground that the accused had thereby waived his right not to
have the same considered as part of the evidence against him
PEOPLE V. LAUGA
People V. Samus 615 SCRA 548
(389 SCRA 93)
FACTS: Lagua, being the father of AAA with lewd design, with the use of
FACTS: Appellant was fund guilty of murder for the killing of Dedicacion force and intimidation, did then and there, willfully, unlawfully and
Balisi, 62 yrs old and her 6 year old grandson, John Arden Balisi in criminally have carnal knowledge with aforementioned daughter, a 13
Calamba Laguna. The police, 8 days after the commission of the crime, year[s]old minor against her will. After the deed was done, AAA
arrested the appellant who was sighted in Sta. Rosa, Laguna. According recounted what happened with her brother BBB, and then told her
to the prosecution, appellant was brought to Camp Vicente Lim where grandfather and uncle, and after which they sought the assistance of
he was informed of his constitutional rights. The following morning, Moises Boy Banting, a bantay bayan.
appellant, assisted by Atty. Juliano, gave his statement admitting the
killings. The appellant denied the charges, as well as the version of the Moises Boy Banting found appellant in his house wearing only his
police as to how he was arrested, He alleged that he was arrested underwear. He invited appellant to the police station, to which
without warrant, was tortured and forced to admit the killing of the appellant obliged. At the police outpost, he admitted to him that he
victims, and that he did not know Atty. Juliano nor did he talk to him. raped AAA because he was unable to control himself – said confession
On appeal, he raises among the issues the admission of his extrajudicial being something which he denied in his testimony. The RTC found him
confession. guilty and convicted him, and upon appeal said conviction was modified
by the CA (by rendering Lagua ineligible for parole and increasing the
HELD: While it is true that the confessions of appellant were made indemnity and moral damages to be paid), hence this petition.
without benefit of counsel, they are still admissible in evidence because
of   the   appellant’s   failure   to   make   timely objections before the trial ISSUE: WON his alleged confession with  a  “bantay  bayan”  is  admissible  
court. If only the defense had proffered them on time, the prosecution in evidence.
could have been warned of the need to present additional evidence to
support  its  case.  To  disregard  a  major  portion  of  the  prosecution’s  case   RULING: NO. This Court is convinced that barangay-based volunteer
at a late stage during an appeal goes against the norms of fundamental organizations in the nature of watch groups, as in the case of the
fairness. "bantay bayan," are recognized by the local government unit to
perform functions relating to the preservation of peace and order at
The apprehending officers contends that the constitutional rights of the barangay level. Thus, without ruling on the legality of the actions
appellant were not violated, since they were not the ones who had taken by Moises Boy Banting, and the specific scope of duties and
investigated and elicited evidentiary matters from him. The Court is not responsibilities delegated to a "bantay bayan," particularly on the
persuaded. The events narrated by the law enforcers in court are too authority to conduct a custodial investigation, any inquiry he makes has
good to be true. Their sworn statements given a day after the arrest the color of a state-related function and objective insofar as the
contradict their testimonies and raise doubts on their credibility. entitlement of a suspect to his constitutional rights provided for under
Evidence to be believed, must not only proceed from the mouth of a Article III, Section 12 of the Constitution, otherwise known as the
credible witness, but must be credible in itself-such as the common Miranda Rights, is concerned, therefore finding the extrajudicial
experience of mankind can approve as probable under the confession of appellant, which was taken without a counsel,
circumstances. inadmissible in evidence. (This does not change anything either way as
Lagua’s   guilt   was   not   deduced solely from said confession but from
confluence   of   evidence   showing   his   guilt   beyond   reasonable   doubt…  
Duty of Subscribing Authorities
but  that’s  another  issue.)

In some instances, the extrajudicial confessions may have to be


Chapter 15
sworn to before authorities, such as public prosecutors or
Rights of the Accused
investigating judges. To ensure that no force, torture or
violence was inflicted on the confessant, the Court required
“No   person   shall   be   held   to   answer   for   a   criminal   offense  
said judges and prosecutors to take some steps to assure
without due process of law.
themselves that there was no maltreatment leading to the
confessions
“In   all   criminal   prosecutions,   the   accused   shall   be   presumed  
innocent until the contrary is proved, and shall enjoy the right
Administrative Investigation
to be heard by himself and counsel, to be informed of the
nature and cause of the accusation against him, to have a
The Miranda rights inhere only in custodial investigations.
speedy, impartial, and public trial, to meet the witnesses face
Accordingly, they may not be invoked in administrative
to face, and to have compulsory process to secure the
investigations conducted by an employer
attendance of witnesses and the production of evidence in his
behalf. However, after arraignment, trial may proceed
While others may view the rights accorded to a suspect as an
notwithstanding the absence of the accused: Provided, that he
added difficulty imposed on police work in solving crimes, the
same should be seen in the republican state over string-arm
tactics which might be better suited for totalitarian countries.

99 | P LATON
has been duly notified and his failure to appear is
22
unjustifiable.” Salonga V. Cruz Pano
134 SCRA 438 (1985)
“All   persons,   except   those   charged   with   offenses   punishable  
FACTS: There was a rash of bombings occurred in Metro Manila from
by reclusion perpetua when evidence of guilt is strong, shall,
August to October 1980. On September 6, Victor Lovely, Jr, almost
before conviction, be bailable by sufficient sureties, or be killed himself and injured his younger brother, Romeo as a result of an
released on recognizance as may be provided by law. The right explosion of a small bomb inside his room at YMCA building in Manila.
to bail shall not be impaired even when the privilege of the writ Found in his possession were several pictures taken sometime in May
of habeas corpus is suspended. Excessive bail shall not be 1980 at the birthday party of former Congressman Raul Daza held at
23
required.” the  latter’s  residence  in  LA.  Petitioner  Jovito  Salonga  and  his  wife  were  
among the guests, including lovely. Lovely was brought by military and
Once a criminal investigation develops to one where there is a police authorities, as a result of his injuries, at the AFP Medical Center
where he was placed in the custody and detention of Col. Roman
person charged in court, a different set of rights sets in
Madella, under the overall direction of General Fabian Ver, Head of
National Intelligence and Security Authority. Shortly, Lovely and his 2
Criminal Due Process brothers, Romeo and Baltazar, were charged with subversion, illegal
possession of explosives and damage to property. The petitioner had
The basic difference between the two provisions lies in the fact been linked to the various bombings in MM. Arrest, Search and Seizure
that while Section 1 speaks of due process in general, both in its orders (ASSO) were issued against persons implicated by Lovely, among
substantive and procedural aspects, Section 14 refers to the them, was herein petitioner. On October 21, while confined in a
procedural component only, i.e., the manner in which the hospital, the arresting officer showed petitioner the ASSO form which
however did not specify the charges against him. His lawyers were also
finding of guilt or innocence would be had, which basically
not permitted to visit him in his hospital room and on November 2, he
means the presence of a court of competent jurisdiction, notice was transferred against his objections from his hospital arrest to an
and hearing, acquisition of jurisdiction over the person of the isolation room without windows in an army prison camp, in Fort
accused, and judgment rendered after trial. Section 14 Bonifacio, Makati. The petitioner was not informed why he was
catalogues the essentials of due process in a criminal transferred and detained, nor was he ever investigated or questioned
prosecution by any military or civil authority. On or around March 26, the counsel
for petitioner was furnished a copy of an amended complaint signed by
General Considerations Gen. Prospero Olivas dated march 12, 1981, charging the petitioner,
along with 39 others with violation of RA 1700, as amended. Hearings
for preliminary investigation were conducted. On October 15, the
The fundamental principle of due process necessarily means counsel for petitioner filed a motion to dismiss the charges against
that a person must be heard before being condemned. The due petitioner for failure of the prosecution to establish a prima facie case
process   requirement   is   part   of   a   person’s   basic   rights,   not   a   against him. The respondent judge denied the motion and on January 4
mere formality that may [be] dispensed with or performed 1982, he issued a resolution ordering the filing of an information for
perfunctorily.   In   its   broader   aspect,   “Due   process   prohibits   violation of the revised Anti-subversion Act, as amended, against 40
criminal stability from shifting the burden of proof to the people, including herein petitioner. the resolutions of the respondent
accused, punishing wholly passive conduct, defining the crimes judge are now the subject of the petition. It is the contention of the
petitioner that no prima facie has been established by the prosecution
in vague or overbroad language and failing to grant fair warning
to justify the filing of an information against him.
of  illegal  conduct”
HELD: The petitioner invokes the constitutionally protected right to life
In criminal proceedings then, due process is satisfied if the and liberty guaranteed by the due process clause, alleging that no
accused   is   “informed   as   to   why   he   is   proceeded   against   and   prima facie has been established to warrant the filing of an information
what charge he has to meet, with his conviction being made to for subversion against him. Petitioner asks this court to prohibit and
rest on evidence that is not tainted with falsity after full prevent the respondents from using the iron arm of the law to harass,
opportunity for him to rebut it and the sentence being imposed oppress and persecute him, a member of the democratic opposition in
the Philippines. In this case, respondents agree with our earlier finding
in accordance with a valid law. It is assumed, of course, that the
that the prosecution evidence miserably fails to establish a prima facie
court that rendered the decision is one of competent case against the petitioner, either as a co-conspirator of a
jurisdiction” destabilization plan to overthrow the government or as an officer or
leader of any subversive organization. They have taken the initiative of
While that right is statutory rather than constitutional in its dropping the charges against the petitioner. We reiterate the rule,
fundament, since it has in fact been established by statute, it is however, that this Court will not validate the filing of an information
a component part of due process in criminal justice. The right based on the kind of evidence against the petitioner found in the
to have a preliminary investigation conducted before being records. WHEREFORE, the petitioner is DISMISSED for having become
moo and academic.
bound over to trial for a criminal offense and hence formally at
risk of incarceration or some other penalty, is not a mere
formal or technical right; it is a substantive right In Allado v. Diokno, the Court declared: The sovereign power
has the inherent right to protect itself and its people from
Part of the guarantee of criminal due process is the assurance vicious acts which endanger the proper administration of
that persons are not simply made to respond to any criminal justice; hence, the State has every right to prosecute and
proceeding on the basis of flimsy or insubstantial grounds. For punish violators of the law. This is essential for its self-
as long as no sufficient basis exists for summoning a person to preservation, nay, its very existence. But this does not confer a
answer a criminal charge, he should not be vexed, annoyed or license for pointless assaults on its citizens
harassed or otherwise unduly compelled to put up with a
Olivas v. Office of the Ombudsman
baseless suit.
239 SCRA 283 (1994)
22 CONSTITUTION, Art. III, § 14(1) and (2); Sixth Amendment
23 CONSTITUTION, Art. III, § 13; Eight Amendment
Petitioner was Commanding General of the PC Metrocom and was

100 | P LATON
retired from the Armed Forced of the Philippines effective Feb 26, the  so  called  “desistance”  of  the private complainant.
2986.He assails the order of respondent requiring him to submit his
affidavit and those of his witnesses at the preliminary investigation of a Jurisprudence acknowledges that the due process in criminal
case for unexplained wealth despite the fact that the PCGG as proceedings, in particular, require a) that the court or tribunal trying
complainant had not reduced its evidence in the form of affidavits and the case is properly clothed with judicial power to hear and determine
submitted supporting documents. In August 1986, petitioner, was the matter before it. b) that jurisdiction is lawfully acquired by it over
informed that there was a freeze order issued covering his bank the person of the accused c) that the accused is given an opportunity to
accounts and a hold order issued against him by PCGG. be heard and d) that judgment is rendered only upon lawful hearing.
The above constitutional and jurisprudential postulates, by now
Olivas V. Office of the Ombudsman elementary and deeply imbedded in our own criminal justice system
239 SCRA 283 (1994) are mandatory and indispensable. The principles find acceptance and
are tersely expressed in the oft-quoted statement that procedural due
FACTS: Petitioner assails the order of respondent requiring him to process  cannot  possibly  be  met  with  out  a    ”law  which hears before it
submit his affidavit and those of his witnesses at the preliminary condemns, which proceeds upon inquiry and renders judgment only
investigation of a case for unexplained wealth despite the facts the after  trial.”
PCGG, as complainant, had not reduced its evidence in the form of
affidavits and submitted supporting documents. Shortly after petitioner Hildawa v. Enrile
retired last February 26, 1986, anonymous letters were sent to PCGG 138 SCRA 146 (1985)
charging him with violations of the Anti Graft and Corrupt Practices
Act and Unexplained Wealth Act. He is among those AFP personnel Facts: Petitioners Hildawa and Valmonte in these civil actions for
being investigated. The issue is whether the petitioner may be declaration of Nullity of Executive/ Administrative Order creating Secret
compelled to file his counter affidavit notwithstanding the fact the no Marshals formed to conduct a concentrated campaign against criminal
sworn complaint or affidavit has been field against him. elements preying on passengers of jeepneys, buses, taxis and all forms
of public conveyance. They allege that such secret marshals which has
HELD: We find for the petitioner. The lack of complaint and affidavits the authority to kill thieves, hold uppers, robbers, pickpockets and
cannot be excused on the plea that this case originate in anonymous other same kind are violative of the several provisions of the
letters sent to the PCGG. Because of leads furnished by those letters, it constitution such as those on due process in regards of the accuse.
would seem that the PCGG has found sufficient evidence justifying its
demand to the petitioner to explain. It is incumbent upon the Issue: Whether or not such formation of secret marshals is violative
complainant to reduce the evidence into affidavits. This is a against the due process which is vested by the constitution.
requirement not only of Rule II, S4 (a) of the Rules of Procedure but
also of due process in an adversary proceedings. Held: According to Relova J.. there is nothing wrong in the creation and
deployment of special operation teams to counter the resurgence of
Alonte v. Savellano Jr. criminality. The problem is the allege use of violence in the
287 SCRA 245 (1998) implementation of objectives of the special squads. Due process of law
is requires that the accused must be heard under the orderly process of
Alonte, then the Mayor of Binan, Laguna was charged with rape law. Wherefore the respondents are directed to exercise strict
together with Concepcion, who was alleged to have brought the supervision and control over these special operation teams, ensuring
complainant, Punongbayan, a minor, to Alonte. The case was originally that these teams will not use unnecessary force and will comply strictly
raffled to a branch of RTC in Binan, but complainant sought a change in with the law. If should death or injury result, responded is hereby
venue. While awaiting action on her request, the complainant, assisted immediately report matter to their superior officers and the National
by her parents and counsel, executed an affidavit of desistance, Police Commission (NAPOLCOM) for investigation and proper action
adverting to the delay and inconvenience to her and to her family. The
petitioners then moved for the dismissal of the case which was Informed Participation in the Proceedings
opposed by the public prosecutor. Shortly thereafter, the SC approved
the request of change in venue and was raffled to the respondent
For the purpose of satisfying the due process requirements, it is
judge. The complainant reiterated before the trial court her decision to
abide by her affidavit of desistance. In the meantime, the judge found also necessary that the accused have an understanding of what
probable cause for the issuance of the warrants of arrests of the the proceeding is all about. Accordingly, he would have to be
petitioners. On arraignment, both pleaded not guilty. According to the assisted and informed in such a language and in a manner that
judge, both parties agreed to proceed while Alonte said that the judge he can understand and comprehend what is being conveyed.
allowed the prosecution to proceed to present evidence relative only to This means that he has to be given, for instance, a counsel with
the question of voluntariness and validity of the affidavit of desistance. whom he could communicate and a qualified and competent
The parents and the complainant affirmed their decision to desist. The interpreter to assist him. Also, an accused should be proceeded
prosecutor then manifested that in light of the decision of the
against with a particular understanding of his state or condition
complainant and her parents not to pursue the case, the State had no
further evidence against the accused to prove the guilty of the accused, of mind. One exhibiting signs of unstable mental condition
and moved for the dismissal of the case against both petitioner. Then, should not be treated like any other sane person if the
the trial judge promulgated his decision, convicting both accused. They guarantee of due process us to be accorded substance and
filed the instant petition, assailing the decision, for among others, being meaning
violative of the guarantee of due process.
People v. Crisologo
HELD: The Court must admit that it is puzzled by the somewhat strange 150 SCRA 653(1987)
way the case has proceeded below. Per Judge Savellano after the
waiver by the parties of the pre-trial stage: The trial of the case did Facts: Appellant, a deaf – mute, was charged with robbery with
proceed on the merits but that—“the   two   accused   did   not   present   homicide committed on May 1, 1976. The accused was allegedly
countervailing evidence during the trail. They did not take the witness informed of the charge against him through sign language by Special
stand to refute or deny under oath the truth of the contents of the Policeman Alejandro Munoz, a childhood acquaintance. Subsequently
private   complainant’s   aforementioned   affidavit   which   she   expressed   Munoz entered a plea of guilty on behalf of the accused. The pleas was
affirmed and confirmed in court, but instead, thru their respective disregarded upon the objection of the counsel and arraignment was
lawyers, they rested and submitted the case for decision merely on the rescheduled until such time the court could avail of the service of an
basis   of   the   private   complainant’s   so   called   “desistance”   which,   to   expert in the sign language from the school of the deaf and dumb. In
them, was sufficient enough for their purposes. They left everything to 1979 through another presiding judge the court reset arraignment as

101 | P LATON
no expert in sign language was available. In 1982 after five years under Courts are the proper fora before which criminal proceedings
another presiding judge directed the school of the deaf and dumb Bago are supposed to be heard. It is before the courts that certain
Gallera, Talmo District, Davao City be availed of to the enable the minimum guarantees are assured, not the least of which is the
accused to intelligently express his understanding of the plea of guilty
presence of an impartial magistrate steeped in the delicate task
or not guilty. Finally in February 10, 1986 without the service of the
expert in sign language, accused was found guilty and sentence to die in
of determining guilt established by proof beyond reasonable
electrocution. doubt. Administrative bodies could not qualify for such task,
especially so as they only require substantial evidence for the
Issue: Whether or not rendering the decision without the aid of an purpose of supporting their determinations. As for military
expert in sign language deprives the accused of due process of law. tribunals, they may be better fitted for and attuned to the
disciplinary needs within the military structure but not
Held: Absence of an interpreter in sign language deprived the accused necessarily for civilians outside. However, during the time when
of the full and fair trial and reasonable opportunity to defend himself.
martial law was in effect, and when former President Marcos
The absence of a qualified interpreter in sign language and of any other
means, whether in writing of otherwise, to inform the accused of the
was still in power, the Court did not find anything wrong with
charges against him denied the accused his fundamental right to due trial of civilians by military tribunals, only to shed off that
process of a law. Aside from the unfair setting of circumstance in which doctrine the moment Marcos was cast away in Hawaii
the accused was convicted, insufficiency of evidence to warrant a
finding guilty beyond reasonable doubt also led the court to set aside Scoty’s  Department  Store  v.  Micaller
the conviction. 99Phiul.762(1956)

People v. Estrada Facts:   Nena   Micaller   a   salesgirl   of   Scoty’s   Department   Store   in   Scolta  
333 SCRA 699(2000) Manila organized a union among her co – employees which was later
affiliated to NLU. NLU sent petition to the store with several demands,
Facts: On December 27, 1994 a confirmation was being held at the St. the petitioners subjected Micaller and her com – employees to
John’s  Cathedral,  Dagupan  City,  when  the  appellant  Roberto  Estrada sat questioning with respect to their union membership activities. Micaller
on   the   Bishop’s   chair.   Some   churchgoers   summoned   Mararac,   the   hired temporary employees equal in number to the old. Latter was
security guard of the Cathedral to request the appellant to vacate the charged with for alleged threats and slander before being dismissed for
Bishop’s  Chair.  Mararac  tapped  appellant’s  hand  with  a  night  stick  but   insulting the owner of the store and for talking to the girls inside the
the appellant did not badge. When Mararac attmpt to tapped the store during business hours. Micaller filed charges of unfair labor
appellant’s  hands  again,   the   appellant   drew   a  knife   from   his  back   and   practice with the Court of Industrial Relations (CIR), alleging that she
stubbed the gurad. After the commotion appellant got up, went to the was dismissed because of her union activities. CIR found the petitioners
mocirophone and shouted in the local dialect that no one can beat him guilty of unfair labor practice, and ordered them to pay a fine of
there. Mararac expired a few minutes after arrival in the hospital. 100php. In their appeal, the petitioners raise as one of the issues
Appellant was charged with murder. whether they can be legally punished by a fine of 100php.

Counsel for the appellant filed an Urgent Motion to suspend Issue: Whether or not petitioners can be legally punished by a fine of
arraignment and to commit the accused to the Psychiatric Ward at 100php
Baguio General Hospital because he was suffering mental illness.
Motion was opposed for the reason that the appellant answered the Held: Court reach the conclusion that word cannot refer to the Court of
questions intelligently. Jail warden of Dagupan City wrote the trial the Industrial Relations for to give that meaning would be a violation of
judge   informing   him   of   the   accused   appellant’s   unsual   behaviour   and   the safeguards guaranteed to every accused by our constitution,
requesting that he be examined at the hospital to determine whether referring  to  those  which  postulate  that  “no  one  shall  be  held  to  answer  
he should remain in jail or to other institution, judge ignored the letter. for   a   criminal   offense   without   due   process   of   law.”     Procedure   laid  
One year later accused – appellant  counsel  filed  a  “  Motion  to  Confine   down to be observed by the CIR in dealing with unfair labor practice
Accused  for  Physical  ,  Mental  and  Psychiatric  Examination.”  Attached  to   cases negates those constitutional guarantees to the accused. Rules
this motion was a second letter by the new Jail Warden of Dagupan City and evidence prevailing to the court of the law or equity shall not be
accompanied by letter complaint of the Bukang Liwayway Association controlling and it is the spirit and intention of the act the court and its
City Jail. The judge still ignored the motion. The trial Court instead members shall use all reasonable means to ascertain the facts and
found the appellant guilty of murder and sentenced hin to death. without the regards to technicalities of law and procedure. Court shall
not be bound solely by evidence presented during the hearing but may
Issue: Whether or not the appellant was deprived of a fair trial, and a avail itself of all other means such as ocular inspection and questioning
violation to the basic requirements to due process. of well – informed persons.

Held: The trial court took solely upon itself to determine the sanity of The power to impose penalties provided for section 25 of Republic Act
the accused – appellant. The trial judge is not a psychiatrist or No. 875 (the fine of 100php) is lodged in ordinary courts and not in CIR.
psychologist   or   some   of   the   person’s   mental   health.   By   depriving   the   The decision appealed from is modified by eliminating the fine of
appellant of a mental examination, the trial court effectively deprived 100php in all other respects
appellant   for   a   fair   trial.   The   trial   court’s  negligence   was  a  VIOLATION  
OF THE BASIC REQUIREMNTS OF THE DUE PROCESS. There is a need to OLAGUER V. MILITARY COMMISSION NO. 34
reconstitute the records in accordance with the procedure outlined in 150 SCRA 144 (1987)
the law, in order to dispel and doubt as to the integrity of the records
that were lost and recovered in the office of the Solicitor General. Every Habeas Corpus
aspect of the right to due process must be afforded the accused – In 1979, Olaguer and some others were detained by military personnel
appellant, and this includes the right to examine and assail the veracity and they were placed in Camp Bagong Diwa. Logauer and his group are
of every piece of evidence contained in the records. Sentence to death all civilians. They were charged with (1) unlawful possession of
is vacated and the case is remanded to the court a quo for the conduct explosives and incendiary devices; (2) conspiracy to assassinate
proper mental examination on the accused – appellant. President and Mrs. Marcos; (3) conspiracy to assassinate cabinet
members Juan Ponce Enrile, Francisco Tatad and Vicente Paterno; (4)
Administrative and Military Tribunals conspiracy to assassinate Messrs. Arturo Tangco, Jose Roño and Onofre
Corpus; (5) arson of nine buildings; (6) attempted murder of Messrs.
Leonardo Perez, Teodoro Valencia and Generals Romeo Espino and
Fabian Ver; and (7) conspiracy and proposal to commit rebellion, and

102 | P LATON
inciting to rebellion. On August 19, 1980, the petitioners went to the SC The fact is that both majority and minority reports were one in
and filed the instant Petition for prohibition and habeas corpus. rejecting the military version stating that "the evidence shows to the
contrary that Rolando Galman had no subversive affiliations. Only the
ISSUE: Whether or not the petition for habeas corpus be granted. soldiers in the staircase with Sen. Aquino could have shot him; that
Ninoy's assassination was the product of a military conspiracy, not a
HELD: The petition for habeas corpus has become moot and academic communist plot. Only difference between the two reports is that the
because by the time the case reached the SC Olaguer and his majority report found all the twenty-six private respondents above-
companions were already   released   from   military   confinement.   “When   named in the title of the case involved in the military conspiracy; "
the release of the persons in whose behalf the application for a writ of while the chairman's minority report would exclude nineteen of them.
habeas corpus was filed is effected, the Petition for the issuance of the
writ becomes moot and academic. 18 Inasmuch as the herein Then Pres. Marcos stated that evidence shows that Galman was the
petitioners have been released from their confinement in military killer.
detention centers, the instant Petitions for the issuance of a writ of
habeas corpus should be dismissed for having become moot and Petitioners pray for issuance of a TRO enjoining respondent court from
academic.”   But   the   military   court   created   to   try   the   case   of   Olaguer rendering a decision in the two criminal cases before it, the Court
(and the decision it rendered) still continues to subsist. resolved by nine-to-two votes 11 to issue the restraining order prayed
for. The Court also granted petitioners a five-day period to file a reply
ISSUE2: The issue is then shifted to: Whether or not a military tribunal to respondents' separate comments and respondent Tanodbayan a
has the jurisdiction to try civilians while the civil courts are open and three-day period to submit a copy of his 84-page memorandum for the
functioning. prosecution.

HELD: The SC nullified for lack of jurisdiction all decisions rendered by But ten days later, the Court by the same nine-to-two-vote ratio in
the military courts or tribunals during the period of martial law in all reverse, resolved to dismiss the petition and to lift the TRO issued ten
cases involving civilian defendants. A military commission or tribunal days earlier enjoining the Sandiganbayan from rendering its decision.
cannot try and exercise jurisdiction, even during the period of martial The same Court majority denied petitioners' motion for a new 5-day
law, over civilians for offenses allegedly committed by them as long as period counted from receipt of respondent Tanodbayan's
the civil courts are open and functioning, and that any judgment memorandum for the prosecution (which apparently was not served on
rendered by such body relating to a civilian is null and void for lack of them).
jurisdiction on the part of the military tribunal concerned.
Thus, petitioners filed a motion for reconsideration, alleging that the
Res inter alios judicatae nullum aliis praejudicium faciunt. dismissal did not indicate the legal ground for such action and urging
“Matters  adjudged  in  a  cause  do  not  prejudice  those  who  were   that the case be set for a full hearing on the merits that the people are
entitled to due process.
not  parties  to  it.”  (54  C.J.  719)  It  is  a  cardinal  rule  of  procedure  
that   a   court’s   judgment   of   order   in   a   case shall not adversely However, respondent Sandiganbayan issued its decision acquitting all
24
affect persons who were not parties to the self same case the accused of the crime charged, declaring them innocent and totally
absolving them of any civil liability. Respondents submitted that with
Due Process for the Prosecution and the Offended Party the Sandiganbayan's verdict of acquittal, the instant case had become
moot and academic. Thereafter, same Court majority denied
While the language of the due process provision seems to be a petitioners' motion for reconsideration for lack of merit.
guarantee only for the accused, the assurance of fair play
Hence, petitioners filed their motion to admit their second motion for
actually runs both ways. There could be no due process if only
reconsideration alleging that respondents committed serious
the accused is treated fairly for, in the ultimate analysis, that irregularities constituting mistrial and resulting in miscarriage of justice
would hardly be considered fair or just at all and gross violation of the constitutional rights of the petitioners and
the sovereign people of the Philippines to due process of law.
“A  day  in  court  is  the  touchstone  of  the  right  to  due  process  in  
criminal justice. It is an aspect of the duty of the government to Issues:
follow a fair process of decision-making when it acts to deprive (1) Whether or not petitioner was deprived of his rights as an accused.
a person of his liberty. But just as an accused is accorded this (2) Whether or not there was a violation of the double jeopardy clause.
constitutional protection, so is the State entitled to due process
in criminal prosecutions. It must similarly be given the chance Held: Petitioners' second motion for reconsideration is granted and
to  present  its  evidence  in  support  of  a  charge” ordering a re-trial of the said cases which should be conducted with
deliberate dispatch and with careful regard for the requirements of due
GALMAN V. SANDIGANBAYAN process.
144 SCRA 43 (1986)
Deputy Tanodbayan Manuel Herrera (made his expose 15 months later
Facts: Assassination of former Senator Benigno "Ninoy" Aquino, Jr. He when former Pres. was no longer around) affirmed the allegations in
was killed from his plane that had just landed at the Manila the second motion for reconsideration that he revealed that the
International Airport. His brain was smashed by a bullet fired point- Sandiganbayan Justices and Tanodbayan prosecutors were ordered by
blank into the back of his head by an assassin. The military investigators Marcos to whitewash the Aquino-Galman murder case. Malacañang
reported within a span of three hours that the man who shot Aquino wanted dismissal to the extent that a prepared resolution was sent to
(whose identity was then supposed to be unknown and was revealed the Investigating Panel. Malacañang Conference planned a scenario of
only days later as Rolando Galman) was a communist-hired gunman, trial where the former President ordered then that the resolution be
and that the military escorts gunned him down in turn. revised by categorizing the participation of each respondent; decided
that the presiding justice, Justice Pamaran, (First Division) would
President was constrained to create a Fact Finding Board to investigate personally handle the trial. A conference was held in an inner room of
due to large masses of people who joined in the ten-day period of the Palace. Only the First Lady and Presidential Legal Assistant Justice
national mourning yearning for the truth, justice and freedom. Lazaro were with the President. The conferees were told to take the
back door in going to the room where the meeting was held,
presumably to escape notice by the visitors in the reception hall waiting
to see the President. During the conference, and after an agreement
24 See Pages 285-288
103 | P LATON
was reached, Pres. Marcos told them 'Okay, mag moro-moro na lamang Sandiganbayan should not have precipitately issued its decision of total
kayo;' and that on their way out of the room Pres. Marcos expressed his absolution of all the accused pending the final action of this Court. All
thanks to the group and uttered 'I know how to reciprocate'. of the acts of the respondent judge manifest grave abuse of discretion
on his part amounting to lack of jurisdiction which substantively
The Court then said that the then President (code-named Olympus) had prejudiced the petitioner.
stage-managed in and from Malacañang Palace "a scripted and
predetermined manner of handling and disposing of the Aquino- With the declaration of nullity of the proceedings, the cases must now
Galman murder case;" and that "the prosecution in the Aquino-Galman be tried before an impartial court with an unbiased prosecutor.
case and the Justices who tried and decided the same acted under the Respondents accused must now face trial for the crimes charged
compulsion of some pressure which proved to be beyond their capacity against them before an impartial court with an unbiased prosecutor
to resist. Also predetermined the final outcome of the case" of total with all due process.
absolution of the twenty-six respondents-accused of all criminal and
civil liability. Pres. Marcos came up with a public statement aired over The function of the appointing authority with the mandate of the
television that Senator Aquino was killed not by his military escorts, but people, under our system of government, is to fill the public posts.
by a communist hired gun. It was, therefore, not a source of wonder Justices and judges must ever realize that they have no constituency,
that President Marcos would want the case disposed of in a manner serve no majority nor minority but serve only the public interest as they
consistent with his announced theory thereof which, at the same time, see it in accordance with their oath of office, guided only the
would clear his name and his administration of any suspected guilty Constitution and their own conscience and honor.
participation in the assassination. such a procedure would be a better
arrangement because, if the accused are charged in court and Demurrer to Evidence, Acquittal and Civil Liability
subsequently acquitted, they may claim the benefit of the doctrine of
double jeopardy and thereby avoid another prosecution if some other
When a criminal case is filed, it generally carries with it the
witnesses shall appear when President Marcos is no longer in office.
institution of the civil liability arising from the offense. Thus,
More so was there suppression of vital evidence and harassment of when the accused is found guilty, he is also adjudged civilly
witnesses. The disappearance of witnesses two weeks after Ninoy's liable. It may happen, however, the he may get exonerated
assassination. According to J. Herrera, "nobody was looking for these from the criminal liability due to reasonable doubt and yet may
persons because they said Marcos was in power. The assignment of the still be found liable for damages. Ordinarily, this would mean
case to Presiding Justice Pamaran; no evidence at all that the that the judgment comes after both the prosecution and the
assignment was indeed by virtue of a regular raffle, except the defense would have presented their respective evidence
uncorroborated testimony of Justice Pamaran himself. The custody of
the accused and their confinement in a military camp, instead of in a
civilian jail. The monitoring of proceedings and developments from In Salazar v. People, the accused was acquitted in a case for
Malacañang and by Malacañang personnel. The partiality of estafa (swindling) arising from her endorsement of a check
Sandiganbayan betrayed by its decision: That President Marcos had which bounced. She was, however, order to pay the amount
wanted all of the twenty-six accused to be acquitted may not be represented by the check. She questioned that part of the
denied. In rendering its decision, the Sandiganbayan overdid itself in judgment, claiming that she was denied due process since she
favoring the presidential directive. Its bias and partiality in favor of the was not given an opportunity to adduce evidence to prove that
accused was clearly obvious. The evidence presented by the she was not civilly liable. The Supreme Court agreed:
prosecution was totally ignored and disregarded.

The record shows that the then President misused the overwhelming If the demurrer is granted and the accused is acquitted by the
resources of the government and his authoritarian powers to corrupt court, the accused has the right to adduce evidence on the civil
and make a mockery of the judicial process in the Aquino-Galman aspect of the case unless the court also declares that the act or
murder cases. "This is the evil of one-man rule at its very worst." Our omission from which the civil liability may arise did not exist. If
Penal Code penalizes "any executive officer who shall address any order the trial court issues an order or renders judgment not only
or suggestion to any judicial authority with respect to any case or granting the demurrer to evidence of the accused and
business coming within the exclusive jurisdiction of the courts of acquitting him but also on the civil liability of the accused to the
justice."
private offended party, said judgment on the civil aspect of the
Impartial court is the very essence of due process of law. This criminal case would be a nullity for the reason that the constitutional
collusion as to the handling and treatment of the cases by public right of the accused to due process is thereby violated
respondents at the secret Malacañang conference (and revealed only
after fifteen months by Justice Manuel Herrera) completely disqualified Right to Bail
respondent Sandiganbayan and voided ab initio its verdict. The courts
would have no reason to exist if they were allowed to be used as mere Once a person has been charged he may find himself already
tools of injustice, deception and duplicity to subvert and suppress the the object of a warrant of arrest with the consequent threat of
truth. More so, in the case at bar where the people and the world are
momentary loss of his liberty. To secure his freedom, he would
entitled to know the truth, and the integrity of our judicial system is at
stake. have to post bail or otherwise obtain the same through
recognizance. Bail is the security given for the release of a
There was no double jeopardy. Courts' Resolution of acquittal was a person in custody of the law. It may be in the form of corporate
void judgment for having been issued without jurisdiction. No double surety, property bond, cash deposit, or recognizance
jeopardy attaches, therefore. A void judgment is, in legal effect, no
judgment at all. By it no rights are divested. It neither binds nor bars A recognizance is an obligation of record entered into before
anyone. All acts and all claims flowing out of it are void. some court or magistrate duly authorized to take it, with the
condition to do some particular act, the most usual condition in
Motion to Disqualify/Inhibit should have been resolved ahead. In this
case, petitioners' motion for reconsideration of the abrupt dismissal of criminal cases being the appearance of the accused for
their petition and lifting of the TRO enjoining the Sandiganbayan from trial...[A]   ‘contract   between   the   sureties   and   state   for   the  
rendering its decision had been taken cognizance of by the Court which production  of  the  principal  at  the  required  time’
had required the respondents', including the Sandiganbayan's,
comments. Although no restraining order was issued anew, respondent General Considerations

104 | P LATON
Obosa was charges with 2 counts of murder, a capital offense for the
In order for one to be entitled to apply for bail, he must already ambush  slaying  of  Sec  of  Interior  and  Local  Gov’t  Jaime  Ferrer  and  his  
be in the custody of the law or otherwise deprived of his driver Jesus Calderon. On the day the trial court promulgated its
decision, Obosa manifested his intention to appeal and ask the court to
liberty. If he is not, then there could hardly be any basis for the
allow him to post bail.
court in granting him liberty as the court would not yet have Is petitioner entitled to bail as a matter of right?
acquired any jurisdiction over him. Jurisdiction acquired may be
by reason of an arrest, or simply by surrendering himself to the Held: Before conviction, every person is bailable except if charged with
court capital offenses when the evidence of guilt is strong.

One who is detained though not yet formally charged may We rule that bail cannot be granted as a matter of right even after an
apply for bail; otherwise, to afford bail to one already charged accused, who is charged with the capital offense, appeals his convition
for a non-capital crime. In ruling against bail, the lower court found that
and yet deny it to one who has not yet been determined by the
treachery attended the killing thereby justifying its action. Hereby
authorities whether to charge or not would result in an petition is denied.
anomalous and absurd situation. On the other hand, the
accused need not be arraigned first before being granted bail Paderanga v CA
247 SCRA 741(1995)
Bail is generally available only in criminal proceedings.
Accordingly, it may not be availed of in administrative Petitioner was belatedly charged in an amended information as co-
proceedings, like deportation or extradition proceedings conspirator in a crime of multiple murder in Cagayan de oro for the
killing of bucag family of which petitioner was mayor at that time.
The Court also held that the right to bail has traditionally not Felizardo was implicated in the crime. He hired petitioner as counsel
but later during preliminary investigation, he implicated petitioner as
been recognized and is not available to those in the armed
the mastermind of the massacre. Before arrest warrant could be
forces given the unique structure of the military granted to him, his counsel filed a motion to bail. RTC granted bail.CA
reversed.
Right to Bail in General
The main purpose of bail is to relieve an accused from the rigors of
GR: A person charged with a crime has the right to bail imprisonment until hi conviction and yet secure his appearance at the
trial. The same cannot be posted before custody over him has been
XPN: If he is charged with a crime punishable by reclusion acquired either by lawful arrest or voluntary surrender. The rationale of
this rule is that it discourages and prevents resort to the former
perpetua or life imprisonment, or death, and the evidence
pernicious practice whereby an accused could just send another in his
against him is really strong stead to post his bail. As a paramount requisite, only those who have
been arrested detained or deprived of liberty can post bail. This
In regard to appeals from convictions by the Regional Trial however is subject to the limitation that the applicant is in the custody
Court respecting offenses not punishable by death, reclusion of the law. He was constructively in the custody of the law even if he
perpetua or life imprisonment, bail may also be availed of, but was not physically arrested because he was hospitalized.
then not anymore as a matter of right. It becomes discretionary
with the court Section Section 13, Article III of the Constitution lays down the rule that
before conviction, all indictees shall be allowed bail, except only those
charged with offenses punishable by reclusion perpetua when the
PEOPLE V. DONATO
evidence of guilt is strong. In pursuance thereof, Section 4 of Rule 114,
198 SCRA 130 (1991)
as amended, now provides that all persons in custody shall, before
conviction by a regional trial court of an offense not punishable by
Private respondent Salas, chairman of the communist party of the
death, reclusion perpetua or life imprisonment, be admitted to bail as a
Philippines and head of the military arm of the NPA, with co-accused
matter of right. The right to bail, which may be waived considering its
was charged of rebellion. The information was filed and they were in
personal nature 21 and which, to repeat, arises from the time one is
military custody. Private respondent filed a petition for bail.
placed in the custody of the law, springs from the presumption of
innocence accorded every accused upon whom should not be inflicted
Whether or not the right to bail may, under certain circumstance be
incarceration at the outset since after trial he would be entitled to
denied to a person with an otherwise bailable offense, and whether
acquittal, unless his guilt be established beyond reasonable doubt.
such right may be waived.
Thus, the general rule is that prior to conviction by the regional trial
Held: Bail is a matter of right even in cases of capital offenses, unless
court of a criminal offense, an accused is entitled to be released on bail
the proof of guilt is evident or the presumption thereof is great. The
as a matter of right, the present exceptions thereto being the instances
prosecution does not have the right to oppose the bail where bail is a
where the accused is charged with a capital offense or an offense
matter of right. However, in cases where the grant of bail is
punishable by reclusion perpetua or life imprisonment 23 and the
discretionary, due process requires that the prosecution must be
evidence of guilt is strong. Under said general rule, upon proper
allowed to present evidence, within a reasonable time, the evidence
application for admission to bail, the court having custody of the
presented should resolve the motion for bail.
accused should, as a matter of course, grant the same after a hearing
conducted to specifically determine the conditions of the bail in
Private respondent has waived his right to bail when he entered into an
accordance with Section 6 (now, Section 2) of Rule 114. On the other
agreement that he will remain in legal custody and face trial before the
hand, as the grant of bail becomes a matter of judicial discretion on the
court having custody of his person. Custody has been held to mean
part of the court under the exceptions to the rule, a hearing,
nothing less than actual imprisonment. The right to bail is another of
mandatory in nature and which should be summary or otherwise in the
the constitutional rights which can be waived. It is a right which is
discretion of the court, is required with the participation of both the
personal to the accused.
defense and a duly notified representative of the prosecution, this time
to ascertain whether or not the evidence of guilt is strong for the
OBOSA v. CA provisional liberty of the applicant. Of course, the burden of proof is on
266 SCRA 281 (1997) the prosecution to show that the evidence meets the required
quantum.

105 | P LATON
SC reinstated bail. In requiring that petitioner be first arraigned before he could be
granted bail, the trial court apprehended that if petitioner were
Bail Hearing released on bail he could, by being absent, prevent his early
arraignment and thereby delay his trial until the complainants got tired
and lost interest in their cases. Hence, to ensure his presence at the
Part of the requirement in the grant or denial of bail, especially
arraignment, approval of petitioner's bail bonds should be deferred
in non-bailable cases, is the necessity for a hearing. The hearing until he could be arraigned. After that, even if petitioner does not
is summary in nature, meaning such brief and speedy method appear, trial can proceed as long as he is notified of the date of hearing
of receiving and considering the evidence of guilt as is and his failure to appear is unjustified, since under Art. III, §14(2) of the
practicable and consistent with the purpose of the hearing Constitution, trial in absentia is authorized. This seems to be the theory
which is merely to determine the weight of evidence for of the trial court in its May 16, 1997 order conditioning the grant of bail
purposes of bail to petitioner on his arraignment.

This theory is mistaken. In the first place, as the trial court itself
It  has  also  been  pointed  out  that:  “A   bail  application  does  not  
acknowledged, in cases where it is authorized, bail should be granted
only involve he right of the accused to temporary liberty, but before arraignment, otherwise the accused may be precluded from
likewise the right of the State to protect the people and the filing a motion to quash. For if the information is quashed and the case
peace of the community from dangerous elements. These two is dismissed, there would then be no need for the arraignment of the
rights must be balanced by a magistrate in the scale of justice, accused. In the second place, the trial court could ensure the presence
hence, the necessity for hearing to guide his exercise of of petitioner at the arraignment precisely by granting bail and ordering
discretion” his presence at any stage of the proceedings, such as arraignment.
Under Rule 114, §2(b) of the Rules on Criminal Procedure, one of the
conditions of bail is that "the accused shall appear before the proper
Further, arraignment is not a prerequisite to the conduct of
court whenever so required by the court or these Rules," while under
hearings on a petition for bail. A person is allowed to petition Rule 116, §1(b) the presence of the accused at the arraignment is
for bail as soon as he is deprived of his liberty by virtue of his required.
arrest or voluntary surrender--he need not await his
arraignment On the other hand, to condition the grant of bail to an accused on his
arraignment would be to place him in a position where he has to
RE RELEASE BY JUDGE MANUEL T. MURO OF AN ACCUSED IN A NON- choose between (1) filing a motion to quash and thus delay his release
BAILABLE OFFENSE on bail because until his motion to quash can be resolved, his
367 SCRA 285 arraignment cannot be held, and (2) foregoing the filing of a motion to
quash so that he can be arraigned at once and thereafter be released
Yu yuk lai is one of the accused for sale and delivery of 3 kg of shabu, on bail. These scenarios certainly undermine the accused's
was reasrested bby PAOCTF while playing baccarat without jail guards constitutional right not to be put on trial except upon valid complaint
at the holiday inn casino. At that time he was supposed to be detained or information sufficient to charge him with a crime and his right to
at the manila city jail since his bail was denied. It was published in bail.8
Philippine star that Yu yuk lai has obtained an order signed by muro
allowing her hospital confinement for medical and humanitarian It is the condition in the May 16, 1997 order of the trial court that
reasons. Muro was directed to submit pertinent documents showing "approval of the bail bonds shall be made only after arraignment,"
that the order was regularly issued. He complied. He was suspended which the Court of Appeals should instead have declared void. The
and investigated. condition imposed in the trial court's order of May 16, 1997 that the
accused cannot waive his appearance at the trial but that he must be
Trial judge may rely on statements of someone knowledgeable in the present at the hearings of the case is valid and is in accordance with
subject, it should not be to the extent that such reliance would amount Rule 114. For another condition of bail under Rule 114, §2(c) is that
to a surrender of his authority to decide. "The failure of the accused to appear at the trial without justification
despite due notice to him or his bondsman shall be deemed an express
As shown by the records, respondent judge was guilty of gross waiver of his right to be present on the date specified in the notice. In
misconduct constituting violations of the code of judicial conduct for such case, trial shall proceed in absentia."
being utterly inefficient and for manifesting partiality. Despite request
that he be confined in PGH, judge preferred Manila Doctors on the Petitioner could delay the proceedings by absenting himself from the
ground   that   it   is   yu   yuk’s   choice.   He   subjected   the   length   of   medical   arraignment. But once he is arraigned, trial could proceed even in his
confinement to the will of her physicians. And he did not make incisive absence. So it thought that to ensure petitioner's presence at the
inquiry to ascertain the true state of facts despite the opposition by arraignment, petitioner should be denied bail in the meantime. The fly
another prosecutor. He threw all his cautions in the wind in favor of yu in the ointment, however, is that such court strategy violates
yuk petitioner's constitutional rights
Judge was dismissed. The Court of Appeals declared conditions (a) and (b) invalid but
declined to pass upon the validity of condition (d) on the ground that
the issue had become moot and academic. Although this condition is
Lavides v Court of appeals
invalid, it does not follow that the arraignment of petitioner on May 23,
324 SCRA 321 (2000)
1997 was also invalid. Contrary to petitioner's contention, the
arraignment did not emanate from the invalid condition that "approval
Petitioner was arrested for child abuse. His arrest was made without
of the bail bonds shall be made only after the arraignment." Even
warrant as a result of entrapment conducted by police. He was caught
without such a condition, the arraignment of petitioner could not be
with 16-year old complainant. Within a month, nine more informations
omitted. In sum, although the condition for the grant of bail to
were filed against him by complainant and 3 others who had been
petitioner is invalid, his arraignment and the subsequent proceedings
exploited in prostitution given money as payment for sexual
against him are valid.
intercourse. He was entitled to bail with conditions that 1. He shall not
be entitled to waiver of appearance and must be present at all hearings
WHEREFORE, conditions are valid, with the exception of condition (d)
2. If he did not, his bail shal be automatically forfeited, warrants shall
(making arraignment a prerequisite to the grant of bail to petitioner),
be immediately issued and cases will proceed trial in absentia 3. hold
which is hereby declared void.
departure-order stands 4. Approval of bail shall be done after
arraignment for courts to acquire jurisdiction over him.

106 | P LATON
In Serapio v. Sandiganbayan, the Court held that no
inconsistency exists between application of an accused for bail Facts: In accordance to the existing RP-US Extradition Treaty, the US
and his filing of a motion to quash.   “These   two   reliefs   have   Government sent to the RP documents requesting the extradition of
Mark B. Jimenez a.k.a. Mario Batacan Crespo. SFA transmitted the
objectives which are not necessarily antithetical to each other.
documents to the SOJ for appropriate action. Then the DOJ filed with
Certainly, the right of an accused to seek provisional liberty the RTC the appropriate petition for Extradition in relation to charges
when charged with an offense not punishable by death, involving conspiracy to defraud the US, tax evasion, wire fraud, false
reclusion perpetua or life imprisonment, or when charged with statements, and illegal campaign contributions. To prevent the flight of
an offense punishable by such penalties but after due hearing, Jimenez, the petition prayed   for   an   order   for   his   ‘’immediate   arrest’.  
evidence of his guilt is found not to be strong, does not Nevertheless, prior to such petition, the Jimenez had already filed and
preclude his right to assail the validity of the Information urgent manifestation or ex-parte motion which prayed that the
charging him with such offense application for his arrest warrant be set for hearing. RTC granted the
motion and  eventually   issued  a  warrant   for   Jimenez’s  arrest   and  fixed  
bail at P1 Min cash. After he surrendered his passport and posted the
Reasonableness of Bail required cash bond, Jimenez was granted provisional liberty by the
court.
The directive not to require excessive bail could thus be read to
mean that bail should be just and equitable and not the Issues: WON prospective extraditees are entitled to notice and hearing
product of capricious or whimsical exercise of discretion before warrants for their arrest can be issued
WON they are entitled to the right to bail and provisional liberty while
DE LA CAMARA V ENAGE the extradition proceeding s are pending
41 SCRA 1
Held: Conviction is only applicable when a person has been arrested
Petitioner, Ricardo de la Camara, a town mayor in Misamis Oriental, and detained for violation of Phil. criminal laws. It is not applicable to
was arrested and detained Magsaysay, Misamis Oriental was arrested extradition proceedings because extradition courts do not render
and detained at the provincial jail for his alleged participation in the judgments of conviction or acquittal. The constitutional provision on
killing of 14 and wounding off 12 other laborers.(multiple frustrated bail is also not applicable to extradition proceeding s because
murder and multiple murder) petitioner filed for bail premised on the presumption of innocence is not an issue therein. Extradition
assertion that there was no evidence to link him with such incident. proceedings are separate and distinct from the trial for the offense for
Respondent judge granted bail admitting that the prosecution failed to which Jimenez is charged. He should apply for bail before the courts
prove that petitioner would flee even if he had an opportunity but fixed trying the criminal cases against him, not before the extradition court.
1.195 M +. Exceptions to the No Bail Rule: Upon a clear and convincing showing (1)
that once granted bail, the applicant will not be a flight risk or a danger
The bail of petitioner is repugnant to the constitutional mandate to the community, and (2) that there exist special, humanitarian and
prohibiting excessive bail. The case is moot and academic since compelling circumstances including those cited by the highest court in
petitioner escaped. However for the guidance of the lower courts, here the requesting state when it grants provisional liberty in extradition
are the doctrines that should be observed in fixing the amount of bail: cases therein.
1. Before conviction, every person is bailable except if charged with
capital offenses when the evidence of guilt is strong. Such right flows Regarding the alleged disenfranchisement that Jimenez contends, the
from the presumption of innocence. Court is not persuaded since it was already of public knowledge that
2. Where the right of bail exist, it should not be rendered nugatory by the US was requesting for his extradition prior to his running for and
requiring a sum that is excessive. The sole permissible function of winning of a congressional seat in Manila. His constituents were or
money   bail   is   to   assure   the   accused’s   presence   in   trial   and   bail   set   at   should have been prepared for the consequences of his extradition
higher figure than amount reasonably calculated to fulfil this purpose is case. Also, the disenfranchisement argument was already debunked by
excessive. the SC in People v. Jalosjos.
3. The guidelines in fixing bail:
1. Ability of the accused to give bail In response to the contention of the respondent about anticipated
2. Nature of offense delay, the court says that extradition proceedings are summary in
3. Penalty for offense nature and they are only resorted to determine WON the petition and
4. Character and reputation of accused its annexes conform to the Extradition treaty law.
5. Character and strength of evidence
6. Probability of appearing in trial Although the SC admits that respondent has not actually fled during the
7. Forfeiture of other bonds preliminary stages of the request for his extradition, it does not mean
8. WON the accused was fugitive when arrested that he will not flee as the process moves forward to its final decision.
9. If accused is under bond for other cases Petition is GRANTED; the RTC order is declared NULL and VOID, the
challenge order which granted bail to Jimenez, is SET ASIDE, and the
bail posted by him is CANCELLED.
Extradition and Bail
Presumption of Innocence
As noted earlier, the right to bail has been considered as
referring only to criminal proceedings and not extending to
The starting point in any criminal prosecution is the
administrative cases. In extradition proceedings, the Court said
presumption of innocence. This presumption stands as a
that the right to avail of bail would be only in exceptional
fundamental principle of both constitutional and criminal law,
circumstances. Thus, after a potential extradite has been
imposing a rule of evidence, a degree of proof that demands no
arrested or placed under the custody of the law, he may apply
less than total compliance. The accused goes before the court
for, and be granted, bail as an exception only upon a clear and
without any prejudgment that he is guilty, or any obligation to
convincing showing that (1) once granted bail, he will not be a
establish his innocence. Instead, it is the responsibility of the
flight risk or a danger to the community, and (2) there exist
State to prove with the requisite amount of evidence--proof
special, humanitarian and compelling circumstances
beyond reasonable doubt--that he is guilty
Government of the USA v. Purganan
389 SCRA 623 (2002)

107 | P LATON
Then   also,   “the   presumption   of   innocence   has   never   been   ‘’if  done  by  an  adult  would  constitute  the  crime  or  crimes  of  Larceny”.  
intended as evidence of innocence of the accused but only to The judge relied on the NY Family Court Act which required a standard
shift the burden of proof that he is guilty to the prosecution. If of proof based on a preponderance of evidence and not on the
appellant’s   contention   that   proof   must   be   established   beyond   a  
‘accusation  is  not  synonymous  with  guilt,’  so  is  the  presumption  
reasonable doubt as required by the 14th amendment. The appellant
of innocence not a proof thereof. It is one thing to say that the was then ordered to be placed in a training school for an initial period
accused is presumed to be innocent in order to place on the of 18 months subject to annual extension of his commitment until his
prosecution the burden of proving beyond reasonable doubt 18th birthday (6 yrs in his case). NY CA affirmed and sustained the
that the accused is guilty. It is quite another thing to say that he constitutionality of the provision of the NY Family Court Act.
is innocent   and   if   he   is   convicted   that   he   has   been   ‘unjustly  
convicted’ Issue: WON the standard of proof beyond a reasonable doubt is
required in adjudicatory stage of delinquency proceedings
Proof Beyond Reasonable Doubt
Held: The observance of the standard of proof beyond a reasonable
doubt will not compel the State to abandon or displace any of the
A primordial consideration of due process when it comes to substantive  benefits  of  the  juvenile  process.  The  Court  rejects  the  CA’s  
criminal proceedings is the requirement that the presumption suggestion that there is only a questionable difference between
of innocence could only be overcome by a quantum of evidence reasonable doubt and preponderance standards. The constitutional
that is based on proof beyond reasonable doubt. Any other safeguard of proof beyond a reasonable doubt is also required during
lesser standard of proof would run counter to the due process the adjudicatory stage of delinquency proceedings because those are
guarantee. This higher requirement is basically dictated by the constitutional   safeguards.   The   Court   agrees   with   Chief   Judge   Fuld’s  
dissenting   opinion   ‘’that   where   a   12-yr old is charged with an act of
fact that more substantial and important societal values are
stealing which renders him liable to confinement for as long as 6 yrs,
involved--liberty and even, possibly, life then as a matter of due process, the case against him must be proved
beyond  a  reasonable  doubt.”
Proof beyond reasonable doubt, while indeed requiring more
than the other standards of proof, does not, of course, mean Estrada v. Sandiganbayan
such degree of proof that would produce absolute certainty. 369 SCRA 394 (2001)
Moral certainty only is required, or that degree of proof which
produces conviction in an unprejudiced mind Facts: Petitioner seeks the invalidation of the Anti-Plunder Law and
contends that the law allegedly dispensed with the requirement of
“The   sea   of   suspicion   has   no   shore,   and   the   court   embarks   proof beyond reasonable doubt for conviction.
upon  it  is  without  rudder  compass”
Issue: WON the Anti-Plunder Law violates the Due Process Clause

It must necessarily involve genuine and irreconcilable Held: The   use   of   the   ‘’reasonable   doubt’’   standard  is   indispensable   to  
contradictions based, not on suppositional thinking, but on the command the respect and confidence of the community in the
hard facts constituting the elements of the crime. It is not mere application of criminal law. Such standard has acquired such high
possible doubt, because everything relating to human affairs is stature in the realm of constitutional law since it gives life to the Due
open to some possible or imaginary doubt. It should not be Process Clause which protects the accused against conviction except
vague, speculative or whimsical, but intelligent, reasonable upon proof beyond reasonable doubt of every fact necessary to
constitute the crime with which he is charged.
and impartial and based on a careful examination and
conscious consideration of all the evidence in the case. A What the prosecution needs to prove beyond reasonable doubt is only
reasonable doubt is not such a doubt as any man may start by a number of acts necessary to form a combination or series which
questioning for the sake of a doubt; nor a doubt suggested or would constitute a pattern and involving an amount of at least P50 M.
surmised without foundation in facts of testimony, for it is There is no need to prove each and every other act alleged in the
possible always to question any conclusion derived from Information. Thus, the prosecution is not required to make a deliberate
testimony, but such questioning is not what is reasonable doubt and conscious effort to prove pattern.

In an American case, it was held that an instruction to the jury People v. Mendoza
which   equated   reasonable   doubt   with   a   “grave   uncertainty”   231 SCRA 264 (1994)
and  an  “actual  substantial  doubt”  required  less  proof  than  was  
Facts: The Office of the Provincial Fiscal of Malaybalay, Bukidnon filed
consistent  with  the  due  process  guarantee.  “It  is  plain  to  as  that   the instant petition for certiorari and mandamus assailing the judgment
the   words   ‘substantial’   and   ‘grave,’   as   they   are   commonly of respondent judge   Mendoza   because   of   the   latter’s   acquittal   of   the  
understood, suggest a higher degree of doubt that is required accused Juan Magalop of the crime of robbery with force upon things
for acquittal under the reasonable doubt standard. When those despite his plea of guilt. Based on the evidence, on Jan. 20, 1987, the
statements   are   then   considered   with   the   reference   to   ‘moral   storeroom of Bukidnon National School of Home Industries in
certainty,’   rather   than   evidentiary   certainty,   it   becomes   clear   Maramag, Bukidnon was ransacked. Since there was a loss of identity of
that a reasonable juror could have interpreted the instruction the culprit/s, responsibility was laid on the accused Juan Magalop,
Petronilo Fernandez, and Ricarte Dahilan. Magalop pleaded guilty,
to allow a finding of guilt based on a degree of proof below that
Fernandez   pleaded   non   guilty,   Dahilan’s   arraignment   was deferred
required by the Due Process Clause” because he was not mentally well. The court a quo conducted a hearing
and acquitted Fernandez and Magalop because the prosecution failed
In Re Winship to prove that the 3 accused were responsible even if some of the stolen
397 US 358, 25 L Ed 2d, 90 S Ct 1068 (1970) articles were found in the possession of Barbie Tan to whom they were
allegedly sold and such person refused to testify. Also, it was not shown
Facts: Sec 12 of the NY Family Court Act defines a juvenile delinquent as how Magalop and Fernandez conspired. The RTC denied the motion for
‘’a  person  over  7  and  less  than  16  yrs.  of  age  who  does  any  act  which,  if   reconsideration.
done  by  an  adult,  would  constitute  a  crime’’.  During  a  hearing  in  1967,  
the judge found that appellant, then a 12-yr old boy had entered a Issue: WON respondent judge was correct in acquitting the accused
locker  and  had  stolen  $112  from  a  woman’s  pocketbook,  an  act  which  

108 | P LATON
Held: The petition is defective since only the OSG may bring and defend when the case is one where the supreme penalty of death may be
actions on behalf of the People of the Philippines once such actions are imposed. Courts should take extra efforts to assure themselves that the
brought before the CA or the SC. However, in this case, petitioner filed accused, if ever he pleads guilty, does so with the all the attendant
it directly with the Provincial Fiscal of Malaybalay, Bukidnon without safeguards by which he could give an informed, intelligent, knowing
coursing it first through the OSG. It is evident that from the start that and voluntary acknowledgment of his guilt.
the case of the prosecution against the 3 accused was non-existent
because the stolen articles were found in possession of Barbie Tan and The Informed Plea
yet the prosecution did not summon her to the witness stand. Although
the respondent judge erred in the process, since it can be fairly
When an accused is asked to state whether he pleads guilty or
concluded that there was no standing plea at the time the court
rendered its judgment of acquittal, such acquittal was a nullity. not guilty, the same must be done in a manner that ensures
However, the Court cannot allow such procedural error to prevail over that he actually know what he is talking about. And this aspect
the constitutional right of the accused to be presumed innocent until becomes more critical when the case is one where the supreme
the contrary is proved. Since there was no evidence against Magalop, penalty of death may be imposed. Courts should take extra
his acquittal must be sustained. Petition is DISMISSED for lack of merit efforts to assure themselves that the accused, if ever he pleads
and the acquittal of the accused is sustained. guilty, does so with all the attendant safeguards by which he
could give an informed, intelligent, knowing and voluntary
PEOPLE V. FRAGO acknowledgment of his guilt
232 SCRA 653 (1994)
PEOPLE V. CHUA
Facts: Appellant, an ice cream vendor, was charged with attempted
366 SCRA 283 (2001)
rape and rape. According to the version of the prosecution, the accused
went to the residence of nine-year-old Ronalyn Pastera. The accused
Facts: Appellant was charged by his 13-year-old daughter, Chenny, of
entered her bedroom, fanned her face with his handkerchief then lifted
rape.   At   arraignment,   he   pleaded   “not   guilty”   but   at   the   pretrial  
her. He was about to take her out of the room when she suddenly woke
conference appellant, through counsel, manifested that he was
up and screamed for help. As a consequence, the accused drop Ronalyn
withdrawing  his  plea  and  changing  it  to  “guilty”  as  charged.  The  pretrial
on the floor and run out of the house. The prosecution would seem to
was then reset to another date, on which resumption the court
infer that after running, he went to the boarding house of Jicelyn. He
propounded to appellant several questions to determine the
carried Jicelyn to a nearby house where he had carnal knowledge while
voluntariness  of  appellant’s  change  of  plea  and  of  his  comprehension  of  
she was deeply asleep. When she narrated of the harrowing
its  consequences.  Satisfied  with  appellant’s  response, the court ordered
experience, Jicelyn and her mother went to the hospital where she was
his rearraignment. Appellant, with the assistance of counsel, withdrew
examined. The results indicated the presence spermatozoa and
his   plea   of   “not   guilty”   and   entered   a   plea   of   “guilty”   as   charged.  
“physical  virginity  lost”.  On  the  same  day,  Ronalyn’s  father  and  Jicelyn  
Thereafter, the court ordered the prosecution to present its evidence.
reported the incidents to the police authorities then filed their formal
The prosecution presented Chenny who testified on how the alleged
complaints against the accused. Appellant interposed denial and alibi.
rape was committed.
After trial, the court a quo found the accused guilty of rape but
acquitted him in the attempted rape case. In his appeal, he imputes
The trial court found appellant guilty and sentenced him to death.
error to the trial courts conviction on the basis of an identification
made   without   a   counsel   and   according   to   Jicelyn’s   story, which he
Issue: WON the trial court observed the mandatory procedure in
believes fantastic, thereby denying his constitutional right to be
ascertaining the guilt of the accused.
presumed innocent until proven beyond reasonable doubt.
Ruling: When the accused enters a plea of guilty to a capital offense,
Issue: WON   Orlando   Frago’s   conviction   in   the   rape   case   was   proved  
the trial court must do the following: (1) conduct a searching inquiry
beyond reasonable doubt.
into  the  voluntariness  of  the  plea  and  the  accused’s  full  comprehension  
of the consequence thereof; (2) require the prosecution to present
Ruling: The testimony of Jicelyn shows that she has no reliable basis for
evidence to prove the guilt of the accused and the precise degree of his
pointing to the accused as the person who raped her. The only
culpability; and (3) ask the accused if he desires to present evidence in
evidence of sexual intercourse is the result of the medical examination.
his behalf and allow him to do so if he desires. This procedure is
There was no positive identification of Orlando Frago by Jicelyn.
mandatory and a judge who fails to observe it commits grave abuse of
discretion.
There seems to be no question that, on the part of the Pastera sisters,
they may have recognized appellant positively because their room was
The essence of a plea of guilty is that the accused admits his guilt freely,
lighted with a lamp and he was not wearing anything on his face. Since
voluntarily and with full knowledge of the consequence and meaning of
they were neighbors, there is the possibility of Jicelyn conferring with
his act and with a clear understanding of the precise nature of the
the   Pastera   sisters   the   identity   of   the   accused.   Thus,   the   Court’s  
crime charged in the complaint or information. The court must conduct
conclusion   that   Jicelyn’s   identification   of   Orlando   Frago   was   merely  
a searching inquiry. The inquiry is not a simple question and answer
patterned after the identification made by the Pastera sisters. This is
exercise;  it  must  be  searching.  To  “search”  means  “to  look  into  or  over  
then a derivative, not positive, identification. The identification then
carefully or thoroughly in an effort to find something”.
of appellant by Jicelyn is doubtful.
The judge should, first of all, consider the age, personality, educational
Appellant’s   denial   and   alibi   are   inherently   weak,   but   the   prosecution  
background, socio-economic status and other personal circumstances
cannot rely on their frailty to enhance its cause. The prosecution must
of the accused had been coerced or placed under a state of duress
draw its strength from its own evidence.
either by actual threats of physical harm coming from malevolent or
avenging quarters, or by mistaken impressions given, wittingly or
Wherefore, Orlando Fargo is acquitted as his guilt has not been
unwittingly, by authorities or parties; whether the accused had the
proved beyond reasonable doubt.
assistance of competent counsel during the custodial and preliminary
investigations; and whether he understood the charges against him. In
all cases, the bottom line is that the judge must fully convince himself
THE INFORMED PLEA
that: (1) the accused, in pleading guilty, is doing so voluntarily; and (2)
he, in so doing, is truly guilty, and that there exists a rational basis for a
When an accused is asked to state whether he pleads guilty or not
finding of guilt, based on his testimony.
guilty, the same must be done in a manner that ensures that he actually
know what he is talking about. And this aspect becomes more critical

109 | P LATON
It is clear that the court a quo did not probe carefully and thoroughly and   possessed   a   firearm,   described   in   the   indictment,   “which   firearm  
into  the  reasons  for  applelant’s  change  of  plea  and  his  comprehension   had theretofore been shipped and transported in interstate
of the consequences of said plea. Further, the trial court did not commerce.”   The   Circuit   Court   of   Appeals   reversed   the   conviction   on
bother to explain the essential elements of the crime with which each count.
appellant was charged. The trial court did not only neglect to make
the searching inquiry, it also failed to inquire from appellant whether Ruling: These cases involve the construction and validity of section 2(f)
he desired to present evidence in his behalf. of the Federal Firearms Act, which is:

There is also another element of the crime overlooked by the trial It shall be unlawful for any person who has been convicted of a crime
court. There is no law that specifically defines and penalizes incest. The of violence or is a fugitive from justice to receive any firearm or
case involves rape. Force or intimidation is an element of the crime of ammunition which has been shipped or transported in interstate or
rape. There must, therefore, be proof beyond reasonable doubt that foreign commerce, and the possession of a firearm or ammunition
the victim did not resist her defloration due to the moral ascendancy was shipped or transported or received, as the case may be, by such
of the accused. person in violation of this Act.

Wherefore, the case is remanded to the trial court for rearraignment. The Government seems to argue that there are two alternatives tests
(PUNO, J.) of the validity of a presumption created by statute. The first is that
there be a rational connection between the facts proved and the fact
presumed; the second, that of comparative convenience of producing
PRESUMPTIONS OF INNOCENT AND GUILT evidence of the ultimate fact. These are not independent tests, but the
first is controlling, and the second but a corollary.
The Constitution presumes innocence but this not necessarily
incompatible with the presumption of guilt arising from facts and Doubtless the defendants in these cases knew better than anyone else
circumstances proven and from which culpability could be inferred. whether they acquired the firearms or ammunition in interstate
commerce. It would, therefore, be a convenience to the Government to
When it comes to presumptions, there must be a rational connection rely upon the presumption and cast on the defendants the burden of
between the facts proven and the ultimate facts presumed so that the coming forward with evidence to rebut it. (Justice Roberts)
inference of one from proof of the other is not irrational and arbitrary
because of lack of connection between the two in common experience. HIZON V. COURT OF APPEALS
As the Supreme Court reminded lower courts: “The   courts,   265 SCRA 517 (1996)
nonetheless, are advised to take caution in applying the presumption of
regularity. It should not by itself prevail over the presumption of FACTS: In response to the reports of illegal fishing operations in the
innocence and the constitutionally-protected rights of the individual. coastal waters received by the Philippine National Police Maritime
Command of Puerto Princesa City, Palawan, the city mayor organized
The constitutional presumption of innocence can only be overturned by Task Force Bantay Dagat to assist the police in the detection and
competent and credible proof and never by mere disputable apprehension of violations of the laws on fishing. There was a report
presumptions. that   a  boat   and  several   small   craft   were   fishing   by   “muro   ami”   within  
the shoreline of Puerto Princesa. The police proceeded to the area and
Presumptions of Innocence and Guilt found several men fishing in motorized sampans and a big fishing boat.
They boarded and inspected the boat. The police saw two foreigners
The Constitution presumes innocence but this is not necessarily having only photocopied passports and discovered a large aquarium full
of live lapu-lapu and assorted fish. The license of the boat and its
incompatible with a presumption of guilt arising from the facts
fishermen were in order, but nonetheless they were brought to Puerto
and circumstances proven and from which culpability could be Princesa for further investigation. The crew and fishermen were
inferred charged with several violations including conducting fishing operations
without   mayor’s   permit,   employing   excess   fishermen   on   board, and
When it comes to presumptions, there must be a rational having two Hongkong nationals on board without original passports.
connection between the facts proven and the ultimate facts
presumed so that the inference of one from proof of the other The following day, the police directed the boat captain to get random
is not irrational and arbitrary because of lack of connection samples of fish for laboratory examination. It was found that they
contained sodium cyanide. The PNP Maritime Command of Puerto
between the two in common experience
Princesa City filed the complaint against the owner and operator
Richard Hizon, the boat captain, Silverio Gargar, the boat engineer,
The constitutional presumption of innocence can only be Ernesto Andaya, two other crew members, the two Hongkong nationals
overturned by competent and credible proof and never by and 23 fishermen for the offense of illegal fishing with the use of
mere disputable presumptions obnoxious or poisonous substance penalized under Sections 33 and 38
of P.D. No. 704, the Fisheries Decree of 1975. After trial, they were
TOT V. UNITED STATES found guilty and affirmed by the Court of Appeals.
319 U.S. 463, 87 L Ed 1519, 63 Ct 1241 (1943)
On appeal to the Supreme Court, petitioners question the admissibility
Facts: The two accused in these cases were prosecuted under section of  the  evidence  which  were  seized  without  warrant,  and  the  appellate’s  
2(f) of the Federal Firearms Act. court’s   failure   to   hold   that   the   statutory   presumption   of   guilt   under  
Section 33 of P.D. 704 cannot prevail over the constitutional
In No. 569, Tot was convicted upon an indictment which charged that presumption of innocence.
he, having been previously convicted of two crimes of violence, a
burglary and an assault and battery, with intent to beat, wound, and ill- Ruling: Given the evidence admitted by the trial court, the next
treat, at Newark, New Jersey, knowingly, unlawfully, and feloniously question now is whether petitioners are guilty of the offense of illegal
received  a  described  firearm  which  “had  been  shipped  and  transported fishing with the use of poisonous substances. Again, the petitioners,
in  interstate  commerce  to  the  said  City  of  Newark.”  The  Circuit  Court  of   joined by the Solicitor General, submit that the prosecution evidence
Appeals affirmed the judgment. cannot convict them. SC agreed.
In the other case, No. 636, Delia the respondent, was convicted upon
two counts. The first alleged that, he was a person previously convicted The offense of illegal fishing is committed when a person catches, takes
of a crime of violence – robbery while armed – and that he received or gathers or causes to be caught, taken or gathered fish, fishery or

110 | P LATON
aquatic products in Philippine waters with the use of explosives, prosecution and the accused, the latter should be acquitted.
electricity, obnoxious or poisonous substances. The law creates a This is the so-called equipoise rule. Under this rule, where the
presumption that illegal fishing has been committed when: (a) evidence on an issue of fact is in equipoise, or there is doubt on
explosives, obnoxious or poisonous substances or equipment or device
which side the evidence preponderates, the party having the
for electric fishing in a fishing are found in a fishing boat or in the
possession of a fisher-man; or (b) when fish caught or killed with the
burden of proof loses.
use of explosives, obnoxious or poisonous substances or by electricity
are found in a fishing boat. Under these instances, the boat owner, Right to Be Heard and to Counsel
operator or fishermen are presumed to have engaged in illegal fishing.
The right to be heard would be a farce if it did not include the
The validity of laws establishing presumptions in criminal cases is a right to counsel. This right to have assistance of a lawyer is
settled matter. principally designed to afford an accused an opportunity to
present his side as well as to protect his interests
The prosecution failed to explain the contradictory findings on the fish
samples and this omission raises a reasonable doubt that the one ton of
fishes in the cage were caught with the use of sodium cyanide. The Right to Be Heard
apprehending officers who boarded and searched the boat did not find
any sodium cyanide nor any poisonous or obnoxious substance. The Basic and fundamental to the idea of due process is the
only basis for the charge of fishing with poisonous substance is the opportunity to be heard in defense of oneself before he is
result of the first NBI laboratory test on the four fish specimens. deprived of his life, liberty or property
Petitioners are acquitted. Rock v. Arkansas
483 U.S. 44, 97 L Ed 2d 37, 107 S Ct 2704 (1987)
People v. Godoy
250 SCRA 676 (1995) Rock got into an argument with her abusive husband. She called the
police, who arrived at the scene to find the husband dead, and Rock
Accused-appellant Danny Godoy, a married high school teacher, was holding a gun, completely hysterical.Rock professed that she could not
charged in two separate informations with rape and kidnapping with remember what happened. Her attorney took her to a hypnotist, and
serious illegal detention of Mia Taha allegedly committed in January under hypnosis, she stated that she was holding the gun, but never
1994. The complainant testified as to how the accused allegedly raped actually pulled the trigger. A gun expert was brought in, and he found
her several times and how she was kept for three days and nights in a that the gun was defective and prone to firing when knocked around.
hotel. Godoy denied the alleged rape and kidnapping. He also At trial, Rock attempted to introduce the hypnotically-derived
presented evidence that during his detention following Mia`s filing of a testimony.The prosecution objected on the grounds that the testimony
complaint, Mia`s cousin delivered to him on different occasions two was inadmissible.The prosecution argued that a defendant's testimony
letters from Mia. Said letters reaffirmed her love for him. After the trial, is limited to matters that they can prove were remembered before
the court rendered judgement finding appellant guilty of the crimes and hypnosis. The Trial Judge excluded the testimony, but allowed the gun
sentenced him to the maximum penalty in both cases expert's analysis into evidence. Arkansas has a State law that excluded
It frequently happens that in a particular case two or more hypnotically-derived testimony on the grounds that it was inherently
presumptions are involved. Sometimes the presumptions conflict, one unreliable.
tending to demonstrate the guilt of the accused and the other his The Trial Court convicted Rock of manslaughter. She appealed. The
innocence. In such case, it is necessary to examine the basis for each Arkansas Supreme Court affirmed. Rock appealed.Rock argued that the
presumption and determine what logical or social basis exists for each limitations on her testimony violated her right to present a defense.
presumption, and then determine which should be regarded as the However, the Arkansas Supreme Court found that "the dangers of
more important and entitled to prevail over the other. It must however admitting this kind of testimony far outweigh whatever probative value
be remembered that the existence of a presumption indicating guilt it may have."
does not in itself destroy the presumption against innocence unless the The US Supreme Court reversed and remanded for a new trial.
inculpating presumption together with all of the evidence, or the lack The US Supreme Court found that under the 6th Amendment a
thereof, is sufficient to overcome the presumption of innocence by defendant has a right to testify in their own defense. The Court found
proving   the   defendant’s   guilt   beyond   a   reasonable   doubt.   Until   the   that the Arkansas Supreme Court failed to perform a constitutional
defendant’s   guilt   is   shown   in   this   manner,   the presumption of analysis.While the US Supreme Court noted that hypnotically-derived
innocence continues.The presumption of innocence is founded upon testimony is often incorrect, it isn't always incorrect, and so Arkansas'
the first principles of justice, and it is not a mere form but a substantial absolute bar to hypnotically-derived testimony was an impermissible
part of the law. It is not overcome by mere suspicion or conjecture, a and arbitrary violation of the 6th Amendment right to testify in your
probability that the defendant committed the crime, nor by the fact own defense.The Court noted that in this case, the gun expert's findings
that he had the opportunity to do so. Its purpose is to balance the corroborated Rock's hypnotically-derived memory, which is a good
scales in what would otherwise be an uneven contest between the lone argument for admissibility.
individual pitted against the People and all the resources at their In a dissent, it was argued that an individual's right to represent
command. Its inexorable mandate is that, for all the authority and evidence is always subject to reasonable restrictions and that the right
influence of the prosecution, the accused must be acquitted and set to present evidence is not absolute. There is a convincing argument
free if his guilt cannot be proved beyond the whisper of a doubt. This is that hypnotically-derived testimony is inherently unreliable, and
in consonance with the rule that conflicts in evidence must be resolved therefore it is reasonable for Arkansas to bar it from admissibility.
upon the theory of innocence rather than upon a theory of guilt when it
is possible to do so,
Right to Assistance of Counsel
WHEREFORE, the judgement appealed from is hereby REVERSED and
SET ASIDE, and accused-appellant DANNY GODOY is hereby ACQUITTED The right to be heard may not really be of much help if the
of the crimes. accused himself were the only one who managed his own
defense. He is not steeped in the intricacies of criminal
Equipoise Rule courtroom practice and procedure, not to speak of the nuances
and other complexities in the law itself
With the presumption of innocence favouring the accused, it
follows that if the evidence is equally balanced between the

111 | P LATON
“It  is  said  that  a  little  learning  is  a  dangerous  thing;  and  he  who   Appeals.
acts  as  his  own  lawyer  has  a  fool  for  a  client”  (In Re: Joaquin T.
Borromeo, 241 SCRA 405 [1995]) SAYSON v. PEOPLE
166 SCRA 680 (1988)
It is likewise noteworthy that while the right to assistance of
In March 1972, petitioner was charged with the crime of attempted
counsel in regard to custodial investigation may basically be to
Estafa through Falsification of a Commercial Document. When
help the suspect in keeping intact his right to remain silent so arraigned in December 1972, he pleaded not guilty. In October 1974,
as not to incriminate himself, the right to counsel as part of the after several postponements, the prosecution rested its case. At the
trial proper is meant to assist the accused get his version of the hearing of 9 December 1974, when the defense was scheduled to
story across for consideration and appreciation by the court present its evidence, only the petitioner appeared. He said that his
counsel had another case in a different court. In the morning of the said
It is also elementary that the accused himself has the primary day, his lawyer also sent a telegram to the court requesting cancellation
right to choose his own counsel. However, considering the of the hearing because he was sick. The court denied the motion for
postponement and the case was considered submitted for decision
State’s  and  the  offended  party’s  right  to  speedy  and   adequate
without   petitioner’s   evidence.   The   trial   court   rendered   judgment   in  
justice, the court may restrict his option to retain a private January 1975, finding the accused guilty. The Court of Appeals affirmed
counsel if the accused insists on an attorney he cannot afford, but modified the penalty by imposing a lower imprisonment term and
or if the chosen counsel is not a member of the bar, or if the eliminating the fine.
attorney declines to represent the accused for a valid reason
Here, the trial court had been liberal in granting the postponements
In any event, the right guaranteed by the Constitution is to an secured by the petitioner himself, at the same time admonishing the
effective counsel, not necessarily an intelligent one latter to be ready with his present counsel or another counsel [Original
Records, p.430.] Notwithstanding this admonition, the petitioner kept
on attending the hearings without securing another lawyer to
In regard to line-ups, while, as discussed in Chapter 14 on rights substitute his present counsel who was constantly absent during the
of the suspects, there is no need for giving the Miranda hearings. Still, as admitted by petitioner in his memorandum, the trial
warning before conducting such identification process, it has court, at the December 9, 1974 hearing, allowed him to look for a
been held in the United States that counsel should be present lawyer but no one was available at the time [Rollo, p.94.] These steps
in  such  a  proceeding,  as  the  same  constitutes  a  “critical”  stage   undertaken by the trial court removes any doubt that its order was
in the criminal prosecution, specially in regard to the possibility tainted with grave abuse of discretion.
of suggestiveness
People v. Santoclides, Jr.
In   assessing   the   effectiveness   of   counsel’s   assistance,   the   321 SCRA 310 (1999)
Strickland standard...is too stringent for application in
Appellant was charged with and found guilty of the crime of rape. On
Philippine judicial setting. Strickland only seeks to ensure that appeal, he assigns as one of the errors his being represented at trial by
the adversarial testing process is present in a case by requiring a person not authorized to practice law, amounting to denial of due
that   the   assistance   rendered   by   counsel   be   “effective.”     The   process.
presence of an adversarial testing process, in other words,
ensues that the trial is fair by according the accused due W/N the misrepresentation of his counsel not authorized to practice
process  through  the  “effective”  assistance  of  counsel would amount to deprivation of his right to counsel and thus result to
his acquittal.
UNITED STATES v. ASH
Judgment is SET ASIDE; case is REMANDED to the trial court for new
413 U.S. 300
trial.
A robbery was committed at a bank by a man wearing a stocking mask,
When an accused was represented by not a member of the Philippine
waiving a pistol, and another entered, gathering the money. An
Bar during trial, the judgment should be set aside and case must
informer told authorities that he had discussed the robbery with Ash
remanded to the trial court for a new trial. The person who
and photos, including one of Ash, were shown to four witnesses. All
misrepresents himself as a lawyer shall be held liable for indirect
four made uncertain identifications. In preparation for trial, the
contempt.
witnesses were shown a photographic display to determine whether
the witnesses could make in-court identifications.
The right to counsel is of such primordial importance that even if an
accused was represented by three successive counsels from PAO, the
The  Court  of  Appeals  held  that  Ash’s  right  to  counsel  was  violated  when  
court has ordered the remand of a rape case when it found that
his attorney was not given the opportunity to be present at the
appellant was not properly and effectively accorded the right to
photographic displays.
counsel.
Did the Court of Appeals err in finding that Ash’s   Sixth   Amendment  
The presence and participation of counsel in criminal proceedings
rights had been violated?
should never take lightly. Even the most intelligent or educated man
may have no skill in the science of the law, particularly on rules of
Yes. The Court found that Wade recognized times where the
procedure and without a counsel he may convicted not because he is
subsequent trial would cure a one-sided confrontation between
guilty but because he does not know how to establish his innocence.
prosecuting authorities and an uncounseled defendant. Such stages
The right of an accused to counsel is guaranteed to minimize the
were not critical. Since the accused himself is not present at the time
imbalance in the adversarial system where the accused is pitted against
of the photographic display, no liability arises that the accused might be
the awesome prosecutory machinery of the State. Such a right
misled by his lack of familiarity with the law or overpowered by his
proceeds from the fundamental principle of due process which basically
professional adversary. We are not persuaded that the risks inherent in
means that a person should be heard before being condemned.
the use of photographic displays are so pernicious that an extraordinary
system of safeguards is required.
Jurisprudence   has   also   held   that   “the   right   to   practice   law   is   not   a  
natural right or constitutional right but is in the nature of a privilege or
The Court reversed and remanded the judgment of the Court of
franchise. It is limited to persons of good moral character with special

112 | P LATON
qualifications duly ascertained and certified. The right does not In essence, the right to be heard by counsel simply refers to the right to
presuppose in its possessor integrity, legal standing and attainment, bit be assisted by counsel for the purpose of ensuring that an accused is
also the exercise of a special privilege, highly personal and partaking of not denied the collateral right to due process, a fundamental right
the  nature  of  a  public  trust.” which cannot be waived by an accused. The underlying basis for due
process is the concept of fairness, without which there can be no
People v. Liwanag justice. In other words, there can be no due process accorded an
363 SCRA 62 (2001) accused if he is not given the right to be heard through counsel or
assisted by counsel. It follows that in order to be heard, and therefore
Complainant Hernandez was on her way home at around 1:00 am, be accorded due process, the assistance given by counsel must be
when   she   was   offered   by   tricycle   terminal   driver   Vendibil   a   “special   “effective”  as  implied  in  the  rationale  of  Art.  III,  Sec.  14(2).  In  this  sense,  
trip”.   She   would   be   brought   about   to   leave,   when   Simbulan   and   this  court  subscribes  to  American  jurisprudence  when  it  held  that  “the  
Liwanag also rode the same tricycle behind the driver. Then, Liwanag right of an accused to counsel is beyond question of fundamental right.
then entered the sidecar and sat beside the complainant, pointed an Without counsel, the right to a fair trial itself would be of little
instrument at the side of her neck and declared hold-up. The consequence, for it is through counsel that the accused secures his
complainant   told   that   she   only   has   “P60,00”   on   her   pocket.   Liwanag   other rights. In other words, the right to counsel is right to effective
told   her   since   they   can’t   get   anything   from   her,   she   would submit assistance of counsel”
herself to them. The malefactors succeeded on raping her. After
satisfying their lust, they talked of killing her. She pleaded for her life Contention of the accused that his counsel de officio failed to safeguard
and in desperation, offered money in exchange for her life. From initial his right to be secure against unreasonable search and seizures. He was
demand  of   “P10,000”,   they   finally   agreed  on  “P2,000”,   and  instructed   arrested without warrant and his counsel should contested. Argument
her to deliver the money at Guadalupe, Makati at 11:30 am the same is not well-taken. It was reiterated in People v. Costelo:
morning. After she was released, she went home and told the whole
incident to her mother and proceeded to the police station, to devise Appellant’s   failure   to   quash   the   information,   his   participation   in   the  
an entrapment operation. The police succeeded on arresting the trial and presenting evidence in his behalf, placed him in estoppel to
suspects and thus accused of highway robbery with multiple rape. make such challenge. He has patently waived any objection or
During  arraignment,  all  the  suspects  pleaded  “not  guilty”  on  the  crime   irregularities and is deemed as having submitted himself to the
charged. Simbulan and Vendabil were earlier released on recognizance, jurisdiction of the court. It should be noted that the legality of arrest
but then re-arrested for failure to appear during the trial. The trial affects only jurisdiction of the court over the person of the accused.
convicted Liwanag. On appeal, he contends, among others, that he was Consequently, if objection on such ground is waived, the illegality of the
deprived of his constitutional right to effective and competent counsel. arrest is not sufficient reason foe setting aside an otherwise valid
He claims that from the time he was arrested up to the time of his judgment rendered after the trial, free from error. The technicality
conviction, he was deprived of his other constitutional rights, cannot render the subsequent proceedings void and deprived the State
particularly his right to be secure in his person against unreasonable of its right to convict the guilty when the facts on the record point to the
search and seizures, his right to preliminary investigation, and his right culpability of the accused.
to bail. In addition, he claims that the assistance extended to him by his
former counsel was ineffective to the extent that the private Any objection involving a warrant of arrest must be made before he
complainant and two prosecution witnesses were hardly cross- enters his plea, otherwise the objection is deemed waived.
examined, while the medico-legal officer was not cross-examined at all.
He changed counsels three times from the time of the investigation Contention of the accused that he was deprived of right to bail. If his
until his trial. counsel should be effective, he would filed for proper motion. This is
without merit. It was ruled in People v. Manes:
W/N he was denied his constitutional guaranteed right to be heard by
himself and counsel. The issue of bail has been rendered academic by the conviction of the
accused. When an accused is charged with capital offense, or an
There was no dispute that accused-appellant was provided with offense punishable by Reclusion Perpetua, or life imprisonment or
counsel de officio who assisted him during the arraignment and death, and evidence of guilt is strong, bail must be denied, as it is
conducted the cross-examination of all prosecution witnesses as well as neither a matter of right nor discretion.
his direct examination. Thereafter, from the time of he was cross-
examined up to the presentation of other defense witnesses; he was The contention of the accused that his counsels show ineffectiveness is
assisted by a counsel of his choice. not convincing. As noted by the Office of the Solicitor General:

Jurisprudence   defined   the   meaning   of   “effective   counsel”   only   in   the   The pertinent transcripts of stenographic notes would show that
light of Art. III, Sec. 12(1) of the Constitution, which refers to the right appellant’s   counsel   de   officio,   Atty.   Uy,   cross-examined the private
of persons under custodial investigation. The rationale was elucidated complainant extensively as well as two other prosecution witnesses.
on People v. Lucero: That said counsel opted not to cross-examine the prosecution expert
witness, Dr. Nario, is of no moment because said witness merely
The 1987 Constitution requires that a person under investigation for explained in court her findings and conclusions that she had arrived at
commission of a crime should be provided with counsel. We have after conducting the medical examination on the private complainant.
constitutionalized the right to counsel because of our hostility against In fact, at one point, Atty. Uy raised an objection to the private
the use of duress and other undue influence in extracting confessions prosecutor’s   question   on   how   private   complainant’s   genital   injuries  
from a suspect. Force and fraud tarnish confessions and render them were sustained for being incompetent to answer, which objection was
inadmissible. In providing for said right, this Court has held in the same impliedly sustained by the trial judge.
case that when the Constitution requires right to counsel, it did not
mean any kind of counsel but effective and vigilant counsel. The The accused invoke the Strickland standard which is too stringent for
requirements of effectiveness and vigilance of counsel during the stage application in Philippine judicial setting. The standard seeks to ensure
before arraignment were for the purposes of guarding against the use that the adversarial testing process is present in a case by requiring that
of duress and other undue influence in extracting confessions which the assistance rendered  by  counsel   be  “effective”.  The  presence  of  an  
may taint them and render them inadmissible. adversarial testing process, in other words, ensures that the trial is fair
by  according  the  accused  due  process  through  “effective”  assistance  of  
Art. III, sec. 14(2) of the 1987 Constitution requires that the accused counsel.
shall enjoy the right to be heard by himself and counsel. The ratio was
explained in People v. Holgado: Fairness is likewise the object of Art. III, Sec. 14(2) of the Philippine
Constitution, assistance afforded by counsel to an accused in light of

113 | P LATON
the Philippine constitutional requirement need only be in accordance violated when upon the acquittal of the former crime does not
with the pertinent provisions of Rules of Court, Code of Professional preclude him convicted on the crime of attempted estafa through
Responsibility and Canons of Professional Ethics. falsification of official and commercial documents.

The   presumption   that   counsel’s   performance   was   reasonable,   as   long   1st issue:
as the trial was fair, accorded the accused of due process by means of
effective assistance of a counsel, the constitutional requirement that an Contention without merit. Such right, an ancient bulwark of the
accused shall have the right to be heard by himself and counsel is liberties of men, has its origin in the Bill of Rights which the people of
satisfied.   The   only   instance   when   the   quality   of   counsel’s   assistance   Great Britain demanded and received from the Prince and Princess of
cane be questioned is when an accused is deprived of due process. Orange on Feb. 13, 1688. it was adopted by the US Constitution and
extended to Philippines by Jones Law and enshrined in 1935, 1973 and
Based on the findings, accused-appellant was not deprived of his right 1987 Constitution. Has its objectives:
to heard by himself and counsel.
1. To furnish the accused with such description of the charge
Right to Be Informed against him as will enable him to make defense.
2. To avail himself of his conviction or acquittal for protection
against further prosecution for the same cause.
An element of fair play in the prosecution of any person would
3. To inform the court pf the facts alleged, so that it may
be the requirement that he be adequately informed of what he decide whether they are sufficient in law to support a
is really charged with, of what acts or omission he may have conviction, if one should had.
done or failed to do which amounted to a crime
Rules of Court has prescribed the appropriate rules. It was provided on
General Considerations Sec. 4, Rule 120 of Rules of Court.

Aside from providing an accused with information as to his What determines the real nature and cause of accusation against an
accused is the actual recital of facts stated in the information or
alleged criminal misconduct, the right to be informed also
complaint and not the caption or preamble of the information or
affords him an opportunity to set up any defense that might complaint nor the specification of the provision of law alleged to have
have been brought about by a prior acquittal, conviction, or been violated, they being conclusions of law. An incorrect caption is not
earlier proceeding arising from the same facts or offense. At a fatal mistake.
the same time, it also informs the court of the facts alleged so
that such tribunal may determine if they are sufficient in law to 2nd issue:
support a conviction
There is no sufficient circumstantial evidence to prove conspiracy
between the petitioner and Catre to commit the crime. Neither there is
Relevant herein is the so-called variance doctrine under which,
active participation in the commission of the crime. The concordant and
in spite of the difference between the crime that was charged cumulative effect of the acts of the petitioner as proven by the
and that which was eventually proved, the accused still may be prosecution’s  evidence   fails  to  satisfy  the  requirements  of  Sec.  4,  Rule  
convicted of whatever offense that was proved even if not 133 of the Rules of Court.
specifically set pout in the Information provided it was included
in what was charged Motion for reconsideration is granted. Acquitted.

Pecho v. People
262 SCRA 518 (1996)
Right to be Informed and the Death penalty
Petitioner was charged with violation of a special law, RA 3019 in
relation to an attempted underdeclaration of items to be taken out The imperative to provide the accused knowledge of what he is
from Customs, automotive diesel engines were attempted to be passed
being charged with, together with the possible consequence of
off as agricultural disc blades and irrigation water pumps. Petitioner
was convicted by Sandiganbayan. On appeal, Supreme Court modified conviction, carries more urgency when the crime charged is
the judgment by holding him guilty of the complex crime of attempted capital. The punishment being irrevocable and irreversible, the
estafa through falsification of official and commercial documents. Court Court has to be careful and certain that the accused really
said that the special law penalizes only consummated offenses and the understood what he was told, and that if ever he pleaded
offense charge was not consummated. Petitioner files for motion to guilty, then it should have been under the safeguards that
reconsideration that since he has been acquitted of violation of the would provide the assurance that he really knew what he was
special law, he could not be convicted anymore a crime punishable by responding to
the RPC, a general law, otherwise the constitutional provision on
double jeopardy would be violated. The acquittal of the crime charged
precludes conviction for attempted estafa through falsification of People v. Alicando
official and commercial documents. The Assistant Solicitor General and 251 SCRA 293 (1995)
a Solicitor disagreed with the petitioner of his asserted double
jeopardy, considering that no new information for estafa through Appellant was accused of raping and killing a four year old girl. At
falsification of public document was filed against the petitioner, but arraignment, he pleaded guilty. After his plea of guilt, the trial court
agrees for his plea of acquittal of the latter case for another ground, ordered the prosecution to present its evidence. It also set the case for
insufficiency of evidence. When the court directed the Sol Gen to reception of evidence for the appellant, if he so desired. The evidence
inform the court whether he agreed or not, to the recommendation of shows that after a witness pointed to the appellant as one who
the Asst. Sol Gen, he not only expressly agree, but added that there was committed the crime, he was arrested and interrogated by the police.
no overt act conclusively attributed to petitioner. He verbally confessed his guilt without the assistance of counsel. On
the basis of this uncounseled verbal confession and follow up
W/N the evidence adduced by the prosecution had established beyond interrogations,  the  police  came  to  know  and  recovered  from  appellant’s  
reasonable doubt the guilt of the petitioner. house,  the  victim’s  green  slippers,  a  pair  of  gold  earrings,  a  buri  mat,  a  
stained pillow, and a stained t-shirt all of which were presented as
W/N   the   petitioner’s   constitutional   right   against   double   jeopardy   was   evidence for the prosecution. After trial, he was convicted and

114 | P LATON
sentenced to death. In his brief, appellant assails the decision of the Concepcion, this court held that no valid judgment can be rendered
trial court as a travesty of justice. upon an invalid arraignment.

We find the decision of the trial court sentencing the appellant to death The arraignment is void, the judgment is also void. There is no
is shot full of errors, both substantive and procedural. The conviction is philosophy that allows the State to kill without any semblance of
based on an amalgam of inadmissible and incredible evidence fairness and justice.
supported by scoliotic logic.
People V. Ramos
1. The arraignment was null and void. 296 SCRA 559 (1998)

Section1a of rule 116 on arraignment Appellant was accused of having raped Elizabeth T. Ramos, 14-year old
Section 1, Arraignment and plea, how made.- minor through force, violence and intimidation.
(a) The accused must be arraigned before the court where
the complaint or information has been filed or A re-reading of the accusatory portion of the information reveals that
assigned for trial. The arraignment must be made in appellant was charged with rape on its simple form with the additional
open court by the judge or clerk by furnishing the allegation that the victim was 14 years old. Such act is punishable by
accused a copy of the complaint or information with reclusioin perpetua under article 335 of rpc .
the list of witnesses, reading the same in the language
or dialect known to him and asking him whether he In RA 7659 if the offender is the parent of the victim, the penalty is
plead guilty or not guilty. The prosecutor may, death.
however, call at the trial witness other than those
named in the complaint or information. The trial court arrive at the conclusion that the penalty for that kind of
rape is reclusion perpetua to death and considering the relationship as
Records do not reveal that the information was read against the a generic aggravating circumstance, imposed him a penalty of death
appellant in the language or dialect known to him. The information was (the higher of the two indivisible penalties.
written in English, it is unknown if appellant understands English nor is
it known what dialect is understood by the appellant. The case of people vs bayot gave the reminder that a qualifying
Note: after reading the information to the accused, the accused pleads circumstance or an inherent aggravating circumstance should not be
guilty. mistaken for a generic aggravating circumstance. The court
distinguishes the two by stating that a generic aggravating
2. The plea of guilt is likewise null and void. circumstance, not offset by any mitigating circumstance, increases the
penalty which should be imposed to the maximum period without
Section 3 of rule 116 exceeding the limit prescribed by law, while a qualifying circumstance
Section 3. Plea of guilty to capital offense; reception of evidence.- not only gives the crime its proper and exclusive name but also imposes
When the accused pleads guilty to a capital offense, the court shall the author thereof no other penalty but that specially prescribed by law
conduct a searching inquiry into the voluntariness and full for the said crime.
comprehension of the consequences of his plea and require the
prosecution to prove his guilt and precise degree of culpability. The Held: since the relationship was in a nature of a qualifying
accused may also present evidence in his behalf. circumstance, and not merely a generic aggravating circumstance, it
must be alleged in the information.
The bottom line of the rule is that the plea of guilt must be based on a
free and informed judgment. Thus the questions should focus on:
In People v. Aquino, the Court said that the words  “aggravating/  
1. Voluntariness of the plea
2. Full comprehension of the consequences of the plea. qualifying,”   “qualifying,”   “qualified   by,”   “aggravating,”   or  
“aggravated   by”   need   not   be   expressly   stated   as   long   as   the  
Neither was satisfied during the searching inquiry. The records do not particular attendant circumstances are specified in the
reveal any information about the personality profile of the appellant Information
which can serve as a trustworthy index of his capacity to give a free and
informed plea of guilt. The age, socio-economic status and educational Right to be Informed and Aggravating Circumstances
background were not plumbed by the trial court. The questions were
framed in English yet there is no inkling that he had a nodding
Under the Revised Rules of Criminal Procedure which took
acquaintance of English. It must also be noted that the trial court did
not bother to explain to the appellant the essential elements of the effect on 1 December 2000, it is now required that aggravating
crime rape with homicide. circumstance, whether generic or qualifying must be alleged in
the Information or Complaint in order to be appreciated
Also the trial court did not ask when he was arrested, by whom, how
and where he was interrogated, whether he was medically examined Right to Speedy Trial
before and after the examination.
To ensure that justice be done, it is necessary that there be no
It warned the appellant that he would get the mandatory penalty of
unwarranted, vexatious, capricious, and oppressive delays in
death without explaining the word mandatory. It did not inform the
appellant of the indemnity he has to pay for the death of the victim. It the criminal proceedings. Otherwise, it would be animate the
cautioned the appellant that there will be some effects to his civil rights familiar dictum that justice delayed is justice denied
without telling him what those effects are.
Delay in proceedings not only could give rise to denial of justice
The  appellant’s  plea  of  guilt  is  void  and  the  trial  court  erred  in  using  it   in its general application. It could also work undue vexation and
to sentence the accused to death. oppression upon the accused and other characters involved,
from the government itself to the private offended parties
3. Some prosecution evidence, offered independently of the
plea of guilt of the appellant were inadmissible yet, were
considered by the trial court in convicting the appellant If he needs to be declared innocent and free, then he should
have it at the earliest opportunity instead of being harassed by
In binabay vs people, et al., a ponencia of Mr. Chief Justice R. prolonging his hardships. If he is to be convicted, then it would

115 | P LATON
also be to his advantage that the matter be determined soonest trial. He also argues that the CA erred in not sustaining the submission
in order that he may be able to take whatever legal steps are of the Solicitor General for his acquittal. Required to comment, the
available to him, or to enable him to simply accept the verdict Office of the Solicitor General adopted its Motion and Manifestation
filed before the CA and recommended the grant of the petition for
instead of being agonized by a sword of Damocles hanging over
Certiorari averring that the findings and conclusions of the CA are not
his head supported by substantial evidence.

Further, even as the right to a speedy trial is guaranteed to the In his first assignment of error, he claims lack of due process due to the
accused, the same should not be utilized to deprive the State of unusual speed of the trial court.
a reasonable opportunity of fairly indicting and prosecuting
criminals Applicable to this case is General Order no. 39 amending General Order
no 12, datedSept.30, 1972.
Conde vs Rivera
45 Phil 650 (1924) “23.  Crimes where the offended party is a tourist or a transient.

Aurelia Conde, formerly a municipal midwife in Lucena, Tayabas. She The civil court shall have concurrent jurisdiction with the military
was forced to respond to no less the five information for various crimes tribunals over the said crimes, provided that civil courts shall dispose of
and misdemeanors, and appeared with her witnesses and counsel at such cases w/in 24 hrs after the filing thereof by the arresting officer.
hearings no less than on 8 different occasions only to see the cause The court or tribunal that first assumes jurisdiction shall exercise
postponed. jurisdiction  shall  exercise  jurisdiction  to  the  exclusion  of  all  others.”

She then filed this present petition for mandamus and prohibition. He was not denied due process, the trial court merely deferred such
conference till after the prosecution had presented its witnesses. (it is
Philippine organic and statutory law expressly guarantee that in all to be noted further that defense counsel was not totally unprepared for
criminal prosecutions the accused shall enjoy the right to speedy trial in the trial for it was ready with 2 witnesses when asked by the court).
order that if innocent, she may go free, and she has been deprived of Moreover, after the prosecution had rested its case, trial was resumed
that right in defiance of law. Dismissed from her humble position, and the next day, thereby giving the defense enough time to prepare for
compelled to dance attendance on courts while investigations and trials the presentation of its direct evidence.
are arbitrarily postponed without her consent, is palpably and openly The trial was not indicative of inordinate haste. On the contrary, they
unjust to her and a detriment to the public. By the use of reasonable should be commended for their punctilious compliance with the explicit
diligence, the prosecution could have settled upon the appropriation of mandate of the law.
the information, could have attended the formal preliminary
examination, and could have prepared the case for trial free from That the complainant was not a transient visitor, his having been in
vexations, capricious and oppressive delays. Olongapo for a number of times does not make him any less than a
transient   or   one   whose   stay   is   “of   uncertain   duration”.     Also,  
We lay down the legal prosecution that, where a prosecuting officer, notwithstanding the number of times he has been to the country, he is
without good cause, secures postponements of the trial of the still a tourist, or one who travels from place to place for pleasure and
defendant against his protest beyond a reasonable period of time, as in culture.
the instance for more than a year, the accused is entitled to relief by a
proceeding in mandamus to compel a dismissal of the information, or if Also,   on  Solicitor   General’s  position   that   the   guilt   was  not   established  
hr be restrained of his liberty, by habeas corpus to obtain his freedom. beyond reasonable doubt, the complainant was categorical in his
identification of the petitioner-accused  and  emphatic  as  to  the  latter’s  
Dismissed, cost against respondent fiscal. direct and active involvement in the robbery.

Magat v. CA DACANAY V. PEOPLE


116 SCRA 283 (1982) 240 SCRA 490 (1995)

Petitioner was charged with robbery for divesting of his money an FACTS : In 1985 , petitioner was the vice-president of the National
American businessman who went to Olongapo as a tourist. Sugar Trading Corporation (NASUTRA) . In 1986, a criminal complaint
for economic sabotage through smuggling, with regard to the
The information was filed July 26 1979 at 3pm, a week after the importation of raw sugar by NASUTRA, was filed with the Tanodbayan
incident complained of. against the principal officers of the said corporation including the
petitioner. In 1988, a resolution was issued by Special Prosecutors
Magat was charged together with four others. The case was Gervacio and Kallos recommending the dismissal of the complaint
immediately raffled and set for arraignment and trial. Only the against petitioner and his co-accused for lack of sufficient evidence and
petitioner was arraigned at 4:35 the same afternoon. the withdrawal of the information filed in court. In 1989, the resolution
was reviewed by special prosecutor Orencia, who recommended its
After arraignment, it proceeded to trial. Counsel asked for time, an disapproval and such recommendation was approved by Acting Special
hour at least, to confer with his client, the fiscal said he had no Prosecutor Guerrero and by the Ombudsman. The motion for
objection to having a conference between counsel and his witness reconsideration filed by co-accused Unson and adopted by petitioner
provided  it  be  done  after  he  would  have  already  submitted  the  People’s   was denied. In 1991, petitioner filed a motion for immediate and
case . The court said it would allow counsel to consult his client before, separate trial invoking his constitutional right to a speedy trial.
and from time to time, during the cross-examination. Respondent People opposed the said motion on the ground that a
separate trial for petitioner would entail a lengthy and repetitious
The following day, trial was resumed at 830 am. At 1135, judgement of proceeding . The petitioner filed a motion for reconsideration setting
conviction was promulgated. forth as grounds therefor his advanced age and protection of his
reputation.   The   Sandiganbayan   denied   petitioner’s   motion   for  
He appealed to CA. CA promulgated its decision August 21 1981 reconsideration .
affirming but reduced the penalty.
ISSUE: whether or not petitioner is entitled to a separate trial.
In his appeal to the SC, petitioner raises the issues of having been
denied due process, specifically the right to be heard, including ample DECISION: The respondents are ORDERED to proceed with the trial of
opportunity for the accused and the counsel to confer and prepare for petitioner in Criminal Case No. 11957, separately if it need be.

116 | P LATON
the original setting that the accused suddenly became zealous of
REASON: the main objective of respondent is that such a procedure safeguarding his right to speedy trial and disposition.
would entail a repetitive presentation of evidence. But the resulting
inconvenience and expense on the part of the Government cannot be While this court recognizes the right to speedy disposition quite
given preference over the right to speedy trial and the protection to a distinctly from the right to a speedy trial, and although this court has
person’s   life,   liberty   or   property   accorded   by   the   constitution. A always zealously espouse protection from oppressive and vexatious
separate trial is in consonance with the right of an accused to a speedy delays not attributable to the party involved, at the same time, we hold
trial as guaranteed to him by the 1987 constitution under Art. III sec. that  a  party’s  individual  rights  should  not  work  against  and  preclude  the  
14(2). The primordial purpose of this constitutional right is to prevent people’s  equally  important  right  to  public  justice.  Three  people  died  as  
the oppression of an accused by delaying criminal prosecution for an a result of the crash of the airplane that the accused was flying. It
indefinite period of time. It has been eight years since the information appears to us that the delay in the disposition of the case prejudiced
against petitioner was filed, but the case against him has yet to be not just the accused but the people as well. Since the accused has
tried. The long delay has clearly prejudiced petitioner, who is now more completely failed to assert his right seasonably and inasmuch as the
than seventy-three years of age. respondent judge was not in a postion to dispose of the case on the
merits due to the absence of factual basis, we hold it proper and
GUERRERO V. CA equitable to give the parties fair opportunity to obtain substantial
257 SCRA 703 (1996) justice in the premises.

FACTS: In 1971, an information for triple homicide through reckless PEOPLE V. TEE
imprudence was filed against petitioner before the Court of First 395 SCRA 419 (2003)
Instance in Caloocan, alleging that on or about May 13 1969, the
petitioner, then the pilot of a non-commercial aircraft, without taking FACTS: Appellant was charged with violation of the Dangerous Drugs
the necessary care and precaution to avoid accidents or injuries to Act. After his conviction, he raised as one of the issues the several
persons, and without ascertaining as to whether the quantity of fuel in postponements allowed by the trial court by reason of the repeated
the tanks of his aircraft was sufficient for the flight, flew said aircraft absence   of   the   prosecution’s   witness   in   the   person   of   Abratique.
with four passengers on board. When the engine quit twice indicating Appellant  insists  that  such  lapse  on  the  prosecution’s  part  violated  ‘the  
that there was no more fuel, accused made an emergency landing and speedy   trial   act   of   1998.’   On   record,   the   trial   court   found   that  
in  doing  so  the  aircraft’s  landing  gear  collided  with  a  dike  and  trees  near   prosecution witness Danilo Abratique failed to appear in no less than
a fishpond in Malabon, resulting in the death of three passengers. twenty hearings. No less than five warrants of arrest were issued
against him to compel him to testify. The NBI agent who supposedly
Due to several postponements, all filed by the petitioner, the had him in custody was found guilty of contempt of court for failing to
prosecution was only able to start presenting its evidence in 1972. In produce Abratique at said hearings and sanctioned. Nothing on the
1975, the prosecution finally rested its case while the defense did so in record discloses the reason for Abratique aforecited absences.
1978. In 1989, court administrator ordered the re-raffling of the case Appellant now stresses that the failure of Abratique to appear and
from the RTC of Caloocan City to the RTC of Navotas-Malabon which testify   on  twenty   hearing   dates  violated  appellant’s  constitutional   and  
had jurisdiction over the case. In March 1990, Judge Aquino ordered statutory right to a speedy trial.
the parties to follow-up and complete the transcript of stenographic
notes within 30 days considering that the same was found to be ISSUE: whether or not failure of Abratique to appear and testify on
incomplete. When the parties were not able to complete the transcript twenty   hearing   dates   violated   appellant’s   constitutional   and   statutory  
of stenographic notes, the court ordered the retaking of the right to a speedy trial.
testimonies of the witnesses.
DECISION: no. his right was not violated
In October 1990 the presiding judge set the retaking on the 24th of the
month, however, on the said date the retaking was reset to Nov. 9 due REASON: the speedy trial act of 1998, provides that the trial period for
to  the  petitioner’s  failure  to  appear  on  the  scheduled  hearing.  On  Nov.   criminal cases in general shall be 180 days. However, in determining the
7, petitioner filed a motion to dismiss on the ground that his right to right of an accused to speedy trial, courts should do more than a
speedy trial has been violated. The judge denied the motion to dismiss mathematical computation of the number of postponements of the
and reconsideration and reset the retaking. The petitioner then filed a scheduled hearings of the case. The right to speedy trial is deemed
petition for certiorari, prohibition and mandamus with the court of violated only when: (1) the proceedings are attended by vexatious,
appeals, anchored on the alleged violation of his right to speedy trial. In capricious, and oppressive delays; or (2) when unjustifiable
its decision in 1991, the appellate court dismissed the petition and also postponements are asked for and secured; or (3) when without cause
denied the motion for resconsideration. Before the SC, petitioner or justifiable motive a long period of time is allowed to elapse without
complains of undue prejudice as a result of the protracted litigation and the party having his case tried.
of violation of his right to speedy trial.
In the present case, although the absence of prosecution witness
ISSUE: does the constitutional right to speedy trial include the right to Abratique totaled twenty hearing days, there is no showing that
prompt disposition and judgment prosecution   capriciously   caused   Abratique’s   absences   so   as   to   vex   or  
oppress appellant an deny him his rights. Nor do we find a delay of
DECISION: the petition is DENIED twenty hearing days to be an unreasonable length of time. Delay of less
than two months has been found, in fact, to be not an unreasonably
REASON: In the determination of whether or not the right to a speedy lengthy period of time. Moreover, nothing on the record shows that
trial has been violated and on whether or not a person officially appellant objected to inability of the prosecution to produce its
charged with the administration of justice has violated the speedy witness. Under the Rules, appellant could have moved the trial court to
disposition of case guarantee, certain factors may be considered and require that witness Abratique post bail to ensure that the latter would
balanced against each other such as length of delay, reason for the testify when required and also he could have moved to have Abratique
delay, assertion of the right or failure to assert it, and prejudice caused found in contempt and duly sanctioned but he did neither. It is a bit too
by the delay. In this case, the petitioner merely sat and waited after the late in the day for appellant to invoke now his right to speedy trial.
case submitted for resolution in 1979. It was only in 1989 when the
case was re-raffled and only after respondent trial judge ordered the No  persuasive  reason  supports  appellant’s  claim  that  his  constitutional  
parties to follow-up and complete the transcript of stenographic notes right to speedy trial was violated. One must take into account that a
that matters started to get moving towards the resolution of the case. trial is always subject to postponements and other causes of delay. But
More importantly, it was only after the new trial judge reset the in the absence of a showing that delays were unreasonable and
retaking  of  the  testimonies  because  of  the  petitioner’s  absence  during   capricious, the State should not be deprived of a reasonable

117 | P LATON
opportunity of prosecuting an accused. because of the constitutional presumption of innocence. Needless to
stress, this right is available to every accused, whatever his present
Right to Impartial Trial and Tribunal circumstances and no matter how dark and repellent his past.

In any event, convictions are based not on the mere appearance of the
It is a basic requirement of due process that the judge or
accused but on his actual commission of the crime, to be ascertained
tribunal hearing a case should be impartial. The case would with pure objectivity of the true judge who must uphold the law for all
have to be determined in accordance with the evidence without favor or malice and always with justice. Considering the way
presented and not by any personal bias or prejudice on the part they were tried, we now declare that they should not be detained in jail
of the court. Lady Justice is supposed to be above the in a minute longer. While this is not to say that the accused are not
importunities of the parties and blindfolded as well in order guilty, it does mean that, because their constitutional rights have been
that she can decide cases based strictly on the merits and not violated, their guilt, if it exists, has not been established beyond
on the personalities of the parties involved reasonable doubt and so cannot be pronounced.

Tabuena v. Sandiganbayan
The guiding rule is that a judge must not only render a just,
268 SCRA 332 (1997)
correct and impartial decision but should also do so in such a
manner as to be free from any suspicion as to his fairness, In January 1986, President Marcos instructed over the phone, Luis
impartiality and integrity Tabuena, the general manager of Manila International Airport
Authority   (MIAA),   to   pay   directly   to   the   President’s   Office   what   the  
PEOPLE V. OPIDA MIAA owed the Philippine National Construction Corporation (PNCC). A
142 SCRA 295 (1986) week later, Tabuena received from Mrs. Fe Roa-Gimenez, the private
secretary of the President, the same instruction given by Marcos over
FACTS: several persons ganged up on Fabian Galvan, stoned and hit him the phone. Tabuena, with the help of co-accused Dabao, assistant
with beer bottles until finally one of the stabbed him to death. The manager of MIAA, then caused the release of 55 million pesos, went to
actual knife-wielder was identified as Mario del Mundo. Nonetheless, the office of Gimenez and gave the money. No receipt was issued.
appellants Opida and Marcelo was charged with murder as conspirators
and, after trial, sentence to death. The basis of their conviction by the Appellants were charged of malversation. For the prosecution, they
trial court was the testimony of two prosecution witnesses, neither of said that there were no outstanding obligations in favour of PNCC at
whom positively said that the accused were at the scene of the crime, the time of the disbursement of 55 Million. As for the defense, they
their extrajudicial confessions which were secured without the said that they acted in good faith, that they were merely complying
assistance of counsel, and corroboration of the allege conspiracy under with  the  President’s  instructions.
the theory of interlocking confessions.
Appellants were charged before the Sandiganbayan with malversation.
What was striking about this case is the way the trial judge conducted After trial, they were convicted, the court disregarding their defense of
his interrogation of the two accused and their lone witness, Lilian having acted in good faith.
Layug. Reading the transcripts, one gathers the impression that the
judge had allied himself with the prosecution to discredit at the outset Held: Good faith is a defense in a prosecution for malversation.
the credibility of witnesses for the defense. Opida is a police character,
admittedly a member of the Commando gang and with a string of However,  what  amounted  to  acquittal  is  the  violation  of  the  accused’s  
convictions for robbery, theft and vagrancy. It is worth noting that the basic constitutional right to due process.
judge took special interest in his tattoos, required him to remove his
shirt so they could be examined, and even describe them in detail for The SC is strucked by the way Sandiganbayan actively took part in the
the record. The interrogation of Virgilio Marcelo, the other accused, questioning of a defense witness and of the accused
was conducted almost wholly by the judge who started cross-examining themselves.Questions   from   the   court   after   Tabuena’s   cross-
the witness even before the defense counsel could ask his first examination totalled 67 questions, this is more than 5 times the
question, and took over from the prosecatuion the task of impeaching prosecutor’s   questions,   more   than   double   the   total   of   direct  
Marcelo’s  credibility.  The  judge  asked  him  about  his  drug  addiction,  his   examination and cross examination.
membership in the Commando gang, his tattoos, his parentage, his
activities, his criminal record – all when he was supposed to be under The SC acknowledged the right of a trial judge to question witnesses,
examination by his own lawyer. but this is limited to clarificatory questions. In this case, this limitation
was not observed.The cold neutrality of an impartial judge requirement
ISSUE: violation of the right to impartial trial and tribunal and of due process was certainly denied Tabuena and Peralta when the
inadmissibility of the extrajudicial confessions of the accused under the court, with its overzealousness, assumed the dual role of magistrate
due process guarantee of the constitution and advocate.

DECISION: the conviction of Opida and Marcelo is reversed and they Marcos v. Sandiganbayan
are hereby ordered released immediately. 297 SCRA 95 (1998)

REASON: Given the obvious hostility of the judge toward the defense, it Petioner Imelda Marcos was the minister of Human Settlement while
was inevitable that all the protestations of the accused in this respect Jose P. Dans, Jr. Was the minister of Transporation and
would be, as they in fact were, dismissed. And once the confessions Communication.The two served as ex officio chairman and vice
were admitted, it was easy enough to employ them as corroborating chairman of Light Rail Transport Authority. Marcos was also the
evidence of the claimed conspiracy among the accused. chairman of Philippine General Hospital Foundation. The two entered in
a lease agreement, Marcos acting as the chairman of PGHF and Dans as
The accused are admittedly notorious criminals who were probably vice chairman of LRTA, LRTA leased to PGHF a lot at a monthly rental of
even proud of their membership in the Commando gang even as they 102,760 for 25 years.. On june 1984, Marcos entered a sublease
flaunted their tattoos as a badge of notoriety. Nevertheless, they were agreement wherein Transnational Construction Corporation
entitled to be presumed innocent until the contrary was proved and represented by President Ignacio Gimenez, wherein said lessee rented
had a right not to be held to answer for a criminal offense without due the same area for 734,000 a month for 25 years. Petitioner and Dans
process of law. The judge disregarded these guarantees and was in fact were indicted in 1992 for conspiring and confederating with each other
all too eager to convict the accused. The scale of justice must hang in entering into lease agreement alleged to be manifestly and grossly
equal and, in fact, should even be tipped in favor of the accused disadvantageous to the government, in violation of Anti Graft and

118 | P LATON
Corrupt Practices Act.
Right to Public Trial
The case was raffled to First Division of Sandiganbayan, with presiding
Justice Francis Garchitorena as Chairman, Justices Balajadia and Atienza
While it is part of the guarantee of criminal due process that
as members. Failing to comply with the requirement of unanimity with
the dissent of Justice Narcisio Atienza, Garchitorena requested to have
there be a public trial in order that there be an assurance that
a Special Division of 5, designating Justices amores and del Rosario as the case may not be railroaded or otherwise unfairly dealt with
additional members. Justice Amores wrote Garchitorena that he be in secrecy, too much publicity may also create its own problems
given 15 days to send his manifestation. However, on the same day, the and difficulties
two original members of the division of 3 agreed with the opinion of
Justice Atienza. Garchitorena then issued an order dissolving the What constitutes a public trial? A trial possesses that character
division of 5. Said agreement was arrived over lunch in an unnamed when anyone interested in observing the manner a judge
Quezon City restaurant that day among Justices Garchitorena,
conducts the proceedings in his courtroom may do so, be he a
Balajadia, Del Rosario, and the other who was not a member of the
Special Division. The three conviceted the petitioner and Dans,JR.
relative, a friend or a complete stranger to the proceedings.
Petitioner appealed to the SC. However, the mere fact that trial is held in the air-conditioned
chambers of the judge does not violate the right to public trial.
Held: The procedural flaws committed by the Sandiganbayan are fatal And   if   the   trial   is   so   held   in   the   judge’s   chamber,   then   the  
to   the   validity   of   its   “decision”   convicting   the   petitioner   for   the   accused is entitled to be inside the chamber and not simply by
following reasons: That sessions of the Sandiganbayan shall be held in the door
the principal office, that rules of Sandiganbayan do not allow
unscheduled discussion of cases, that they do not allow informal
General Considerations
discussion of cases, that they do not allow the presence of a non-
member in the deliberation of cases, and that they do not allow
exclusion of a member of a division. These irregularities violated the Secrecy may suggest something sinister, or suppression and
right of the petitioner to be tried in a collegial court. Moreover, Justice oppression. It might as well be if trials were held in private, far
Francisico   observed   “the   court   questions   were   so   numerous   which   as   from the view and observation of the public. For by then,
per petitioner Dans count totalled 179 compared to prosecutor nobody really knows what goes on within the walls of silence
Querubin’s  questions  which  numbered  merely  73.  More  noteworthy  is   except those inside
that the court propounded leading, misleading, and baseless
hypothetical  questions  rolled  into  one.”
This does not preclude the courts from excluding the public,
Marcos’s  motion  for  reconsideration  was  granted  and was acquitted.
however, where the subject matter is sensitive or otherwise
dictated by the nature of the proceedings, as in rape cases
Webb v. People
where the audience might be more interested as salacious
voyeurs than disinterested spectators
Petitioners were charged with the crime of rape with homicide for
allegedly raping Carmela Vizconde, killing carmela herself, her mother,
and her sister. The case was raffled to RTC of Paranaque presided by
respondent Tolentino. Webb and his co-accused, Bion, sought the Publicity and Prejudice
disqualification of respondent judge. The respondent judge allegedly
told  the  media  that  “failure  to  surrender  following  the  issuance  of  the  
When it comes to the issue of impartiality of trials, the matter f
warrant  of  arrest  is  an  indication  of  guilt.”  Respondent  judge denied the
motion. In September 1995, Biong filed another motion to disqualify
the adverse and contagious effect of publicity may also come
respondent judge on the ground of bias and impartiality. This was into the picture. Publicity could have a distorting effect on trial
likewise denied by the respondent judge. The petitioners filed separate such that judges may not see the case for what it really is but as
petitions for bail. On nov. 9, 1995, petitioners filed a motion to distilled through the noise outside, or the pens and lenses of
disqualify the judge due to bias and prejudice, the judge denied the the ubiquitous mass media. It is in this area that there comes
motion   for   lack   of   merit.   The   respondent   judge   denied   petitioners’   again the need to carefully weigh conflicting values of society--
petitions for bail. the right of the people to know and of the press to disseminate
newsworthy materials, on one hand, and the right of the
The present petition principally anchored on the alleged error of the CA
in declaring that no sufficient ground existed for the disqualification of
accused to a fair and impartial trial, on the other hand
respondent judge and on the issue of lack of fairness and impartiality
on the part of the respondent judge. WEBB v. DE LEON
247 SCRA 652 (1995)
Held: To disqualify a judge on the basis of bias and prejudice, the
movant must prove the same by clear and convincing evidence. This is a Hubert Webb, Michael Gatchalian, Antonio Lejano and six (6) other
heavy burden and petitioners failed to discharge their burden of proof. persons, were charged with the crime of Rape with Homicide in relation
to rape and killing of Carmela Vizconde, her mother Estrellita, and her
To be disqualifying, the bias and prejudice must be shown to have sister Jennifer in their home in Paranaque. The DOJ Panel conducted a
stemmed from an extrajudicial sources and result in an opinion on the preliminary investigation and, in an issued Resolution, found probable
merits on some basis other than what the judge learned from his cause to hold respondents for trial. Information for rape with homicide
participation in the case. Extrinsic evidence is required to establish bias, was filed against the petitioners with Paranaque RTC. Warrants of
bad faith, malice or corrupt purpose, in addition to the palpable error Arrests were issued against them. Petitioners Webb, Gatchalian, and
which may be inferred from the decision or order. Although the Lejano voluntarily surrendered to the police authorities and filed
decision may seem so erroneous as to raise doubts concerning the petitions questioning the proceeding against them and the issuance of
judge’s  integrity,  absent  extrinsic  evidence,  the  decision  itself  would  be   warrants, assailing, the prejudicial publicity that attended their
insufficient to establish a case against the judge. Petioners simply lean preliminary investigation considering the highly sensationalized nature
on the alleged series of adverse rulings of the respondent judge which of the case.
can be characterized as palpable errors. This is not enough.
Conflicting  demands  of  freedom  of  speech  and  of  the  press,  the  public’s  
right  to  information,  and  the  accused’s  right  to  a  fair  and  impartial  trial  
Bracy v. Gramley

119 | P LATON
collide and compete for prioritization. The possibility of media abuses and accurately W/N his trial will be fair and impartial.
and their threat to fair trail notwithstanding, criminal trials cannot be
completely closed to the press and the public. Panganiban, J.
It is technologically possible to uphold the right of the people to public
To warrant a finding of judicial publicity there must be allegation and information without violating the right of the accused to due process
proof that the judges have been unduly influenced, not simply that they and without impeding the orderly administration of justice.
might be, by the barrage of publicity. (Martelino, et al. Vs. Alejandro et
al.) In the case at bar, nothing in the records will prove that the tone Media and judiciary are natural partners, because they are bound
and content of publicity that attended the investigation of petitioners together by the same reasons for being- the search for truth, the
fatally infected fairness and impartiality of DOJ panel. DOJ panel is protection  of  the  people’s  rights,  and  the  defense  for  the  basic  norms  
composed of an Asst. Chief State Prosecutor and Senior State of society. By proscribing live coverage of trials and hearings, the Court
Prosecutors, their long experience in criminal investigation is a factor to has lost an indispensable teammate in discovering, processing and
be considered whether they can easily be blinded by the klieg lights of reporting the raw, unadulterated and unvarnished truth.
publicity. The 26-page Resolution carries no indubitable indicia of bias
for it does not appear that they considered any extra-record evidence RESOLUTION ON MOTION FOR RECONSIDERATION
except evidence properly adduced by the parties. Also, the length of 365 SCRA 62 (2001)
time the investigation was conducted despite its summary nature and
generosity in accommodating the motions of petitioners speak well of The Secretary of Justice filed a motion for reconsideration arguing that
their fairness. there is really no conflict between the right of the people to public
information and the freedom of the press, on one hand, and on the
Petitions are dismissed for lack of showing of grave abuse of discretion other, the right of the accused to a fair trial; that if there is a clash
on the part of respondents. between these rights, it must be resolved in favour of the right of the
people and the press because the people, as the repository of
RE: REQUEST FOR RADIO-TV COVERAGE OF THE TRAIL IN THE sovereignty, are entitled to information; and, that live media coverage
SANDIGANBAYAN OF THE PLUNDER CASES AGAINST THE FORMER is a safeguard against attempts by any party to use the courts as
PRESIDENT JOSEPH E. ESTRADA instruments for the pursuit of selfish interests. Estrada reiterates his
360 SCRA 248 (2001) objection to the live coverage of his trial. Based on his experience with
the impeachment trial. Live media coverage will only pave way for so-
The Kapisanan ng mga Brodkaster ng Pilipinas (KBP) requested the SC to called  “expert  commentary”  which  can  trigger  massive  demonstrations  
allow media coverage via live radio and television broadcast of the aimed at pressuring the Sandiganbayan. He further contends that the
anticipated trial before the Sandiganbayan of the trial of the plunder right of people to information may be served through other means less
cases against former president Joseph E. Estrada, in order to assure the distracting, degrading, and prejudicial than live coverage.
public full transparency in the proceedings of an unprecedented case in
our history.Sec. of Justice filed the instant petition seeking live radio After due deliberation, SC finds no reason to alter or modify its decision
and television coverage of the proceedings grounded on the right of the on prohibiting live coverage of the trial of the former president. By a
people to be informed on matters of public concern as well as desire vote of 9-6, the court denies the MR of the Sec. of Justice.
for transparency in administration of justice. Estrada, expressed his
opposition, and so did the IBP. In lieu of the live TV and radio coverage, the court resolved, by the vote
of 8-7, to order the audiovisual recording for documentary purposes.
The proprietary of granting or denying the petition involve the weighing
out of constitutional guarantees of freedom of the press and the right Reasons for televised recording: 1)the hearings are of historic
to public information, on the other hand, and the fundamental rights of significance. 2)Estrada cases involve matters of vital concern to our
the accused, on the other hand, along with the constitutional power of people who have a fundamental right to know how their government is
the court to control its proceeding in ensuring a fair and impartial trial. conducted. 3)audio-visual presentation is essential for the education
When these rights race against one another, jurisprudence tells us that and civic training of people.
the right of the accused must be preferred to win.
Conditions of ausio-visual recording: a)trial shall be recorded in its
A public trial is not synonymous with publicized trial; it only implies that entirety, excepting such portions thereof as the Sandiganbayan
the court doors must be open to those who wish to come, sit in the determine should not be held public; b)cameras shall be installed
available seats, conduct themselves proper decorum and observe the inconspicuously inside the courtroom and the movement of TV crews
trial process. shall be regulated; c)audio-visual recordings shall be madefor
documentary purposes only and shall be made without comment
The television does not simply mirror or reflect events as they unfold. except such annotations of scenes depicted therein as may be
The images transmitted onscreen are the end products of a series of necessary to explain them; d)live broadcast of the recordings before
technical modifications employed by TV editors and cameramen; the Sandiganbayan shall have rendered its decision in cases against
limiting  the  public’s  perception  of  the  events  being  covered. former president Estrada shall be prohibited under the pain of
contempt of court and other violations; e)audio-visual recording of the
With intense coverage, the media becomes less of a defensive force proceedings shall be made under the supervision and control of the
against injustice and more of an offensive force by intimidating Sandiganbayan or its Division; f)simultaneously with the release of the
witnesses, distracting the lawyers and distorting the unfolding drama in audio-visual recordings for public broadcast, the original thereof shall
the courtroom. Thus, until and unless the media can secure the rights be deposited in the National Museum and the Records Management
of the accused and eliminate all the adverse effects, specifically on the and Archive Office for preservation and exhibition in accordance with
general public, the TV should remain outside of the courtroom. law.

DISSENTING: Related to the issue of prejudicial publicity is the prohibition of


Puno, J. certain fictionalized representation of matters which might be
It will to a large degree, throttle the right of the press to access to
the subject of on-going court proceedings
information and choke the flow of knowledge to the people. it is the
people who govern in a democracy and they can only govern well if
they are fully informed. A people kept in the dark by the blindfold of Right of Confrontation
ignorance will only govern with mistakes.
By outlawing the TV in the trial of former President Estrada, the Another element of fairness is the right of whoever is charged
majority has denied our people the opportunity to know completely to have the opportunity to confront whoever it is whose
120 | P LATON
testimony or evidence may lead the former to lose his liberty or
even life (d) The Confrontation Clause commands that reliability be assessed in a
particular manner: by testing in the crucible of cross-examination.
Roberts allows a jury to hear evidence, untested by the adversary
It is the province of [cross-examination] to test the credibility of
process, based on a mere judicial determination of reliability, thus
the witnesses, expose falsehood or half-truth, uncover the replacing the constitutionally prescribed method of assessing reliability
truth which rehearsed direct-examination testimonies may with a wholly foreign one.
successfully suppress, and demonstrate inconsistencies on
substantial matters which create reasonable doubt. In short, (e) Roberts' framework is unpredictable. Whether a statement is
cross-examination is an indispensable instrument of criminal deemed reliable depends on which factors a judge considers and how
justice to give substance and meaning to the Constitutional much weight he accords each of them. However, the unpardonable vice
right of the accused to confront the witnesses against him and of the Roberts test is its demonstrated capacity to admit core
testimonial statements that the Confrontation Clause plainly meant to
to show that the presumption of innocence has remained
exclude.
steadfast and firm
(f) The instant case is a self-contained demonstration of Roberts'
It has also been held that the admission of hearsay evidence in unpredictable and inconsistent application. It also reveals Roberts'
criminal cases would be a violation of the constitutional failure to interpret the Constitution in a way that secures its intended
guarantee of the right of confrontation constraint on judicial discretion. The Constitution prescribes the
procedure for determining the reliability of testimony in criminal trials,
Mere opportunity and not actual cross-examination is the and this Court, no less than the state courts, lacks authority to replace
it with one of its own devising.
essence of the right

CRAWFORD V. WASHINGTON PEOPLE V. PIDO


541 U.S. 36, 158 L Ed 2d 177, 124 Ct 1354 (2004) 200 SCRA 45 (1991)

Petitioner was tried for assault and attempted murder. The State Accdg. To Prosecution: Teresita was awakened at around 1-2:00 by an
sought to introduce a recorded statement that petitioner's wife Sylvia intruder who she identified as the appellant. Holding a sharp bladed
had made during police interrogation, as evidence that the stabbing instrument in his right hand he threatened to kill Teresita if she would
was not in self-defense. Sylvia did not testify at trial because of shout. He told her to remove her panty and afterwards placed himself
Washington's marital privilege. Petitioner argued that admitting the on top of her, and with his left hand inserted his penis into his genital
evidence would violate his Sixth Amendment right to be "confronted organ. Thereafter, he left through the window, leaving Teresita crying.
with the witnesses against him." Under Ohio v. Roberts, 448 U. S. 56, Meanwhile, Lydia, who was also staying in the same house came out of
that right does not bar admission of an unavailable witness's statement her room and when she switched on the light, she saw Teresita shaking
against a criminal defendant if the statement bears "adequate `indicia and crying. She later told Lydia that she was raped by the appellant.
of reliability,'" a test met when the evidence either falls within a "firmly Accdg. To the Appellant: While admitting that he had sexual intercourse
rooted hearsay exception" or bears "particularized guarantees of with Teresita, claimed that their tryst was with her consent. He
trustworthiness." The trial court admitted the statement on the latter declared that Teresita was his former girlfriend and on the night in
ground. The State Supreme Court upheld the conviction, deeming the question, he attended the despedida party of the husband of Lydia at
statement reliable because it was nearly identical to, i. e., interlocked their backyard. In the course of their drinking spree, he saw Teresita
with, petitioner's own statement to the police, in that both were looking out the window and waving her hand at him, asking him to
ambiguous as to whether the victim had drawn a weapon before come up. They simultaneously took off their clothes and made love,
petitioner assaulted him. and while they were in the act, Lydia came out of her room and saw
them. Both were embarrassed, and because of their embarrassment,
The State's use of Sylvia's statement violated the Confrontation Clause she cried. He claimed that Teresita filed the complaint for rape because
because, where testimonial statements are at issue, the only indicium Lydia threatened that she will   report   what   she   saw   to   Teresita’s  
of reliability sufficient to satisfy constitutional demands is common-law husband.
confrontation. After trial, appellant was found guilty. The TC characterized the
testimony  of  the  complainant  that  appellant  raped  her  as  “positive  and  
(a) The Confrontation Clause's text does not alone resolve this case, so categorical”,  and  “clear,  positive,  and  convincing”.  It  concluded that the
this Court turns to the Clause's historical background. That history credibility of the declaration of the complainant and her witness, Lydia,
supports two principles. First, the principal evil at which the Clause was “have  not  been  shaken  much  less  destroyed  by  the  searching  questions  
directed was the civil-law mode of criminal procedure, particularly the of the cross-examiner”
use of ex parte examinations as evidence against the accused. The Upon reviewing the case, the Court find that the prosecution failed to
Clause's primary object is testimonial hearsay, and interrogations by discharge its duty to establish the guilt of the accused beyond
law enforcement officers fall squarely within that class. Second, the reasonable doubt. The TC merely hurried itself to the sweeping
Framers would not have allowed admission of testimonial statements conclusion   that   the   “credibility   of   their   declarations   have   not   been  
of a witness who did not appear at trial unless he was unavailable to shaken much less destroyed by the searching questions of the cross-
testify and the defendant had had a prior opportunity for cross- examiner.”   This   is   not   supported   by   facts   unfurled   during   the   cross-
examination. English authorities and early state cases indicate that this examination.
was the common law at the time of the founding. And the "right . . . to This case is an exception to the rule that appellate courts will generally
be confronted with the witnesses against him," Amdt. 6, is most not disturb the factual findings of the trial court considering that it is in
naturally read as a reference to the common-law right of confrontation, better position to decide the question , having heard the witnesses
admitting only those exceptions established at the time of the themselves and observed their deportment and manner of testifying.
founding. Two special considerations in the case justify such departure: 1) it was
another judge (Judge Lazaro) who heard and received the whole
(b) This Court's decisions have generally remained faithful to the testimony . Judge Makasinar, who decided the case, did not have
Confrontation Clause's original meaning. sufficient   base=is   to   form   an   opinion   as   to   the   complainant’s  
deportment an manner of testifying; 2)the TC had ignored or
(c) However, the same cannot be said of the rationales of this Court's overlooked substantial facts and circumstances:
more recent decisions. The Roberts test departs from historical D
principles because it admits statements consisting of ex By a direct, positive and unequivocal statement, complainant
parte testimony upon a mere reliability finding. categorically asserted on direct examination that:

121 | P LATON
When he [accused] was on top of me, he inserted his private organ to A Not so loud, sir.
my private organ. Q You are (sic) crying because of happiness, am I right?
with the use of his left hand, which she repeated in detail at one point A No, sir.
of the cross-examination, thus: Q After making love with you, do you know if this intruder
“Q After you have removed your panty, what did the accused dressed-up?
do? A I  do  not  know  anymore  sir,  I  was  crying  because  of  fear.”
A He removed his hand . . . left hand on my neck and moved xxx xxx xxx
his body a little to the back and he held his private organ and inserted it “Q   So, you want to impress this Honorable Court that while you
into my private organ. were making love with the accused this Lydia Sulit passed by?
Q Which hand did he use in inserting his organ to yours? A Yes,  sir.”
A He used his left hand. xxx xxx xxx
However, in another point during cross-examination, she affirmed that “Q   While you were making love with the accused, do you
the accused was beside her, and she was the one who took hold of his remember if the party is (sic) still going on downstairs?
penis and put it into her private organ: A Yes, sir.
Q Have you felt his body when he was beside you at that right? Q So, there are (sic) still many people at the ground floor of
A Yes, sir. your house during that night when you were making love with the
Q For how many minutes have (sic) you felt his body when he accused?
was beside you? A No sir, that is not true.
A Just for a short while, sir. Q Which is not true?
Q Because you immediately took hold of his penis and put into A The people were already asleep, sir.
your vagina? Q After making love with the accused, do you know where the
A Yes, sir. accused passed thru in going down?
which she did for the following reason: A I do not know, sir.
Q And so that you can immediately finish his intercourse? Q Is it not true that the accused jumped over the window?
A Yes,  sir.” A I  do  not  know  sir,  I  was  crying  because  I  was  afraid.”
The lame excuse for such conduct offered by her on redirect This claim that she did not then know where the accused passed
examination was because she was nervous as she was threatened by through is of course inconsistent with her assurance on direct
accused who told her "you insert it otherwise I will kill you including examination that the accused used the window for his exit:
your children." He resorted to this because "at first he was not able to “A After he put on his panty or shorts, he left through the
insert it"; yet, on direct examination, she categorically stated that when window. This window is at the second floor of the house where he
he was on top of her he inserted his organ, and although she did not jumped.
see how, she knew that it was already inside. xxx xxx xxx
E A He went out of the window where the stairs is just nearby
While on direct examination, she wanted to convey that she was and he stepped  on  it.  There  he  already  went  down.”
reluctant to remove her panty, and in an earlier cross-examination, she xxx xxx xxx
wanted the court to believe that she spread her legs preparatory to the “A   Well, I don't know how he entered, Your Honor, but he went
intercourse because she was allegedly ordered by the accused; yet out through the window.
subsequently, she was candid enough to admit that she immediately then she confirmed on such cross-examination, thus:
removed her panty and raised her dress for the intercourse, thus: Q So, you were then lying when you testified on January 18,
“Q   So, when this intruder allegedly told you to remove your 1982 that the accused jumped over the window?
panty, you immediately removed your panty? A It was not our window that opened in front of where we are
A Yes, sir, I obeyed him because I was afraid. (sic) sleeping, sir.
Q And after that you immediately raised your dress so that this Q But you are telling that the window is (sic) still open since
intruder can immediately have sexual intercourse? you slept and making (sic) love with the accused?
A Yes, sir. A That window was really open because it is (sic)  warm,  sir.”
Q And immediately this intruder immediately (sic) penetrated If the references to "making love" were without basis, or that the
his penis over (sic) your vagina? questions based thereon were misleading, the prosecuting fiscal should
A Yes, sir. have objected. None was made, and there is no showing that the
Q And while he was penetrating (sic) his over your vagina, he is prosecuting fiscal is incompetent. After this "love-making" rendezvous,
(sic) making love with you? the accused promised to return to make love again.
A No  sir,  straight.” “Q   And after finishing the intercourse what did this intruder do
However, in another portion of the cross-examination, complainant, by if he did anything?
slip of her tongue, admitted that she in fact voluntarily removed her A He told me that he will return, sir.
panty, thus: Q To make love again with you?
“Q And you never even bothered to go out of the mosquito net A Yes, sir.
and shout? Q And you said "yes" because you like it?
A I stood up, sir. A No sir, I was crying.
Q You stood up because you want (sic) to wear again your Q Because you are (sic) afraid that your husband might know
panty which you voluntarily removed? it?
A Yes, sir. A No sir, I was still startled because of my fear.”
Q You never even bothered to go to the window and asked G
(sic) help from your neighbors? This fear, however, seemed to be more imagined than real in the light
A No.  sir.”  (Emphasis  supplied) of the above findings clearly demonstrating her consent to the act. As a
F matter of fact, there is enough evidence to show that she signaled to
On cross-examination, the cross examiner succeeded in showing that the accused to spend time with her and that she prepared for the tryst.
the sexual liaison was the product of mutual consent, or the For reasons only known to the prosecution, it opted not to rebut the
complainant and the accused were "making love." The following damaging testimony of the accused that among others, complainant
questions and answers are enlightening: signaled to him to come up; she thereafter took a bath; when he came
“Q   Now, you said that after less than 5 minutes making up, she ordered him to get inside the mosquito net, and to give him
love with this alleged intruder, he left you at the sala, am I right? space beside her, she even moved her youngest child; and then they
A Yes sir, he went away but I could not recall if he left because simultaneously took off their clothes and made love. While they were
I was crying. in the act, Lydia Sulit came out of her room and saw them. Because of
Q Are (sic) you crying aloud? embarrassment, complainant cried. He then dressed up and returned

122 | P LATON
to the party. The accused in a criminal case is guaranteed the right of confrontation.
Such right has two purposes: first, to secure the opportunity of cross-
What then seems to be clear is that Lydia did not arrive after the act, examination; and, second, to allow the judge to observe the
but as testified by the accused and corroborated by complainant deportment and appearance of the witness while testifying. This right,
herself, she saw both inside the mosquito net. The complainant was so however, is not absolute as it is recognized that it is sometimes
embarrassed that she had to concoct the story of rape. impossible to recall or produce a witness who has already testified in a
previous proceeding, in which event his previous testimony is made
PEOPLE V. NARCA admissible as a distinct piece of evidence, by way of exception to the
275 SCRA 696 (1997) hearsay rule.

Accused-appellants were charged with murder fo the killing of Mauro The adoption by the Makati TC of the facts stated in the decision of the
Reglos, Jr. They filed a motion to quash the information, and when this Paranaque TC does not fall under the exception to the right of
failed,  they  filed  a  motion  for  bail.  During  the  bail  hearing,  the  victim’s   confrontation as the exception contemplated by law covers only the
wife, Elizabeth Reglos, who was with him on that fateful night, testified utilization of testimonies of absent witnesses made in previous
on direct examination. Defense counsel requested the court that his proceedings, and does not include utilization of previous decisions or
cross-examination of Elizabeth be conducted on the next hearing, the judgments.
following month. However, such cross-examination never took place
because Elizabeth died before the hearing. After hearing, lower court Prosecution did not offer the testimonies made by complainants.
denied bail, and after trail, convicted the appellants. Instead, what was offered, admitted in evidence, and utilized as a basis
for conviction in the case for illegal recruitment in large scale was the
Issue:   W/N   Elizabeth’s   testimony be given credence considering that previous decision in the estafa case. A previous decision or judgment,
she was nit cross-examined. while admissible in evidence may only prove that an accused was
previously convicted of a crime. It may not be used to prove that the
Held: Where death prevents cross-examination under such accused is guilty of a crime charged in a subsequent case.
circumstances that no responsibility of any sort can be ascribed to the
plaintiff or the witness, it seems a harsh measure to strike out all that For insufficiency of evidence and in the absence of the third element of
has obtained in the direct examination. illegal   recruitment   in   large   scale:   “the   offense   is   committed   against  
three   or   more   persons”,   the   court   cannot   affirm   the   conviction   for  
Besides, mere opportunity and not actual cross-examination is the illegal recruitment in large scale but agrees that appellant illegally
essence of the right to cross-examine. Right to cross-examination is a recruited Marasigan, for which she ust be held liable for the lesser
personal one which may be waived expressly or impliedly by conduct offense of simple illegal recruitment.
amounting to a renunciation of the right of cross-examination. Thus,
where a party has had the opportunity to cross-examine a witness but Relevant to the right of an accused to confront the witness
failed to avail himself of it, he necessarily forfeits the right to cross- against him is the issue as to whether to disclose the identity of
examine and the testimony given on direct examination of witness will confidential informers or to present them at the witness stand.
be received or allowed to remain in record.
This necessarily involves again the balancing of conflicting
Sec.  1  (f)  of  Rule  115  provides  that  “either  party  may  utilize  as  part  of  
societal values
its evidence the testimony of a witness who is deceased ***given in
another  case  or  proceeding”  and  under  Sec 8, Rule 114 as amended by The Court is sharply aware of the compelling considerations
circular 12-94,   “evidence   presented   during   the   bail   hearings,”   like   the   why confidential informants are usually not presented by the
testimony   of   the   deceased   witness   Elizabeth,   are   “considered   prosecution. One is the need to hide their identity and preserve
automatically  reproduced  at  trial”. their invaluable service to the police. Another is the necessity
to protect them from objects or targets of revenge by the
PEOPLE v. ORTIZ-MIYAKE criminals they implicate once they become known. All these
279 SCRA 180 (1997) considerations, however, have to be balanced with the right of
an accused to a fair trial
Accused – appellant Lanie Ortiz-Miyake was charged with illegal
recruitment in large scale in RTC of Makati on a complaint initiated by
Elenita Marasigan, Imelda Generillo and Rosamar del Rosario. In It was held that the scope of the privilege is limited by its
addition, she was indicted for estafa by means of pretenses in the same underlying purpose. Thus, where the disclosure of the contents
court, the offended party being Elenita Marasigan alone. of the communication will not tend to reveal the identity of an
informer, the contents are not privileged. Likewise, once the
Of the three complainants in the case for illegal recruitment in large identity of the informer has been disclosed to those who would
scale, Marasigan was the only one who testified at the trial. The two have cause to resent the communication, the privilege is no
other complainants, Generillo and Del Rosario, were unable to testify as
longer applicable
they were then abroad. in lieu of their testimonies, the prosecution
presented as witnesses Lila Generillo, the mother of Imelda, and Victor
Amin, the sister of Del Rosario. The final witness for the prosecution A further limitation on the applicability of the privilege, which
was Riza Balberte, POEA representative, testified that appellant was not arises form the fundamental requirements of fairness was
authorized or licensed to recruit workers for overseas employment. The emphasized.  Where  the  disclosure  of  an  informer’s  identity,  or  
prosecution sought to prove that although two of the three the contents of his communication, is relevant and helpful to
complainants were unable to testify, appellant was guilty of committing the defense of an accused, or is essential to a fair determination
the offense against all three and, therefore, be convicted as charged. of a cause, the privilege must give way
The TC convicted appellant of both crimes as charged. In convicting
appellant of illegal recruitment in large scale, the lower court adopted a
In sum, there is no fixed rule with respect to disclosure of the
previous decision (conviction for estafa in 1993 wherein complainats identity of an informer. The problem has to be resolved on a
Generillo and Del Rosario charged the appellant) of MTC of Paranaque case to case basis and calls for balancing the state interest in
as a basis for the judgment. RTC adopted the facts and conclusions protecting   people   from   crimes   against   the   individual’s   right   to  
established in the earlier decision as its own findings of facts and as its prepare his defense. The balance must be adjusted by giving
rationale for the conviction of the case before the court. due weight to the following factors, among others: (1) the

123 | P LATON
crime charged, (2) the possible defenses, (3) the possible Appeals for the Armed Forces reversed, finding the evidentiary
significance  of  the  informer’s  testimony,  and  (4)  other  relevant   exclusion to be unconstitutional. The United States appealed and the
factors Supreme Court granted certiorari.

Does Military Rule of Evidence 707, excluding the admission of


Right of Compulsory Process polygraph results into evidence, violate a defendant's Sixth Amendment
right to present a fair defense?
This guarantee assures the accused an opportunity to compel
the production of evidence and the attendance of witnesses on No. In an 8-to-1 decision, the Court held that Rule 707 was consistent
his behalf in order that he may be adequately equipped to with the legitimate interest of state and federal authorities to admit
present his side. Also, compared to the 1935 Constitution which only reliable evidence. In addition to noting the even-handed scope of
only guaranteed compulsory process to secure the attendance Rule 707, excluding from evidence both favorable and unfavorable
polygraph results, the Court emphasized the poor reliability of
of witnesses, the 1973 and 1987 Constitutions expanded the
polygraph evidence as a whole. In the absence of sounder detection
same by now including the right to secure the production of methods, the Court noted that the fundamental premise of the criminal
evidence  in  one’s  behalf  as  well.  Accordingly, justice system is that juries are the ultimate and most reliable
evaluators of credibility and truthfulness.
By analogy, U.S. v. Ramirez which laid down the requisites for
compelling the attendance of witnesses, may be applied to this Trials In Absentia--Waiver of Appearance
expanded concept. Thus, the movant must show: (a) that the
evidence is really material; (b) that he is not guilty of neglect in While the accused has the right to be heard, he cannot hold the
previously obtaining the production of such evidence; (c) that proceedings hostage, however, by absenting himself or refusing
the evidence will be available at the time desired; and (d) that to   participate.   Thus,   the   Constitution   provides   that   “after  
no similar evidence could be obtained arraignment, trial may proceed notwithstanding the absence of
the accused provided that he has been duly notified and his
In Rolito Go v. Court of Appeals,  “the  right  to  have  a  preliminary   failure  to  appear  is  unjustifiable”
investigation conducted before being bound over for trial for a
criminal offense, and hence formally at risk of incarceration or Nor is it only the due process guarantee that calls for the
some other penalty, is not a mere or technical right; it is a accused being duly arraigned. As noted, it is at the stage where
substantive  right” in the mode and manner required by the Rules, an accused, for
the first time, is granted the opportunity to know the precise
In the 1963 watershed case of Brady v. Maryland the United charge tat confronts him. It is imperative that he is thus made
States   Supreme   Court   held   that   “suppression of evidence fully aware of possible loss of freedom, even of his own life,
favorable to an accused upon request violates due process depending on the nature of the crime imputed to him
where the evidence is material to guilt or punishment,
irrespective  of  the  good  faith  or  bad  faith  of  the  prosecution” Clearly, the innovation introduced by the present Constitution
goes no further than to enable a judge to continue with the trial
The right to the production of all evidence at a criminal trial even if the accused is not present under the conditions therein
similarly has constitutional dimensions. The Sixth Amendment specified. It does not give him the right to jump bail.
explicitly confers upon every defendant in a criminal trial the
right  “to  be  confronted  with  the  witnesses  against  him”  and  “to   People v. Salas
have  compulsory  process  for  obtaining  witnesses  in  his  favor.” 143 SCRA 163
Moreover, the Fifth Amendment also guarantees that no
person shall be deprived of liberty without due process of law. Mario Abong was originally charged with homicide in the Court of First
It is the manifest duty of the courts to vindicate those Instance of Cebu but before he could be arraigned the case was
reinvestigated on motion of the prosecution. As a result of the
guarantees, and to accomplish that it is essential that all
reinvestigation, an amended information was filed, with no bail
relevant and admissible evidence be produced recommended, to which he pleaded not guilty. Trial commenced, but
while it was in progress, the prisoner, taking advantage of the first
We conclude that, when the ground for asserting privilege as to information for homicide, succeeded in deceiving the city court of Cebu
subpoenaed materials sought for use in a criminal trial is based into granting him bail and ordering his release; and so he escaped. The
only on the generalized interest in confidentiality, it cannot respondent judge, learning later of the trickery, cancelled the illegal bail
prevail over the fundamental demands of due process of law in bond and ordered Abong's re-arrest. But he was gone. Nonetheless, the
the fair administration of criminal justice. The generalized prosecution moved that the hearing continue in accordance with the
constitutional provision authorizing trial in absentia under certain
assertion of privilege must yield to the demonstrated, specific
circumstances. The respondent judge denied the motion, however, and
need for evidence in a pending criminal trial suspended all proceedings until the return of the accused. The order of
the trial court is now before us on certiorari and mandamus. The judge
US v. SCHEFFER erred. He did not see the woods for the trees. He mistakenly allowed
523 U.S. 303 himself to be tethered by the literal reading of the rule when he should
have viewed it from the broader perspective of its intendment
While defending himself before a military court martial on, among
other things, substance abuse charges, airman Edward G. Scheffer The trial judge is directed to investigate the lawyer who assisted Mario
sought to introduce his polygraph examination results. The results Abong in securing bail from the city court of Cebu on the basis of the
indicated there was "no deception" in Scheffer's denial that he used withdrawn information for homicide and to report to us the result of
drugs while enlisted. Relying on Military Rule of Evidence 707 ("Rule his investigation within sixty days.
707"), prohibiting the use of polygraph results in court-martial
proceedings, the military judge refused Scheffer's request to admit his The order of the trial court dated December 22, 1983, denying the
results into evidence. On successive appeals, following his conviction on motion for the trial in absentia of the accused is set aside. The
all charges, the Air Force Court of Appeals affirmed but the Court of respondent judge is directed to continue hearing the case against the

124 | P LATON
respondent Mario Abong in absentia as long as he has not reappeared, the allegation in the information and proof adduced during trial shall be
until it is terminated. No costs fatal to the criminal case if it is material and prejudicial to the accused
so  much  so  that  it  affects  his  substantial  rights.”
It could thus be seen that the Bill of Rights has set up a lot of
guarantees and safeguards by which life and liberty, as well as No matter the victim, accused still guilty! That might have been the
mindset of the trial court judge.
property of individuals are protected from being taken by the
State,  or  otherwise  lost.  A  person’s  entitlement  to  his  freedom  
and to his life and what other things he may own or possess
could not simply be forfeited without having to satisfy the
constitutional shield and requirements, starting with the
presumption of innocence to the quantum of proof needed to
Chapter 16
secure a conviction. In between are the other procedural
Privilege of the Writ of Habeas Corpus
hurdles designed to ensure fairness in order that only the
guilty, as much as humanly possible, be punished and that the
“The   privilege   of   the   writ   of habeas corpus shall not be
innocent be set free to continue to enjoy life and liberty and to
suspended except in cases of invasion or rebellion, when the
bask in the wide open and exhilarating field of freedom 25
public  safety  requires  it.”
Additional Cases
Liberty being the rule instead of the exception, there must be
(N) Rights of Suspects [7]
an effective and speedy manner by which any deprivation of
such freedom be swiftly looked into and remedied

Indeed,   it   has   been   called   the   “great   writ   of   liberty”   by   which  


the   legality   of   one’s   detention   or   deprivation   of   freedom   of  
movement  may  be  inquired  into.  “Vindication  of  due  process,  it  
has been well said, is precisely the historic office of the Great
Re: Conviction of Judge Adoracion G. Angeles, RTC, Br. 121, Caloocan
Writ.”   In   a   nutshell,   it   is   a   writ   directed   to   a   person   detaining  
City in Criminal Cases Nos. Q-97-69655 to 56 for Child Abuse another, commanding the former to produce the body of the
543 SCRA 196 (2008) latter at a designated time and place. The objective of the writ
is to determine whether the confinement or detention is valid
Does the presumption of innocence come to an end when there is and lawful. If it is, the writ cannot be issued
conviction by the trial court, even if there is an appeal taken? In this
case, the Court echoed what was said Trillanes – the presumption lost More specifically, its vital purposes are to obtain immediate
only when there   is   a   final   judgment.   Accordingly,   “the   fact   of   [the  
relief from illegal confinement, to liberate those who may be
judge’s]   conviction   by   the   RTC   does   not   necessarily   warrant   her  
suspension. We agree with [her] argument that since her conviction the
imprisoned without sufficient cause, and to deliver them from
crime of child abuse is currently on appeal before the CA, the same has unlawful custody. It is then essentially a writ of inquiry and is
not yet attained finality. As such, she still enjoys the constitutional granted to test the right under which a person is detained
presumption of innocence. It must be remembered that the existence
of a presumption indicating the guilt of the accused does not in itself General Considerations
destroy the constitutional presumption of innocence unless the
inculpating presumption, together with all the evidence, or the lack of The writ of habeas corpus extends to all cases of illegal
any   evidence   or   explanation,   proves   the   accused’s   guilt   beyond   a  
confinement by which any person is deprived of his liberty.
reasonable  doubt.”
“The  extraordinary  writ  of  habeas corpus has long been a haven
of relief for those seeking liberty from any unwarranted denial
Andaya v. People
493 SCRA 539 of  freedom  of  movement.”  Indeed,  habeas corpus embraces so
broad a dimension--aside from being thorough and complete--
In this case, the accused was charged with Falsification of Private it affords prompt relief from unlawful imprisonment of any
Documents but no damage to the offended party was proved. The kind, and under all circumstances
accused was still convicted, though, for intent to cause damage to
government through evasion of tax.   Despite   the   trial   court’s Further,  it  has  also  been  held  that  “[i]t  is  not  physical  restraint  
concession that the allegedly offended party (a savings and loan alone which is inquired into by the writ of habeas corpus.
association) suffered no damage, it still convicted the accused,
Reservation in the form of restrictions attached to the
reasoning out that the third essential element of falsification of private
document was present because the falsification of the voucher was
temporary release of a detainee constitutes restraints on his
done with criminal intent to cause damage to the government liberty and limits his freedom of movement of petitioner
considering that its purpose was to lower the tax base of the recipient
of the money, allowing him to evade payment of taxes. Habeas Corpus could also be utilized to invoke the beneficial
effects of a law which was subsequently enacted. Hence, where
The Court saw the act done by the judge unjustified. “We  find  ourselves   the decision convicting the accused is already final, the
unable to agree with this ratification of the trial court because it appropriate remedy of the convict who invokes the retroactive
violates the constitutional right of petitioner to be informed of the
application of a statute is to file a petition for habeas corpus,
nature   and   cause   of   the   accusation   against   him.”   It   must   be  
remembered   that   “[n]o   matter   how   conclusive and convincing the not a motion for reconsideration with modification of sentence
evidence of guilt may be, an accused cannot be convicted of any
offense unless it is charged in the information on which he is tried or is The  privilege  of  the  writ  could  also  be  availed  of  to  secure  one’s  
necessarily included therein. To convict him of a ground not alleged liberty from the restraint imposed by a private person, as
while he is concentrating his defense against the ground alleged would
plainly be unfair and underhanded. The rule is that a variance between 25 CONSTITUTION, Art. III, § 15
125 | P LATON
exemplified in the case of Caunca v. Salazar. It is likewise
available in regard to custody of children but not for the Ilusorio v Bildner
purpose of compelling a spouse to live with the other 332 SCRA 169

The wife of Potenciano Ilusorio filed a petition for Habeas Corpus of his
In relation to other procedural remedies, in Velasco v. Court of
86 year old husband. After living for 30 years under the same house,
Appeals, the Court said, relative to the availability of both the couple separated in 1972. The younger daughter of the two filed
certiorari and habeas corpus: apetition for custody of the father because of his failing physical and
mental health. Later on, the daughter allegedly prohibited the mother
While ordinarily, the writ of habeas corpus will not be granted from seeing the father and living with her. Potenciano filed a motion to
when there is an adequate remedy by writ of error or appeal or enjoin a previous motion granted for visitation rights.
by writ of certiorari, it may, nevertheless, be available in
exceptional cases, for the writ should not be considered WON Writ of Habeas Corpus can compel a husband to live with his wife.
subservient to procedural limitations which glorify over
No court is empowered as a judicial authority to compel a husband
substance to live with his wife. In order to justify the grant of the writ of habeas
corpus, the restraint of liberty must be in the nature of an illegal and
Gumabon v Director of Prisons involuntary deprivation of freedom of action. The illegal restraint of
37 SCRA 420 liberty must be actual and effective, not merely nominal or moral.

Petitioners Mario Gumabon, Blas Bagolbagol, Gaudencio Agapito, The law provides that the husband and the wife are obliged to live
Epifanio Padua and Paterno Palmares were charged and convicted of together, observe mutual love, respect and fidelity. The sanction
the complex crime of rebellion with murder. They were imposed the therefore is the "spontaneous, mutual affection between husband and
penaltyof Reclusion Perpetua. At the time of the petition each suffered wife and not any legal mandate or court order" to enforce consortium.
more than 13 years of imprisonment.Subsequently, the Court ruled in Obviously, there was absence of empathy between spouses Erlinda
PEOPLE VS. HERNANDEZ that the information against the accused in and Potenciano, having separated from bed and board since 1972.
thatcase for rebellion complexed with murder, arson and robbery was
not warranted under Article 143 of theRPC, there being no such
Resolution on Motion for Reconsideration
complex offense. Petitioners thus invoke that the ruling in Hernandez
be applied tothem. Petitioners contend that he has served more than
Erlinda K. Ilusorio claimed that she was not compelling Potenciano to
the maximum penalty that could have been imposedupon them and is
live with her in consortium and that Potenciano's mental state was not
thus entitled to freedom, his continued detention being illegal.
an issue. One reason why Erlinda K. Ilusorio sought custody of her
husband was that respondents Lin and Sylvia were illegally restraining
WON Habeas Corpus can be granted to convicted inmates.
Potenciano Ilusorio to fraudulently deprive her of property rights out of
pure greed.
Petition for Habeas Corpus Granted, petitioners ordered released. Once
a deprivation of a constitutional right is shown to exist, the court that
Motion denied Again
rendered the judgment is deemed ousted of jurisdiction and Habeas
Corpus is the appropriate remedy to assail the legality of the detention.
Clearly, Erlinda cannot now deny that she wanted Potenciano Ilusorio
The essential purpose of the writ of habeas corpus is to inquire into all
to live with her.
manner of involuntary restraint and to relieve therefore if such
The fact of illegal restraint has not been proved during the hearing at
restraint is illegal. The exception of jurisdiction, excluding habeas
the Court of Appeals on March 23, 1999.16 Potenciano himself declared
corpus to sentenced prisoners, is not found here because of the
that he was not prevented by his children from seeing anybody and
retroactivity of the Hernandez rulings. What is required under the
that he had no objection to seeing his wife and other children whom he
equal protection of law is the uniform operation of legal norms so that
loved. We were not convinced that Potenciano Ilusorio was mentally
all persons under similar circumstances would be accorded the
incapacitated to choose whether to see his wife or not. Again, this is a
same treatment both in the privileges conferred and the liability
question of fact that has been decided in the Court of Appeals.
imposed. For the principle that equal protection and security shall be
given to every person under circumstances, which if not identical are
analogous. If law be looked upon in terms of burdens or charges, those Suspension of the Privilege of the Writ, Bail and Damages
that fall within a class shall be treated in the same fashion, whatever
restrictions cast on some in the group equally binding on the rest. This particular Bill of Rights guarantee is meant to delimit the
circumstances in which the privilege of the writ may be
Subayno v Enrile suspended, namely, in cases of invasion or rebellion when the
145 SCRA 282 public safety requires it

Ybanez, an officer of the education arm of the BAYAN movement was This should also be taken in conjunction with the other
abducted in Cebu City. Alleging tha this abductors was the military,
restrictions contained in Article VII regarding the strictures in
Subayno who is Ybanez’s  lover,  filed a petition for Habeas Corpus to the
court. A line up was conducted on the military of Camp Lapu-Lapu place whenever the President suspends the privilege or
which yielded no identification of the perpetrators. declares martial law. It states:

WON a motion for Habeas Corpus can be granted by mere accusation Section 18. The President shall be the Commander-in-Chief of all armed
of illegal detention. forces of the Philippines and whenever it becomes necessary, he may
call out such armed forces to prevent or suppress lawless violence,
The petition must be dismissed for lack of proof, without prejudice to invasion or rebellion. In case of invasion or rebellion, when the public
the filing of another petititon. Under the foregoing circumstances, the safety requires it, he may, for a period not exceeding sixty days,
return of the writ must be taken on its face value considering that, suspend the privilege of the writ of habeas corpus or place the
unless it is in some way traversed or denied, the facts stated therein Philippines or any part thereof under martial law. Within forty-eight
must be taken as true. Moreover, a writ of habeas corpus should not hours from the proclamation of martial law or the suspension of the
issue where it is not necessary to afford the petitioner relief or where it privilege of the writ of habeas corpus, the President shall submit a
would be ineffective. The writ of Habeas corpus cannot be used as report in person or in writing to the Congress. The Congress, voting
a means of obtaining evidence. jointly, by a vote of at least a majority of all its Members in regular or

126 | P LATON
special session, may revoke such proclamation or suspension, which
revocation shall not be set aside by the President. Upon the initiative of ISSUES:
the President, the Congress may, in the same manner, extend such (1) Whether or not the suspension of the privilege of the writ of
proclamation or suspension for a period to be determined by the habeas corpus bars a civil action for damages for illegal searches
Congress, if the invasion or rebellion shall persist and public safety conducted by the military personnel and other violations of rights
requires it. and liberties guaranteed under the Constitution.
(2) Who can be held liable for such violations: only the military
The Congress, if not in session, shall, within twenty-four hours following personnel directly involved and/or their superiors as well?
such proclamation or suspension, convene in accordance with its rules
without need of a call. THE  COURT’S  RULE:  Accordingly,  we  grant  the  petition  and  ANNUL  and
SET ASIDE the resolution of the respondent court. Let the case be
The Supreme Court may review, in an appropriate proceeding filed by remanded to the respondent court for further proceedings.
any citizen, the sufficiency of the factual basis of the proclamation of
martial law or the suspension of the privilege of the writ of habeas REASON:   (1)   We   find   merit   in   the   petitioner’s   contention   that   the  
corpus or the extension thereof, and must promulgate its decision suspension of the privilege of the writ of habeas corpus does not
thereon within thirty days from its filing. destroy   petitioners’   right   and   cause   of   action   for   damages   for   illegal  
arrest and detention and other violations of their constitutional rights.
A state of martial law does not suspend the operation of the The suspension does not render valid an otherwise illegal arrest or
Constitution, nor supplant the functioning of the civil courts or detention. What is suspended is merely the right of the individual to
legislative assemblies, nor authorize the conferment of jurisdiction on seek release from detention through the writ of habeas corpus as a
military courts and agencies over civilians where civil courts are able to speedy means of obtaining his liberty.
function, nor automatically suspend the privilege of the writ of habeas
corpus. (2)Doctrine of state immunity from suit cannot be construed as a
blanket license or a roving commission untrammelled by any
The suspension of the privilege of the writ of habeas corpus shall apply constitutional restraint, to disregard or transgress upon the rights and
only to persons judicially charged for rebellion or offenses inherent in, liberties of the individual citizen enshrined in and protected by the
or directly connected with, invasion. Constitution.

During the suspension of the privilege of the writ of habeas corpus, any Article 32 of the Civil Code which renders any public officer or
person thus arrested or detained shall be judicially charged within three employee or any private individual liable in damages for violating the
days, otherwise he shall be released. Constitutional rights and liberties of another, does not exempt
respondents from responsibility.
Related to this is the declaration in the Bill of Rights itself that
the  “right  to  bail  shall  not  be  impaired  even  when  the  privilege   It is also noteworthy that in the United States, the suspension
26
of the writ of habeas corpus is  suspended” of the privilege of the writ is vested in the Congress and not in
the President, through the former may authorize the latter to
The only significant consequence of the suspension of the writ do  so.  “Only  in  the  rarest  of  circumstances  has  Congress  seen  fit  
of habeas corpus is to divest the courts of the power to issue to   suspend   the   writ…At   all   other   times,   it   has   remained   a  
the writ whereby the detention of the person is put in issue. Or, critical check on the Executive, ensuring that it does not detain
in the language of the Court in Aberca v. Ver,   “What   is   individuals  except  in  accordance  with  law”
suspended is merely the right of the individual to seek release
from the detention through the writ of habeas corpus as a If society is really to value liberty and freedom, it must provide
speedy  means  of  obtaining  his  liberty” the means and the method by which the validity of any
deprivation of liberty, any detention may be challenged and
The suspension is supposed to enable the Government to deal scrutinized at the earliest opportunity and in the most
more effectively with the rebellion or invasion that might be expeditious manner. Here, the office is filled up by the writ of
afoot. Even this advantage to the authorities has been limited, habeas corpus
however, by the requirement that those arrested or detained
pursuant to such suspension be judicially charged within three Additional Cases
days or otherwise they should be released (O) Habeas Corpus [7]

Finally, the suspension of the privilege could not be seized Martinez v. Mendoza
upon, however, as a license to violate the rights of the persons
who might be affected thereby. There must still be adherence FACTS: Petitioners are mother and wife of Michael Martinez, who was
allegedly abducted and taken away by 7 persons last November 19,
to the Rule of Law even as the privilege might not be fully
2001 at Sun Valley, Paranaque City. Martinez, is being implicated in the
available in the meantime killing of Dorothy Jones, aka Nida Blanca. It appears that on the evening
on November 19, 2001, the CIDG presented before the media a certain
ABERCA v. VER Philip Medel, Jr who allegedly executed a statement confessing to his
160 SCRA 590 (1988) participation in the killing, naming Michael Martinez as the person who
introduced him to Rod Lauren Strunk, the husband of Nida Blanca and
FACTS: This case stems from alleged illegal searches and seizures and alleged mastermind in the killing. In a televised interview, Medel
other violations of the rights and liberties of plaintiffs by various narrate that he saw Michael Martinez at the CDG at Cam Crame where
intelligence units of the Armed forces of the Philippines, known as Task he was being detained. The respondents vehemently deny any
Force Makabansa (TFM),   ordered   by   General   Fabian   Ver   “to   conduct   participation or involvement in the alleged abduction of Martinez and
pre-emptive strikes against known communist-terrorist underground said that the alleged victim was never confined and detained by them
houses in view of increasing reports about CT plans to sow disturbances or  in  their  custody  at  any  given  time.    The  CA  did  not  believe  Medel’s  
in  Metro  Manila.”  The  defendants  through  their  counsel,  filed  a  motion narration and said that his credibility was highly suspect since, he had
to dismiss which was favorably acted upon by the trial court. contradicted himself as to material facts, as the Court opined. The issue
here is whether the court erred in reversing the trail court and
26 CONSTITUTION, Art. III, § 13 dismissing the petition for habeas corpus.

127 | P LATON
HEL:  The  SC  held  that  the  petitioner’s  anchor  for  the  present case is the
disappearance of Martinez. The matter of his alleged detention is, at
best, merely consequential of his disappearance. The ultimate purpose
for the writ of habeas corpus is to relieve a person from unlawful
restraint. It is a remedy intended to determine whether the person
under detention is held under lawful authority. If the respondents are
neither detaining nor restraining the person on whose behalf the Republic of the Philippines
petition for habeas corpus has been filed, then it should be dismissed. SUPREME COURT
Considering that respondents have persistently denied having Martinez Manila
in their custody, and absent any decisive proof to rebut their denial, the
SC   is  restrained  to   affirm   the   CA’s   dismissal   of   the   petition  for   habeas   A.M. No. 07-9-12-SC (25 September
corpus. 2007)

THE RULE ON THE WRIT OF AMPARO


Manalo V. Calderon
SECTION 1. Petition. – The petition for a writ of amparo is a remedy
FACTS: On May 125, 2007, 5 unidentified men forcibly entered Bgy. available to any person whose right to life, liberty and security is
Pinagbayanan Elementary School in Taysan, Batangas, which served as violated or threatened with violation by an unlawful act or omission of
a polling area for the 2007 National and Local elections. They entered a public official or employee, or of a private individual or entity.
polling precinct 76-A and poured gasoline over a ballot box in and set it The writ shall cover extralegal killings and enforced disappearances or
ablaze. In the investigation that ensued, several eyewitnesses threats thereof.
identified some of petitioners as the perpetrators of the school burning
The investigation also yielded that all petitioners, who are members of SEC. 2. Who May File. – The petition may be filed by the aggrieved
the PNOP regional special operations group, failed to timely respond to party or by any qualified person or entity in the following order:
the incident . Acing on the report, the PNP hierarchy issued 3 successive
memoranda.   The   petitioners   contend   that   these   memoranda   “defines   a. Any member of the immediate family, namely: the spouse,
and   circumscribes”   the   scope   of   petitioner’s   restrictive   custody”   that  
children and parents of the aggrieved party;
“although   technically speaking, they are not detained or imprisoned,
their physical movements are however, limited within Camp Vicente b. Any ascendant, descendant or collateral relative of the
Lim; they cannot go home to their respective families and if they would aggrieved party within the fourth civil degree of
leave Camp Vicente Lim, they need to be escorted. They said that such consanguinity or affinity, in default of those mentioned in
is degrading and their restrictive custody status is illegal. Without the preceding paragraph; or
necessarily giving due course to the petition, the Court required c. Any concerned citizen, organization, association or
respondents to comment. In lieu of the comment, the OSG manifested
institution, if there is no known member of the immediate
that 2 of the memoranda be recalled.
family or relative of the aggrieved party.
HELD: Notwithstanding the mootness of the issues on restrictive
custody and monitoring movements of petitioners, we opt to resolve The filing of a petition by the aggrieved party suspends the right of all
them given a) the paramount public interest involved b) their other authorized parties to file similar petitions. Likewise, the filing of
susceptibility of recurring yet evading review and c) the imperative the petition by an authorized party on behalf of the aggrieved party
need to educate the police community on the matter. suspends the right of all others, observing the order established herein.

The release of petitioners by respondents in a petition for habeas SEC. 3. Where to File. – The petition may be filed on any day and at any
corpus does not automatically abate a decision on the case. Similarly, a time with the Regional Trial Court of the place where the threat, act or
recall of the custody order challenged by petitioners will not necessarily omission was committed or any of its elements occurred, or with the
call for a dismissal on the ground of mootness alone. Although, the Sandiganbayan, the Court of Appeals, the Supreme Court, or any justice
general rule is mootness of the issue warrants a dismissal, there are of such courts. The writ shall be enforceable anywhere in the
well defined exceptions. Courts will decided cases, otherwise moot and Philippines.
academic if 1) there is a grave violation of the constitutions 2)
exceptional character of the situation and paramount public interest is When issued by a Regional Trial Court or any judge thereof, the writ
involved 3) when constitutional issue raised requires formulation of shall be returnable before such court or judge.
controlling principles to guide the bench, bar and public and 4) the case
if capable of repetition yet evading review. When issued by the Sandiganbayan or the Court of Appeals or any of
their justices, it may be returnable before such court or any justice
There is no illegal restraint in the restrictive custody and monitored thereof, or to any Regional Trial Court of the place where the threat,
movements of police officers under investigation. A petition for habeas act or omission was committed or any of its elements occurred.
corpus will be given due course only if it shows that petitioner is being
detained or restrained of his liberty unlawfully. A restrictive custody When issued by the Supreme Court or any of its justices, it may be
and monitoring of movements of whereabouts of police officers under returnable before such Court or any justice thereof, or before the
investigation by their superiors is not a form of illegal detention or Sandiganbayan or the Court of Appeals or any of their justices, or to any
restrained of liberty. Regional Trial Court of the place where the threat, act or omission was
committed or any of its elements occurred.
The ultimate purpose of the write of habeas corpus is to relieve a
person from unlawful restrained. The writ cannot and will not issue SEC. 4. No Docket Fees. – The petitioner shall be exempted from the
absent a showing that petitioners are deprived of their liberty. Neither payment of the docket and other lawful fees when filing the petition.
can it relive petitioners, who are police officers, from the valid exercise The court, justice or judge shall docket the petition and act upon it
of prescribed discipline over them by the PNP leadership. immediately.

SEC. 5. Contents of Petition. – The petition shall be signed and verified


and shall allege the following:

a. The personal circumstances of the petitioner;

128 | P LATON
b. The name and personal circumstances of the respondent iii. to identify witnesses and obtain statements from
responsible for the threat, act or omission, or, if the name is them concerning the death or disappearance;
unknown or uncertain, the respondent may be described by iv. to determine the cause, manner, location and
an assumed appellation; time of death or disappearance as well as any
c. The right to life, liberty and security of the aggrieved party pattern or practice that may have brought about
violated or threatened with violation by an unlawful act or the death or disappearance;
omission of the respondent, and how such threat or v. to identify and apprehend the person or persons
violation is committed with the attendant circumstances involved in the death or disappearance; and
detailed in supporting affidavits; vi. to bring the suspected offenders before a
d. The investigation conducted, if any, specifying the names, competent court.
personal circumstances, and addresses of the investigating
authority or individuals, as well as the manner and conduct The return shall also state other matters relevant to the investigation,
of the investigation, together with any report; its resolution and the prosecution of the case.
A general denial of the allegations in the petition shall not be allowed.
e. The actions and recourses taken by the petitioner to
determine the fate or whereabouts of the aggrieved party SEC. 10. Defenses not Pleaded Deemed Waived. — All defenses shall
and the identity of the person responsible for the threat, act be raised in the return, otherwise, they shall be deemed waived.
or omission; and
f. The relief prayed for. SEC. 11. Prohibited Pleadings and Motions. – The following pleadings
and motions are prohibited:
The petition may include a general prayer for other just and equitable
reliefs. a. Motion to dismiss;
b. Motion for extension of time to file return, opposition,
SEC. 6. Issuance of the Writ. – Upon the filing of the petition, the court, affidavit, position paper and other pleadings;
justice or judge shall immediately order the issuance of the writ if on its c. Dilatory motion for postponement;
face it ought to issue. The clerk of court shall issue the writ under the d. Motion for a bill of particulars;
seal of the court; or in case of urgent necessity, the justice or the judge
e. Counterclaim or cross-claim;
may issue the writ under his or her own hand, and may deputize any
officer or person to serve it. f. Third-party complaint;
The writ shall also set the date and time for summary hearing of the g. Reply;
petition which shall not be later than seven (7) days from the date of its h. Motion to declare respondent in default;
issuance. i. Intervention;
j. Memorandum;
SEC. 7. Penalty for Refusing to Issue or Serve the Writ. – A clerk of
k. Motion for reconsideration of interlocutory orders or
court who refuses to issue the writ after its allowance, or a deputized
person who refuses to serve the same, shall be punished by the court, interim relief orders; and
justice or judge for contempt without prejudice to other disciplinary l. Petition for certiorari, mandamus or prohibition against any
actions. interlocutory order.

SEC. 8. How the Writ is Served. – The writ shall be served upon the SEC. 12. Effect of Failure to File Return. — In case the respondent fails
respondent by a judicial officer or by a person deputized by the court, to file a return, the court, justice or judge shall proceed to hear the
justice or judge who shall retain a copy on which to make a return of petition ex parte.
service. In case the writ cannot be served personally on the respondent,
the rules on substituted service shall apply. SEC. 13. Summary Hearing. — The hearing on the petition shall be
summary. However, the court, justice or judge may call for a
SEC. 9. Return; Contents. – Within seventy-two (72) hours after service preliminary conference to simplify the issues and determine the
of the writ, the respondent shall file a verified written return together possibility of obtaining stipulations and admissions from the parties.
with supporting affidavits which shall, among other things, contain the The hearing shall be from day to day until completed and given the
following: same priority as petitions for habeas corpus.

a. The lawful defenses to show that the respondent did not SEC. 14. Interim Reliefs. — Upon filing of the petition or at anytime
violate or threaten with violation the right to life, liberty and before final judgment, the court, justice or judge may grant any of the
security of the aggrieved party, through any act or omission; following reliefs:
b. The steps or actions taken by the respondent to determine
(a) Temporary Protection Order. – The court, justice or judge, upon
the fate or whereabouts of the aggrieved party and the
motion or motu proprio, may order that the petitioner or the aggrieved
person or persons responsible for the threat, act or party and any member of the immediate family be protected in a
omission; government agency or by an accredited person or private institution
c. All relevant information in the possession of the respondent capable of keeping and securing their safety. If the petitioner is an
pertaining to the threat, act or omission against the organization, association or institution referred to in Section 3(c) of this
aggrieved party; and Rule, the protection may be extended to the officers involved.
d. If the respondent is a public official or employee, the return
The Supreme Court shall accredit the persons and private institutions
shall further state the actions that have been or will still be that shall extend temporary protection to the petitioner or the
taken: aggrieved party and any member of the immediate family, in
accordance with guidelines which it shall issue.
i. to verify the identity of the aggrieved party; The accredited persons and private institutions shall comply with the
ii. to recover and preserve evidence related to the rules and conditions that may be imposed by the court, justice or judge.
death or disappearance of the person identified
in the petition which may aid in the prosecution (b) Inspection Order. — The court, justice or judge, upon verified
motion and after due hearing, may order any person in possession or
of the person or persons responsible;

129 | P LATON
control of a designated land or other property, to permit entry for the SEC. 18. Judgment. — The court shall render judgment within ten (10)
purpose of inspecting, measuring, surveying, or photographing the days from the time the petition is submitted for decision. If the
property or any relevant object or operation thereon. allegations in the petition are proven by substantial evidence, the court
The motion shall state in detail the place or places to be inspected. It shall grant the privilege of the writ and such reliefs as may be proper
shall be supported by affidavits or testimonies of witnesses having and appropriate; otherwise, the privilege shall be denied.
personal knowledge of the enforced disappearance or whereabouts of
the aggrieved party. SEC. 19. Appeal. – Any party may appeal from the final judgment or
order to the Supreme Court under Rule 45. The appeal may raise
If the motion is opposed on the ground of national security or of the questions of fact or law or both.
privileged nature of the information, the court, justice or judge may The period of appeal shall be five (5) working days from the date of
conduct a hearing in chambers to determine the merit of the notice of the adverse judgment.
opposition. The appeal shall be given the same priority as in habeas corpus cases.

The movant must show that the inspection order is necessary to SEC. 20. Archiving and Revival of Cases. – The court shall not dismiss
establish the right of the aggrieved party alleged to be threatened or the petition, but shall archive it, if upon its determination it cannot
violated. proceed for a valid cause such as the failure of petitioner or witnesses
to appear due to threats on their lives.
The inspection order shall specify the person or persons authorized to
make the inspection and the date, time, place and manner of making A periodic review of the archived cases shall be made by the amparo
the inspection and may prescribe other conditions to protect the court that shall, motu proprio or upon motion by any party, order their
constitutional rights of all parties. The order shall expire five (5) days revival when ready for further proceedings. The petition shall be
after the date of its issuance, unless extended for justifiable reasons. dismissed with prejudice upon failure to prosecute the case after the
lapse of two (2) years from notice to the petitioner of the order
(c) Production Order. – The court, justice or judge, upon verified motion archiving the case.
and after due hearing, may order any person in possession, custody or
control of any designated documents, papers, books, accounts, letters, The clerks of court shall submit to the Office of the Court Administrator
photographs, objects or tangible things, or objects in digitized or a consolidated list of archived cases under this Rule not later than the
electronic form, which constitute or contain evidence relevant to the first week of January of every year.
petition or the return, to produce and permit their inspection, copying
or photographing by or on behalf of the movant. SEC. 21. Institution of Separate Actions. — This Rule shall not preclude
The motion may be opposed on the ground of national security or of the filing of separate criminal, civil or administrative actions.
the privileged nature of the information, in which case the court, justice
or judge may conduct a hearing in chambers to determine the merit of SEC. 22. Effect of Filing of a Criminal Action. – When a criminal action
the opposition. has been commenced, no separate petition for the writ shall be filed.
The reliefs under the writ shall be available by motion in the criminal
The court, justice or judge shall prescribe other conditions to protect case.
the constitutional rights of all the parties. The procedure under this Rule shall govern the disposition of the reliefs
available under the writ of amparo.
(d) Witness Protection Order. – The court, justice or judge, upon
motion or motu proprio, may refer the witnesses to the Department of SEC. 23. Consolidation. – When a criminal action is filed subsequent to
Justice for admission to the Witness Protection, Security and Benefit the filing of a petition for the writ, the latter shall be consolidated with
Program, pursuant to Republic Act No. 6981. the criminal action.
The court, justice or judge may also refer the witnesses to other When a criminal action and a separate civil action are filed subsequent
government agencies, or to accredited persons or private institutions to a petition for a writ of amparo, the latter shall be consolidated with
capable of keeping and securing their safety. the criminal action.
After consolidation, the procedure under this Rule shall continue to
SEC. 15. Availability of Interim Reliefs to Respondent. – Upon verified apply to the disposition of the reliefs in the petition.
motion of the respondent and after due hearing, the court, justice or
judge may issue an inspection order or production order under SEC. 24. Substantive Rights. — This Rule shall not diminish, increase or
paragraphs (b) and (c) of the preceding section. modify substantive rights recognized and protected by the Constitution.
A motion for inspection order under this section shall be supported by
affidavits or testimonies of witnesses having personal knowledge of the SEC. 25. Suppletory Application of the Rules of Court. – The Rules of
defenses of the respondent. Court shall apply suppletorily insofar as it is not inconsistent with this
Rule.
SEC. 16. Contempt. – The court, justice or judge may order the
respondent who refuses to make a return, or who makes a false return, SEC. 26. Applicability to Pending Cases. – This Rule shall govern cases
or any person who otherwise disobeys or resists a lawful process or involving extralegal killings and enforced disappearances or threats
order of the court to be punished for contempt. The contemnor may be thereof pending in the trial and appellate courts.
imprisoned or imposed a fine.
SEC. 27. Effectivity. – This Rule shall take effect on October 24, 2007,
SEC. 17. Burden of Proof and Standard of Diligence Required. – The following its publication in three (3) newspapers of general circulation.
parties shall establish their claims by substantial evidence.
The respondent who is a private individual or entity must prove that Chapter 17
ordinary diligence as required by applicable laws, rules and regulations Speedy Disposition of Cases
was observed in the performance of duty.
The respondent who is a public official or employee must prove that
extraordinary diligence as required by applicable laws, rules and “All  persons  shall  have  the  right  to  a  speedy  disposition of their
regulations was observed in the performance of duty. cases before all judicial, quasi-judicial, or administrative
27
The respondent public official or employee cannot invoke the bodies.”
presumption that official duty has been regularly performed to evade
responsibility or liability.

27 CONSTITUTION, Art. III, § 16


130 | P LATON
In order to ensure the right to expeditious determination of
cases is not limited to trials in criminal proceedings, the TATAD V. SANDIGANBAYAN
Constitution has explicitly provided for the same also in 159 SCRA 70 (1988)
proceedings before judicial, quasi-judicial and administrative
FACTS: Antonio de los Reyes, charged petitioner, who was then
bodies.  “This  protection  extends  to  all  citizens,  including  those  
Secretary and Head of the Department of Public Information, with
in the military and covers the periods before, during and after alleged violations of RA 3019, otherwise known as the Anti-graft and
trial, affording broader protection that Section 14(2) which Corrupt Practices Act. In January 1980, the resignation of petitioner was
guarantees  merely  the  right  to  a  speedy  trial.”  In  any  event,  the   accepted by President Marcos. Four months later, the Tanodbayan
rationale for both Section 14(2) and Section 16 of Article III of referred the complaint of delos Reyes to the Criminal Investigation
the  Constitution  is  the  same,  “justice  delayed  is  justice  denied” Servive (CIS) for fact-finding investigation. The CIS investigator
submitted his Investigation Report concluding that evidence indicates
The importance of speedy disposition of cases could not be that petitioner had violated Sec3(e) and Sec 7 of RA 3019, and
recommended appropriate legal action on the matter. In October 1982,
overstated.  “The  office  of  a  judge  exists  for  one  solemn  end--to
all affidavits and counter-affidavits were with the Tanodbayan for final
promote the ends of justice by administering it speedily and disposition but it was only in July 1985 when the Tanodbayan approved
impartially.”Nevertheless,   the   desirability   of   speed   and   a resolution recommending that several informations be filed against
expediency in the disposition of cases should not ride the petitioner before the Sandiganbayan. Petitioner filed with the
roughshod on the rights of the litigants Sandiganbayan a consolidated motion to quash the information
predicated on several grounds including deprivation of due process of
In any event, the guarantee of speedy disposition of cases does law and of the right to a speedy disposition of the cases filed against
not mean a mechanical act of simply counting days or months. him.
It is a relative and flexible concept that accommodates and
ISSUE: Whether or not petitioner was deprived of his constitutional
adapts to the various circumstances that may go into the right  to  due  process  and  the  right  to  “speedy  disposition”  of  the  cases  
proceedings and the issues they bring. What it simply ensures is against him.
freedom from arbitrary, oppressive, vexatious and
unreasonable delays, not absolute right to a specific period. THE   COURT’S   RULE:   After a careful review of the facts and
Expounding on this right which was first explicitly provided for circumstances of this case, we are constrained to hold that the
in the 1973 Constitution, the Court aid in Caballero v. Alfonso, inordinate delay in terminating the preliminary investigation and the
Jr.: filing of the information in the instant case is violative of the
constitutionally granted right of the petitioner to due process and to a
speedy disposition of the cases against him. Accordingly, the
The guarantee of the right to "a speedy disposition of cases," which the
informations in the Criminal Cases should be DISMISSED.
Constitution expressly provides, recognizes the truism that justice
delayed can mean justice denied. Likewise, the broad sweep that the
REASON:
guarantee comprehends, when it provides that the right is available
A painstaking review of the facts cannot but leave the impression
before all judicial, quasi-judicial or administrative bodies, confirms that
that political motivations played a vital role in activating and propelling
the application of the immunity from arbitrary and oppressive delays is
the prosecutorial process in this case.
not limited to an accused in a criminal proceeding but extends to all
The long delay in resolving the case under preliminary investigation
parties and in all cases. Hence, under the constitutional provision, any
cannot be justified on the basis of the facts on record. The law (PD 911)
party to a case may demand expeditious action on the part of all who
prescribes a ten-day period for the prosecutor to resolve a case under
are officially tasked with the proper administration of justice.
preliminary investigation by him from its termination.
A delay of close to three years cannot be deemed reasonable or
However, "speedy disposition of cases" is a relative term. Just like the
justifiable in the light of the circumstance obtaining in the case at bar.
constitutional guarantee of "speedy trial" accorded an accused in all
criminal proceedings, "speedy disposition of cases" is a flexible
concept. It is consistent with delays and depends upon the CADALIN  V.  POEA’S  ADMINISTRATOR
circumstances. What the Constitution prohibits are unreasonable, 238 SCRA 721 (1994)
arbitrary and oppressive delays which render rights nugatory.
FACTS:   Petitioner’s   Cadalin,   Amul   and   Evagelista,   in   their   own   behalf  
In the determination of whether or not the right to a "speedy trial" has and on behalf of 728 other OCWs what instituted a class suit for money
been violated, certain factors may be considered and balanced against claims before the POEA arising from their recruitment by Asia
each other. These are length of delay, reason for the delay, assertion of International Builders Corporation (AIBC) and overseas employment by
the right or failure to assert it, and prejudice caused by the delay. The Brown & Root International Inc. (BRII). In 1989, the POEA rendered its
same factors may also be considered in answering judicial inquiry decision. This was appealed to the NLRC which promulgated its decision
whether or not a person officially charged with the administration of more than two years later. One of the issues raised before the SC is the
justice has violated the "speedy disposition of cases" guarantee. alleged violation of the right to speedy disposition of cases.

To strike down a law on the ground that it violates the guarantee of HELD:
"speedy disposition of cases" requires more than a citation of what may Speedy disposition of cases is a flexible concept, what the
be a misfeasance or malfeasance of a public officer whose duty and Constitution prohibits are unreasonable, arbitrary and oppressive
responsibility it is to apply and administer the law. The challenge must delays which render rights nugatory.
be based on a clear showing that it is the law, or its operation, and not The final disposition in the administrative level after seven years
merely its administration, which invades and impairs constitutionally from their inception, cannot be said to be attended by unreasonable,
protected personal or property rights. In the case at bar, it is true that arbitrary and oppressive delays (complaints undergoing several
the referral of cases to the Department of Agrarian Reform opens the amendments, 1,767 claimants, fights between lawyers of complainants)
door to more bureaucratic red tape and, perhaps, more opportunities as to violate the constitutional rights to a speedy disposition of the
for corrupt practices. The defects in the bureaucratic system do not, cases of complainants.
however, constitute valid arguments against the merits of legislative
policy intended to protect the legitimate tenant-tiller. Besides, it is not Well, one could only speed up so much without burning
for this Court to determine the wisdom of PD 1038. This is a matter left oneself. Depending on the circumstances, each case would
for Congress to re-examine in the exercise of its legislative authority. have to be viewed in its own pace and not simply rushed for the

131 | P LATON
sake of speed. And, as illustrated by Cadalin, delay could be due under duress. Thus, it would not be implicated if it is physical
to the proceedings themselves but also to in-fighting or evidence that is sought to be taken from a person or otherwise
internecine strife among lawyers on the same side of the fence sought to be produced, except when these are personal papers.
“The seizure of books and documents by means of a search
Chapter 18 warrant, for the purpose of using them as evidence in a criminal
Privilege Against Self-Incrimination case against the person in whose possession they were found,
is unconstitutional because it makes the warrant unreasonable,
“No   person   shall   be   compelled   to   be   a   witness   against   and it is equivalent to a violation of the constitutional provision
28
himself” prohibiting the compulsion of an accused to testify against
himself.”   What   is   actually   proscribed   is   the   use   of   physical   or  
Respect  for,  and  recognition  of,  man’s  individuality  and  right  to   moral compulsion to extort communication from the accused
be left alone may be said to be implicated in this right. It is a and not the inclusion of his body in evidence when it may be
form   of   “a   civilized   protest against the use of torture in material
extorting   confessions.”   It   prescribes   an   ‘option   of   refusal   to  
answer incriminating questions and not a prohibition of VILLAFLOR V. SUMMERS
inquiry.” 41 Phil. 62 (1920)

FACTS: Emeteria Villaflor and Florentino Souingco are charged with the
History, Development and Policy Considerations
crime of adultery. The court ordered the defendant Villaflor, herein
petitioner, to submit her body to the examination of one or two
The privilege is one of those advances in the progress of man, competent doctors to determine if she was pregnant or not.
from the barbarous methods of extracting confessions from
unwilling lips to the more humane ways of procuring evidence ISSUE: Whether or not compelling a woman to submit her body to be
which do not rely on the person sought to be prosecuted. It examined by physicians to determine if she was pregnant violates the
may as well be reflective of the direction by which a provision that no person shall be compelled in any criminal case to be a
government deals with and treats its citizens and inhabitants witness against himself.

THE  COURT’S   RULE:   Although  the   order   of   the   trial   judge,   acceding   to  
It was established on the grounds of public policy and the request of the assistant fiscal for an examination on the person of
humanity--of policy, because if the party were required to the defendant by physicians is phrased in absolute terms, it should,
testify, it would place the witness under the strongest nevertheless, be understood as subject to the limitations herein
temptation to commit the crime of perjury, and of humanity, mentioned, and therefore, legal. The writ of habeas corpus prayed for
because it would prevent the extorting of confessions by duress is hereby DENIED.

It had its origin in a protest against the inquisitorial methods of REASON:


Once again we lay down the rule that the constitutional guaranty,
interrogating the accused person, which had long obtained in
that no person shall be compelled in any criminal case to be a witness
the continental system against himself, is limited to a prohibition against compulsory
testimonial self-incrimination.
The privilege against self-incrimination  “registers  an  important   It is a reasonable presumption that in an examination by reputable
advance in the development of our liberty--one of the great and disinterested physicians due care will be taken not to use violence
landmarks  in  man’s  struggle  to  make  himself  civilized and not to embarrass the patient any more than is absolutely
necessary.
Scope of Privilege
BELTRAN V. SAMSON
In determining the circumstances in which the privilege may be 53 Phil. 570 (1929)
invoked, one would have to take into account the nature of the
FACTS:  For  the  purpose  of  comparing  the  petitioner’s  handwriting and
evidence, the personality of the person invoking it, and the determining whether or not it was he who wrote certain documents
proceeding involved. Parenthetically, it is available to one who supposed to be falsified, the fiscal petitioned the lower court to order
might be guilty or one who simply professes innocence. Not herein petitioner to appear before the former to take dictation in
because one claims no wrongdoing that he can not invoke the petitioner’s   own   handwriting.   Petitioner   then   filed a petition for
privilege to remain silent prohibition seeking to enjoin the order of the lower court.

The right, while ordinarily available only in criminal ISSUE:  Whether  the  writing  from  the  fiscal’s  dictation  by  the  petitioner  
for   purpose   of   comparing   the   latter’s   handwriting   and   determining  
prosecutions, extends to all other government proceedings--
whether he wrote certain documents supposed to be falsified,
including civil actions, legislative investigations, and constitutes evidence against himself within the scope and meaning of
administrative proceedings that possess a criminal or penal the constitutional provision under examination.
aspect--but not to private investigations done by private
individuals THE  COURT’S  RULE:  Wherefore,  we  find  the  present  action  well  taken,  
and it is ordered that the respondents and those under their orders
Testimonial v. Physical Evidence desist and abstain absolutely and forever from compelling petitioner to
take down dictation in his handwriting for the purpose of submitting
the latter for comparison.
The privilege is basically directed at testimonial evidence or any
evidence communicative in nature acquired from the accused REASON:
As to its scope, the privilege is not limited precisely to testimony, but
28CONSTITUTION, Art. III, § 17. In the American Bill of Rights, this is among extends to all giving or furnishing of evidence.
those rights guaranteed by the Fifth Amendment,   which   provides:   “No   It is the duty of the courts liberally to construe the prohibition in
person...shall be compelled in any criminal case to be a witness against favor of personal rights, and to refuse permit any steps tending toward
himself,...”
132 | P LATON
their invasion. Hence, there is the well-established doctrine that the hence, incompetent evidence.
constitutional inhibition is directed not merely to giving of oral The constitutional privilege has been defined as a protection against
testimony but embraces as well the furnishing of evidence by other testimonial compulsion, but this has been since then extended to any
means than by word of mouth, the divulging, in short, of any fact which evidence   “communicative   in   nature”   acquired   under   circumstances   of  
the accused has a right to hold secret. duress.
In the case at bar, the question deals with something not yet in
existence, and it is precisely sought to compel the petitioner to make, *This should be distinguished from mechanical acts the accused is
prepare, or produce by this means, evidence not yet in existence; in made to execute not meant to unearth undisclosed facts but to
short, to create this evidence which may seriously incriminate him. ascertain physical attributes determinable by simple observation
(extract virus from body, pregnancy test, expectorate morphine from
* The privilege not to give self-incriminating evidence, while absolute his mouth, footprinting test). In this case, the accused does not speak of
when claimed, may not be waived by any one entitled to invoke it. his guilt.

Related to the foregoing is the issue as to whether requiring a Diverse Proceedings, Different Roles--Accused, Respondents and
suspect to say something in order to hear and identify his voice Witnesses
may violate the privilege. In United States v. Wade, the U.S.
Supreme Court again held that there was no such violation, The privilege applies to all kinds of proceedings in which an
holding thus: incriminating statement may be obtained. Thus, while primarily
it has direct application in criminal proceedings, the privilege
We have no doubt that compelling the accused merely to against self-incrimination may also be invoked in civil or
exhibit his person for observation by a prosecution witness administrative proceedings if the questions asked would tend
prior to trial involves no compulsion of the accused to give to elicit an incriminating answer. It is available to him in a
evidence having testimonial significance. It is compulsion of the pending criminal case in court, or, in a preliminary investigation
accused to exhibit his physical characteristics, not compulsion before   the   public   prosecutor’s   office.   Insofar   as   accused   and  
to disclose any knowledge he might have witnesses are concerned, the privilege means that the former
may absolutely refuse to take the witness stand while the latter
We held in Schmerber,...that the distinction to be drawn may only refuse to answer incriminatory questions when asked
between   an   accused’s   “communications,”   in   whatever   form,  
vocal   or   physical,   and   “compulsion   which   makes   a   suspect   or   An accused when he takes the witness stand correspondingly
accused  the  source  of  ‘real  or  physical  evidence’ waives his right against self-incrimination, at least in so far as
relating to those matters in which he testifies about. Thus, he
Reenactments may be cross-examined on matters he testified about and he
cannot thereafter refuse to answer questions just because they
A person who is made to re-enact a crime may also right fully may tend to incriminate him. He simply cannot eat his cake and
invoke his privilege against self-incrimination, because by his have it, too. Nevertheless, if he is asked an incriminatory
conduct of acting out how the crime was supposedly question in regard to another crime, he may avail of his
committed, he thereby practically confesses his guilt by action privilege against self-incrimination
which is as eloquent, if not more so, than words
Another exception to the right of an accused not to take the
PEOPLE V. OLVIS witness stand is when he is covered by an immunity statute, in
154 SCRA 513 (1987) which case he may not refuse to answer incriminatory
questions
FACTS: Following the report on September 9, 1975 by the siblings of
Deosdedit Bagon that he was missing, an inquiry was conducted by the
police of Polanco, Zamboanga del Norte. After a volunteer informed the
It has also been held that this privilege is something that the
police that Bagon was last seen together with Sorela, one of the person invoking it is supposed to know. Thus, it is not
accused appellants, the police went to pick him up. Then, Sorela incumbent on the judge to remind or warn him about possibly
admitted having participated together with Romulo Villarojo and incriminating questions and answers that may come his way.
Leonardo Cademas(co-accused) in the killing of the missing Bagon. The The constitutional provision--
policemen thereafter made the three re-enact the crime. Initial findings
of investigators disclosed that the threesome of Solero, Villarojo and does not impose on the judge, or other officer presiding over a
Cademas executed Bagon on orders of accused Anacleto Olvis, then the
trial, hearing or investigation, any affirmative obligation to
mayor of Polanco. The three appellants, together with Olvis, were all
charged with murder. After trial, Olvis was acquitted and the appellants
advise a witness of his right against self-incrimination. It is a
were found guilty and sentenced to death. right that a witness knows or should know, in accordance with
the well known axiom that every one is presumed to know the
ISSUE: Whether or not the three accused-appellants’   EXTRAJUDICIAL   law, that ignorance of the law excuses no one. Furthermore, in
CONFESSIONS are inadmissible in evidence. the very nature of things, neither the judge nor the witness can
be expected to know in advance the character or effect of a
THE   COURT’S   RULE:   The   accused   appellants   Leonardo   Cademas   and   question to be put to the latter
Dominador Sorela are ACQUITTED on the ground of reasonable doubt.
The accused-appellant Romulo Villarojo is found guilty of homicide, and
is sentenced to suffer an indeterminate penalty of eight years and one
The right against self-incrimination is not self- executing or
day of prision mayor as minimum, to fourteen years, eight months and automatically operational. It must be claimed. If not claimed by
one day of reclusion temporal as maximum. or in behalf of the witness, the protection does not come into
play. It follows that the right may be waived, expressly, or
REASON: impliedly, as by a failure to claim it at the appropriate time.
Forced re-enactments, like uncounselled and coerced confessions
come within the ban against self-incrimination. We hold that evidence
based on such a re-enactment to be in violation of the Constitution and

133 | P LATON
early  Philippine  case,  the  Court   said:  “[T]o  compel  a  person  to  
produce his private papers to be used in evidence against him
would be equivalent to compelling him to be a witness against
himself”

Corporations and Self-Incrimination

“It  is  elementary  that  the  right  against  self-incrimination has no


application  to  juridical  persons.”  The  basic  reason  for  this  is  the  
Regulatory Reporting Requirements fact that corporations are mere creatures by sufferance of the
State. They could have no existence or personality if they were
In an increasing complex society, the government may require not allowed and recognized by the government. The U.S.
certain reports to be made every now and then for the more Supreme Court explained:
efficient management and ordering of life for everyone, or
simply out of necessity and inevitable consequence of having Historically, private corporations have been subject to broad
multifarious activities and concerns. Thus, papers and reports visitorial power, both in England and in this country. And it long
have to [be] filed and made, income and other taxable has been established that Congress may exercise wide
transactions and occurrences have to be recorded and investigative power over them, analogous to the visitorial
reported, and other studies undertaken and submitted for power of the incorporating state, when their activities take
monitoring, revision or approval by the appropriate place within or affect interstate commerce. Correspondingly, it
government agencies. In doing so, some aspect of such has been settled that corporations are not entitled to all of the
requirements may be perceived as providing incriminating constitutional protections which private individuals have in
information these and related matters. As has been noted, they are not at
all within the privilege against self-incrimination, although this
Court more than once has said that the privilege runs very
closely with the Fourth Amendment's search and seizure
Silence and Guilt provisions. It is also settled that an officer of the company
cannot refuse to produce its records in his possession upon the
The privilege should be understood to mean that invoking it is plea that they either  will  incriminate  him  or  may  incriminate  it…
not a sign or admission of guilt. Silence should not be read as an
implied incrimination in itself, otherwise the right would Immunity Statutes
become meaningless since he means by which one tries to keep
himself away from prison would become the very vehicle which While the Bill of Rights guarantee may give the impression that
he is delivered the privilege against self-incrimination is absolute, the courts
have recognized that a person may still be compelled to testify
It   has   also   been   held   that   “[e]xercise   of   such   privilege   can   even if it will incriminate him in the process. In exchange of
neither be equated with guilt nor be treated as a forbidden allowing the government to obtain necessary evidence which
failure to cooperate with a proper inquiry and used by may lead to an admission by a person of his criminal
government as the basis for adverse treatment, including denial wrongdoing, it is required that a protection equivalent or co-
of a public benefit extensive to that guaranteed by Self-Incrimination Clause be
accorded the witness. This comes in the form of the so-called
Nevertheless, the foregoing should be distinguished from a immunity statutes,  which  may  either  be  in  the  nature  of  a  “use  
situation in which a person keeps quiet when incriminating or  derivative  use”  immunity  or  a  “transactional”  immunity.  The  
statements are being made by others and there is a failure to former simply means that whatever s elicited from the witness,
say anything in refutation as well as any other evidence which the investigators were led
to because of the testimony given, would not be admissible in
Rule 130, Section 32 of the Rules of Court provides that an act evidence against the witness. In the latter, the witness is
or declaration made in the presence and within the hearing or immunized from prosecution in relation to the crime in which
observation of a party who does or says nothing when the act he was compelled to provide testimony
or declaration is such as naturally to call for action or comment
if not true, and when proper and possible for him to do so, may
be given in evidence against him

The problem here is in determining when silence is golden, and


when it is obviously counterproductive. It should ever be
remembered, however, that the privilege of silence is there to
be availed of and not to provide unnecessarily adverse In Commission on Elections v. Tagle, the Supreme Court upheld
inferences simply because it was exercised the right of the Commission on Elections to grant immunity to
those who voluntarily give information against those
Self-Incrimination and Unreasonable Searches responsible for vote-buying

The privilege against self-incrimination is also directly related to Incrimination in Foreign Jurisdictions
the right against unreasonable searches and seizures. In an

134 | P LATON
May the privilege be invoked if the possibility of incrimination is a) those committed while he was in the service of the Marcos
in regard to criminal laws of other countries and not within the government; b) those committed in behalf of the Marcos government;
jurisdiction of the State in which the proceedings are held? In and c) any other act revealed by him in the course of his cooperation
with the PCGG.
United States v. Balsys, the U.S. Supreme Court said no--at
2.   Yes.  EO   No.   14   defines  “the   jurisdiction   over   cases  involving   the   ill-
least, for the moment gotten wealth of former President Ferdinand E. Marcos, Mrs. Imelda R.
Marcos, members of their immediate family, close relatives,
subordinates, close and/or business associates, dummies, agents and
nominees.”   SECTION   5,   as   amended   authorizes   the   PCGG   to   grant  
The Bill of Rights guarantees the freedom to freely speak but immunity from criminal prosecution. iT also does not provide any
recognizes as well the moment when one may be better off express limitations as to the scope of immunity from criminal
being silent--for his own protection. Either way, he could pick prosecution that the PCGG is authorized to grant. The qualifications
on the menu of rights and freedoms with which to feast as he that Section 5 do provide relate to the character of the information or
testimony before the PCGG of the grantee of immunity.
goes along enjoying or simply living life in a libertarian society
3. No. Section 5 of E.O. 14-A does not make any qualification as to
classes of criminal acts, offenses, or cases, it COMPLETELY immunizes
Additional Cases from prosecution.
(P) Self-Incrimination Clause [3]
P3. Disini v. Sadiganbayan
GR. No. 180564 June 22 2010

P2. Tanchanco v. Sandiganbayan Facts: In 1989 respondent RP, represented by PCGG wanted petitioner
476 SCRA 202 (2005) Jesus P. Disini to testify for his government in its case against
Westinghouse and an arbitration case which the latter and others filed
Facts: Tanchanco was the NFA Administrator from 1972-1986 during against RP before the International Chamber of Commerce Court of
the Marcos regime and his co-petitioner, Romeo Lacson was the Arbitration. Petitioner was an executive in the companies of his 2nd
Deputy Administrator of the NFA. On May 6, 1988, Tanchanco and the cousin, Herminio T. Disini, from 1971-1984. RP believed that
PCGG entered in to a Cooperation Agreement wherein Tanchanco Westinghouse's contract for the construction of the Bataan Nuclear
would cooperate with the RP in locating and pursuing government Power Plant, brokered by one of Herminio's companies, had been
properties stolen by the Marcos family. It was also agreed that RP attended by anomalies. On Feb 16, 1989, RP and petitioner entered
would dismiss all cases against Tanchanco pending before the into an Immunity Agreement wherein the latter undertook to testify for
Sandiganbayan, lift any sequestration orders against Tanchanco's his gov't and provide its lawyers the necessary documents and
properties, if any, rescind hold orders it may have issued against his/her information. RP also guaranteed that it shall not compel Disini to testify
actions, and shall not bring any civil or criminal charges against him in any other proceeding brought by RP against Herminio. Petitioner
arising from his service for the Marcos government and any other complied with the said agreement however after 18 yrs, or on Feb 27,
actions revealed by him. Thereafter, Tanchanco was called upon to 2007, upon application of RP, respondent Sandiganbayan, issued a
testify in a case against Imelda Marcos for violation of the RICO Act subpoena duces tecum and ad testificandum against Disini
wherein he elicited concerning the transfer of P10 M rebate obtained commanding him to testify and produce documents before such court
by the NFA from the Philippine National Lines to the Security Bank. on March 6 and 30 2007 in a case filed by RP against Herminio. Disini
Although he cooperated, a criminal case was filed against him with the filed a motion to quash subpoena invoking his immunity agreement
Sandiganbayan for malversation of public funds amounting to P10 M with the Republic, but respondent Sandiganbayan ignored the motion
from the PNB. He filed a Motion for Reinvestigation, wherein he argued and issued a new subpoena, directing him to testify before it on May 6
that the case should be dismissed as he had been granted immunity by and 23, 2007. On July 19, 2007 the PCGG issued a resolution revoking
the PCGG. The case was dismissed. In 1997, Tanchanco was charged and nullifying the Immunity Agreement between petitioner Disini and
with Malversation of Public Funds and Failure of Accountable Officer to RP since it prohibited the latter from requiring Disini to testify against
Render Accounts. He filed a motion to quash because the Cooperation Herminio. On August 16, 2007 respondent Sandiganbayan denied
Agreement granted immunity to Tanchanco from criminal prosecution. Disini's motion to quash subpoena,which made the latter take recourse
Motion was denied. Sandiganbayan claimed that Sec 5 of E.O. to this Court.
14 empowered the PCGG to grant immunity from criminal prosecution
which pertained only to offenses which may arise from the act of a Issues:
person testifying or giving information in connection with the recovery WON the PCGG acted within its authority when it revoked and nullified
of supposed ill-gotten wealth and that charges of malversation and the Immunity Agreement between respondent RP and petitioner Disini
failure to render an accounting could not be considered as falling within WON respondent Sandiganbayan gravely abused its discretion when it
the immunity because they were not related or connected to the denied petitioner Disini's motion to quash the subpoena addressed to
testimony or information furnished by Tanchanco in a proceeding him.
concerning the recovery of the purported ill-gotten wealth of the
Marcoses. Sandiganbayan declared in the motion for reconsideration Held: The guarantee given by PCGG covers only immunity from civil or
that the grant of immunity to crimes to which petitioners were charged criminal prosecution and not immunity from providing evidence in
are beyond the authority and mandate of the PCGG. court. The immunity from the need to testify in other ill-gotten wealth
cases would defeat the very purpose of EO 1 which charged the PCGG
Issues: with the task of recovering all ill-gotten wealth of former President
WON the grant of immunity under the Cooperation Agreement Marcos. Moreover, if he refuses to testify in those other cases, he
encompassed the malversation and failure to render accounts charges. would face indirect contempt and would be penalized. His refusal to
Whether the PCGG, in entering into the Cooperation Agreement, acted testify as ordered by the Sandiganbayan would result in prosecution for
within the scope of its statutory authority to extend immunity in the criminal contempt.
first place
Whether the available immunity from criminal prosecution relates only RP invokes Sec 15 , Art XI of the 1987 Constitution which provides: "The
to the prosecution of the grantee in like minded cases. right of the State to recover properties unlawfully acquired by public
officials or employees from them or from their nominees, or
Held: transferees, shall not be barred by prescription, laches or estoppel."
1. Yes. The undertakings expressed in the Cooperation Agreement are However, the estoppel which Disini invokes does not have the effect of
quite clear-cut, even if broad in scope. It is composed of 3 classes of denying the state its right to recover whatever ill-gotten wealth
actions committed by Tanchanco: Herminio may have acquired under the Marcos regime. And although

135 | P LATON
the government cannot be barred by estoppels, such principle cannot United States v. Pompeya
apply to this case since PCGG acted within its authority when it 31 Phil. 245 (1915)
provided Disini with a guarantee against having to testify in other cases.
Petition is GRANTED and PCGG resolution is ANNULLED. The Facts: For his alleged failure, without justifiable motive, to render
service on patrol duty, Silvestre Pompeya was charged with the
Chapter 19 violation of a municipal ordinance enacted pursuant to Section 40 of
Political Prisoners and Involuntary Servitude the Municipal Code (Act. No. 82, as amended by Act No. 1309) of Iloilo,
which required each able-bodied male resident of the municipality,
when so required by the municipal president to assist in the
“No   person   shall   be   detained   solely   by   reason   of   his   political   maintenance of peace and good order in the community, by
beliefs and aspirations. apprehending ladrones, etc., as well as by giving information of the
existence of such persons in the locality. Upon arraignment, he
“No   involuntary   servitude   in   any   form   shall   exist   except as presented a demurrer on the ground that the acts charged in the
punishment for a crime whereof the party shall be duly complaint do not constitute a crime. He argued that the municipal
29 ordinance alleged to be violated is unconstitutional because it is
convicted.”
repugnant to the Organic Act of the Philippines which guarantee the
liberty of the citizens. The lower court sustained said demurrer and
Consistent with the underlying philosophy of a society which
ordered the dismissal of said complaint. The prosecuting attorney
values individual freedom and self-determination and appealed.
guarantees freedom of thought and of the mind, it is only
logical that nobody should be imprisoned solely because of The Issue: Whether or not the ordinance upon which said complaint
political beliefs and aspirations was  based  (paragraph  “m”  of  section  40  of  the  Municipal  Code)  which  
was adopted in accordance with the provisions of Act No. 1309 is
And in a society that also values liberty, involuntary servitude constitutional.
should be anathema. Individuals are free to make their own
Held: To authorize the municipal president to require able-bodied male
decisions, to move about and realize their dreams. They could
residents of the municipality, between ages of eighteen and fifty years,
not   be   held   in   bondage.   “Slavery   and   involuntary   servitude,   to assist, for a period not exceeding five days in any one month, in
together   with   their   corollary,   peonage,   all   denote   ‘a   condition   apprehending ladrones, robbers and other lawmakers and suspicious
of  enforced,  compulsory  service  of  one  to  another.’ characters, and to act as patrols for the protection of the municipality,
not exceeding one day in each week.
The only time when they may be forced to do things, in
accordance with the language of the Constitution, is when they The question we have to consider is whether or not the facts stated in
have been found guilty of a crime for which they may rightfully the complaint are sufficient to show a (a) cause of action under the said
law; and (b) whether or not said law is in violation of the provisions of
be compelled to do things in accordance with their punishment.
the Philippine Bill in depriving citizens of their rights therein
Another exception would be in regard to service for the guaranteed.
defense of the country, i.e.,   “[t]he   Government   may   call   upon  
the people to defend the State and, in the fulfillment thereof, We are of the opinion, and so hold, that the power exercised under the
all citizens may be required, under conditions provided by law, provisions of Act No. 1309 falls within the police power if the state and
to   render   personal   military   or   civil   service.”   Related   to   this   is   that the state was fully authorized and justified in conferring the same
the concept of posse comitatus, where able-bodied me may be upon the municipalities of the Philippine Islands, and that, therefore,
called upon to contribute their share in services for the the provisions of said act and not in violation nor derogation of the
rights of the persons affected thereby.
maintenance of peace and order in their own community
With reference to the first question presented by the appeal, relating to
In Aclaracion v. Gatmaitan, a court stenographer, after he had the sufficiency of the complaint, it will be noted that Act No. 1309
ceased to be a court stenographer, was compelled to transcribe authorized the municipal governments to establish ordinances
his stenographic notes despite hi contention that forcing him to requiring (a) all able-bodied male residents, between the ages 18 and
do so would constitute involuntary servitude. The Court 55 [50], and (b) all householders, under certain conditions, to do certain
dismissed such argument by declaring that the situation of things.
enforced, compulsory service of one to another or the
condition of one who is compelled by force, coercion, or Of course, in the ultimate analysis, men and women are only as
imprisonment, and against this will, to labor for another, free as they can let themselves be. The Constitution is there to
whether he be paid or not, simply does not obtain guarantee the right but they must take the necessary steps to
assert and exercise their freedoms, safeguard their liberties--as
From the earliest historical period, the contract of the sailor has well as pay the price for their own actions and decisions
been treated as an exceptional one, and involving, to a certain
extent, the surrender of his personal liberty during the life of Chapter 20
the contract. Indeed the business of navigation could scarcely Excessive Fines and Cruel Punishments
be carried on without some guaranty, beyond the ordinary civil
remedies upon contract that the sailor will not desert the ship “Excessive   fines   shall   not   be   imposed, nor cruel, degrading or
at a critical moment or leave her at some place where seamen inhuman punishment inflicted. Neither shall death penalty be
are impossible to be obtained--as Molloy forcibly expresses it, imposed, unless, for compelling reasons involving heinous
“to  rot  in  her  neglected  brine” crimes, the Congress hereafter provides for it. Any death
penalty already imposed shall be reduced to reclusion
perpetua.
29 CONSTITUTION, Art. III, § 18(1) and (2). Its counterpart in the American
Constitution  is  the  Thirteenth  Amendment  which  states:  “Neither  slavery  nor   “The   employment   of   physical,   psychological,   or   degrading  
involuntary servitude, except as a punishment for the crime whereof the party punishment against any prisoner or detainee or the use of
shall have been duly convicted, shall exist within the United States, or any
place subject to their jurisdiction
136 | P LATON
substandard or inadequate penal facilities under subhuman punishment rather than its severity in respect of duration or
30
conditions  shall  be  dealt  with  by  law.” amount, and apply to punishments which never existed in
America or which public sentiment has regarded as cruel or
Society, to be viable, has to have order and peace in the obsolete, such as those inflicted at the whipping post, or in the
community. For this, purpose, it has to come up with rules of pillory, burning at the stake, breaking on the wheel,
conduct by which certain acts or activities are either required disemboweling, and the like
to be done, or prohibited from being engaged in. If these rules
are violated, and after the transgressor shall have been Bail, fines, and punishment traditionally have been associated
proceeded against in accordance with the proper procedure, with the criminal process, and by subjecting the three to
the corresponding punishment may then be imposed on them parallel limitations the text of the Amendment suggests an
as   a   way   of   vindicating   society’s   rights,   and   those   of   the   intention to limit the power of those entrusted with the
offended party, or for the purpose of deterring future wrong criminal-law function of government. An examination of the
doings or infractions. But not because the State has the right history of the Amendment and the decisions of this Court
to criminalize certain acts or omissions and penalize construing the proscription against cruel and unusual
infractions of its rules and regulations does it mean that it punishment confirms that it was designed to protect those
may impose any and all kinds of penalties or punishments, convicted of crimes. We adhere to this longstanding limitation
irrespective of their reasonableness, severity, or and hold that the Eighth Amendment does not apply to the
proportionality to the offense committed.  “There  is  also  merit   paddling of children as a means of maintaining discipline in
in the view that punishment inflicted beyond the merit of the public schools
offense  is  so  much  punishment  of  innocence.”  Speaking  of  the  
Eight Amendment, the U.S. Supreme Court said that the basic In Zaldivar v. Sandiganbayan, the Court held:
concept underlying the constitutional guarantee is nothing less
than the dignity of man x x x “The   indefiniteness   of   the   respondent's   suspension,   far   from  
being 'cruel' or 'degrading' or 'inhuman' has the effect of
It regulates the power of the State to punish. Thus, it prohibits placing, as it were, the key to the restoration of his rights and
the imposition of excessive fines and infliction of cruel, privileges as a lawyer in his own hands. That sanction has the
degrading or inhuman punishments. While it allows the effect of giving respondent the chance to purge himself in his
imposition of the death penalty, it limits it to heinous crimes own good time of his contempt and misconduct by
only acknowledging such misconduct, exhibiting appropriate
repentance and demonstrating his willingness and capacity to
Origin and Development live up to the exacting standards of conduct rightly demanded
from every member of the bar and the officer  of  the  courts”
"The prohibition in the Philippine Bill against cruel and unusual
punishments is an Anglo-Saxon safeguard against governmental In Trop v. Dulles, the U.S. Supreme Court held that expatriation
oppression of the subject, which made its first appearance in or denationalization as a punishment is barred by the Eight
the reign of William and Mary of England in 'An Act declaring Amendment.   It   declared:   “There   may   be   involved   no   physical  
the rights and liberties of the subject, and settling the mistreatment, no primitive torture. There is, instead, the total
succession of the crown,' passed in the year 1689. It has been destruction of the individual's status in organized society. It is a
incorporated into the Constitution of the United States (of form of punishment more primitive than torture, for it destroys
America) and into most constitutions of the various States in for the individual the political existence that was centuries in
substantially the same language as that used in the original the  development”
statute. The exact language of the Constitution of the United
States is used in the Philippine Bill" Weems v. United States
217 U.S. 349, 54 L Ed 793, 30 S Ct 544 (1910)
In   the   1935   Constitution,   there   was   reference   to   “cruel   and
The Facts: Weems, a disbursing officer of the Coast Guard and
unusual   punishment”   but   in   the   1973   Constitution   it   was  
Transportation of the United States Government of the Philippine
modified   to   “cruel   or unusual   punishment.”   Then,   in   the   1987   Islands was found guilty of falsification of a public and official document
Constitution   it   was   written   as   “cruel,   degrading,   or inhuman namely, a cash book of the captain of the port of Manila, by entering as
punishment.”   “The   Bill   of   Rights   Committee   of   the   1986   paid  out  208  and  408  pesos,  as  “wages  of  employees  of  the  lighthouse  
Constitutional Commission read the 1973 modification as service  of  the  United  States  Government  of  the  Philippine  Islands.”  He  
prohibiting  ‘unusual’  punishment  even  if  not  ‘cruel.’  It  was  thus   was   sentenced   to   the   “penalty   of   fifteen   years   of cadena,”   together  
seen as an obstacle to experimentation in penology. with the accessories and to pay a fine of 4,000 pesetas. The judgment
Consequently, the Committee reported out the present text and sentence were affirmed by the Philippine Supreme Court. He
appealed to the U.S. Supreme Court. In his fourth assignment of error,
which prohibits 'cruel, degrading or inhuman punishment' as
he contends that the penalty imposed on him is violative of the
more consonant with the meaning desired and with guarantee of the Philippine Bill of Rights, as contained in the Act of 1
jurisprudence on the subject." July 1902, against cruel unusual punishments of the Philippine Bill of
Rights.
Excessive Fines and Cruel Punishments
The Issue: Whether   or   not   that   a   punishment   of   fifteen   years’   of  
This is primarily addressed to the lawmakers, however, subject cadena together with the accessories and to pay a fine of 4,000 pesetas
to judicial review. The prohibition of cruel and unusual was a cruel and unusual punishment?
punishments is generally aimed at the form or character of the
Held: “Those  sentenced  to  cadena temporal and cadena perpetua shall
labor for the benefit of the state. They shall aways carry a chain at the
30CONSTITUTION, Art. III, § 19(1) and (2). The U.S. counterpart is found in the ankle, hanging from the wrists; they shall be employed at hard and
Eight Amendment which decrees:   “Excessive   bail   shall   not   be   required,   not  
painful labor, and shall receive no assistance whatsoever from without
excessive fines imposed, nor cruel and unusual punishments inflicted
137 | P LATON
the  institution.”   (1) that the death penalty law is unconstitutional per se for having
been enacted in the absence of compelling reasons therefore; and
These provisions are attacked as infringing that provision of the Bill of (2) that the death penalty for rape is a cruel, excessive and inhuman
Rights of the islands which forbids the infliction of cruel and unusual punishment In violation of the constitutional proscription against
punishment. punishment of such nature.
What constitutes a cruel and unusual punishment has not been exactly
decided. It has been said that, ordinarily, the terms imply something “We   have   already   demonstrated earlier in our discussion of heinous
inhuman and barbarous – torture and the like. crimes that the forfeiture of life simply because life was taken, never
was a defining essence of the death penalty in the context of our legal
“The   general   rule   that   a   judgment   rendered   by   a   court   in   a   criminal   history and cultural experience; rather, the death penalty is imposed in
case must conform strictly to the statute, and that any variation from heinous crimes because the perpetrators thereof have committed
its provisions, either in the character or the extent of punishment unforgivably execrable acts that have so deeply dehumanized a person
inflicted,  renders  the  judgment  absolutely  void.” or criminal acts with severely destructive effects on the national efforts
to lift the masses from abject poverty through organized governmental
“In  cases  where  the  statute  makes  hard  labor  a  part  of  the  punishment,   strategies  based  on  a  disciplined  and  honest  citizenry.”
it  is  imperative  upon  the  court  to  include  than  in  its  sentence.”
“Capital   punishment   ought   not   to   be   abolished   solely   because   it   is  
The Philippine Code unites the penalties of cadena temporal, principal substantially repulsive, if infinitely less repulsive than the acts which
and accessory, and it is not in our power to separate them, even if they invoke it. Yet the mounting zeal for its abolition seems to arise from a
are separable, unless their independence is such that we can say that sentimentalized hyper fastidiousness that seeks to expunge from the
their union was not made imperative by the legislature. society all that appears harsh and suppressive. If we are to preserve the
humane society, we will have to retain sufficient strength of character
It follows from these views that, even if the minimum penalty of and will to do the unpleasant in order that tranquility and civility may
cadena temporal had been imposed, it would have been repugnant to rule comprehensively. It seems very likely that capital punishment is a
the Bill of Rights. In other words, the fault is in the law; and, as we are *** necessary, if limited factor in that maintenance of social tranquility
pointed to no other under which sentence can be imposed, the and ought to be retained on its ground. To do otherwise is to indulge in
judgment must be reversed, with directions to dismiss the proceedings. the luxury of permitting a sense of false delicacy to reign over the
necessity  of  social  survival.”
Death Penalty
WHEREFORE, in view of all foregoing, the Motion for Reconsideration
and the Supplemental Motion for Reconsideration are hereby DENIED
One of the most contentious issues when it comes to
for LACK OF MERIT.
punishments is the constitutionality or perhaps the advisability
and wisdom of the death penalty. The language of the Echegaray v. Secretary of Justice
Constitution itself implies that the supreme penalty may be 297 SCRA 754 (1998)
provided for by Congress. The issue therefore boils down to the
conditions which must attend the enactment and the The Facts: Following the decision in People v. Echegaray, supra.,
subsequent application of such a law. Before such a law be petitioner filed a Petition for Prohibition, Injunction and/or Temporary
enacted, it is required that there be compelling reasons and Restraining Order to enjoin respondents Secretary of Justice and
that it is for heinous crimes. The problem in determining what Director of Bureau of Prisons from carrying out execution of petitioner
by lethal injection.
would   constitute   “compelling   reasons”   and   defining   what   are  
“heinous   crimes.”   Then   even   after   one   may   have   succeeded,   Contending that these are unconstitutional and void for being, inter
there is still the question as to whether the manner or method alia, cruel, degrading and inhuman punishment per se as well as
of execution is constitutional, or whether it is within arbitrary, unreasonable and a violation of due process.
constitutional limits to apply the death penalty to certain types
of persons Earlier, in 1996, Congress had seen it fit to change the mode of
execution of the death penalty from electrocution to lethal injection,
People v. Echegaray and passed R.A. No. 8177.
267 SCRA 682 (1997)
The Issue: Whether or not execution by lethal injection is
The Facts: Accused – appellant Leo Echegaray was found guilty if raping unconstitutional and void for being, inter alia, cruel, degrading and
his ten-year-old daughter for he was sentenced to death. The Supreme inhuman punishment per se as well as arbitrary, unreasonable and a
Court affirmed the same on appeal. He filed a motion for violation of due process.
reconsideration. Subsequently, he discharged his defense counsel and
retained the services of Anti-Death Penalty Task Force of the Free Legal Held:
Assistance Group (FLAG). Thereafter, FLAG filed a Supplemental Motion I. LETHAL INJECTION, NOT CRUEL, DEGRADING OR INHUMAN
for Reconsideration on behalf of Echegaray. It raised, among the PUNISHMENT UNDER SECTION 19, ARTICLE III OF THE 1987
grounds for reversal of the death sentence, the issue of the CONSTITUTION
constitutionality of R.A. 7659 (The Death Penalty Law), which took
effect on 31 December 1993, in that it violates the constitutional ‘An  Act  declaring  the  rights  and  liberties  of  the  subject,  and  settling  the  
prohibition on cruel, inhuman and excessive punishments. succession  of  the  crown’

The Issue: Whether or not R.A. 7659 (The Death Penalty Law), which It is well-settled in jurisprudence that the death penalty per se is not a
took effect on 31 December 1993, violates the constitutional cruel, degrading or inhuman punishment.
prohibition on cruel, inhuman and excessive punishments.
This  Court  held  that  “[p]unishments  are  cruel  when  they  involve  torture  
Held: “The  penalty  complained  of  is  neither  cruel,  unjust  nor  excessive.     or a lingering death; but the punishment of death is not cruel, within
Punishments are cruel when they involve torture or a lingering death, the meaning of that word as used in the constitution. It implies there
but punishment of death is not cruel, within the meaning of that word something inhuman and barbarous, something more than mere
as used in the constitution. It implies there something inhuman and extinguishment  of  life.”
barbarous,  something  more  than  the  mere  extinguishment  of  life.”
Third paragraph of R.A. No. 8177 which requires that all personnel

138 | P LATON
involve in execution proceedings should be trained prior to the for their crimes.
performance of such task.
Our dependent evaluation of the issue reveals no reason to disagree
Any infliction of pain in lethal injection is merely incidental in carrying with the judgment  of  “the  legislatures  that  have  recently  addressed  the  
out the execution of the death penalty and does not fall within the matter”   and   concluded   that   death   is   not   a   suitable   punishment   for   a  
constitutional proscription against cruel, degrading or inhuman mentally retarded criminal. We are not persuaded that the execution of
punishment. mentally retarded criminals will measurably advance the deterrent or
the retributive purpose of the death penalty. Construing and applying
The cruelty against which the Constitution protects a convicted man is the   Eight   Amendment   in   the   light   of   our   “evolving   standards   of  
cruelty inherent in the method of punishment, not necessary suffering decency,”  we  therefore  conclude  that  such  punishment  is  excessive  and  
involved in any method employed to extinguish life humanely. that   the   Constitution   “places   a   substantive   restriction   on   the   State’s  
power  to  take  the  life”  of  a  mentally  retarded  offender.
We  find  that  the   legislature’s  substitution  of   the  mode  of  carrying  out  
the death penalty from electrocution to lethal injection infringes no Prison  Life  and  Inmates’  Rights
constitutional rights of petitioner herein.
Being sent to prison does not necessarily and effectively cut off
II. REIMPOSITION OF THE DEATH PENALTY LAW DOES NOT VIOLATE
INTERNATIONAL TREATY OBLIGATIONS all  of  a  person’s  entitlement  to  rights  and  freedom  enshrined  in  
the Constitution. Its most immediate effect would, of course,
Second Optional Protocol to the International Covenant on Civil and be the loss of liberty. As the Court said, in People v. Jalosjos:
Political Rights, Aiming at the Abolition of the death Penalty was
adopted by the General Assembly on December 15, 1989. The More explicitly, "imprisonment" in its general sense, is the
Philippines neither signed nor ratified said document. Evidently, restraint of one's liberty. As a punishment, it is restraint by
petitioner’s   assertion   of   our   obligation   under   the   Second Optional judgment of a court or lawful tribunal, and is personal to the
Protocol is misplaced.
accused. The term refers to the restraint on the personal liberty
WHEREFORE, the petition is DENIED insofar as petitioner seeks to of another; any prevention of his movements from place to
declare the assailed statute (Republic Act No. 8177) as unconstitutional. place, or of his free action according to his own pleasure and
will. Imprisonment is the detention of another against his will
Atkins v. Virginia depriving him of his power of locomotion and it "[is] something
536 U.S. 304, 153 L Ed 2d 335, 122 S Ct 2242 (2002) more than mere loss of freedom. It includes the notion of
restraint within limits defined by wall or any exterior barrier."
The Facts: Petitioner, Daryl Renard Atkins, was convicted of abduction,
armed robbery and capital murder, and sentenced to death. Atkins did “A  prisoner  does  not  shed  such  basic  First  Amendment  rights  at  
not argue before the Virginia Supreme Court that his sentence was
the  prison  gate.  Rather,  he  ‘retains  all  the  rights  of  an  ordinary  
disproportionate to the penalties imposed for similar crimes in Virginia,
but   he   did   contend   “that   he   is   mentally   retarded   and   thus   cannot   be  
citizen except those expressly, or by necessary implication,
sentenced   to   death.”   The   Court   was   “not   willing   to   commute   Atkins’   taken  from  him  by  law.’”
sentence  of  death  to  life  imprisonment  merely  because  his  IQ  score.”
In Wolf v. McDonell, [i]t then declared that the following
The Issue: The American public, legislators, scholars, and judges have safeguards should be present: (1) advance written notice of
deliberated over the question whether the death penalty should ever charges; (2) written statement by the fact-finders as to the
be imposed on a mentally retarded criminal. The evidence relied on and reasons for the disciplinary action; and
consensus reflected in those deliberation informs our answer to the
(3) the prisoner should be allowed to call witnesses and present
question presented by this case: whether such executions   are   “cruel  
and  unusual  punishments”  prohibited  by  the  Eight  Amendment  to  the  
documentary evidence in his defense if permitting him to do so
Federal Constitution. will not jeopardize institutional safety or correctional goals

Held: “The   basic   concept   underlying   the   Eight   Amendment   is   nothing   With respect to prison mail censorship, it was also ruled that
less   than  the   dignity   of   man…The  Amendment  must   draw   its  meaning   the following criteria be met: First, the regulation or practice in
from the evolving standards of decency that mark the progress of a question must further an important or substantial
maturing  society.” governmental interest unrelated to the suppression of
expression. Second, the limitation of First Amendment
Mentally retarded persons frequently know the difference between
right and wrong and are competent to stand trial. Because of their
freedoms must be no greater than is necessary or essential to
impairments, however, by definition they have diminished capacities to the protection of the particular governmental interest involved.
understand and process information, to communicate, to abstract form In this regard, it must be remembered that censorship of
mistakes and learn from experience, to engage in logical reasoning, to prisoner mail implicates not only the   inmate’s   freedom   to  
control impulses, and to understand the reactions of others. communicate  but  also  the  addressee’s  own  constitutional  right

Their deficiencies do not warrant an exemption from criminal The  Court’s  task  then  “is  to  formulate  a  standard  of  review  for  
sanctions, but they do diminish their personal culpability.
prisoners’   constitutional   claims   that   is   responsive   both   to   the  
In light of these deficiencies, our death penalty jurisprudence provides
‘policy  of  judicial  restraint  regarding  prisoner complaint and to
two reasons consistent with the legislative consensus that the mentally the   need   to   protect   constitutional   rights.’”   It   proceeded   to  
retarded should be categorically excluded from execution. First, there is conclude that a lesser standard of review than a strict scrutiny
a serious question as to whether either justification that we have is appropriate on determining the constitutionality of prison
recognized as a basis for the death penalty applies to mentally retarded rules.   “When   a   prison   regulation   impinges   on   inmates’  
offenders. constitutional rights, the regulation is valid I it is reasonably
related   to   legitimate   penological   interests.”   However,   strict  
Mentally retarded defendants may be less able to give meaningful
scrutiny is the proper standard of review when it relates to a
assistance to their counsel and are typically poor witnesses, and their
demeanor may create an unwarranted impression of lack of remorse
policy of prisoner segregation based on race

139 | P LATON
Depending on the behavior of the prisoner, he may be made to
Procunier v. Martinez serve out the full term of his prison sentence or temporarily be
416 U.S. 396, 40 L Ed 2d 224, 94 S Ct 1800 (1974) set free earlier. The latter is accomplished by means of parole.
This temporary liberty, however, is subject to certain conditions
The Facts: Appellees, prison inmates, brought this action challenging
such that if he violates any of them he may soon find himself
prisoner mail censorship regulations issued by the Director of the
California Department of Corrections and the ban against the use of law
back in prison garb. What are the procedural safeguards
students and legal paraprofessionals to conduct attorney-client attendant to these parole revocation proceedings? The
interviews with inmates. The regulations also directed inmates not to American Supreme Court dealt with this issue in Morrissey v.
write  letters  in  which  they  “unduly  complain”  or  “magnify  grievances,”   Brewer. In the process at arriving at the answer, it had to
including contraband writings, namely,  those  “expressing  inflammatory,   analyze the personal liberty stakes of the parolee as against the
political,   racial,   religious   or   other   views   of   beliefs.”   The   regulations   societal interests.
further   provided   that   inmates   “may   not   send   or   receive   letters   that  
pertain to criminal activity; are lewd, obscene, or defamatory; contain We turn to an examination of the nature of the interest of the parolee
foreign   matter   or,   are   otherwise   inappropriate.”   Prison   employees   in his continued liberty. The liberty of a parolee enables him to do a
screened both incoming and outgoing personal mail for violations of wide range of things open to persons who have never been convicted
these regulations. of any crime. The parolee has been released from prison based on an
evaluation that he shows reasonable promise of being able to return to
The Issue: The Issue before us is the appropriate standard of review for society and function as a responsible, self-reliant person. Subject to the
prison regulations restricting freedom of speech. conditions of his parole, he can be gainfully employed and is free to be
Whether or not prisoner mail censorship regulations constitute a with family and friends and to form the other enduring attachments of
violation of the Constitutional guaranty against excessive fines and normal life. Though the State properly subjects him to many restrictions
cruel punishments as well as a violation of any other rights enshrined in not applicable to other citizens, his condition is very different from that
the Constitution. of confinement in a prison. He may have been on parole for a number
of years, and may be living a relatively normal life at the time he is
Held: Applying the teachings of our prior decisions to the instant faced with revocation. The parolee has relied on at least an implicit
context, we hold that censorship of prisoner mail is justified if the promise that parole will be revoked only if he fails to live up to the
following criteria are met. First, the regulation or practice in question parole conditions. In many cases, the parolee faces lengthy
must further an important or substantial governmental interest incarceration if his parole is revoked.
unrelated to the suppression of expression. Second, the limitation of
First Amendment freedoms must be no greater than is necessary or We see, therefore, that the liberty of a parolee, although
essential to the protection of the particular governmental interest indeterminate, includes many of the core values of unqualified liberty
involved. and its termination inflicts a "grievous loss" on the parolee and often on
others. It is hardly useful any longer to try to deal with this problem in
On the basis of this standard, we affirm the judgment of the District terms of whether the parolee's liberty is a "right" or a "privilege." By
Court. The regulations invalidated by that court authorized, inter alia, whatever name, the liberty is valuable, and must be seen as within the
censorship   of   statements   that   “unduly   complain”   or   “magnify   protection of the Fourteenth Amendment. Its termination calls for
grievances,”   expression   of   “inflammatory   political,   racial,   religious   or   some orderly process, however informal.
other   views,”   and   matter   deemed   “defamatory”   or   “otherwise  
inappropriate.”   These   regulations   fairly   invited   prison   officials   and   Turning to the question what process is due, we find that the State's
employees to apply their own personal prejudices and opinions as interests are several. The State has found the parolee guilty of a crime
standards for prisoner mail censorship. against the people. That finding justifies imposing extensive restrictions
on the individual's liberty. Release of the parolee before the end of his
We also agree to the decision of the District Court that the decision to prison sentence is made with the recognition that, with many prisoners,
censor or withhold delivery of a particular letter must be accompanied there is a risk that they will not be able to live in society without
by minimum procedural safeguards. The interest of prisoners and their committing additional antisocial acts. Given the previous conviction and
correspondents in uncensored communication by letter, grounded as it the proper imposition of conditions, the State has an overwhelming
is in the First   Amendment,   is   plainly   a   “liberty”   interest   within   the   interest in being able to return the individual to imprisonment without
meaning of the Fourteenth Amendment even though qualified of the burden of a new adversary criminal trial if, in fact, he has failed to
necessity by the circumstances of imprisonment. As such, it is protected abide by the conditions of his parole.
from arbitrary governmental invasion. The District Court required that
an inmate be notified of the rejection of a letter written by or Yet the State has no interest in revoking parole without some informal
addressed to him, that the author of that letter be given a reasonable procedural guarantees. Although the parolee is often formally
opportunity to protest that decision, and that complaints be referred to described as being "in custody," the argument cannot even be made
a prison official other than the person who originally disapproved the here that summary treatment is necessary as it may be with respect to
correspondence. These requirements do not appear to be unduly controlling a large group of potentially disruptive prisoners in actual
burdensome, nor do appellants so contend. custody. Nor are we persuaded by the argument that revocation is so
totally a discretionary matter that some form of hearing would be
The constitutional guarantee of due process of law has a corollary the administratively intolerable. A simple factual hearing will not interfere
requirement that prisoners be afforded access to courts in order to with the exercise of discretion. Serious studies have suggested that fair
challenge unlawful convictions and to seek redress for violations of treatment on parole revocation will not result in fewer grants of parole.
their constitutional rights. This means that inmates must have a
reasonable opportunity to seek and receive the assistance of attorneys. This discretionary aspect of the revocation decision need not be
Regulations and practices that unjustifiably obstruct the availability of reached unless there is first an appropriate determination that the
professional representation or other aspects of the right of access tot individual has, in fact, breached the conditions of parole. The parolee is
the courts are invalid. not the only one who has a stake in his conditional liberty. Society has a
stake in whatever may be the chance of restoring him to normal and
The judgment is Affirmed. useful life within the law. Society thus has an interest in not having
parole revoked because of erroneous information or because of an
Beyond Prison Walls erroneous evaluation of the need to revoke parole, given the breach of
parole conditions... And society has a further interest in treating the
parolee with basic fairness: fair treatment in parole revocations will

140 | P LATON
enhance the chance of rehabilitation by avoiding reactions to On this matter, the factors or criteria set out in Kennedy v.
arbitrariness. Mendoza-Martinez have been generally referred to in
ascertaining whether an act of Congress is penal or regulatory
The Court then turned its focus on the nature of the process in nature. These factors are:
that is due, bearing in mind that the interest of both State and
parolee will be furthered by an effective but informal hearing. Whether the sanction involves an affirmative disability or restraint,
In analyzing what is due, it referred to two important stages in whether it has historically been regarded as a punishment, whether it
the typical process of parole revocation--(2) the arrest of comes into play only on a finding of scienter, whether its operation will
parolee and preliminary hearing, and (b) the revocation hearing promote the traditional aims of punishment - retribution and
itself. Each stage has its own set of procedural requirements in deterrence, whether the behavior to which it applies is already a
crime, whether an alternative purpose to which it may rationally be
order not to offend the Due Process Clause.
connected is assignable for it, and whether it appears excessive in
relation to the alternative purpose assigned are all relevant to the
In the first stage, some minimal inquiry in the nature of a "preliminary inquiry, and may often point in differing directions. Absent conclusive
hearing" should be conducted ate or reasonably near the place of the evidence of congressional intent as to the penal nature of a statute,
alleged parole violation or arrest and as promptly as convenient after these factors must be considered in relation to the statute on its face.
arrest to determine whether there is probable cause or reasonable
ground to believe that the arrested parolee has committed acts that
would constitute a violation of parole conditions. This determination If any trend could be discerned from the rights and guarantees
should be made by someone not directly involved in the case. This accorded accused and convicts, it is the ever-evolving
independent officer need not be a judicial officer. The granting and humanization of the law and of punishments. The
revocation of parole are matters traditionally handled by administrative transgressions of the rules of an organized society for its own
officers. The parolee should be given notice that the hearing will take protection and self-preservation may call for the imposition of
place and that its purpose is to determine whether there is probable penalties but these would have to be in accordance with the
cause to believe he has committed a parole violation. The notice should mandates of an enlightened body politic. In the future, some
state what parole violations have been alleged. At the hearing, the
forms of punishments that are taken for granted today may be
parolee may appear and speak in his own behalf; he may bring letters,
documents, or individuals who can give relevant information to the treated or seen in different light and thus pose other questions
hearing officer. On request of the parolee, a person who has given of constitutionality. As Atkins graphically   illustrates,   “evolving  
adverse information on which parole revocation is to be based is to be standards  of  decency”  could  put  on  a  different  gloss  to  certain  
made available for questioning in his presence. However, if the hearing past practices and punishments. And in the same manner that
officer determines that an informant would be subjected to risk of from what were recognized as rights of the accused developed
harm if his identity were disclosed, he need not be subjected to constitutionally encrusted rights of suspects, in the not too
confrontation and cross-examination. The hearing officer shall have the distant future, there might also come to be more clearly
duty of making a summary, or digest, of what occurs at the hearing in
defined, and guaranteed rights of convicts
terms of the responses of the parolee and the substance of the
documents or evidence given in support of parole revocation and of the
parolee's position. Based on the information before him, the officer Chapter 21
should determine whether there is probable cause to hold the parolee Non-Imprisonment for Debts
for the final decision of the parole board on revocation. Such a
determination would be sufficient to warrant the parolee's continued “No   person   shall be imprisoned for debt or non-payment of a
detention and return to the state correctional institution pending the 31
poll  tax.”
final decision.
Another marker in the advance of man from barbarism to more
With respect to the second stage--the probation revocation civilized and humane treatment of individuals is the
hearing--the Court said: proscription against imprisonment for debt. And, for the
benefit of those less fortunate members of society, there is also
We cannot write a code of procedure; that is the responsibility of each
the guarantee against incarceration simply because of their
State. Most States have done so by legislation, others by judicial
decision usually on due process grounds. Our task is limited to deciding failure to pay a poll tax
the minimum requirements of due process. They include (a) written
notice of the claimed violations of parole; (b) disclosure to the parolee Imprisonment for Debts
of evidence against him; (c) opportunity to be heard in person and to
present witnesses and documentary evidence; (d) the right to confront
and cross-examine adverse witnesses (unless the hearing officer
specifically finds good cause for not allowing confrontation); (e) a Poll Tax
"neutral and detached" hearing body such as a traditional parole board,
members of which need not be judicial officers or lawyers; and (f) a
written statement by the factfinders as to the evidence relied on and
Chapter 22
reason for revoking parole. We emphasize there is no thought to Double Jeopardy
equate this second stage of parole revocation to a criminal prosecution
in any sense. It is a narrow inquiry; the process should be flexible “No  person  shall  be  twice  put  in  jeopardy  of  punishment  for  the
enough to consider evidence including letters, affidavits, and other same offense. If an act is punished by a law and an ordinance,
material that would not be admissible in an adversary criminal trial. conviction or acquittal under either shall constitute a bar to
32
another  prosecution  for  the  same  act.”
Factors in Determining Whether Sanction is Civil or Penal

Relevant to the issue of punishment is the determination as to 31CONSTITUTION, Art. III, § 20


whether a particular sanction imposed by aw may be 32CONSTITUTION, Art. III, § 21
considered as merely civil or penal In the U.S., the guarantee against double jeopardy is found in the Fifth
Amendment:   “No person shall be held to answer for a capital, or otherwise
infamous crime, unless on a presentment or indictment of a Grand Jury, except
in cases arising in the land or naval forces, or in the Militia, when in actual
141 | P LATON
This guarantee against jeopardy of being tried twice for the In a nutshell, before the defense of double jeopardy may be
same offense gives a person the freedom and the right not to invoked, the following requisites must first be present:
be unnecessarily vexed or threatened with repeated
prosecutions. The rule protects the accused not against the (a) first jeopardy which had attached prior to the second;
peril of second punishment, but against being again tried for
the   same   offense.   “The   fundamental   philosophy   behind the (b) first jeopardy had been terminated; and,
constitutional proscription against double jeopardy is to afford
the defendant, who has been acquitted, final repose and (c) the prosecution is for the same offense
safeguard him from government oppression through the
abuse of criminal process.”   Once   prosecuted,   whatever   result   As to when legal jeopardy is deemed to have attached, the
that ensues should already be a bar to any subsequent attempt presence of the following conditions must be shown:
to relitigate the same alleged criminal conduct--a  “res judicata
dressed   in   prison   grey.”   As   the   Court   explained   in   People v. (a) a valid indictment;
Ylagan:
(b) a competent court;
“Without   the   safeguard   this   article   establishes   in   favor   of   the  
accused, his fortune, safety, and peace of mind would be (c) an arraignment had;
entirely at the mercy of the complaining witness, who might
repeat his accusation as often as dismissed by the court and (d) a valid plea entered; and
whenever he might see fit, subject to no other limitation or
restriction than his own will and pleasure. The accused would (e) the case dismissed or otherwise terminated without the
never be free from the cruel and constant menace of a never- express consent of the accused
ending charge, which the malice of the complaining witness
might hold indefinitely suspended over his head, were it not Valid Indictment
that the judiciary is exclusively empowered to authorize, by an
express order to that effect, the repetition of a complaint or It is a requirement for jeopardy to attach that there must be a
information once dismissed in the cases in which the law valid complaint or information to start with. This essentially
requires  that  this  be  done…  Thanks  to  this  article,  the  accused,   means that the said indictment must be filed by a duly
after being notified of the order rest dismissing the complaint authorized person or officer. It is a valid information signed by a
may, as the case may be, either rest assured that he will not be competent officer, among other things, which confers
further molested, or prepare himself for the presentation of a jurisdiction on the court over the person of the accused and the
new complaint. In either case, the order gives him full subject  matter  of  the  accusation.  “An  invalid  information is no
information as to what he may hope or fear, and prevents his information at all and cannot be the basis for criminal
reasonable hopes from being dissipated as the result of an proceedings.”   Hence,   dismissal   of   the   first   information   would  
equivocal and indefinite legal situation. To this much, at least, not be a bar to a subsequent prosecution--jeopardy does not
one who has been molested, possibly unjustly, by prosecution attach where a defendant pleads guilty to a defective
on  a  criminal  charge,  is  entitled.” indictment that is voluntarily dismissed by the prosecution

But as simple as the language of the guarantee appears to be, Arraignment and Valid Plea
the assurance is only against a second or subsequent
prosecution for the same offense. Thus, it is possible that a It is a requirement that for legal jeopardy to attach, there
single act may give rise to one or more offenses, unless the said must first be an arraignment and a valid plea because without
act constitutes a violation of both a national law and a local the same the court cannot have jurisdiction to proceed to
ordinance, in which event conviction or acquittal under either render a valid and binding judgment. A void judgment would
shall constitute a bar to another prosecution under the other then have no force and effect. Consequently, no jeopardy could
for the same act. Another matter to be considered in regard to also be deemed to have attached to preclude another
constitutional safeguard is the fact that subsequent prosecution
prosecution may be had if the earlier prosecution was not
decided on the merits and therefore did not end in an outright
acquittal or conviction, such as when the dismissal was due to
the motion of the accused himself, or with his express consent

General Considerations

Like most of the Bill of Rights guarantees, this protection Dismissals and Finality of Judgments
against double jeopardy is also of American origin. It was
introduced into the country through the Philippine Bill of 1902 Once a criminal case terminates in acquittal, conviction, or is
and  the  Jones  Law  of  1916.  “This principle is founded upon the dismissed without the consent of the accused, he cannot be
law of reason, justice and conscience. It is embodied in the prosecuted anew for the same offense. It is important to
maxim of the civil law non bis in idem, in the common law of determine, however, whether the court that rendered the
England, and undoubtedly in every system of jurisprudence, decision had jurisdiction throughout, or might have been
and instead of having specific origin it simply always existed.” ousted of such authority along the way. Further, for an order of
dismissal to be effective, it must first be reduced to writing and
service in time of War or public danger; nor shall any person be subject for the
same offense to be twice put in jeopardy of life or limb;…”
142 | P LATON
signed by the Judge. Otherwise, a verbal order of dismissal can or amendment to correct a manifest substantial error, even if
not be the valid basis for claiming double jeopardy unwittingly committed by the trial court through oversight or an
initially erroneous comprehension, can be made only with the consent
or upon the instance of the accused. Errors in the decision cannot be
Where there is a valid information and the accused has been
corrected unless the accused consents thereto, or himself moves for
arraigned, an order of dismissal issued by the court, motu reconsideration of, or appeals from, the decision.
proprio, in the course of a trial of a criminal case, whether
based on the merits or for failure of prosecution witnesses to It must be stressed, however, that the protection against double
appear, has the effect of a judgment of acquittal and double jeopardy in the foregoing rules may be waived by the accused. Thus,
jeopardy attaches. The order is also immediately executory. when the accused himself files or consents to the filing of a motion for
However, this order of dismissal must be written in the official reconsideration or modification, double jeopardy cannot be invoked
language, personally and directly prepared by the judge and because the accused waived his right not to be placed therein by filing
such motion. His motion gives the court an opportunity to rectify its
signed by him conformably with the provisions of Rule 120,
errors or to reevaluate its assessment of facts and conclusions of law
Section 2 of the Rules of Court. In the instant case, it is very and make them conformable with the statute applicable to the case in
clear that the order was merely dictated in open court by the the new judgment it has to render. The raison   d’etre is to afford the
trial judge. There is no showing that this verbal order of court a chance to correct its own mistakes and to avoid unnecessary
dismissal was ever reduced to writing and duly signed by him. appeals from being taken. In effect, a motion for reconsideration or
Thus, it did not yet attain the effect of a judgment of acquittal, modification filed by or with consent of the accused renders the entire
so that it was still within the powers of the judge to set it aside evidence open for the review of the trial court without, however,
and enter another order, now in writing and duly signed by conducting further proceedings, such as the taking of additional proof.
him, reinstating the case
Clearly, therefore, appellants cannot dictate upon the trial court which
aspects of the judgment of conviction should be reviewed. Having filed
It goes without saying that the dismissal should be one in a a timely motion for reconsideration asking the court to acquit, or in the
case already pending in court, not one before the public alternative, convict them of the lesser offense of homicide, appellants
prosecutor’s   office.   “Since   a   preliminary   investigation   is   not   waived the defense of double jeopardy and effectively placed the
part of the trial, the dismissal of a case by the fiscal will not evidence taken at the trial open for the review of the trial
constitute double jeopardy and hence there is no bar to the court…Hence,   the   court a quo is not only empowered but also under
filing  of  another  complaint  for  the  same  offense” obligation to rectify its mistake in appreciating the qualifying
circumstance  of  abuse  of  superior  strength  instead  of  treachery…

Bustamante V. Maceren (p. 741)


48 SCRA 155 (1972)

FACTS: Petitioner was charged with the crime of murder in Laguna.


Motions for Reconsideration and Appeals by the Accused Upon arraignment on Dec 14 1970, he entered a plea of guilty and was
sentenced to one year imprisonment. On the very same day was
A judgment of conviction could only be reconsidered or promulgated, petitioner made an express waiver of his right to appeal.
appealed at the initiative of the accused. When the accused Accordingly, Judge Coquia who rendered the aforesaid judgment,
decides to simply serve his sentence or pay his fine, that is the issued a commitment order on the same day the petitioner started to
served his sentence. Three days later, petitioner withdrew his plea of
end of the case. The prosecution may not move for
guilty and the judge ordered the trial of the case on merits. Judge
reconsideration or appeal the judgment as the same would Coquia, subsequently was transferred and the case was reassigned to
place the accused in double jeopardy. The prosecution could the respondent judge who eventually rendered a new judgment against
not also appeal for the purpose of increasing the penalty the petitioner finding him guilty of the crime of homicide and convicting
him to an indeterminate sentence of 6 years and one day to twelve
However, when the accused himself appeals, he stands the years and one day. A motion for reconsideration was filed questioning
chance of having his penalty increased since an appeal in a the jurisdiction of the court to try his case anew after he had fully
criminal case throws open the whole case for review, including served the judgment rendered by Judge Coquia. Petitioner, argued that
the judgment of Judge Coquia against him had already become final
the penalty imposed. Given the rule for appeals, how about if
when he started serving his sentence and that therefore, the court lost
he simply moves for reconsideration? Can he delimit the extent jurisdiction over his case and that no amount of waiver or consent on
to which the court may review the decision for the purposes of his part could bestow on said court jurisdiction that it had already lost
reconsideration so that it may not modify the judgment The judge said that he could not nullify an order of another judge of
adversely against him? This was he thinking of the accused in equal rank and that only a higher court had the authority to nullify said
People v. Astudillo but the Court disagreed: order but that he would hold his ruling in abeyance until petitioner
could file a certiorari before the Supreme court. Hence, this petition.
Anent the qualifying circumstance of treachery, we find no merit in
appellants’  contention  that  the  trial  cannot  validly  appreciate  the  same   HELD: We agree and grant the certiorari. Petitioner is entitled to the
in its amended decision because the attendance of treachery was not relief prayed for. With the judgment of conviction not only
one of the issues raised in their motion for reconsideration. Otherwise promulgated but actually carried out with petitioner having started to
stated, appellants posit that the reconsideration of the judgment of serve his sentence, no such order reopening the case should have been
conviction should be limited only to the issues raised in their motion for issued. That was not in accordance with the controlling doctrine on the
reconsideration, i.e., their guilt or innocence and/or the propriety of constitutional right against being twice put in jeopardy. It is true that he
appreciating   the   qualifying   circumstance   of   “abuse   of   superior   had in fact contributed to bringing such judicial deviation from the
strength”  which  was  not  alleged in the information. correct norm, That did not forfeit though his right to a remedy to which
he is entitled.
*** The requisite consent of the accused to such motion for
reconsideration or modification is intended to protect the latter from People v. Magat (p. 745 Vol. 2)
having to defend himself anew from more serious offenses or penalties 332 SCRA 517 (2000)
which the prosecution or the court may have overlooked. Accordingly,
once the judgment has been validly promulgated, any reconsideration FACTS: Appellant Anotonio Magat was accused of raping his daughter

143 | P LATON
twice when she was 17 yrs old and when she was 19 yrs. Old. Upon court a quo if not acquire jurisdiction over the case. The error of the
arraignment on Jan 19, 1997, he pleaded guilty but bargained a for a trial judge was in confusing the concept of jurisdiction with that of
lesser penalty for each case. The court issued a sentence of insufficiency on substance of an indictment. We also held that the
imprisonment term of 10 years or each crime. After 3 months, the application of the sister doctrines of waiver and estoppel required two
cases were revived at the instance of he complainant on the ground sine qua non conditions: first, the dismissal must be sought or induced
that  the  penalty  imposed  was  “too  light”.  As  a  consequence,  he  was  re- by the defendant personally or though his counsel and second, such
arraigned where he entered a plea of not guilty but subsequently he dismissal must not be on the merits and must not necessarily amount
entered anew a plea of guilty. He was sentenced to death by lethal to an acquittal. Indubitably, the case at bar falls squarely within the
injection. On automatic review o the SC, appellant contends that the periphery of the said doctrines which have been preserved unimpaired
trial court erred re-arraigning and proceeding to trail despite the fact in the corpus of our jurisprudence. The case is hereby remanded to the
that he was already convicted earlier based on his plea of guilt. He also court of origin for further proceedings in accordance with law.
argues that when the court rendered judgment convicting him, the
prosecution did not appeal nor more fro reconsideration or took steps Dismissals, Insufficient Evidence and Speedy Trials
to set aside the order he also posits that the re-arraignment and trial on
the same information violated his right against double jeopardy.
As shown above, the termination of a case other than on
HELD: The January 1997 order of the trial court convicting the accused acquittal or conviction of the accused, which dismissal is
appellant on his own plea of guilt is void ab initio on the ground that brought about by the accused himself, is not a bar to further
accused-appellant’s   plea   is   not   the   plea   bargaining   contemplated   and   prosecution. The defense of double jeopardy could not be
allowed by law and the rules of procedure. The only instance where a invoked. The exception, however, would be in cases of
plea bargaining is allowed under the rules is when an accused pleads demurred to evidence being sustained and dismissal is actually
guilty to a lesser offense. It must be emphasized that accused appellant in the nature of an acquittal. Thus, there could be no more
did not plead to a lesser offense but pleaded guilty to the rape charges subsequent prosecution for the same offense
and only bargained for a lesser penalty. In short, he did not plea bargain
but made conditions on the penalty to be imposed. This is erroneous
because by pleading guilty to the offense charged, accused appellant On a demurred to evidence being favorably acted upon, the
should be sentenced to the penalty to which he pleaded. Nonetheless, acquittal results from the fact that the prosecution has not
whatever procedural infirmity in the arraignment of the accused really presented that amount of evidence sufficient to
appellant was rectified he was re-arraigned and entered a new plea. overcome the presumption of innocence. Where the
Accused appellant did not question the procedural errors in the first prosecution evidence could not sustain a conviction, there is no
arraignment and failing failed to do so, he is deemed to have point in still requiring the accused to present his evidence. On
abandoned his right to question the same and waived the errors in the right to speedy trial, when this right is upheld it means that
procedure.
the accused is now free because the prosecution failed to abide
by its duty to prosecute without unwarranted delays. In effect,
Waiver and Estoppel
the State has failed to prove its case within a reasonable period
of time. Nevertheless, and paradoxically, when there is, in
Just like most rights, the right against double jeopardy may be
dismissing a case based on right to speedy trial, an act of
waived or otherwise barred by estoppel. If the case did not
overspeeding on the part of the court, this may lead to a
proceed to either acquittal or conviction because the accused
contrary result
along the way sought the dismissal of the case in the
meantime, then this may be considered as a waiver, because by Salcedo V. Mendoza (p. 755)
so doing he chose to abort the process. At the same time, if by 88 SCRA 811 (1979)
his allegations he may have misled the court into dismissing the
case on an invalid ground, such as want of jurisdiction, then he FACTS: A criminal information for homicide through reckless
could not subsequently be heard to assert that the court was imprudence against the petitioner was filed. Upon arraignment, he
wrong in going along with him. He is estopped to claim entered a plea of not guilty and the case was then set for trial. When
otherwise the case was called for trial on the scheduled date, the provincial fiscal
asked for and was granted postponement because the accused failed to
appear. On the next scheduled hearing, they once more, moved for
People v. Obsania (p. 747 Vol. 2)
postponement. The 3rd time it was called for trial, no prosecuting fiscal
23 SCRA 1249
appeared for the prosecution. The private prosecutor, who withdrew
his appearance from the case and reserved the right to file a separate
FACTS: A day after the occurrence of the crime of rape with robbery,
civil action, moved for its postponement in order to give the
Erlina Dollente, 14 year old victim and her parents filed in the municipal
prosecution a another chance because they intended to request the
court of Balungao, Pangasinan, a complaint against the defendant.
Ministry of Justice to appoint a special prosecutor to handle the case.
After the case was remanded to the Court of the First Instance of
The trial court denied the same. Whereupon, the petitioner, though
Pangasinan for further proceedings, the assistant provincial fiscal filed
counsel, moved for the dismissal for the criminal case invoking his right
an information for rape against the accused, employing the same
to speedy trial. The respondent judge dismissed the case . A motion to
allegations with an additional averment that the offense was
reconsider the above order was filed the same day but was denied the
committed   with   “lewd   designs”.   Upon arraignment, the accused
following day. A second motion was filed and was set for hearing,
pleaded not guilty and moved for the dismissal of the case contending
however, the parties failed to submit their pleadings, The respondent
that the complaint was totally fatally defective for failure to allege lewd
Judge issued an order setting aside the order of dismissal and ordering
design and that the subsequent information filed did not cure the
that the case be set for trial. Petitioner filed a motion for
jurisdictional infirmity. The Court a quo granted the motion and
reconsideration alleging that the dismissal of the criminal case against
ordered the dismissal of the action. The fiscal appealed.
him was equivalent to an acquittal and reinstatement of the same
would place him twice in jeopardy for the same offense. The trial court
HELD: Two issues are tendered for resolution, namely: First, are lewd
denied the motion. In the instant petition for certiorari, the Solicitor
design an indispensable element which should be alleged in the
General, in his Comment, agrees   with   the   petitioner   that   “   a  
complaint? Second, does the present appeal place the accused in
reinstatement of this case would operate to violate his right against
double jeopardy? We held that the trial judge erred in dismissing the
double  jeopardy”.
case on the grounds that the complaint was defective for failure to
allege lewd design and as a consequence of such infirmity that the
HELD; The stand of the petitioner and the Solicitor General is well

144 | P LATON
taken. We have said that the dismissal of the criminal case predicated manipulating the trial of those accused in the killing of former
on the right of the accused to speedy trial, amounts to an acquittal on Senator Aquino and the supposed assassin, Galman. The Court
the merits which bars the subsequent prosecution of the accused for explained why there was no double jeopardy:
the same offense. In the present case the respondent judge dismissed
the criminal case upon the motion of the petitioner involving his
1. No double jeopardy.--It is settled doctrine that double jeopardy
constitutional right to speedy trial because the prosecution failed to
cannot be invoked against this Court's setting aside of the trial courts'
appear on the day of the trial on March 28, 1978 after it had previously
judgment of dismissal or acquittal where the prosecution which
been postponed twice. The setting aside of the respondent judge of the
represents the sovereign people in criminal cases is denied due process.
order of dismissal of March 28 1978 and thereby reviving criminal case
As the Court stressed in the 1985 case of People vs. Bocar--
places the petitioner twice in jeopardy in the same offense. The
respondent judge therefore committed a grave abuse of discretion in
“Where  the prosecution is deprived of a fair opportunity to prosecute
issuing the order of May 9 1978 setting aside the order of dismissal
and prove its case its right to due process is thereby violated.
issued on March 28 1978.
“The   cardinal   precept   is   that where there is a violation of basic
People V. Declaro (p. 757) constitutional rights, courts are ousted of their jurisdiction. Thus, the
170 SCRA 142 violation of the State's right to due process raises a serious
jurisdictional issue which cannot be glossed over or disregarded at will.
FACTS: Private respondent, Edgar Ibabao was charged with slight Where the denial of the fundamental right of due process is apparent, a
physical injuries through reckless imprudence as a result of a traffic decision rendered in disregard of that right is void for lack of
accident. The case was docketed as Criminal case no 1028-N wherein a jurisdiction. Any judgment or decision rendered notwithstanding such
certain Crispin Conanan was the offended party. Again, an information, violation may be regarded as a "lawless thing, which can be treated as
docketed as Criminal case no 1421 was filed against the same accused an outlaw and slain at sight, or ignored wherever it exhibits its head."
by Eduardo Salido as the offended party. The second case arose from
the same incident. Upon the arraignment of the accused in the first “Respondent  Judge's dismissal order dated July 7, 1967 being null and
case , he entered a plea of not guilty. The hearing was set for hearing void for lack of jurisdiction, the same does not constitute a proper
but the offended party and the prosecuting fiscal failed to appear at the basis for a claim of double jeopardy.
scheduled hearing despite due notice. After a series of motion filed, the
court dismissed the case for lack of interest on the part of the xxx xxx xxx
prosecution. Since the case was dismissed, the petitioner filed a motion
to dismiss the second case on the ground that the dismissal for the “Legal  jeopardy  attaches  only  (a)  upon  a  valid  indictment,  (b)  before  a  
prior case is a bar to the prosecution. The court dismissed the second competent court, (c) after arraignment, (d) a valid plea having been
case on the ground of double jeopardy. Hence, this petition with the entered; and (e) the case was dismissed or otherwise terminated
petitioner contending that double jeopardy have not set in because a.) without the express consent of the accused. The lower court was not
the dismissal of the first case was at the instance and the express competent as it was ousted of its jurisdiction when it violated the right
consent of the accused and his counsel and b.) the second offense of the prosecution to due process.
charged is not the same as the fist, nor is it an attempt to commit the
same or a frustration thereof nor does it include in the firs and c.) the “In  effect  the  first jeopardy was never terminated, and the remand of
first case is not yet terminated. the criminal case for further hearing and/or trial before the lower
courts amounts merely to a continuation of the first jeopardy, and does
HELD; The petitioner is impressed with merit. Although there are not  expose  the  accused  to  a  second  jeopardy.”
criminal cases which were dismissed upon motion of the accused
because the prosecution was not prepared for trial sine the More so does the rule against the invoking of double jeopardy hold in
complainant or his witnesses did not appear at the trail or where the the cases at bar where as we have held, the sham trial was but a mock
court held that the dismissal is equivalent to an acquittal that would bar trial where the authoritarian president ordered respondents
further prosecution of the defendant for the same offense. However, Sandiganbayan and Tanodbayan to rig the trial and closely monitored
the facts and the circumstance of the present case does not warrant the entire proceedings to assure the pre-determined final outcome of
similar ruling. Double jeopardy will apply even if the dismissal is made acquittal and total absolution as innocent of an the respondents-
with the express consent to the accuse or upon his own motion only if accused.33
it is predicated in either of 2 grounds i.e. insufficiency of he evidence or
denial of the right to a speedy trial. In both cases, the dismissal will Respondents-accused's contention that the Sandiganbayan judgment of
have the effect of the acquittal. Since the dismissal in this case does acquittal ends the case which cannot be appealed or re-opened,
not fall under either of these 2 instances, and it was made with the without being put in double jeopardy was forcefully disposed of by the
express consent of the accused, it would not thereby be a bar to Court in People vs. Court of Appeals, which is fully applicable here, as
another prosecute for the same offense, follows: "That is the general rule and presupposes a valid judgment.
As earlier pointed out, however, respondent Courts' Resolution of
Acquittal, Mistrials and Appeals acquittal was a void judgment for having been issued without
jurisdiction. No double jeopardy attaches, therefore. A void judgment
An acquittal puts an end to the whole case. The accused can is, in legal effect, no judgment at all. By it no rights are divested.
not anymore be imperilled by the prosecutor either seeking a Through it, no rights can be attained. Being worthless, all proceedings
founded upon it are equally worthless. It neither binds nor bars anyone.
reconsideration of the judgment or taking an appeal from it.
All acts performed under it and all claims flowing out of it are void.
Both actions would be considered as violation of the guarantee
against double jeopardy. If there was grave abuse of discretion xxx xxx xxx
which ousted the court of jurisdiction, that would be a different
matter, however. Without jurisdiction, the court cannot "Private respondent invoke 'justice for the innocent'. For justice to
proceed to render a valid judgment. Thus, the prosecution may prevail the scales must balance. It is not to be dispensed for the
elevate the matter to an appellate court by means of the accused alone. The interests of the society, which they have wronged
special civil action of certiorari must also be equally considered. A judgment of conviction is not
necessarily a denial of justice. A verdict of acquittal neither necessarily
spells a triumph of justice. To the party wronged, to the society
In Galman v. Sandiganbayan, the Supreme Court nullified the
acquittal of the accused because of a mistrial brought about by
the acts of then President Marcos in influencing and 33 See Page 762
145 | P LATON
offended, it could also mean injustice. This is where the Courts play a not be placed in Double jeopardy for the reason that from the very
vital  role.  They  render  justice  where  justice  is  due.” beginning lower trial tribunal acted without jurisdiction is in legal
contemplation. Case remanded to the Regional Trial court of Legaspi,
It is only in very rare instances, however, that an acquittal had Public prosecutor is ordered to complete presentation of available
may be allowed to be questioned and reverse. For unless the witness for the prosecution.
violation of the basic dictates is so flagrant and egregious, the
judgment in favor of the accused would stand Appeals and Civil Liability

People v.Velasco (p. 763) The rule that an acquittal cannot be appealed refers only to
340 CRA 207 criminal aspect of the case. The offended party may appeal the
civil component without violating the guarantee against double
FATS: Alex Vinculado was killed while his twin brother Levi lost jeopardy. The proceeding assumes the nature already of a civil,
permanently his vision and their uncle was shot. Information for not criminal, litigation
Homicide and frustrated homicide was filed against Mayor Honorato
Galvez of San Ildefonso, Bulacan and his bodyguard, Godofredo Diaz.
In Manantan v. Court of Appeals, x x x:
Charges were however, withdrawn and a new one for murder and
frustrated murder was filed against the accused. In addition, the mayor
was charged with violation of PD 1866 for unauthorized carrying of Our law recognizes two kinds of acquittal, with different effects on the
firearm outside this residence. The court found them guilty . However, civil liability of the accused. First is an acquittal on the ground that the
it acquitted Galvez of the same charges due to insufficiency of accused is not the author of the act or omission complained of. This
evidence. The acquittal of Mayor Galvez is challenged by the instance closes the door to civil liability, for a person who has been
government in this petition of certiorari. Their position is that the found to be not the perpetrator of any act or omission cannot and can
elevating the issue of criminal culpability to the Supreme court despite never be held liable for such act or omission. There being no delict, civil
acquittal by the trial court should not be considered as violative of the liability ex delicto is out of the question, and the civil action, if any,
constitutional right against double jeopardy.\ which may be instituted must be based on grounds other than
the delict complained of. This is the situation contemplated in Rule 111
HELD: The petition at hand seeks to nullify the decision of the trial of the Rules of Court. The second instance is an acquittal based on
court   acquitting   the   accused   goes   deeply   in   the   court’s   appreciation   reasonable doubt on the guilt of the accused. In this case, even if the
and evaluation in esse of the evidence adduced by the parties. A guilt of the accused has not been satisfactorily established, he is not
reading   of   the   questioned   decision   shows   that   respondent’s   judge   exempt from civil liability which may be proved by preponderance of
considered the evidence received at trial. These consisted among evidence only. This is the situation contemplated in Article 29 of the
others of the testimonies relative to the position of the victims visa vis Civil Code, where   the   civil   action   for   damages   is   “for   the   same   act   or  
the accused and the trajectory, location and nature of the gunshot omission.”   Although   the   two   actions   have   different   purposes,   the  
wounds, and the opinion of the expert witness for the prosecution. matters discussed in the civil case are similar to those discussed in the
While the appreciation may have resulted in possible lapses in evidence criminal case. However, the judgment in the criminal proceeding
evaluation, it does not detract from the fact that the evidence was cannot be read in evidence in the civil action to establish any fact there
considered and passed upon. This consequently exempts the act from determined, even though both actions involve the same act or
the  writ’s  limiting  requirement  of excess or lack of jurisdiction. Hence, it omission. The reason for this rule is that the parties are not the same
becomes an improper object and non reviewable by certiorari. To and secondarily, different rules of evidence are applicable. Hence,
reiterate, errors of judgment are not to be confused with errors in the notwithstanding   herein   petitioner’s   acquittal,   the   Court   of   Appeals   in  
exercise of jurisdiction. determining whether Article 29 applied, was not precluded from
looking   into   the   question   of   petitioner’s   negligence or reckless
imprudence.
Merciales v. Court of Appeals
379 SCRA 345 (2002)
Supervening Fact Doctrine
Facts : On August 12, 1993 six information of rape with homicide in
connection of the death of Maritess Ricafirt Merciales were filed Once a person has been charged and has pleaded, the charge
against private respondents Jeslito Nuada, Patrcik Moral, Adonis may not be amended or revised in a substantial way, otherwise
Nieves, Ernesto Lobelete, Domil Grageda and Ramon Frlores before the this would also be a violation of his right against double
regional trial Court in Legaspi City. Although seven witnesses were jeopardy. In this regard, however, one has to consider whether
presented by the prosecutor, none actually saw the commission of the
the fact that is sought to justify the amendment was already
crime. Only Nuada one of the accused who came forward and express
willingness to turn as state witness. Trial Court required the prosecutor
present when the formal accusation and plea were made, or
to  present  evidence  to  justify  Nuada’s  discharge  as  a  state  witness.  But   came about only subsequently. If before, then the amendment
the latter insisted that there was no need for such proof since Nuada may not be had without violating the constitutional safeguard.
had already admitted into the Witness protection program of the If after, it may be done as the new development is a
Department of Justice. Trial court denies the motion to discharge supervening fact which could not have been included earlier
Nuada. In due time the accused filed demurrer of evidence, the trial
court issued an order acquitting all the accused for lack of sufficient Melo v. People
evidence to prove the guilt of the accused beyond reasonable doubt. 85 Phil. 766 (1950)
Instant petition assails the failure of the court of appeals to nullify the
trial   court’s   order   on   the   ground   that   the   trial   judge   tolerated   or   Facts: Conrado Melo was charge in the Court of first instance of Rizal
committed injustice by failing to require prosecution which is present. with frustrated homicide, inflicted to Benjamin Obillo with a kitchen
knife with intent to kill. Accused pleaded not guilty of the offense
Issue: Whether or not the reopening of the case violates the accused charge in the same they the victim died. Evidence of death was only
right against double jeopardy. available on January 3, 1950 the next they amended information was
filed charging the accused with consummated homicide. Accused file
Held: In the case at Bar the private prosecutor knew that he had not for motion to squash the amended information alleging double
presented sufficient evidence to convict the accused. In this sense he jeopardy but motion was denied.
remiss his duty to protect the interest of the offended parties. Refusing
to comply to present evidence violates Rule 119 section 9 of Rules of Issue: whether or not the denial of the motion denies the accused the
the Court expressly requires presentation of evidence. Accused would right against double jeopardy.

146 | P LATON
second offense charged is exactly the same as the one alleged
Held: Right against double jeopardy does not apply when the second in the first information...And, there is identity between the two
offense was not in existence at the time of the first prosecution for offenses when the evidence to support a conviction for one
simple reason that in such case there is no possibility of accused, during
offense would be sufficient to warrant a conviction for the
the first prosecution, to be convicted for an offense that was inexistent.
Thus where the accused was charged with physical injuries and after
other. Under the Rules there is identity between two offenses
conviction the injured person dies, the charge for homicide against the not only when the second offense is exactly the same as the
same accused does not put him twice in jeopardy. first, but also when the second offense is an attempt to commit
the first or a frustration thereof, or when it necessarily includes
People v. Buling or is necessarily included in the offense charged in the former
107 Phil. 712 (1960) complaint   or   information.   “In this connection, an offense may
be said to necessarily include another when some of the
Facts: Buling was charge of a crime of less serious physical injuries for essential ingredients of the former as alleged in the information
having inflicted wounds on Isidro Balaba, which according to the constitute the latter. And vice-versa, an offense may be said to
complaint requires medical assistance for a period of 10 to 15 days.
be necessarily included in another when all the ingredients of
Accused pleaded guilty and found guilty, then began to serve his
sentence. Balabas injuries however did not heal. Provincial Fiscal filed
the former constitute a part of the elements constituting the
another complaint of serious physical injuries. Information alleges that latter (Rule 116, sec. 5.) In other words, one who has been
the victim requires medical attendance and incapacitated him for a charged with an offense cannot be again charged with the
period of 1 ½ to 2 ½ months. After the trial accused is found guilty of same or identical offense though the latter be lesser or
serious physical injuries as a new sentenced. greater than the former.”

Issue: Whether or not prosecution conviction of Balaba for less physical People v. City Court of Manila, Branch VI
injuries is a bar for the second prosecution for serious physical injuries. 154 SCRA 175 (1987)

Held: the new fact did not supervened, what happened was no X – ray Facts: Gonzales together with Pangilinan was accused of violating Sec.7
examination of wounded hand was made during the first examination, in relation to sec. 11 R.A. 201(3) of RPC which is Consist of indecent or
which was merely superficial. If new fact had been disclosed in the immoral plays scenes, acts or shows, not just motion pictures. And
previous examination may be attributed to the incompetence on the Art.3060 which pertains to public exhibition of any motion picture
part of the examining physician. If the X – ray examination discloses the which has not been previously passed by the motion Board of Censors
existence of a fracture, such fracture should have existed when the first for Motion Pictures that constitutes criminal offense. Gonzales moved
examination wear made. Therefore there is no supervening fact that to squash on the ground of double jeopardy.
could be said to have arisen. Judgement conviction was set aside and
the defendant was acquitted for serious physical injury. Issue: whether or not ground of double jeopardy could be invoked.

“Same  Offense”  v.  Same  Act” Held: It is evident that the two offenses are different. There is no
identity of the offense here involved for which legal jeopardy in one
The constitutional guarantee contemplates two situations. One may be invoked. Evidence required to prove one offense is not the
in which the same act may give rise to two or more offenses in same evidence required to prove another. Double jeopardy cannot
prosper.
which case the rule against double jeopardy does not apply,
and another where even if the same act produces two offenses
Perez v. Court of Appeals
the protection is made available
168 SCRA 236 (1988)

Our Bill of Rights deals with two (2) kinds of double jeopardy. The first Facts: Mendoza charged the petitioner Perez before the court of first
sentence of clause 20, section 1, Article III of the Constitution, ordains instance of Pampanga with consented abduction. Trial court convicted
that "no person shall be twice put in jeopardy of punishment for the Perez who the appealed to the Court of Appeals. Court hold that what
same offense." (Emphasis in the original) The second sentence of said he did was seduction and not abduction. Subsequently the complainant
clause provides that "if an act is punishable by a law and an ordinance, filed another criminal offense against Perez, this time for qualified
conviction or acquittal under either shall constitute a bar to another seduction. Petitioner filed as motion for squash invoking double
prosecution for the same act." Thus, the first sentence prohibits double jeopardy. Motion was denied. He file petition for certiorari and
jeopardy of punishment for the same offense, whereas the second prohibition to the Supreme Court.
contemplates double jeopardy of punishment for the same act. Under
the first sentence, one may be twice put in jeopardy of punishment of Issue: Whether or not the denial of the court violates the right of the
the same act provided that he is charged with different offenses, or the petitioner against double jeopardy.
offense charged in one case is not included in or does not include, the
crime charged in the other case. The second sentence applies, even if Held: The plea cannot therefore be accorded with merit as the two
the offenses charged are not the same, owing to the fact that one indictments are perfectly distinct from the point of the law. Difference
constitutes a violation of an ordinance and the other a violation of a between the two are in consented abduction taking away of the
statute. If the two charges are based on one and the same act offended must be with her consent after cajolery from the offender and
conviction or acquittal under either the law or the ordinance shall bar a the taking must be in lewd design. While in qualified seduction the
prosecution under the other. Incidentally, such conviction or acquittal is crime committed must be in abuse of authority, confidence and
not indispensable to sustain the plea of double jeopardy of punishment relationship and the offender had sexual intercourse with the woman.
for the same offense. So long as jeopardy has attached under one of
the informations charging said offense, the defense may be availed of
Nierras v. Dacuycuy
in the other case involving the same offense, even if there has been
181 SCRA 1 (1990)
neither conviction nor acquittal in either case.
Facts: Petitioner a customer of Pilipinas Shell Corporation, purchase oil
Same  “Offense” products from it. Upon the delivery petitioner issued nine checks. Upon
the presentation of the checks all nine was dishonoured for the reason
Going back to the first situation, the question is: When is there that   petitioner’s   account   was   already   close. Pilipinas Shell repeatedly
“same  offense”?  Simply  put,  there  is  a  same  offense  where  the   demanded for depository checks or direct payment of the products, but

147 | P LATON
the petitioner failed to do so. Petitioner was charged of nine violation prohibition on appointment as provided for in Sec. 6, Art. IX-B of the
of B.P. Blg. 22 or Bouncing checks law. Nine Criminal cases were also Constitution and Sec. 94 (b) of the Local Government Code, mandating
later instituted against him for estafa under Article 315(2-d) of RPC. that a candidate who lost in any election shall not, within one year after
Petitoner filed a motion to squash invoking double jeopardy as such election, be appointed to any office in the Government?
elements of estafa under Article 315(2-d) of RPC are also present in
criminal cases in B.P. Blg. 22. The Court answers in the negative.

Issue: Whether or not the petitioner can be held liable for nine criminal The Sandiganbayan, Fourth Division held that the qualifications for a
cases for violation of B.P. Blg. 22 and separately be held liable for crime position are provided by law and that it may well be that one who
of estafa under article 315(2-d) of RPC. possesses the required legal qualification for a position may be
temporarily disqualified for appointment to a public position by reason
Held: Petitioner is charged of two distinct and separate offenses. of the one-year prohibition imposed on losing candidates. However,
Petitioner failed to answer that deceit and damage is essential to article there is no violation of Article 244 of the Revised Penal Code should a
315(2-d) of RPC but to required to B.P. Blg. 22. Hence filing of two sets person suffering from temporary disqualification be appointed so long
of information does not give rise to double jeopardy. as the appointee possesses all the qualifications stated in the law.

Same  “Act” There is no basis in law or jurisprudence for this interpretation. On the
contrary, legal disqualification in Article 244 of the Revised Penal Code
simply means disqualification under the law. Clearly, Section 6, Article
As noted earlier, when the same act gives rise to prosecution
IX of the 1987 Constitution and Section 94(b) of the Local Government
under a national law and a municipal or local ordinance, Code of 1991 prohibits losing candidates within one year after such
acquittal or conviction under either would be a bar to the election to be appointed to any office in the government or any
other. Thus, since the two offenses are really different because government-owned or controlled corporations or in any of their
they are penalized by two jurisdictions--national and local--if subsidiaries.
the second part of the guarantee were not provided, individuals
could be prosecuted twice for the same act, even if under both Villapando's contention and the Sandiganbayan, Fourth Division's
regulations they ,ay already constitute same offense if they interpretation of the term legal disqualification lack cogency. Article
244 of the Revised Penal Code cannot be circumscribed lexically. Legal
were enacted by the same legislative body
disqualification cannot be read as excluding temporary disqualification
in order to exempt therefrom the legal prohibitions under Section 6,
Article IX of the 1987 Constitution and Section 94(b) of the Local
Government Code of 1991.
Administrative Proceedings
Although this Court held in the case of People v. Sandiganbayan that
The guarantee against double jeopardy refers to criminal once a court grants the demurrer to evidence, such order amounts to
proceedings. It may not, therefore, be invoked elsewhere an acquittal and any further prosecution of the accused would violate
the constitutional proscription on double jeopardy, this Court held in
the same case that such ruling on the matter shall not be disturbed in
Miscellany the absence of a grave abuse of discretion.

A man is entitled to some definite idea as o how a criminal case In this case, the Sandiganbayan, Fourth Division, in disregarding basic
should be determined. There should be some closure that he rules of statutory construction, acted with grave abuse of discretion. Its
can rely on, either in his conviction or acquittal. In either case, interpretation of the term legal disqualification in Article 244 of the
he should be entitled to repose knowing that a particular Revised Penal Code defies legal cogency. Legal disqualification cannot
matter or chapter in his life has come to [an] end, a book that be read as excluding temporary disqualification in order to exempt
therefrom the legal prohibitions under the 1987 Constitution and the
he may now close and put aside
Local Government Code of 1991. We reiterate the legal maxim ubi lex
non distinguit nec nos distinguere debemus. Basic is the rule in
Additional Cases statutory construction that where the law does not distinguish, the
(Q) Double Jeopardy [3] courts should not distinguish. There should be no distinction in the
application of a law where none is indicated.
Q1. People V. Sandiganbayan (Fourth Division)
559 SCRA 449 (2008) Q2. People V. Hernandez
499 SCRA 688 (2006)
During the May 11, 1998 elections, Villapando ran for Municipal Mayor
of San Vicente, Palawan. Tiape (now deceased), a relative of The instant case stemmed from a complaint filed by Aquilino Pimentel,
Villapando's wife, ran for Municipal Mayor of Kitcharao, Agusan del Jr., a senatorial candidate in the May 1995 elections, against private
Norte. Villapando won while Tiape lost. Thereafter, Villapando respondents for allegedly decreasing Pimentel's votes in the Statement
designated Tiape as Municipal Administrator of the Municipality of San of Votes per precinct and in the City Certificate of Canvass for Pasig
Vicente, Palawan. Maagad and Fernandez charged Villapando and Tiape City. The COMELEC filed for violation of Section 27(b) of R.A. No. 6646
for violation of Article 244 of the Revised Penal Code before the Office against private respondents.
of the Deputy Ombudsman for Luzon. The complaint was resolved
against Villapando and Tiape charging the two with violation of Article Section 1, Rule 122 of the Revised Rules of Criminal Procedure provides
244 of the Revised Penal Code filed with the Sandiganbayan. that  “[a]ny  party may appeal from a judgment or final order, unless the
accused will be placed in double jeopardy.”
Article 244 of the Revised Penal Code provides: Unlawful
appointments.-Any public officer who shall knowingly nominate or As a general rule, the prosecution cannot appeal or bring error
appoint to any public office any person lacking the legal qualifications... proceedings from a judgment in favor of the defendant in a criminal
case in the absence of a statute clearly conferring that right. Thus,
There appears to be a dispute. This Court is now called upon to errors of judgment are not appealable by the prosecution. Appeal by
determine whether Orlando Tiape, at the time of [his] designation as the prosecution from the order of dismissal of the criminal case by the
Municipal Administrator, was lacking in legal qualification. Stated trial court may be allowed only on errors of jurisdiction when there was
differently, does "legal qualification" contemplate the one (1) year denial of due process resulting in loss or lack of jurisdiction. This is so as

148 | P LATON
while it is true that double jeopardy will attach in case the prosecution souls since they both apply retroactively to prejudice the
appeals a decision acquitting the accused, an acquittal rendered in person affected
grave abuse of discretion amounting to lack or excess of jurisdiction
does not really "acquit" and therefore does not terminate the case as
Ex Post Facto Laws
there can be no double jeopardy based on a void indictment.

In the case at bar, the trial court dismissed the cases against private While an ex post facto law is basically and generally understood
respondents for the denial of their right to speedy trial. In a long line of as that which punishes retroactively an act which when
cases, we have held that a dismissal on the ground of the denial of the committed was not prohibited, it actually assumes several
accused's right to a speedy trial will have the effect of acquittal that forms. Thus, it is one which:
would bar further prosecution of the accused for the same
offense. Thus, we have held that where after such dismissal the (1) makes criminal an act done before the passage of the law
prosecution moved for the reconsideration of the order of dismissal
and which was innocent when done, punishes such an act;
and the court re-set the case for trial, the accused can successfully
claim double jeopardy as the said order was actually an acquittal, was
final and cannot be reconsidered. Hence, petitioner was correct in (2) aggravates a crime, or makes it greater than it was, when
filing a petition for certiorari under Rule 65, alleging that "respondent committed;
judge committed grave abuse of discretion and/or acted without or in
excess of jurisdiction in issuing the order of dismissal dated November (3) changes the punishment and inflicts a greater punishment
23, 2001 allegedly on account of the speedy trial rule" as an appeal was than the law annexed to the crime when committed;
not available to it. Where the dismissal of the case was allegedly
capricious, certiorari lies from such order of dismissal and does not
(4) alters the legal rules of evidence, and authorizes conviction
involve double jeopardy, as the petition challenges not the correctness
but the validity of the order of dismissal and such grave abuse of
upon less or different testimony than the law required at the
discretion amounts to lack of jurisdiction which prevents double time of the commission of the offense;
jeopardy from attaching.
(5) assuming to regulate civil rights and remedies only, in effect
Q3. Gonzales v. Abaya imposes penalty or deprivation of a right for something which
498 SCRA 445 (2006) when done was lawful; and

On July 27, 2003 at around 1:00 a.m., more than 300 heavily armed (6) deprives a person accused of a crime of some lawful
junior officers and enlisted men of the AFP entered the premises of the protection to which he has become entitled, such as the
Oakwood Premier Luxury Apartments on Ayala Avenue, Makati City,
protection of a former conviction or acquittal, or a
where they disarmed the security guards and planted explosive devices
around the building. They then declared their withdrawal of support
proclamation  of  amnesty.  However,  the  “lawful  protection”   to  
from their Commander-in-Chief and demanded that she resign as which   an   accused   “has   become   entitled”   is   qualified   and   not  
President of the Republic. given a broad scope. It does not embrace the mode of
procedure provided for in the statutory right to appeal. Thus, a
After much negotiation, the group finally laid down their arms. requirement for unanimity of justices for the affirmance of a
Subsequently, an Information for coup d’etat was filed against them death sentence can be subsequently changed without violating
with the RTC, at the same time that they were tried at court martial for the constitutional proscription
conduct unbecoming an officer. They question the jurisdiction of the
court martial, contending that the RTC ordered that their act was not
service-connected and that their violation of Art. 96 of the Articles of
Historical Background
War (RA 7055) was absorbed by the crime of coup d’etat.
The Ex Post Facto Clause refers to criminal laws. However, this
Separate Opinion of Justice Tinga: was   not   always   the   understanding.   “Early   commentators  
understood ex post facto laws to include all laws of
Double Jeopardy--It is very well-settled that double jeopardy attaches if retrospective   application,   whether   civil   or   criminal...”   The  
one is tried by both a military court and a civilian court over the same constitutional inhibition refers only to criminal laws which are
act, notwithstanding the differing natures of both tribunals
given retroactive effect. It does not preclude, therefore, the
retroactive application of an extradition treaty since said
Chapter 23 international agreement is neither a piece of criminal legislation
Ex Post Facto Laws and Bills of Attainder nor a criminal procedural statute
34
“No ex post facto law  or  bill  of  attainder  shall  be  enacted.” The essence of the prohibition that the Ex Post Facto Clause
seeks to prevent is the prejudice that an ex post facto law rings
Another guarantee of fairness, this provision assures that a to the accused. Accordingly, if the new law is actually favorable
person could only be found guilty of a crime if, at the time of to him, then it will be given a retroactive effect
its commission, it was already defined as such by law. The law
can only be prospective, covering events and incidents after its
effectivity, and not one reaching back to punish conduct or acts
which were not considered offenses at the time of their
commission or omission

The other part of the guarantee safeguards against legislative


action by which guilt is declared without the benefit of a judicial Legislative Acts v. Judicial Determinations
proceeding. Ex post facto laws and bills of attainder are closely
related and intertwined. They normally go together like kindred Further, as the language of the Clause makes clear, it is
addressed to the legislature prohibiting it from enacting such
34 CONSTITUTION, Art. III, § 22
149 | P LATON
kinds of laws. Accordingly, it is not directed at judicial banishment, deprivation of the right to vote, confiscation of
determinations. But the spirit of due process precludes similarly property or disqualification from certain employment or
the courts from acting unreasonably by retroactively reaching vocations. However, as intended in the Bill of Rights, it
out to prejudice an accused. The American Supreme Court said encompasses both.
in Marks v. United States:
Although the prohibition against bills of attainder has been addressed
The Ex Post Facto Clause is a limitation upon the powers of the only infrequently by this Court, it is now settled beyond dispute that a
Legislature and does not, of its own force, apply to the Judicial Branch bill of attainder, within the meaning of Art. I, is by no means the same
of government. But the principle on which the Clause is based--the as a bill of attainder at common law. The definition departed from the
notion that persons have a right to fair warning of that conduct which common law concept very early in our history, in a most fundamental
will give rise to criminal penalties--is fundamental to our concept of way. At common law, the bill was a death sentence imposed by
constitutional liberty. As such, that right is protected against judicial legislative Act. Anything less than death was not a bill of attainder, but
action by the Due Process Clause of the Fifth Amendment was, rather, "a bill of pains and penalties." This restrictive definition
was recognized tangentially in Marbury v. Madison, 1 Cranch 137, 179
(1803), but the Court soon thereafter rejected conclusively any notion
that only a legislative death sentence or even incarceration imposed on
named individuals fell within the prohibition. Mr. Chief Justice Marshall
firmly settled the matter in 1810, holding that legislative punishment in
the form of a deprivation of property was prohibited by the Bill of
Bills of Attainder Attainder   Clause:   “A bill of attainder may affect the life of an
individual, or may confiscate his property, or may do both.”
Bills of attainder are legislative acts which inflicts punishment
on named individuals or members of an easily ascertainable Bills of Attainder and the Doctrine of Separation of Powers
group without judicial trial. Its essence is the substitution of a
legislative for a judicial determination of guilt It has also been stated that the Bill of Attainder Clause is
related to the doctrine of separation of powers.   “The best
The U.S. Supreme Court gave a concise background of the available evidence, the writings of the architects of our
constitutional proscription against bills of attainder in United constitutional system, indicates that the Bill of Attainder Clause
States v. Brown: was intended not as a narrow, technical (and therefore soon to
be outmoded) prohibition, but rather as an implementation of
A logical starting place for an inquiry into the meaning of the the separation of powers, a general safeguard against
prohibition is its historical background. The bill of attainder, a legislative exercise of the judicial function, or more simply—
parliamentary act sentencing to death one or more specific persons, trial   by   legislature.”   The   concern   of   the   Framers   of   the   U.S.  
was a device often resorted to in sixteenth, seventeenth and
Constitution   “that   a   legislature   should   not   be   able   unilaterally  
eighteenth century England for dealing with persons who had
attempted, or threatened to attempt, to overthrow the government. to impose a substantial deprivation on one person was
In addition to the death sentence, attainder generally carried with it a expressed not only in this general allocation of power, but also
'corruption of blood,' which meant that the attainted party's heirs could in more specific provisions, such as the Bill of Attainder
not inherit his property. The 'bill of pains and penalties' was identical Clause,... This Clause, and the separation-of-powers doctrine
to the bill of attainder, except that it prescribed a penalty short of generally,  reflect  the  Framers’  concern  that  trial  by  a  legislature  
death, e.g., banishment, deprivation of the right to vote, or exclusion of lacks the safeguards  necessary  to  prevent  the  abuse  of  power.”  
the designated party's sons from Parliament. Most bills of attainder and Nevertheless,   “the Bill of Attainder Clause not only was
bills of pains and penalties named the parties to whom they were to
intended as one implementation of the general principle of
apply; a few, however, simply described them. While some left the
designated parties a way of escaping the penalty, others did not. The fractionalized power, but also reflected the Framers' belief that
use of bills of attainder and bills of pains and penalties was not limited the Legislative Branch is not so well suited as politically
to England. During the American Revolution, the legislatures of all independent judges and juries to the task of ruling upon the
thirteen States passed statutes directed against the Tories; among blameworthiness, of, and levying appropriate punishment
these statutes were a large number of bills of attainder and bills of upon,  specific  persons.”
pains and penalties.
People vs. ferrer
The bill of attainder was a tool for political vendetta in an ever-
changing  landscape  of  temporal  ascendancy:  “Bills  of  attainder   Respondents co and tayag and others were charged with violation of
were typically directed at once powerful leaders of anti-subversion law which outlaws the communist party of he
government. By special legislative Acts, Parliament deprived Philippines and other subversive associations and punishes those who
one statesman after another of his reputation, his property, affiliates becomes or remains a member pf the party or any subversive
organizations.
and his potential for future leadership. The motivation of such
Won the act is a bill of attainder
bills was as much political as it was punitive--and often the
victims were those who had been the most relentless in The essence of bill of attainder is substitution of a legislative for a
attacking their political enemies at the height of their own judicial determination of guilt. In the case at bar the act was
power condemned because it tars and feathers the communist party of the
Philippines as a continuing menace to the freedom and security of the
Bills of Pains and Penalties country.

SC: no. It did not specify the communist party or members to be


The proscription against bills of attainder also includes the so-
punished. What it does is declare the party to be an organized
called bills of pains and penalties, As noted above, it its conspiracy to overthrow the government. Its focus is not on individuals
restrictive sense, the bills of attainder referred to the but on conduct. The court will still be judicially established. Indeed it is
imposition of death as a penalty while the bills of pains and onl when a statute applies either to named individuals or to easily
penalties referred to lesser penalties, such as imprisonment, ascertainable members of a group in such a way as to inflict a

150 | P LATON
punishment on them without a judicial trial does it become a billof appeal.
attainder.nor it is enough that a statute specfy persons or groups in
order that it may fall within the ambit of prohibition against bll of Held: Yes. AO 13 and MO 61 are not ex post facto laws.
attainder,. It is also necessary to that it must apply retroactively and
reach past conduct. If a statute is a bill of attainder it is also an expost The constitutional doctrine that outlaws an ex post facto law generally
facto law. prohibits the retrospectivity of penal laws. Penal laws are those acts of
the legislature which prohibit certain acts and establish penalties for
Nixon V Administrator Of General Services their violations; or those that define crimes, treat of their nature, and
provide for their punishment. The subject administrative and
After appellant had resigned as the president of the united states, he memorandum orders clearly do not come within the shadow of this
executed Nixon-sampson agreement with the administrator of general definition. Administrative Order No. 13 creates the Presidential Ad
services that provided the storage of documents and tape recordings Hoc Fact-Finding Committee on Behest Loans, and provides for its
near his home, with this agreement, neither the administrator nor the composition and functions. It does not mete out penalty for the act of
president can access the materials without each others consent. After granting behest loans. Memorandum Order No. 61 merely provides a
3 years, appellant could withdraw the documents but not the tapes frame of reference for determining behest loans. Not being penal laws,
which can be withdrawn after 5 years. Following the 5 years, the Administrative Order No. 13 and Memorandum Order No. 61 cannot
administrator would destroy such tapes and or at appellants death or be characterized as ex post facto laws. There is, therefore, no basis for
after expiration of 10 years whichever occurred first. The presidential the Ombudsman to rule that the subject administrative and
recordings and materials preservation act wqs enacted by congress to memorandum orders are ex post facto.
abrogate agreement

Appellant argued that it is a bill of attainder. The act was faulted for
singling out appellant as opposed to all the presidents or members of Chapter 24
the government. Citizenship
Appellants contention of meaning of bill of attainder is too much. By
arguing that an individual or defined group is attained whenever he or
he is compelled to bear burdens which individual or group dislikes, the “The  following  are  citizens  of  the  Philippines:
appellant removes the anchor that ties the bill of attainder to realistic
conceptions. His view could cripple the very process of legislating, for “(1)  Those  who  are  citizens of the Philippines at the time of the
every individual tht is made subject of adverse legislation can complain adoption of this Constitution;
that the lawmakers should have defined the relevant affected class at a
greater level of generality the specificity does not automatically offend “(2)   Those   whose   fathers   or   mothers   are   citizens   of   the  
the bill of attainder clause. Philippines;
Congress expressed no interest in punishing or penalizing appellant
butu justified the act according to objectives to preserve judicial “(3)   Those   born   before   January   17,   1973,   of   Filipino   mothers,  
evidence and historical materials. In judging the constitutionality of who elect Philippine citizenship upon reaching the age of
the act, we may only look to its terms, to the intent expressed by majority; and
members of congress and existence of legitinmate explanations.
35
“(4)  Those  who  are  naturalized  in  accordance  with  law.”
If a man is to be punished for what he has done, it must be in
accordance with rules which were already in existence at the “Natural-born citizens are those who are citizens of the
time he acted. A law cannot be passed to reach back to days of Philippines from birth without having to perform any act to
innocence and suddenly, by legal fiat, declare certain acts acquire or perfect their Philippine citizenship. Those who elect
which were then considered perfectly legal as badges of guilt. Philippine citizenship in accordance with paragraph (3), Section
36
Much less can the legislature simply create by its own hands, 1 hereof shall be deemed natural-born  citizens.”
without any judicial determination, a judgment that certain
persons are guilty and visit upon them the corresponding Philippine Citizens
penalty, similarly fashioned just then by them. In any society
that abides by the dictates of due process and republicanism, Citizens by Election
such legislative determinations could never be considered
consistent with expectations of fairness and justice, as well as Application for admission to the Philippine bar, Vicente ching
with   the   overall   design   that   balances   State’s   authority   with  
individuals’  right  to  liberty  and  freedom Vicente Ching,legitimate son of a Chinese citizen and a Filipino was
born 1964.after completing bachelor of laws in st louis baguio he filed
an application to take the 1998 bar examinations. He was permitted to
Additional Cases take the bar subject to the condition that he must submit proof of his
(R) Ex Post Facto Laws [2] Filipino citizenship. He submitted certification by board of accountancy
that he is a CPA and a voters certification and that he was elected as
R1. Salvador v. Mapa, Jr sangguniang bayan of la union. He passed the bar exams but he was not
539 SCRA 34 (2007) allowed to take the oath unless he submits further proof of citizenship.
In the comment filed by OSG, it states that being the legitimate child of
President Ramos issued AO13/MO61 creating the Presidential Ad Hoc Chinese father and a Filipino mother bprn under the 1935 constitution
Fact-Finding Committee on Behest Loans. Several loan accounts were was a chinese citizen and continued to be so until upon reaching the
referred to the Committee for investigation, among others is that of age of majority he elected Philippine citizenship. Ching has not formally
Philippine Eagles Mines, Inc. (PEMI) whose officers were known cronies elected Philippine citizenship and if ever he does it is no longer within
of then President Marcos. The Committee then filed a complaint the reasonable time allowed by jurisprudence.
against the respondents before the Ombudsman for violation of Anti-
Graft and Corrupt Practices Act. The complaint was dismissed due to Can a legitimate child born under the 1935 constitution of a Filipino
prescription and if such orders are to be considered as bases of
charging respondents, they become ex post facto laws. Hence, the 35 CONSTITUTION, Art. IV, § 1
36 CONSTITUTION, Art. IV, § 2
151 | P LATON
mother and an alien father 14 years after he has reached the age of allowed to takenoath of allegiance despite the opposition of republic.
majority?
Won a Chinese woman may be lawfully naturalized as a citizen of the
The reasonable time has been interpreted to mean that the election Philippines separately from her husband also a citizen of China.
should be made within 3 years from reaching age of majority. But this
period may be extended under certain circumstances as when the It is clear that she is a Chinese bgy birt, parentage and by marryiage to
person concerned has always considered himself a Filipino. However a Chinese. And commonwealth act also declares that a woman loses
span of 14 years is way beyond the contemplation of the requirements her citizenship if married to an alien if by vitue of the laws in her
of electing upon reaching the age of majority. Ching offered no reason husbands  country,  she  acquires  the  latter’s  nationality.
why he delayed his election. Philippine citizenship can never be treated
like a commodity that can be claimed when needed and suppressed It is true that our laws do not say that only male alien may seek
when convenient. One who is privileged to elect Philippine citizenship naturalization but we must take into account other provisions on
has only an inchoate right to such citizenship. He should avail his right citizenship. The granting of citizenship by naturalization to a female
with fervor enthusiasm and promptitude. Application to Philippine bar alien applicant while her marriage to another alien is subsisting would
denied. be inoperative because she would still be a citizen of the country of her
husand. The result would be dual citizenship which is believed not
Natural-Born Citizens contemplated under our laws.

Tecson vs. COMELEC Denaturalization

The 3 consolidated cases challenge the qualification of FPJ to hold the Chan Tek Lao v. Republic
highest office of the land on account of his allegedly not being a natural 55 SCRA 1 (1974)
born citizen. His mother was an American and his father a Spanish
national. Petitioner averred that assuming that his father was a Filipino Facts: Petitioner’s  application  for  naturalization  was  denied  by  the  trial  
citizen, this would not be transmitted to him because he is an court but the same was reversed by the SC. More than 10 yrs. later, the
illegitimate child of an alien mother. OSG filed a petition for the cancellation of the certificate of
naturalization, invoking a subsequent issue in Tan Ten Koc v. Republic,
Citizenship is a treasured right conferred to those whom the state since there was no proof that the Nueva Era was a newspaper of
believes are deserving of the privilege. It is a precious heritage, as well general circulation in the province of Tarlac, where the petitioner then
as an inestimable acquisition, that cannot be taken lightly by anyone resided. It must be proved that the petition was published in a
either by those who enjoy it or by those who dispute it. newspaper of general circulation in the province where the proceedings
were held. The SC noted that although it reversed the decision of the
The evidence may not establish conclusively that respondent FPJ is a trial court, it has no jurisdiction to hear the application if Chan Tek Lao
natural born citizen, the evidence on hand still would preponderate in and therefore ordered the cancellation of his certificate of
his favor enough to hold that he cannot be held guilty having made naturalization.
material misrepresentation in his certificate of candidacy.
Issue: WON  petitioner’s  certificate  of  naturalization  should  be  cancelled
Naturalization
Held: As decided in Gan Tsitung v. Republic, no retroactive effect is to
be given a judicial pronouncement that would impose on a party
Qualifications and Disqualifications
proceeded against in a denaturalization proceeding a requirement not
in existence at a time that his application was heard and favorably
Procedure acted on. It would be unfair to set aside a decision that has become
final and has led to the grant of citizenship. Moreover, the status of the
Effect of Naturalization on Spouse and Children petitioner as a national of this country for 13 yrs. has remained
undisturbed. The decision of the lower court ordering the cancellation
BURCA vs republic of the certificate of naturalization of the petitioner is set aside and
reversed.
Petitioner, a Chinese woman filed a petition alleging that she Is married
to a Filipino citizen and possesses all the qualifications and none of the Loss and Reacquisition of Citizenship
disqualifications for naturalization. The SG opposed because it lacks
essential requisites such as formen resisdenc etc. the rtc granted Loss of Citizenship
naturalization. SC agreed with SG and reversed rtc judgement.
Yu v. Defensor-Santiago
The only means by which an alien wife of Filipino be declared as a
169 SCRA 364 (1989)
Filipino is by full compliance with the procedure in naturalization law.
She must also prove that she has all the qualifications and none of
Facts: The case originated from a petition for habeas corpus filed with
disqualifications. She must file petition for naturalization in court,
the SC on July 4, 1988 seeking for the release from detention of herein
allege and prove all the requisites such as continuous residency of
petitioner. Upon its denial of the petition for habeas corpus, in its
atleast 10 years, lucrative income, etc.
November 10, 1988 resolution the SC disposed of the pending issues of
(1) jurisdiction of the Commission on Immigration and Deportation
We declare it to be a sound rule that where the citizenship of a party in
(CID) over a naturalized Filipino citizen and (2) validity of warrantless
a case is resolved by a court or by an administratie agency, as a material
arrest and detention of the same person. SC gave petitioner Yu a non-
issue to the controversy, after full blown hearing, with the sg of his
extendible period of 3 days from notice to explain and prove why he
authorized representative, and this finding is affirmed by court, the
should still be considered a citizen of the Philippines despite his
decision is conclusive f citizenship.
acquisition and use of a Portuguese passport.

Po vs republic Issue: WON   petitioner’s   claim   to   continued   Philippine   citizenship   is  


meritorious
Betty po lim a native born Chinese woman married lim son hue, alsoa
Chinese, filed with the lower court a petition for admission as citizen of Held: Petitioner’s   motion   for   release   from   detention   is   denied. As
the Philippines under the commonwealth act no. 473. She was also petitioner was originally issued a Portuguese passport which he

152 | P LATON
renewed, and despite of his naturalization as a Philippine citizen on Feb repatriated’’   and   ordered   the   cancellation   of   her   alien   certificate   of  
10, 1978, he still applied for and was issued a Portuguese passport by registration. The Provincial Fiscal, however, appealed.
the Consular Section of the Portugal Embassy in Tokyo. Also, while still
a Philippine citizen who had renounced upon his naturalization, Issue: WON petitioner, as judicially repatriated, is already considered a
‘’absolutely   and   for   all   allegiance   and   fidelity   to   any   foreign   prince,   Filipino citizen
potentate   state   or   sovereignty’’   and   pledged   to   ‘’maintain   true   faith  
and allegiance to the  Rep.  of  the  Phils.’’,  he  declared  his  nationality  as   Held: There is no law requiring or authorizing that repatriation should
Portuguese in commercial documents. Such acts were considered by be effected by a judicial proceeding. All that is required for a female
the   SC   amount   to   an   express   renunciation   of   petitioner’s   Philippine   citizen of the Philippines who lost her citizenship to an alien to
citizenship acquired through naturalization. Express renunciation reacquire her Philippine citizenship, upon the termination of her
means a renunciation that is made known distinctly, and explicitly and marital status, is for her to take necessary oath of allegiance to the
not left to inference or implication (Board of Immigration Republic of the Philippines and to register the said oath in the proper
Commissioners v. Go Gallano). civil registry. Decision appealed from is hereby revoked and set aside.

Aznar v. COMELEC Angat v. Republic


185 SCRA 703 (1990) 314 SCRA 438 (1999)

Facts: Private   respondent   Emilio   ‘’Lito’’   Osmena   filed   his certificate of Facts: Petitioner Gerardo Angat was a natural born Filipino citizen who
candidacy for the January 1988 local elections. However, the Cebu PDP- lost such until his citizenship by naturalization in USA. On Mar. 11,
Laban Provincial Council (Cebu-PDP Laban), represented by petitioner 1996, he filed before the RTC of Marikina City a petition to regain his
Jose B. Aznar, the incumbent Provincial Chairman, filed with the status as a Filipino citizen under CA No. 63, RA No. 965 and RA No. 2630
COMELEC, a petition for disqualification of private respondent alleging stating that he was born in 1954 in Tondo, Manila; he lost his Philippine
that that he is not a citizen of the Philippines, but of USA. Aznar citizenship when he got naturalized as an American; he returned to the
submitted then a certificate issued by the CID certifying the private Phils. in 1991; he has the qualifications required in CA No. 63, and RA
respondent is an American and holder of Alien Certificate of Nos. 965 and 2639 to reacquire Philippine citizenship; he possesses
Registration. The COMELEC resolved to order the Board of Canvassers none of the disqualification prescribed in CA No. 473 and he has reside
to continue canvassing but to suspend the proclamation. At the hearing in the Phils. at least 6 mos. immediately preceding the date of the
before the COMELEC, private respondent claims that he is a Filipino petition. He sought to be allowed to take his oath of allegiance to the
citizen since he is a legitimate child of Dr. Emilio D. Osmena, a Filipino RP but the motion was denied. He filed another motion to have the
and son of late President Sergio Osmena, Sr.; that he is a holder of a denial reconsidered and he was allowed by the court a quo. The court
valid and subsisting Phil. Passport; that he has been continuously also ordered him to take his oath of allegiance to the RP. Thereafter,
residing in the Philippines since birth; that he has not gone out of the petitioner was declared as repatriated and a citizen of the Phils.
country for more than 6 months, and that he has been a registered However, the OSG filed a Manifestation and Motion for reconsideration
voter in the Phils. since 1965. Having obtained the highest no. of votes, asserting that the petition should have been dismissed since the court a
private respondent was proclaimed the Provincial Governor of Cebu. quo lacks jurisdiction because the proper forum for it was the Special
COMELEC dismissed the petition for disqualification because it was not Committee on Naturalization. The court a quo found the petition
timely filed and it lacks of sufficient proof. meritorious and declared that it has no jurisdiction and considered the
granting of the petition null and void. Petitioner filed a motion for
Issues: WON private respondent is a Filipino citizen reconsideration since his petition was filed on Mar 14 1996 or months
WON he is allowed to run for and being elected to the office of before the Special Committee on Naturalization was constituted by the
Provincial Governor of Cebu Pres. under AO 285 on Aug. 22, 1996, and stated that the court a quo
had the authority over the case. The RTC denied the motion and
Held: Petitioner failed to present proof that private respondent had lost petitioner   assails   the   lower   court’s   dismissal   of   the   petition   by   giving  
Filipino citizenship by any of the modes provided for under C.A. No. 63 retroactive effect to AO 285.
which includes (1) by naturalization in a foreign country; (2) by express
renunciation of citizenship; and (3) by subscribing to an oath of Issue: WON the court a quo had the   jurisdiction   over   the   petitioner’s  
allegiance to support the Constitution or laws of a foreign country. case
From the evidence, it is clear that private respondent did not lose his
Philippine citizenship by any of the aforementioned modes or any other Held: Under PD No. 725, dated June 5, 1975, amending CA No. 63, an
mode of losing Philippine citizenship. There is neither an express or application for repatriation could be filed by Filipino women who lost
implied renunciation of Philippine citizenship by private respondent; I It their Philippine citizenship by marriage to aliens, as well as by natural
must be noted that in order to lose Philippine citizenship, renunciation born Filipinos who lost their Philippine citizenship with the Special
must be express. Petition for certiorari is DISMISSED and Resolution of Committee on Naturalization. The OSG was right in contending that the
the COMELEC is AFFIRMED. petition should have been filed with the Committee and not with the
RTC which has no jurisdiction. It was also incorrect for the petitioner to
invoke RA Nos. 965 and 2639 since these laws could only apply to
Reacquisition of Citizenship persons who had lost their citizenship by rendering service to or
accepting commission in the armed forces of an allied foreign country
or of the USA. Also, a person who desires to reacquire Philippine
Repatriation
citizenship would not even be required to file a petition in court. All he
has to do is to take an oath of allegiance to the RP and to register that
Jao v. Republic fact with the civil registry in the place of his residence or where he had
121 SCRA 358 (1983) last resided in the Phils. Petition for review is DENIED.

Facts: Jao filed a petition in the CFI of Davao for repatriation alleging
Frivaldo v. COMELEC
that although her father was Chinese, she was a Filipino citizen because
174 SCRA 245 (1989)
her mother was a Filipina who was not legally married to her Chinese
husband; that she lost her Philippine citizenship when she married a
Facts: Petitioner Juan G. Frivaldo was proclaimed to be the governor-
Chinese with whom she had 3 children and that he died in 1962; that
elect of Sorsogon on Jan 22, 1988. However, the League of
her illiterate mother erroneously registered her as an alien with the
Municipalities, represented by its President filed with the COMELEC a
Bureau of Immigration which issued her an Alien Certificate of
petition  for  the  annulment  of  Frivaldo’s  election  and  proclamation  since  
Registration. Petition was not published but notice was served to the
he was not a Filipino citizen having been naturalized in the USA on Jan
Provincial  Fiscal.  The  CFI  subsequently,  declared  petitioner  as  ‘’judicially  
23 1983. Frivaldo admitted that he was naturalized but only due to his

153 | P LATON
protection against Pres. Marcos. He asserted that he also returned to elective public office, and the purpose of the citizenship qualification is
the Phils. after the EDSA revolution. He also claims that the League was none other than to ensure that no alien, i.e., no person owing
not the proper party to file the petition because it is not a voter and allegiance to another nation, shall govern our people and our country
could not sue him. The COMELEC denied   Frivaldo’s   motion   for   or a unit of territory thereof. Now, an official begins to govern or to
reconsideration. He then filed a petition for certiorari and prohibition discharge his functions only upon his proclamation and on the day the
before the SC. He claimed that his oath in his certificate of candidacy law mandates his term of office begin.
that he was a natural-born citizen should be sufficient act of
repatriation and that his active participation in the 1987 congressional Being a former Filipino who has served the people repeatedly, Frivaldo
elections had divested him of American citizenship under the laws of deserves a liberal interpretation of Philippine laws and whatever
the USA , thus restoring his Philippine citizenship. defects there were in his nationality should now be deemed mooted by
his repatriation.
Issue: WON Frivaldo is a citizen of the Philippines under our own laws,
regardless of other nationality laws In sum, we rule that the citizenship requirement in the Local
Government Code is to be possessed by an elective official at the latest
Held: If he really wanted to disavow his American citizenship and as of the time he is proclaimed and at the start of the term of office to
reacquire Philippine citizenship, Frivaldo should have done such in which he has been elected.
accordance the Philippine laws. Under CA no. 63 as amended by CA No.
43 and PD No. 725, Philippine citizenship may be reacquired by direct Laws governing election contest must be liberally construed to the end
act of Congress, by naturalization or by repatriation. Since Frivaldo did that the will of the people in the choice of public officials may not be
not invoke either of the first two methods, he still claims that he has defeated by mere technical objections.
reacquired Philippine citizenship by virtue of a valid repatriation as he
actively participated in the 1987 election. However, such act might Labo, Jr. vs. COMELEC
have forfeited his American citizenship but it does not necessarily mean 176 SCRA 1 (1989)
that he has automatically reacquired Philippine citizenship. Petition is Ponente: Cruz, J.
DISMISSED and petitioner is declared not a citizen of the Philippines
and therefore, disqualified from serving as Governor of Sorsogon. FACTS: Ramon Labo, Jr., married, an Australian citizen in the
Philippines. He was granted Australian citizenship in 1976. In 1980, the
Republic v. De La Rosa marriage was declared void for being bigamous. Labo returned to the
232 SCRA 785 (1994) Philippines in 1980, using an Australian passport, and obtained an Alien
Certificate of Registration (ACR). He later applied for a change in status
It involves three consolidated petitions. The private respondent filed a from immigrant to returning Filipino citizen. However, the Commission
petition for naturalization. The judge granted the petition and on Immigration and Deportation denied his application for the
readmitted Frivaldo as a citizen of the Philippines by naturalization. cancellation of his ACR since he has not applied for reacquisition of his
Frivaldo ran for governor in the May 1992 elections. Raul Lee also ran Filipino citizenship. According to the records of the Australian Embassy
for the same position. The former won and Lee filed a petition with a (as certified by the Australian Consul), Labo was still an Australian
petition to the COMELEC to annul the proclamation on the ground that citizen as of April 12, 1984. Although no direct evidence was presented
the private respondent is not a duly registered voter due to the pending to prove that he took an oath of allegiance as a naturalized Australian
case. The COMELEC dismissed the petition. citizen, the laws of Australia at the time required any person over the
age of 16 years who is granted Australian citizenship to take an oath of
The COMELEC failed to resolve the more serious issue – the allegiance. The wording/text of this oath includes a renunciation of all
disqualification of private respondent to be proclaimed Governor on other allegiance. Labo ran and won as Mayor of Baguio City in the local
grounds of lack of Filipino citizenship. Both the Local Government Code elections held on January 18, 1988. The second-placer, Luis Lardizabal,
and the Constitution require that only Filipino citizens can run and be filed a petition for quo warranto, alleging that Labo is disqualified from
elected to public office. holding public office on the grounds of alienage, and asking that the
latter's proclamation as Mayor be annulled.
Private respondent is declared NOT a citizen of the Philippines and
therefore DISQUALIFIED from continuing to serve as GOVERNOR of the ISSUES: *The original issue raised before the Supreme Court concerned
Province of Sorsogon. He is ordered to VACATE his office and to only the COMELEC's jurisdiction over Lardizabal's petition. Labo
SURRENDER the same to the Vice-Governor of the Province of contended that the petition for quo warranto was not filed on time,
Sorsogon. hence the COMELEC lacks the jurisdiction to conduct an inquiry
regarding his citizenship. However, the SC decided to rule on the merits
Frivaldo v. COMELEC of the case, given that the issue is also of considerable importance (a
257 SCRA 727 (1996) foreign citizen holding public office in the Philippines), and in the
interest of the speedy administration of justice.
Frivaldo filed his Certificate of Candidacy for the office of Governor of
Sorsogon while Raul Lee, another candidate, filed a petition praying 1. Does the COMELEC have the jurisdiction to inquire into Labo's
that  Frivaldo  “be  disqualified  from  seeking  or  holding  any  public  office   citizenship?
or position by reason not  yet  being  a  citizen  of  the  Philippines”. 2. Is Ramon Labo, Jr. a Filipino citizen?
3. Is he qualified to hold public office in the Philippines?
Who should be declared the rightful governor of Sorsogon? 4. If Labo is not eligible to serve as Mayor, can Lardizabal, as the
runner-up in the elections, replace him?
The Local Government Code of 1991 expressly requires Philippine
citizenship as a qualification for elective local officials, including the HELD/RATIO:
provincial governor. Under Philippine law, citizenship may be 1. Yes. Contrary to Labo's claim, the petition for quo warranto was filed
reacquired by direct act of Congress, by naturalization or by on time. Lardizabal did not immediately pay the filing fee because the
repatriation. COMELEC had at first considered the petition as a pre-proclamation
proceeding, which does not require the payment of such a fee. When
Despite his lack of Philippine citizenship, Frivaldo was overwhelmingly the COMELEC reclassified the petition, Lardizabal immediately paid the
elected governor by the electorate of Sorsogon. Twice, he was judicially filing fee -- thus, he still complied with the prescribed 10-day period.
declared a non – Filipino and thus twice disqualified from holding and Furthermore, the Court held that such technicalities should not hinder
discharging his popular mandate. judicial decisions on significant issues, such as the one being decided in
this case.
Philippine citizenship is an indispensable requirement for holding an

154 | P LATON
2. Labo is not a Filipino citizen. He had lost his Philippine citizenship by proceeding established by the law, or the Rules for the judicial
all 3 modes specified in the Constitution: (1) naturalization in a foreign declaration of the citizenship of an individual. Hence, a judicial
country, (2) express renunciation of citizenship, and (3) subscribing to declaration that a person is a Filipino citizen cannot be made in a
an oath of allegiance to support the Constitution or laws of a foreign petition for naturalization because under our laws there can be no
country. He has not reacquired Philippine citizenship by any of the 3 action or proceeding for the judicial declaration of the citizenship of an
methods prescribed in the Constitution: (1) direct act of Congress, (2) individual.  Such  a  declaration  or  pronouncement  is  beyond  the  court’s  
naturalization, and (3) repatriation. jurisdiction.

- Contrary to Labo's claim, his naturalization in Australia did not confer The appealed decision is AFFIRMED and the Commissioner of
him with dual citizenship. The Constitution explicitly states that dual Immigration and Deportation is ordered to CANCEL applicants alien
citizenship is inimical to national interest. certificate of registration.

BENGSON III V. HRET CHIAO BEN LIM V. ZOSA


357 SCRA 545 (2001) 146 SCRA 366 (1986)

Facts: Respondent Teodoro Cruz was a natural-born citizen of the FACTS: The petitioner filed a petition for the correction of an alleged
Philippines. He was born in San Clemente, Tarlac, on April 27, 1960, of wrong entry on the birth records of Kim Joseph describing him as a
Filipino parents. The fundamental law then applicable was the 1935 Chinese national instead of a Filipino citizen. He sought to prove the
Constitution. On November 5, 1985, however, respondent Cruz enlisted error through several pieces of evidence. The respondent judge
in the United States Marine Corps and without the consent of the dismissed the petition and sustained the contention that only clerical
Republic of the Philippines, took an oath of allegiance to the United errors were allowed to be corrected. Substantial issues like citizenship
States. As a Consequence, he lost his Filipino citizenship for under were not covered. In effect, it was held the petition was for a judicial
Commonwealth Act No. 63, section 1(4), a Filipino citizen may lose his declaration of citizenship, which was not allowed under existing rules.
citizenship by, among other, "rendering service to or accepting
commission   in   the   armed   forces   of   a   foreign   country.”   He   was   RULING: Article  412  of  the  Civil  Code  simply  provides:  “No  entry  in  the  
naturalized in US in 1990. On March 17, 1994, respondent Cruz civil  registry  shall  be  changed  or  corrected  without  a  judicial  order.”  In  a  
reacquired his Philippine citizenship through repatriation under number of earlier cases, the Court has ruled that the birth entry
Republic Act No. 2630. He ran for and was elected as the regarding   a  person’s  citizenship  could  not   be   changed under Rule 108
Representative of the Second District of Pangasinan in the May 11, as this would involve substantive rights that the rules of court could not
1998 elections. He won over petitioner Antonio Bengson III, who was “diminish,  increase,  or  modify”  under  the  Constitution.  Rule  108  of  the  
then running for reelection. Rules of Court provides only the procedure or mechanism for the
proper enforcement of the substantive law embodied in Article 412 of
Issue: Whether or Not respondent Cruz is a natural born citizen of the the Civil Code and so does not violate the Constitution.
Philippines in view of the constitutional requirement that "no person
shall be a Member of the House of Representative unless he is a Dual Citizenship and Allegiance
natural-born   citizen.”

Held: Respondent is a natural born citizen of the Philippines. As


distinguished from the lengthy process of naturalization, repatriation
simply consists of the taking of an oath of allegiance to the Republic of Additional Cases
the Philippine and registering said oath in the Local Civil Registry of the (S) Citizenship [5]
place where the person concerned resides or last resided. This means
that a naturalized Filipino who lost his citizenship will be restored to his
prior status as a naturalized Filipino citizen. On the other hand, if he
was originally a natural-born citizen before he lost his Philippine S2. JOEVANIE ARELLANO TABASA vs. HON. COURT OF APPEALS,
citizenship, he will be restored to his former status as a natural-born BUREAU OF IMMIGRATION and DEPORTATION and WILSON SOLUREN.
Filipino.
petitioner Joevanie Arellano Tabasa was a natural-born citizen of the
JUDICIAL DECLARATION AND CORRECTION OF CITIZENSHIP Philippines. By derivative naturalization (citizenship derived from that
The general rule is that there is no judicial proceeding for declaration of of another as from a person who holds citizenship by virtue of
citizenship. Thus, in the past, corrections of entries in birth certificates naturalization), petitioner also acquired American citizenship.
with regard to citizenship were considered not possible as this would
involve making a declaration that a person is a Philippine citizen or not. His passport was revoked by the US because he is the subject of an
However, the Court has relaxed the rules. outstanding federal warrant of arrest for violation of Section 1073,
"Unlawful Flight to Avoid Prosecution," of Title 18 of the United States
Judicial Declaration and Correction of Citizenship Code. He is charged with one count of a felon in possession of a
firearm, in violation of California Penal Code, Section 12021(A)(1), and
Yung Uan Chu V. Republic one count of sexual battery, in violation of California Penal Code,
159 SCRA 593 (1988) Section 243.4 (D).

FACTS: Appellee married to a native – born citizen of the Philippines. The BID ordered   petitioner’s   deportation   to   his   country   of   origin,   the  
Out of said wedlock and at the time of the filing of the petition for United States, on May 29, 1996.
naturalization, the couple had six (6) children, with ages ranging from 6
to 15 years old. All said children were registered as natural born Filipino Tabasa alleged that he was not afforded due process; that no warrant
citizens. Find by the trial court that the appellee possessed of all the of arrest for deportation may be issued by immigration authorities
qualifications and none of the disqualifications of a Filipino citizen and before a final order of deportation is made; that no notice of the
therefore authorized to take her oath of allegiance to the Republic of cancellation of his passport was made by the U.S. Embassy; that he is
the Philippines and to register the same in the proper civil registrar. entitled to admission or to a change of his immigration status as a non-
quota immigrant because he is married to a Filipino citizen as provided
RULING: A careful examination of the records shows that the sole and in Section 13, paragraph (a) of the Philippine Immigration Act of 1940;
only purpose of the petitioner is to have the petitioner declared a and that he was a natural-born citizen of the Philippines prior to his
Filipino citizen. This Court has consistently ruled that there is no derivative naturalization when he was seven years old due to the
naturalization of his father, Rodolfo Tabasa, in 1968.
155 | P LATON
has the power to determine if an applicant for repatriation meets the
Petitioner is alleging that he had acquired Filipino citizenship by requirements of the law for it is an inherent power of the State to
repatriation in accordance with Republic Act No. 8171 (RA 8171), and choose who will be its citizens, and who can reacquire citizenship once
that because he is now a Filipino citizen, he cannot be deported or it is lost. If the applicant, like petitioner Tabasa, fails to comply with said
detained by the respondent Bureau. requirements, the State is justified in rejecting the petition for
repatriation.
Issue:
Whether petitioner has validly reacquired Philippine citizenship under Petitioner Tabasa, whose passport was cancelled after his admission
RA 8171. If there is no valid repatriation, then he can be summarily into the country, became an undocumented alien who can be
deported for his being an undocumented alien. summarily deported. His subsequent "repatriation" cannot bar such
deportation especially considering that he has no legal and valid
No. it provides repatriation of only two (2) classes of persons reacquisition of Philippine citizenship.

Filipino women who have lost their Philippine citizenship by marriage WHEREFORE, this petition for review is DISMISSED, and the August 7,
to aliens and natural-born Filipinos who have lost their Philippine 1996 Decision of the Court of Appeals is AFFIRMED. No costs to the
citizenship, including their minor children, on account of political or petitioner.
economic necessity, may reacquire Philippine citizenship through
repatriation in the manner provided in Section 4 of Commonwealth Act S3. EDISON SO,vs.REPUBLIC OF THE PHILIPPINES
No. 63, as amended: Provided, That the applicant is not a:
(1) Person opposed to organized government or affiliated with any Edison So filed in the RTC a Petition for Naturalization. RTC Granted the
association or group of persons who uphold and teach doctrines petition. Respondent Republic of the Philippines, through the Office of
opposing organized government; the Solicitor General (OSG), appealed the decision to the CA. CA
(2) Person defending or teaching the necessity or propriety of violence, Reversed the RTC hence this petition for review on certiorari.
personal assault, or association for the predominance of their ideas;
(3) Person convicted of crimes involving moral turpitude; or Edison So was born on February 17, 1982, in Manila; he is a Chinese
(4) Person suffering from mental alienation or incurable contagious citizen who has lived in No. 528 Lavezares St., Binondo, Manila, since
diseases. (Emphasis supplied.) birth; as an employee, he derives an average annual income of
around P100,000.00 with free board and lodging and other benefits; he
To reiterate, the only persons entitled to repatriation under RA 8171 is single, able to speak and write English, Chinese and Tagalog.
are the following:
a. Filipino women who lost their Philippine citizenship by marriage to He  was  denied  by  the  CA  because,  according  to  the  CA,  petitioner’s  two  
aliens; and (2) witnesses were not credible because they failed to mention specific
b. Natural-born Filipinos including their minor children who lost their details  of  petitioner’s  life  or character to show how well they knew him;
Philippine citizenship on account of political or economic necessity. they merely "parroted" the provisions of the Naturalization Act without
Petitioner is not entitled to automatic repatriation because he was no clearly   explaining   their   applicability   to   petitioner’s   case.   The   appellate  
longer a minor at the time of his "repatriation" on June 13, 1996. The court likewise ruled that petitioner failed to comply with the
privilege under RA 8171 belongs to children who are of minor age at requirement of the law that the applicant must not be less than 21
the time of the filing of the petition for repatriation. years of age on the day of the hearing of the petition; during the first
hearing on December 12, 2002, petitioner was only twenty (20) years,
Neither can petitioner be a natural-born Filipino who left the country nine (9) months, and twenty five (25) days old, falling short of the
due to political or economic necessity. Clearly, he lost his Philippine requirement. The CA stated, however, that it was not its intention to
citizenship by operation of law and not due to political or economic forever close the door to any future application for naturalization which
exigencies. petitioner would file, and that it believes that he would make a good
Filipino citizen in due time, a decided asset to this country.
In sum, petitioner is not qualified to avail himself of repatriation under
RA 8171. However, he can possibly reacquire Philippine citizenship by WHETHER OR NOT THE HONORABLE COURT OF APPEALS COMMITTED
availing of the Citizenship Retention and Re-acquisition Act of 2003 REVERSIBLE ERROR WHEN IT REVERSED THE DECISION OF THE
(Republic Act No. 9225) by simply taking an oath of allegiance to the REGIONAL TRIAL COURT OF MANILA.
Republic of the Philippines.
Even if petitioner––now of legal age––can still apply for repatriation In its Comment on the petition, respondent countered that R.A. No.
under RA 8171, he nevertheless failed to prove that his parents 9139   (which   took   effect   on   August   8,   2001   and   where   the   applicant’s  
relinquished their Philippine citizenship on account of political or age requirement was lowered to eighteen (18) years old), refers only to
economic necessity as provided for in the law. It is notable that under administrative naturalization filed with the Special Committee on
the Amended Rules and Regulations Implementing RA 8171, the SCN Naturalization; it does not apply to judicial naturalization before the
requires a petitioner for repatriation to set forth, among others, "the court, as in the present case.
reason/s why petitioner lost his/her Filipino citizenship, whether by
marriage in case of Filipino woman, or whether by political or economic The petition is denied for lack of merit.
necessity in case of [a] natural-born Filipino citizen who lost his/her
Filipino citizenship. In case of the latter, such political or economic Naturalization signifies the act of formally adopting a foreigner into the
necessity should be specified." political body of a nation by clothing him or her with the privileges of a
citizen. Under current and existing laws, there are three ways by which
Petitioner contends it is not necessary to prove his political or an alien may become a citizen by naturalization: (a) administrative
economic reasons since the act of  renouncing  allegiance  to  one’s  native   naturalization pursuant to R.A. No. 9139; (b) judicial naturalization
country constitutes a "necessary and unavoidable shifting of his pursuant to C.A. No. 473, as amended; and (c) legislative naturalization
political   allegiance,"   and   his   father’s   loss   of   Philippine   citizenship   in the form of a law enacted by Congress bestowing Philippine
through naturalization "cannot therefore be said to be for any reason citizenship to an alien.
other than political or economic necessity."
Indeed, R.A. No. 9139 was enacted as a remedial measure intended to
Repatriation is not a matter of right, but it is a privilege granted by the make the process of acquiring Philippine citizenship less tedious, less
State. This is mandated by the 1987 Constitution under Section 3, technical and more encouraging. It likewise addresses the concerns of
Article IV, which provides that citizenship may be lost or reacquired in degree holders who, by reason of lack of citizenship requirement,
the manner provided by law. The State has the power to prescribe by cannot practice their profession, thus promoting "brain gain" for the
law the qualifications, procedure, and requirements for repatriation. It Philippines. These   however,   do   not   justify   petitioner’s   contention   that  

156 | P LATON
the qualifications set forth in said law apply even to applications for most that was established was the educational attainment of the
naturalization by judicial act. witnesses; however, this cannot be equated with their credibility. In
fine, petitioner focused on presenting evidence tending to build his
There is nothing from which it can be inferred that C.A. No. 473 was own good moral character and neglected to establish the credibility and
intended to be amended or repealed by R.A. No. 9139. What the good moral character of his witnesses.
legislature had in mind was merely to prescribe another mode of
acquiring Philippine citizenship which may be availed of by native born We   do   not   agree   with   petitioner’s   argument   that   respondent   is  
aliens. The only implication is that, a native born alien has the choice to precluded from questioning the RTC decision because of its failure to
apply for judicial or administrative naturalization, subject to the oppose the petition. A naturalization proceeding is not a judicial
prescribed qualifications and disqualifications. adversary proceeding, and the decision rendered therein does not
constitute res judicata. A certificate of naturalization may be cancelled
In the instant case, petitioner applied for naturalization by judicial act, if it is subsequently discovered that the applicant obtained it by
though at the time of the filing of his petition, administrative misleading the court upon any material fact. Law and jurisprudence
naturalization under R.A. No. 9139 was already available. Consequently, even authorize the cancellation of a certificate of naturalization upon
his application should be governed by C.A. No. 473. grounds or conditions arising subsequent to the granting of the
certificate. If the government can challenge a final grant of citizenship,
Second. If the qualifications prescribed in R.A. No. 9139 would be made with more reason can it appeal the decision of the RTC within the
applicable even to judicial naturalization, the coverage of the law would reglementary period despite its failure to oppose the petition before
be broadened since it would then apply even to aliens who are not the lower court.
native born. It must be stressed that R.A. No. 9139 applies only to
aliens who were born in the Philippines and have been residing here. Thus, petitioner failed to show full and complete compliance with the
requirements of naturalization law. For this reason, we affirm the
Third. Applying the provisions of R.A. No. 9139 to judicial naturalization decision of the CA denying the petition for naturalization without
is contrary to the intention of the legislature to liberalize the prejudice.
naturalization procedure in the country. One of the qualifications set It must be stressed that admission to citizenship is one of the highest
forth in R.A. No. 9139 is that the applicant was born in the privileges that the Republic of the Philippines can confer upon an alien.
Philippines and should have been residing herein since birth. Thus, one It is a privilege that should not be conferred except upon persons fully
who was born here but left the country, though resided for more than qualified for it, and upon strict compliance with the law.
ten (10) years from the filing of the application is also disqualified. On IN LIGHT OF ALL THE FOREGOING, the petition is DENIED for lack of
the other hand, if we maintain the distinct qualifications under each of merit.
the two laws, an alien who is not qualified under R.A. No. 9139 may still SO ORDERED.
be naturalized under C.A. No. 473.
Qualifications and Disqualifications according to:
In any event, petitioner failed to prove that the witnesses he presented C.A. No. 473 (Judicial Act of acquiring Philippine Citizenship)
were competent to vouch for his good moral character, and are Section 2. Qualifications. – Subject to section four of this Act, any
themselves possessed of good moral character. It must be stressed that person having the following qualifications may become a citizen of the
character witnesses in naturalization proceedings stand as insurers of Philippines by naturalization:
the   applicant’s   conduct   and   character.   Thus,   they   ought   to   testify   on  
specific facts and events justifying the inference that the applicant First. He must be not less than twenty-one years of age on the day of
possesses all the qualifications and none of the disqualifications the hearing of the petition;
provided by law.
Second. He must have resided in the Philippines for a continuous period
Petitioner’s   witnesses,   Atty.   Adasa   and   Salcedo,   did   not   testify   on   his   of not less than ten years;
specific acts; they did not elaborate on his traits. Their testimonies do
not convince the Court that they personally know petitioner well and Third. He must be of good moral character and believes in the
are therefore in a position to vouch for his qualifications. As correctly principles underlying the Philippine Constitution, and must have
found  by  the  CA,  the  witnesses’  testimonies  consisted  mainly  of  general   conducted himself in a proper and irreproachable manner during the
statements in answer to the leading questions propounded by his entire period of his residence in the Philippines in his relation with the
counsel. What they conveniently did was to enumerate the constituted government as well as with the community in which he is
qualifications as set forth in the law without giving specific details. The living;
pertinent  portion  of  Atty.  Adasa’s  testimony  follows:
Fourth. He must own real estate in the Philippines worth not less than
In  sum,  petitioner’s  witnesses  clearly  did  not  personally  know  him  well   five thousand pesos, Philippine currency, or must have some known
enough; their testimonies do not satisfactorily establish that petitioner lucrative trade, profession, or lawful occupation;
has all the qualifications and none of the disqualifications prescribed by
law. Fifth. He must be able to speak and write English or Spanish and any
one of the principal Philippine languages; and
In naturalization proceedings, it is the burden of the applicant to prove
not only his own good moral character but also the good moral Sixth. He must have enrolled his minor children of school age, in any of
character of his/her witnesses, who must be credible persons. Within the public schools recognized by the Office of Private Education of the
the purview of the naturalization law, a "credible person" is not only an Philippines (now the Department of Education, Culture and Sports),
individual who has not been previously convicted of a crime; who is not where Philippine history, government and civics are taught or
a police character and has no police record; who has not perjured in the prescribed as part of the school curriculum, during the entire period of
past; or whose affidavit or testimony is not incredible. What must be residence in the Philippines required of him prior to the hearing of this
credible is not the declaration made but the person making it. This petition for naturalization as Philippine citizen.
implies that such person must have a good standing in the community;
that he is known to be honest and upright; that he is reputed to be Section 4. Who are disqualified. – The following cannot be naturalized
trustworthy and reliable; and that his word may be taken on its face as Philippine citizens:
value,  as  a  good  warranty  of  the  applicant’s  worthiness.   (a) Persons opposed to organized government or affiliated with any
association or group of persons who uphold and teach doctrines
The records likewise do not show that the character witnesses of opposing all organized governments;
petitioner are persons of good standing in the community; that they are (b) Persons defending or teaching the necessity or propriety of
honest and upright, or reputed to be trustworthy and reliable. The violence, personal assault, or assassination of the success and

157 | P LATON
predominance of their ideas;
(c) Polygamist or believers in the practice of polygamy;
(d) Persons convicted of crimes involving moral turpitude; Chapter 25
(e) Persons suffering from mental alienation or incurable contagious Suffrage
diseases;
(f) Persons who, during the period of their residence in the Philippines,
have not mingled socially with the Filipinos, or who have not evinced a “Suffrage  may  be  exercised by all citizens of the Philippines not
sincere desire to learn and embrace the customs, traditions, and ideals otherwise disqualified by law, who are at least eighteen years
of the Filipinos; of age, and who shall have resided in the Philippines for at least
(g) Citizens or subjects of nations with whom the United States and the one year, and in the place wherein they propose to vote, for at
Philippines are at war, during the period of such war; least six months immediately preceding the election. No
(h) Citizens or subject of a foreign country other than United States, literacy, property, or other substantive requirement shall be
whose laws do not grant Filipinos the right to become naturalized imposed  on  the  exercise  of  suffrage.”
37
citizens or subjects thereof.

R.A. No. 9139 (administrative act of acquiring Philippine Citizenship) Corollary to the idea of a republic, or a representative
Section 3. Qualifications. – Subject to the provisions of the succeeding government, must necessarily go the attendant presence of a
section, any person desiring to avail of the benefits of this Act must mechanism by which the people can express and manifest their
meet the following qualifications: sovereign will. That is through the exercise of their power of
(a) The applicant must be born in the Philippines and residing therein suffrage.   “The   holding of periodic elections constitutes the
since birth; very foundation of a republican form of government such as
(b) The applicant must not be less than eighteen (18) years of age, at ours.” Through elections, the people can install who they
the time of filing of his/her petition;
choose to momentarily represent them in the governance of
(c) The applicant must be of good moral character and believes in the
underlying principles of the Constitution, and must have conducted their affairs. By means of elections, too, the people can cleanse
himself/herself in a proper and irreproachable manner during his/her the government of people that they do not think deserving of
entire period of residence in the Philippines in his relation with the duly their trust, or by their lethargy and inattention retain or elect
constituted government as well as with the community in which he/she people who may not be deserving of the honor and the office.
is living; At the same time, suffrage also gives them a say in regard to
(d) The applicant must have received hid/her primary and secondary certain issues that may be presented to them for their approval
education in any public school or private educational institution duly or rejection, as exemplified in the conduct of plebiscites,
recognized by the Department of Education Culture and Sports, where
referenda and initiative. Whatever direction the people take
Philippine history, government and civics are taught and prescribed as
part of the school curriculum and whose enrollment is not limited to through the exercise of their right, it is the choice they make
any race or nationality; Provided, That should he/she have minor would not live up their expectations, then it would simply be a
38
children of school age, he/she must have enrolled them in similar mater of getting what they deserve
schools;
(e) The applicant must have a known trade, business, profession or Also, if only to underscore the importance of the political rights
lawful occupation, from which he/she derives income sufficient for of the people, it has been held that among those that may not
his/her support and if he/she is married and/or has dependents, also be the subject matter or object of contracts are certain rights of
that of his/her family; Provided, however, That this shall not apply to
individuals, which the law and public policy have deemed wise
applicants who are college degree holders but are unable to practice
their profession because they are disqualified to do so by reason of to exclude from the commerce of man, such as the political
their citizenship; rights conferred upon citizens, including, but not limited to,
(f) The applicant must be able to read, write and speak Filipino or any of one’s  right  to  vote,  the  right  to  present  one’s  candidacy  to  the  
the dialects of the Philippines; and people and to be voted to public office, provided, however,
(g) The applicant must have mingled with the Filipinos and evinced a that all qualifications prescribed by law obtain
sincere desire to learn and embrace the customs, traditions and ideals
of the Filipino people. Suffrage, A Historical Background
Section 4. Who are disqualified. – The following cannot be naturalized
as Philippine citizens: While the right of suffrage may be taken for granted now, it has
(a) Those opposed to organized government or affiliated with any not always been available, much less to everyone. The
association or group of persons who uphold and teach doctrines Constitution now guarantees the right to all citizens not
opposing all organized governments; otherwise disqualified by law, eighteen years of age, residents
(b) Those defending or teaching the necessity or propriety of violence, of the Philippines for at least one year, and of the place where
personal assault, or assassination of the success and predominance of he or she intends to vote for at least six months immediately
their ideas; preceding the election. He or she need not be literate, not must
(c) Polygamists or believers in the practice of polygamy;
he show any property, or comply with other substantive
(d) Those convicted of crimes involving moral turpitude;
(e) Those suffering from mental alienation or incurable contagious requirements before he or she may cast his or her vote
diseases;
(f) Those who, during the period of their residence in the Philippines, The   1935   Constitution   provided:   “Suffrage   may   be   exercised   by   male
have not mingled socially with the Filipinos, or who have not evinced a citizens of the Philippines not otherwise disqualified by law, who are
sincere desire to learn and embrace the customs, traditions, and ideals twenty-one years of age or over and are able to read and write, and
of the Filipinos; who shall have resided in the Philippines for one year and in the
(g) Citizens or subjects with whom the Philippines is at war, during the municipality wherein they propose to vote for at least six months
period of such war; preceding the election. The National Assembly shall extend the right of
(h) Citizens or subjects whose laws do not grant Filipinos the right to
become naturalized citizens or subjects thereof. 37CONSTITUTION, Art. V, § 1
38“Absolute freedom of choice of the parties and men by whom we shall be
governed, even if only among varying evils, is of the very essence in the
concept of democracy consecrated in the fundamental law of our land.”  
Gonzales v. COMELEC, 27 SCRA 835 (1969)
158 | P LATON
suffrage to women, if in a plebiscite which shall be held for that Elections and Representation
purpose within two years after the adoption of this Constitution, not
less than three hundred thousand women possessing the necessary The exercise of suffrage would have to find expression through
qualifications  shall  vote  affirmatively  on  the  question.”
periodic elections where men and women compete for the
electorate’s  votes  in  order  to  gain  the  right  to  represent  them  
The modern conception of the suffrage is that voting is a
momentarily. In this regard, questions may arise every now and
function of government. The right to vote is not a natural right
then with respect to the determination of what really is the will
but is a right created by law. Suffrage is a privilege granted by
of the people as shown in connection with ballots which may go
the State to such persons or classes as are most likely to
wither way for or against certain candidates. On this score, the
exercise it for the public good. In the early stages of the
Court said:
evolution of the representative system of government, the
exercise of the right of suffrage was limited to a small portion As long as popular government is an end to be achieved and
of the inhabitants. But with the spread of democratic ideas, the safeguarded, suffrage, whatever may be the modality and form
enjoyment of the franchise in the modern states has come to devised, must continue to be the manes by which the great reservoir of
embrace the mass of the audit classes of persons are excluded power must be emptied into the receptacular agencies wrought by the
from the franchise. Among the generally excluded classes are people through their Constitution in the interest of good government
minors, idiots, paupers, and convicts. and the common weal. Republicanism, in so far as it implies the
adoption of a representative type of government, necessarily points to
the enfranchised citizen as a particle of popular sovereignty and as the
Regulation of Suffrage
ultimate source of the established authority. He has a voice in his
Government and whenever called upon to act in justifiable cases, to
Just like any other right, simply because it exists does not mean give it efficacy and not to stifle it. This, fundamentally, is the reason for
that it can be exercised in any manner a person might want it the rule that ballots should be read and appreciated, if not with utmost,
done. It is also subject to regulation for the benefit of everyone. with reasonable, liberality. Counsel for both parties have called our
“In a representative democracy such as ours, the right of attention to the different and divergent rules laid down by this Court on
suffrage, although accorded a prime niche in the hierarchy of the appreciation of ballots. It will serve no good and useful purpose for
rights embodied in the fundamental law, ought to be exercised us to engage in the task of reconciliation or harmonization of these
rules, although this may perhaps be undertaken, as no two cases will be
within the proper bounds and framework of the Constitution
found to be exactly the same in factual or legal environment. It is
and must properly yield to pertinent laws skillfully enacted by sufficient to observe, however, in this connection that whatever might
the Legislature, which statutes for all intents and purposes, are have been said in cases heretofore decided, no technical rule or rules
crafted to effectively insulate such so cherished right from should be permitted to defeat the intention of the voter, if that
ravishment and preserve the democratic institutions our people intention is discoverable from the ballot itself, not from
have, for so long, guarded against the spoils of opportunism, evidence aliunde.
debauchery   and   abuse.”   Thus,   one   may   not   simply   go   to   the  
polling precinct and cast his or her vote one election day, even To prevent the establishment of political dynasties is not the
if he or she did not bother registering earlier only policy embodied in the constitutional provision in
question. The other policy is that of enhancing the freedom of
The act of registration is an indispensable precondition to the right of choice of the people. To consider, therefore, only stay in office
suffrage. For registration is part and parcel of the right to vote and an regardless of how the official concerned came to that office –
indispensable element in the election process...Proceeding from the whether by election or by succession by operation of law –
significance of registration as a necessary requisite to the right to vote, would be to disregard one of the purposes of the constitutional
the State undoubtedly, in the exercise of its inherent police power, may
provision in question
then  enact  laws  to  safeguard  and  regulate  the  act  of  voter’s  registration  
for the ultimate purpose of conducting honest, orderly and peaceful
election, to the incidental yet generally important end, that even pre- Related to the provisions on term limits is the constitutional
election activities could be performed by the duly constituted directive   with   regard   to   political   dynasties:   “The   State   shall  
authorities in a realistic and orderly manner--one which is not guarantee equal access to opportunities for public service, and
indifferent and so far removed from the pressing order of the day and prohibit political dynasties as may be defined  by  law.”  This  has  
the prevalent circumstances of the times. been apparently consciously ignored by Congress, however

Of course, it may not be lost sight of that registration is merely Moya v. Del Fierro
a means, not the end. One must not be mistaken for the other. 69 Phil. 199 (1939)
“Registration  regulates  the  exercise  of  the  right  of  suffrage.  It  is  
not  a  qualification  for  such  right” Petitioner Moya and respondent Del Fierro were both candidates for
the office of mayor of Paracale, Camarines Norte. Petitioner was
proclaimed as the elected mayor by 91 votes which was contested by
One of the limitations on the right to exercise suffrage is
the respondent. On appeal, the CA declared the respondent the winner
conviction of a crime punishable with a penalty that includes by three votes
disqualification from the exercise of the right to vote
WON the appellate court erred in its act of admitting and counting
The right of the State to deprive persons to the right of suffrage by certain ballots in favor of respondent
reason of their having been convicted of crime, is beyond question.
"The manifest purpose of such restrictions upon this right is to preserve Held: No. As long as popular government is an end to be achieved and
the purity of elections. The presumption is that one rendered infamous safeguarded, suffrage, whatever may be the modality and form
by conviction of felony, or other base offense indicative of moral devised, must continue to be the manes by which the great reservoir of
turpitude, is unfit to exercise the privilege of suffrage or to hold office. power must be emptied into the receptacular agencies wrought by the
The exclusion must for this reason be adjudged a mere disqualification, people through their Constitution in the interest of good government
imposed for protection and not for punishment, the withholding of a and the common weal. (supra)
privilege and not the denial of a personal right.
Maquera v. Borra
159 | P LATON
15 SCRA 7 (1965) promise to reside in the Philippines in the future to be qualified
to voted immediately--a  class  of  “promissory  note  voters.”  Even  
RA   4421’s   (which   requires   “all   candidates   for   national,   provincial,   city   as they might be considered residents-to-be n the promised
and municipal offices...to post surety bond equivalent to one-year
future, the Court has upheld the validity of the law, considering
salary...if   the   candidate   fails   to   obtain   at   least   10%   of   the   votes”)  
constitutionality is in question on the ground that the same is
it within the power of the legislature, pursuant to the
undemocratic and contrary to the letter and spirit of the Constitution. Constitution itself, to determine who are qualified
Moreover, it has the effect of imposing property qualifications in order
that a person could run... Macalintal v. COMELEC
405 SCRA 614 (2003)
The avowed purpose of RA 4421 in requiring a candidate to post a bond
equal  to  a  year’s  salary  of  the  office  for  which  he  will  run  is  to  curb  the   Petitioner, as a taxpayer and a lawyer, challenges the constitutionality
practice of so-called nuisance candidates of certain provisions of RA 9189 (The Overseas Absentee Voting Act of
2003), among others, Section 5(d) which allows the registration of
Held: Yes. A democratic form of government requires that political immigrant or permanent residents...who execute, upon registration, an
rights be enjoyed by the citizens regardless of social or economic affidavit declaring that they shall resume actual physical permanent
distinctions residence in the Philippines not later than three (3) years from approval
of their registration, contrary to Section 1. Art. V of the Constitution
It is within the power of Congress, however, to prescribe the manner of which requires that the voter must be a resident in the Philippines for
exercising political rights so long as it does not run counter to the at least one year and in the place where he proposes to vote for at least
Constitution six months immediately preceding an election

The imposition of unwarranted restrictions and hindrances precluding The seed of the present controversy is the interpretation that is given
qualified candidates from running, is, therefore, violative of the to   the   phrase,   “qualified   citizens   of   the   Philippines   abroad”   as   it  
constitutional guaranty of freedom in the exercise of elective franchise appears in RA 9189

Nuisance candidates, as an evil to be remedied, do not justify the Held: No. RA 9189 was enacted in obeisance to the mandate of the first
adoption of measures that would bar poor candidates from running for paragraph of Section 2, Article V of the Constitution that Congress shall
office provide a system for voting by qualified Filipinos abroad...In the
absence of restrictions, Congress is presumed to have duly exercised its
Elections,  Ballot  Sanctity  and  the  People’s  Will function as defined in Article VI (The Legislative Department) of the
Constitution

“The   Congress   shall   provide   a   system for securing the secrecy Ordinarily, an absentee is not a resident and vice versa; a person
and sanctity of the ballot as well as a system for absentee cannot be at the same time, both a resident and an absentee. However,
voting by qualified Filipinos abroad. under our election laws and the countless pronouncements of the
Court pertaining to elections, an absentee remains attached to his
“The   Congress   shall   also   design   a   procedure   for   the   disabled   residence in the Philippines as residence is considered synonymous with
and the illiterates to vote without the assistance of other domicile
persons. Until then, they shall be allowed to vote under existing
laws and such rules as the Commission on Elections may Additional Cases
promulgate  to  protect  the  secrecy  of  the  ballot.”
39 (T) Suffrage [1]

The object of elections being the selection of persons who are


really chosen by the people to be their representatives, it is
imperative that there be an accurate and trustworthy “Education  is  what  remains  after  one  has  forgotten  everything  
determination   of   the   people’s   will.   If   not,   then   the   citizenry   one  learned  in  school.”  -Albert Einstein
may simply end up having in power persons or pretenders who
were not really their choice. One way of ensuring that the
people are able to really express their choice is to see to it that
the secrecy and sanctity of the ballot are kept intact

The   ballot,   as   an   expression   of   the   voter’s   will,   should   also   be  


inviolate  just  like  the  person’s  right  to  privacy

Absentee Voting

An acknowledgment of the fact that quite a number of Filipinos


have gone abroad for work or otherwise, the Constitution has
also thought it advisable and desirable that they not be
deprived of the chance to express themselves in the political
field simply because they might be physically outside the
Philippines. Thus, the Charter has directed Congress has
directed to provide a system of absentee voting for qualified
overseas Filipinos. Congress, in so complying, however, appears
to have gone overbroad by including even those who would not
otherwise be considered residents. This Congress did by coming
up with Republic Act No. 9189 which allows even those who
39 CONSTITUTION, Art. V, § 2
160 | P LATON

Anda mungkin juga menyukai